UW Boards 1

Pataasin ang iyong marka sa homework at exams ngayon gamit ang Quizwiz!

Most appropriate response to a patient that tells physician she was sexually abused as a child?

"That sounds like an awful experience; are you comfortable talking about it?' Must respond with clear empathy and support. Acknowledge the sexual abuse and gently ask patient is she would like to discuss it further. Follow the patient's lead in exploring the sensitive area. If not ready to talk about it then acknowledge the trauma and convey willingness to discuss it when the patient is ready

Physician delivers news to a pregnant woman that her fetus has down syndrome. Patient begins to cry and says 'this is all my fault, God is punishing me'. Most appropriate response?

'I understand that this news is hard to hear. Please tell me why you feel that this is your fault' Gently explore her belief that the situation is her fault. This will help asses her level of understanding of down syndrome and allow the physician to address the specific reasons behind her self beam (allieviating some of her immediate distress). Be empathetic and use a supportive approach, allowing the patient time to express her grief and process the information. During subsequent visits, educate patient about condition and her options using a culturally and religiously sensitive approaching, offering to answer and questions she may have. Referral to a genetic counsoler is NOT an appropriate intervention initially.

Patient tells you that her child is not her husband child. Husband is also your patient. She asks you not to tell her husband. Your response?

'I will not tell your husband' Must be neutral and not force her into actions with which she may be uncomfortable.

In crowded elevator with multiple hospital staff members hospitalist who covers service during the day asks a physician 'did my patient in room 232 get her CT scan'? What is most appropriate response to hospitalist?

'Lets step off of the elevator and talk privately' Must be cautious about discussing information in public places including public settings in the hospital. Do not reply with any patient information & do not reprimand colleague in public, just ask to speak privately.

Best way to approach a patient who is non adherent with medication?

'Many patients find it difficult to take medication everyday, tell me more about what makes it hard for you' Identifying and managing nonadherance is critical to improving outcomes for many chronic conditions, including HTN. Creating an alliance and using an open-ended, nonjudgemental question is the most effective way to initiate a discussion

Risk of wrong site surgery can be reduced by requiring?

'dual identifiers' (usually a nurse and a physician) to independently confirm that they have the correct patient, site, and procedure. Checks must be truly INDEPENDENT to ensure patient safety

Hardy Weinberg equations?

(see question #16 from 5/24)

Drug binds and activates GABA-A receptors in CNS. Resting potential is -70 mV. What membrane potential will be observed from neuron after exposure to new drug?

-75mV Increase in passive transport of chloride into cell interior causes membrane potential to become more hyperpolarized (more negative then resting membrane potential)

FDA will approve new drug if it plus standard care decreases the rate of cancer recurrence by at least 40% compared to standard therapy alone. Recurrence of standard therapy is found to be 8%. In order for FDA to approve new drug, what is maximal incidence of recurrent disease acceptable for people treated with new drug plus standard therapy?

.40 x 8% = 3.2% (how much it needs to be decreased) 8%-3.2% = 4.8% (maximum acceptable recurrence rate)

23 year old man being evaluated for myoclonic epilepsy of recent onset. PE reveals proximal muscle weakness. Gomori trichome stain of muscle biopsy specimen shows muscle fibers with blotchy red appearance. No family history available. Probability patient's one offspring will inherit the disease?

0% (patient has myoclonic epilepsy with ragged red fibers) Blotchy red ('red ragged') muscle fibers on Gomori trichome stain are characteristic of mitochondrial myopathies. Muscle fibers have this appearance because abnormal mitochondria accumulate under the sarcolemma. Mitochondrial diseases show maternal inheritance so there is no chance of this male passing on disease to offspring. The two other important mitochondrial diseases are Weber optic neuropathy (blindness) & mitochondrial encephalopathy with stroke like episodes and lactic acidosis (MELAS).

3 stages of pertussis?

1. Catarrhal: similar to a routine URI (malaise, rhinorrhea, mild fever) 2. Paroxysmal: severe coughing spells with classic inspiratory whoop or post-tussive emesis lasting greater than 2 weeks 3. Convalescent- cough improves Note: consider pertussis in any adult who has not had updated vaccination boosters. CXR will be normal.

14 year old boy experiences severe, prolonged bleeding following a tooth extraction. He has a history of multiple episodes of painful joint swelling following minor trauma. His parents have no bleeding problems. Patient has an inherited disorder and one of his parents is a genetic carrier. His older sister who doesn't have the condition is pregnant, doesn't know the sex of her child, and wants to know what the risk is that her child will be effected. Best estimate that the child will have the disease?

1/8 Patient is a boy with excessive bleeding and hemarthroses suggesting a diagnosis of hemophilia A or B. Both are X-linked recessive coagulation factor deficiencies. Patient's father doesn't carry the mutation on his X chromosome because he would be affected by the disease if he did. This means that the mother carries the mutation on 1 of her X chromosomes Probability sister is carrier= 1/2 Probability offspring will inherit the X chromosome=1/2 Probability child will be a boy (female would only be a carrier) = 1/2 1/2 x 1/2 x 1/2 = 1/8

2 year old girl with ambiguous gentalia. Born with clitoral enlargement & partial fusion of labioscrotal folds. High BP and hypokalemia. Most likely deficienct enzyme?

11 beta hydroxylase (11-deoxycorticosterone accumulates and acts as a weak mineralocorticoid leading to HTN and hypokalemia & excessive androgens cause ambiguous genitalia)

Intense eosinophilic staining of the neuronal cytoplasms ('red neurons') & loss of Nissl substance occurs how long after ischemic injury?

12-24 hours (first microscopic changes)

Consistent use of barrier contraceptives is extremely important for preventing sexually transmitted infections including HPV. Patients infected with what strains of HPV are at risk of squamous cell carcinoma of the cervix?

16, 18, 31

Most common form of congenital adrenal hyperplasia is 21 hydroxyls deficiency. Genetically female infants typically have ambiguous genitalia (virilization) at birth where as males have phenotypically normal genetalia, with salt wasting or precocious puberty appearing later. Elevated levels of what substance are diagnostic?

17-hydroxyprogesterone

Class 1 antiarrythmmics block voltage gated Na+ channels. Bind during open/inactivated states & dislocation of drug occurs during resting state. Tissues undergoing frequent depolarization become more susceptible to blockage (use dependence). What is the order of sodium channel binding strength for the subclasses of class 1 antiarrythmics?

1C>1A>1B Class 1B dissociation from the channels occurs so rapidly that there is minimal cumulative effect over multiple cardiac cycles (little use dependence) making them more selective for ischemic myocardium & ischemia induced ventricular arrhythmias. Class 1C has greatest blockade at higher heart rates making them effective at terminating tachyarrythmias.

Pharyngeal arch in which bony derivatives include maxilla, zygoma, mandible, incus & malleus and muscular derivatives include muscles of mastication & is associated with the trigeminal nerve (CNV)

1st pharyngeal arch

Probability that a child of 2 parents from 2 populations with different mutant allele carrier frequencies will inherit an autosomal recessive disease is?

25% multiplied by the carrier frequencies Example: Carrier frequency in mother population is 1/3 and carrier frequency in father population is 1/100. Probability of affected child is 1/4 x 1/30 x 1/100 = 1/12000

Pharyngeal arch in which bony derivatives include styloid process of temporal bone, lesser horn of hyoid & stapes and muscular derivatives include muscles of facial expression & is associated with facial nerve (CN VII)

2nd pharyngeal arch

Lesser horn of hyoid is derived from what pharyngeal arch?

2nd pharyngeal arch (NOTE: GREATER horn of hyoid derived from 3rd pharyngeal arch)

After vasectomy (transection of vas deferens), viable sperm remain in the portion of the vas deferens distal to the ligation. Patient can still have viable sperm in the ejaculate for how long?

3 months and at least 20 ejaculations Vas deferens functions not only as a transport duct from the epididymis to the ejaculatory duct, but also serves to store and protect sperm following spermatogenesis Another method of birth control should be used after vasectomy until semen analysis confirms azoospermia

Tetracycline, doxycycline, and aminoglycosides are antibiotics that act on what ribosomel subunit?

30S subunit

Glossopharyngeal nerve (CN IX) is derived from what pharyngeal arch?

3rd pharyngeal arch

How many half lives does it take to reach steady state concentration during continuous infusion of a drug metabolized by first order kinetics (constant fraction of drug eliminated per unit time)?

4-5 half lives

Complete mole has no fetal structures & is composed entirely of large, edematous, disordered chorionic vilil that appear grossly as clusters of vesicular structures ('bunch of grapes'). Typically presents with pelvic pain & vaginal bleeding. Uterus larger than expected & B-hCG levels markedly elevated due to trophoblastic hyperplasia. US shows central heterogeneous mass with multiple cystsc areas described as 'swiss cheese' or 'snowstorm'. Risk factors are extremes of lateral age, prior molar pregnancy, prior miscarriage, infertility. Most likely karyotype?

46 XX. Most commonly results from fertilization of an ovum with no maternal chromosomes by one sperm (23X). Chromosomes from haploid sperm are then duplicated forming 46X that only contains paternal DNA. Less commonly 2 sperm can fertilize an empty ovum creating 46 XY.

Two pharyngeal arches that make up the cartilaginous structures of the larynx, including cricoid and thyroid cartilages

4th and 6th pharyngeal arches

Vagus nerve is derived from what pharyngeal arches?

4th and 6th pharyngeal arches

Phenotypically male infant with hypospadias and small phallus. Testes are well developed but reside in inguinal area. Serum testosterone is WNL. Karyotype shows 46, XY. Most likely deficient enzyme in this patient?

5 alpha reductase Conversts T to DHT which mediates development of the external genitalia in the male fetus. Male neonates with 5 alpha reductase deficiency are both with feminized external genitalia that typically masculinize at puberty & a small phallus and hypospadias are common

BPH can be treated with what two classes of medication?

5 alpha reductase inhibitors (finasteride)- inhibit action of androgens on prostate gland, preventing conversion of testosterone to DHT, limiting further prostate enlargement Alpha adrenergic blockers (tamsulosin)- relax the smooth muscle in the bladder neck and prostate gland & are used to control symptoms

Leukotriene B4 stimulates neutrophil migration to sites of inflammation. Other important chemotactic agents include?

5-HETE (leukotriene precursor), complement component C5a, IL-8

Breakdown product of serotonin that is used to screen for carcinoid syndrome

5-hydroxyindoleacetic acid

63 year old man with atherosclerotic narrowing of right renal artery. Flow in the artery is decreased by a factor of 16 compared to the left. What percentage has the radium of the lumen been reduced?

50% Blood flow is directly proportional to the vessel radius to the 4th power. Resistance to blood flow is inversely proportional to the vessel radius raised to the 4th power

Chloramphenicol, clindamycin, linezolid, and macrolides are antibiotics that act on what ribosomal subunit?

50S subunit

What pharyngeal/branchial arch in the developing embryo does not contribute to any adult structures?

5th pharyngeal arch

68-95-99.7 rule?

68% of all observations lie within 1 SD of the mean with half of those above and half of those below the mean. The remaining 32% lie outside 1SD from the mean with half of those above and half of those remaining observations above and the other half below 1SD from the mean. 95% of all observations lie within 2SD and 99.7% lie within 3 SD from the mean SO if the mean is 220 68% lie within 1SD: 220 +/- 10 = 210-230 95% lie within 2SD: 220 +/- 20 - 200-240 99.7 lie within 3SD: 220 +/- 30 = 190-250 TO find out how many people would have greater than 240 in population of 400: because 240 is 2SD from the mean, 5% outside & 2.5% must be below 200 and 2.5% just be above 240. SO if population is 400 then .025 (2.5%) x 400 = 10 people. 10 people would have above 240 and 10 people would have below 200.

Partial moles occur due to the fertilization of an ovum by 2 sperm. Typically contain fetal tissue & normal placental villi intermixed with hydronic villi. What are possible karyotypes?

69 XXX, 69 XXY, 69 XYY

Inhibition of what enzyme reduces the conversion of cholesterol to bile acids and increases the risk of cholesterol stones?

7-alpha-hydroxylase

Sunlight exposure catalyses what vitamin D conversion in the skin?

7-dehydrocholerterol to cholecalciferol

Two parents with sickle cell trait. Probability their child will inherit ONE OR MORE mutant alleles from the parents?

75%

Best and most reliable auscultatory indicator of the degree of mitral stenosis?

A2 (aortic valve closure) to opening snap time interval. A shorter interval indicates mores severe stenosis. The opening snap occurs due to abrupt tensing of the valve leaflets as the mitral valve reaches its maximum diameter during forceful opening. As MS worsens, left atrial pressure increase due to impaired movement of blood into the LV. Higher pressure causes the valve to open more forceful, as a result, the interval becomes shorter as the LA pressure increases. Other auscultatory findings in MS can include a diastolic rumbling murmur with pre systolic accentuation due to left-atrial contraction

Drug class that should be avoided in patients with low C1 esterase inhibitor activity?

ACE inhibitors Patients with hereditary angioedema have increased levels of activated kallikrein and bradykinin levels. ACE converts bradykinin into inactive metabolites so ACE inhibitors can leads to bradykinin accumulation.

For patients dependent on efferent arteriole constriction to maintain renal perfusion (those with renal artery stenosis) what drug class can be detrimental by precipitating acute renal failure?

ACE inhibitors (can cause an acute rise in the serum creatinine by blocking angiotensin mediated efferent arteriole vasoconstriction leading to reduction in filtration fraction)

Risk of progression of diabetic nephropathy in patients with proteinuria can be reduced by appropriate glycemic and BP control. Preferred antihypertensive agents due to their blood pressure independent anti-proteinuric effects?

ACE inhibitors and angiotensin II receptor blockers Microalbuminuria (loss of 30-300mg/day) is indicative of nephropathy in diabetic patients. Only albumin excretion >300mg/day (macroalbuminuria) can be detected by conventional UA .

Long term use of what medications have been show to reduce mortality in HF patients?

ACE inhibitors and angiotensin II receptor blockers. They are recommended in all patients with systolic HF. In addition to improving hemodynamics (improved BP control), they inhibit the chronic angiotensin II mediated adverse cardiac remodeling that occurs with HF (adding additional mortality benefit) Note: diuretics provide symptomatic improvement but don't reduce mortality & digoxin improves symptoms/reduces hospitalization but doesn't reduce mortality.

Peptapeptide with a strong affinity to delta and mu opioid receptors is isolated. What substance is most likely to have a common molecular origin with this pentapeptide?

ACTH Beta-endorphin is an endogenous opioid peptide that is derives from proopiomelanocortin (POMC). POMC is a polypeptide precursor that goes through enzymatic cleavage and modification to produce not only beta endorphins, but also ACTH and MSH. The fact that beta endorphin and ACTH are derived from the same precursor suggest that there may be a close physiological relationship between the stress and the opioid system

An important cause of SIADH is a paraneoplastic effect secondary to small cell carcinoma of the lung. Prodound hyponatremia that occurs can result in headache, weakness, altered mental status, and seizures. SIADH is characterized by low plasma sodium and osmolality, inappropriately concentrated urine, and clinically normal volume status (euvolemic hyponatremia). Mechanism for normal volume status?

ADH causes excessive water absorption by the kidneys leading to a transient subclinical hypervolemia. This increase in ECF volume suppresses the RAAS axis and stimulates the production of natriuretic peptides, leading to excretion of sodium in the urine.

Lead poisoning inhibits what enzymes?

ALA dehydratase and ferrochelatase

Pathogenesis of ARDS?

ARDS is caused by injury of the pulmonary epithelium and/or endothelium and occurs most often due to sepsis or pneumonia. Cytokines recruit neutrophils to the lung tissue which cause capillary damage and leakage of protein rich fluid into the alveoli. Later, cellular proliferation and collagen deposition occurs, and in some patients, leads to irreversible pulmonary fibrosis

Attributable risk percent in the exposed represents the excess risk in the exposed population that can be attributed to the risk factor. It can be derived from the relative risk using the formula?

ARP= 100 x [(RR-1)/RR] RR= risk in exposed/risk in unexposed

Absolute risk reduction formula

ARR = event rate in control group - event rate in treatment group Note: the event rate represents the number of events divided by the number of subjects in that group.

Liver enzyme abnormalities in alcoholic hepatitis?

AST>ALT ratio greater than 2:1

What substance binds directing to potassium channels expressed in pancreatic beta cells to regulate their activity?

ATP Oxidative metabolism of glucose in pancreatic beta cells generate ATP. ATP induced closure of ATP sensitive K+ channel leads to membrane depolarization and subsequent insulin release

Conduction speed of AV node, ventricular muscle, atrial muscle, and purkinje system?

AV node: 0.05 m/s Ventricular muscle: 0.3 m/s Atrial muscle: 1.1 m/s Purkinje system: 2.2 m/s The cardiac action potential condition speed is slowed in the AV node and fastest in the purkinje system. Conduction speed of the atrial muscle is faster than that of the ventricular muscle Cardiac impulses originate in the SA node. SA node depolarization delivers an electrical impulse to the surrounding atrial myocardium which carries the action potential to the AV node at a rate of 1.1. Speed of the AV node is the slowest (0.5) and this delay allows the ventricles to completely fill with blood during diastole. From the AV node action potential enters the His-purkinje system where is travels (2.2) and ensures that the ventricles contract in a bottom up fashion (necessary for efficient propulsion of blood into the pulmonary artery and aorta). From the purkinje fibers, the action potential is transmitted to the ventricular myocardium where is travels at a rate of 0.3.

43 year old man with HIV infection who stopped treatment with HAART 6 months ago is recovering from pneumocystis pneumonia. Testing for HLA-B*57:01 is performed and results are positive. This finding is relevant if the patient is considered for what medication?

Abacavir (nucleoside reverse transcriptase inhibitor used in treatment of AIDS/HIV) Delayed hypersensitivity (type IV) reaction to Abacavir occurs in 2-8% of patients and is strongly associated with HLA-B*57:01 allele. Reaction is due to direct binding of Abacavir to a segment on the HLA-B*57:01 molecule which alters the presentation of self peptides to the immune system. Characterized by fever, malaise, GI symptoms, and delayed rash.

GP IIb/IIIa receptor antagonist which normally promotes platelet binding to fibrinogen. Useful for treatment of unstable angina and acute coronary syndrome, particularly in patients undergoing percutaneous coronary intervention

Abciximab

Monoclonal antibody against platelet GP IIb/IIIa receptor that blocks the final step in platelet aggregation. Often administered during angioplasty in patients with acute coronary syndrome

Abciximab

Testicular torsion, characterized by acute, sever pain with nausea, vomiting, and asymmetrically high riding testis and absent cremasteric reflex, is due to twisting of the testis around the spermatic cord (containing the gonadal artery) which can lead to ischemia. Where does the goal artery arise from?

Abdominal aorta

Where does lymph from testes drain?

Abdominal para-aortic (retroperitoneal) nodes

Patient's with decision making capacity have the right to refuse any treatment. Capacity to refuse treatment requires?

Ability to express a choice, understand the relevant medical information, appreciate the consequences of treatment options, and offer rationale for the decision

4 year old boy with a progressively worsening cough for the past 2 days productive of yellow sputum. Has had multiple episodes of pneumonia. Testing shows a high chloride content in his sweat. What abnormality most likely to be seen in this patient?

Abnormal post-translational processing of a transmembrane protein (improper folding and glycosylation of CFTR) Cystic fibrosis is most commonly due to a 3 base pair deletion in the CF transmembrane conductance regulator (CFTR) gene at amino acid position 508. This mutation impairs post-translational processing of CFTR, resulting in the shunting of CFTR toward the proteosome, with complete absence of the protein on the cell surface. Certain drugs (lumacaftor) can partially correct this folding defect, leading to expression of functional CFTR. Elevated sweat chloride concentrations are found in most patients with CF.

What is dynamic left ventricular outflow tract obstruction due to in patients with hypertrophic cardiomyopathy?

Abnormal systolic anterior motion of the anterior leaflet of the mitral valve toward a hypertrophied interventriuclar septum

Infection in which the virus enters the cell but does not successfully produce new infective virus and therefore is not capable of causing disease

Abortive viral infection

56 year old with chest palpitations. Feels heartbeat is fast and very irregular. PE confirms presence of irregularly, irregular rhythm with HR of 120. Consumed a large amount of alcohol last night and normally only drinks 2-3 times a year. Most likely ECG finding?

Absent P waves Palpitations refer to subjective sensation/awareness of the heartbeat due to rapid arrhythmias or forceful ventricular contractions. Atrial fibrillation is most common cause of an irregularly irregular rhythm and is detected on ECG by an absence of organized P waves and varying R-R intervals. Its precipitate by acute systemic illness (longstanding HTN, HF, hyperthyroidism), increased sympathetic tone, and sometimes excessive alcohol consumption.

Acute intermittent porphyria attacks are due to? Management?

Accumulation of ALA and PBG resulting from inherited PBG deaminase deficiency COMBINED with ALA synthase induction (typically due to meds, alcohol or low calorie diet). Manage with glucose or hemin (inhibits ALA synthase activity)

Ability of a test to measure what it's supposed to measure

Accuracy

Analgesic and antipyretic that reversibly inhibits COX enzymes (primarily in CNS). Lacks anti-inflammatory properties due to weak inhibition of COS in peripheral tissues

Acetaminophen

Metabolic fate of pyruvate

Acetyl CoA directly stimulates pyruvate carboxylase which is the first step in hepatic conversion of pyruvate to glucose (allows pyruvate to be shunted toward acetyl-CoA production when acetyl-CoA levels are low preventing cell from being depleted of energy). When acetyl CoA levels are high (with increased beta oxidation of FA during fasting) pyruvate carboxylase can operate at full capacity & convert most of the pyruvate into oxaloacetate for use in gluconeogenesis

What modification of histones weakens the DNA-histone bond making DNA segments more accessible, enhancing gene transcription?

Acetylation

Isoniazid is metabolized by?

Acetylation The speed with which a patient is able to acetylate drugs depends on whether they are genetically 'fast' or 'slow' acetylators. The presence of fast and slow acetylators within the same population results in a bimodal distribution of the speed of isoniazid metabolism. Slow acetylators are at increased risk of adverse side effects while fast acetylators may require much higher therapeutic doses to achieve a therapeutic effect Note: slow acetylators of isoniazid also metabolize (acetylate) dapsone, hydrazine, and procainamide slowly causing accumulation of these drugs as well

Medullary chromatin cells are modified postganglionic sympathetic neurons that release catecholamines into the bloodstream in response to what?

Acetylcholine released by preganglionic neurons

Neural tube defects most often occur when the neural folds fuse in the region of the anterior or posterior neuropores. Persistent communication between the spinal canal and amniotic cavity allows leakage of what two substances into the amniotic fluid that can be detected on amniocentesis

Acetylcholinesterase and alpha fetoprotein

Acetaminophen causes liver damage (jaundice, elevated liver enzymes) following a latent damage. What is used to treat acetaminophen toxicity?

Acetylcysteine (a glutathione donor)

Autosomal dominant or sporadic (associated with advanced paternal age) disorder involving a defect that results in constitutive activation of FGFR3, which normally has a negative regulatory effect on bone growth. Abnormal endochondral ossification resulting in failure of longitudinal bone growth. Membranous ossification is unaffected so patients have short limbs with normal sized torso & large head. Associated with pectus excavatum, kyphoscoliosis, & genu varum. INTELLECTUALLY NORMAL.

Achondroplasia

In situations in which a parent's presence may interfere with obtaining honest answers from an adolescent patient what should physician do?

Acknowledge parents concerns, politely ask parent to wait outside, and interview patient privately (also important when discussing drugs, alcohol, tobacco and sexual activity with teenagers).

First step in dealing with an angry patient?

Acknowledge their anger with a phrase such as 'I see you are upset, lets discuss your concerns and questions'.

First step in discussion with patient about their non-adherence to their medication?

Acknowledging that is it difficult to take meds daily (always empathize with difficulty of taking medication regularly FIRST to build relationship which may improve patient receptiveness)

Catalyzes the isomerization of citrate to isocitrate in citric acid cycle

Aconitase

MOA of benzodiazepines?

Act by binding to the benzodiazepine binding site, which allosterically modulates the binding of GABA, resulting in an increased FREQUENCY of chloride channel opening. The influence of chloride ions into the neurons causes neuronal hyperpolarization and inhibition of the action potential

55 year old with malaise and cough for 2 months. Reports yellow sputum with occasional streaks of blood. Smokes a pack of cigarettes a day. History of alcohol abuse with prior episodes of binge drinking. Exam shows poor dentition with dental caries, gingivitis, and enlarged submandibular lymph nodes. Coarse rhonchi in right lung. Extensive right lung consolidative process with air bronchograms on CT. Most likely diagnosis?

Actinomycosis Slowly progressive disease caused by gram positive anaerobic bacteria. Organisms typically colonize mouth, colon, and vagina and can be found in dental caries and margins of gums in patients with poor dentition. Most frequently causes cervicofacial abscess, but systemic infection can develop when mucosa is disrupted. PULMONARY actinomyces develops following aspiration which leads to lower lobe consolidation with air bronchograms (air filled bronchi with surrounding alveolar opacification). Diagnosis made by identifying bacteria with unique filamentous, branching patterns and characteristic suffer granules. Treat with Penicillin.

Atrial natriuretic peptide, brain natriuretic peptide and nitric oxide all act via what intracellular signaling mechanism?

Activate guanylyl cyclase and increase conversion of guanosine 5 triphosphate to cGMP. cGMP then activates downstream cGMP dependent protein kinase, leading to relaxation of vascular smooth muscle and subsequent vasodilation. ANP and BNP exert this effect by binding receptors while NO exerts the same effect by binding a cytosolic guanylyl cyclase Note: phosphodiesterase inhibitors (sildenadfil) decreased degradation of cGMP leading to relaxation of vascular smooth muscle and vasodilation. Never combine sildenafil with nitrates as it can lead to excessive cGMP induced vasodilation and severe hypotension

Symptoms of vitamin D intoxication (mental status changes, muscle weakness, constipation, polyuria/polydipsia) are due to hypercalcemia. Granulomatous diseases (sarcoidosis, TB) are also associated with hypercalcemia due to?

Activated macrophages in sarcoidosis and other granulomatous diseases express 1 alpha hydroxylase, leading to the production of excess 1,25 dihydroxyitamin D and hypercalemia (PTH independent conversion of 25 hydroxyvitamin D to 1,25 dihydroxyvitamin D)

Typically occurs in critically ill patients (those with sepsis, severe burns, trauma, immunosuppression) due to gallbladder stasis & ischemia. Clinical findings may be subtle and include fever, RUQ pain, and leukocytosis.

Acute acalculous cholecystitis (inflammation of gallbladder in the absence of gallstones)

Restoration of arterial blow flow to an affected limb floods ischemic tissue with oxygen causing?

Acute compartment syndrome secondary to repercussion injury

HBV infection can produce one of three syndromes: acute hepatitis with complete resolution, chronic hepatitis (with or without cirrhosis and the attendant increased risk of hepatocellular carcinoma), or fulminant hepatitis with massive liver necrosis. By far the most common outcome is?

Acute hepatitis with mild or subclinical symptoms that eventually completely resolve

Trisomy 21 is detectable by cytogenetic karyotype analysis and is the most common genetic cause of congenital mental retardation. Patient's with down syndrome are at risk of developing what two types of cancers?

Acute lymphoblastic and acute myelogenous leukemia

Chronic inflammation is associated with increased circulating pro-inflammatory cytokines (IL-1, IL-6, TNF-alpha) which stimulate the liver to release?

Acute phase reactants (CRP, fibrinogen)

Most statins are metabolized by cytochrome P-450 3A4 (with the exception of pravastatin). Contaminant administration of drugs that inhibit statin metabolism (ex: erythromycin) is associated with increased incidence of statin induced myopathy and rhabdomyolysis and can lead to?

Acute renal failure If patient is on an agent that inhibits CYP-450 3A4, pravastatin is the statin of choice!

Patella fractures are most commonly die to a direct impact to the anterior aspect of the knee. What are physical exam findings associated with fractured patella?

Acutely swollen knee, focal patella tenderness, inability to extend knee against gravity, & palpable gap in extensor mechanism

Impaired beta oxidation of fatty acids causes hypoglycemia after prolonged fasting and inappropriately low levels of ketone bodies. What enzyme catalyzes the first step in the beta oxidation pathway and is the most commonly deficient enzyme?

Acyl-CoA dehydrogenase Note: affected patients may remain asymptomatic for long periods until they experience a significant fast; treatment requires prevention of catabolism- avoid prolonged fasting as well as promptly supply glucose during periods of illness

Intern calls patient by their first name. Most appropriate response by the attending physician?

Address the patient by her surname and ask her preferred form of address Older patients, in particular should be addressed formally unless they indicate otherwise when asked

49 year old man with drug cough that occurs mainly at night, has been intermittent for the past several years. Rerpots frequent sore throat and occasional epigastric discomfort. PMH includes T2DM and obesity. Microscopic findings of lower esophageal biopsy reveal intestinal type columnar epithelium with goblet cells. Due to these findings he is most at risk for?

Adenocarcinoma Barrett esophagus is a metaplastic condition in which the normal squamous epithelium of the distal esophagus is replaced by intestinal type epithelium. It occurs most often in longstanding acid reflux and is associated with increased risk of adneocarcinoma

42 year old female with menorrhagia and dysmenorrhea. Uniformly enlarged uterus with normal appearing endometrial tissue noted on biopsy. Negative urine pregnancy test. Most likely diagnosis?

Adenomyosis- endometrial glandular tissue within the myometrium

Patient with palpitations treated with rapid administration of a medication that results in instantaneous resolution of palpitations but causes short lived flushing, burning in chest, SOB. What medication?

Adenosine Causes hyperpolarization of the nodal pacemaker to briefly block conduction through AV node; effective in initial treatment of paroxysmal SVT.

Phase 4 slow depolarization in cardiac pacemaker cells occurs due to the closure of repolarizing K+ channels, the slow influx of Na+ through funny channels, and the opening of T-type Ca2+ channels. What two substance reduce the rate of spontaneous depolarization in cardiac pacemaker cells by prolonging phase 4?

Adenosine & acetylcholine

Collateral microvessels form adjacent pathways for blood flow to areas distal to an occluded vessel. Selective vasodilators of coronary vessels can cause coronary steal, a phenomenon in which flood flow in ischemic areas is reduced due to arteriolar vasodilation in nonishemic areas. Coronary steal can lead to hypo perfusion and worsening of existing ischemia. What selective coronary vasodilators can cause this?

Adenosine & dipyridamole

Mother and daughter in car crash and both need blood transfusions and are not responsive. Physician spoke to woman's husband & child's biological father on phone who informs physician that they are Jehovah's witnesses and he is on his way to the hospital. Father then hangs up before physician can relay any specifics about their condition and is unable to reached. Most appropriate course of action?

Administer blood products to both patients Adult patients with capacity have the authority to refuse any form of treatment including life saving therapies. In an emergency situation in which the patient doesn't have the capacity, consent is not required to provide life saving treatment. In an emergency, the physician should ALWAYS provide potential life saving therapy to a minor. To expresses their wishes if incapacitated, many Jehovah's witnesses carry a card that identifies their desire to refuse blood product. But if unresponsive and not carrying a blood refusal card, s & communication with the surrogate decision maker is not possible (such as this case, then the physician should transfuse.

Patient with medullary thyroid cancer found to have a gremlin RET mutation should be screened for an abnormality in what oran prior to undergoing thyroidectomy?

Adrenal medulla In patients with undiagnosed pheochromocytoma, induction of anesthesia (preparation for thyroidectomy) can precipitate a catecholamine surge, leading to hypertensive crisis, flash pulmonary edema and atrial fibrillation. Perform 24 hour assay for urinary metanephrines and catecholamines prior to surgery in patients with MTC associated with germline RET mutations

Post-streptococcal glomerulonephritis is most common cause of acute pediatric glomerulonephritis and presents with acute onset of malaise, periorbital edema, hypertension, and microscopic or gross hematuria (cola or tea colored). What characteristic is most likely to indicate a poor long term prognosis?

Adult onset Most children recover completely with anti-hypertensives and salt/water restriction & diuretics for edema, but adult patients have a relatively poor prognosis and higher risk of chronic hypertension, recurrent proteinuria, rapidly progressive glomerulonephritis, and chronic renal insufficiency. Pre-existing kidney disease is another poor prognostic factor.

Cheyne-Stokes breathing is cyclic breathing following by gradually increasing then decreasing tidal volumes until next apneic period and is commonly seen in the setting of what disease?

Advanced congestive heart failure

Woman with advanced directive that specifies no intubation. Both daughter and husband are strongly advocating for intubating the patient. What should physician should follow regarding the decision to intubate?

Advanced directive Advanced directives take precedence over the wishes of family members. Physicians should respect the patient autonomy and adhere to patients' wishes as outlined in advanced directives.

46 year old woman dies in hospital from respiratory failure after a prolonged illness. Had multiple comorbidities, including advanced renal disease. Autopsy reveals multiple small, nondestructive masses attached to the edges of the mitral valve leaflet. Microscopy reveals that the masses are composed of platelet rich thrombi, but cultures reveal no bacterial growth. What disease most likely associated with this patients condition?

Advanced malignancy Nonbacterial thrombotic endocarditis is a form of noninfectious endocarditis characterized by deposition of sterile platelet thrombi on cardiac valves. Its commonly associated with advanced malignancy and can also occur with chronic inflammatory disorders (antiphospholipid syndrome, SLE) and DIC with sepsis. Vegetations can easily dislodge in NBTE and are more likely than infective endocarditis to embolize causing infarction

Afferent and efferent limbs of gag reflex?

Afferent is glossopharyngeal (CN IX) and efferent is vagus (CN X)

Afferent and efferent limbs of corneal reflex?

Afferent is trigeminal (CN V--> nasocilary branch of ophthalmic nerve V1) and efferent is facial (CN VII)

Isolated systolic HTN (systolic pressure >140 with diastolic <90) is due to?

Age related stiffness and decrease in compliance of the aorta and major peripheral arteries Note: increase in sympathetic tone leads to an increase in PVR and increases SBP and DBP (this is an important role in the pathogenesis of HTN in patients less than 50, but stiffening is the major cause of isolated systolic HTN in patients above 60.

Monoamine oxidase is a mitochondrial enzyme that degrades excess monoamine neurotransmitters in presynaptic nerve terminals and detoxifies dietary tyramine in the GI tract. Tyramine hypertensive crisis (severe HTN, headache, blurry vision) can occur in patients taking MAO inhibits following the consumption of food containing high amounts of tyramine such as?

Aged cheeses, cured meats, draft beer

Intravenous acyclovir can cause crystalline nephropathy characterized by an acute elevation of serum creatinine. How can this nephropathy be prevented & treated?

Agressive IV hydration

Major amino acid responsible for transferring nitrogen to liver for disposal?

Alanine (amino groups are transferred from alanine to alpha-ketoglutarate to form glutamate)

Screening of early stage diabetic nephropathy is best accomplished by measuring the urinary concentration of what substance?

Albumin Moderately increased albuminuria (urine albumin levels less than 300 mg/day) is the earliest manifestation of diabetic nephropathy, the leading cause of end stage renal disease in the USA. Screening for early diabetic nephropathy is best achieved through use of albumin specific urine dipsticks.

Minimal change disease is caused by immune dysregulation and overproduction of glomerular permeability factor, which damages podocytes and DECREASES AIONIC properties of the glomerular basement membrane. This results in selective loss of what substance in the urine?

Albumin (in contrast to the nonselective proteinuria seen with other forms of nephrotic syndromes such as membranous nephropathy and focal segmental glomeulosclerosis)

When inserting a central venous catheter, in addition to sterile draping during the procedure, what is most likely to prevent intravascular catheter related infections?

Alcohol based hand scrub prior to beginning the procedure Most important steps for prevention of central venous catheter infections: -Proper hand hygiene (soap and water or alcohol sanitizer) -Full barrier precautions during insertion -Clorhexidine skin disinfection prior to insertion -Avoidance of femoral insertion site (higher rate of infection) -Removal of catheter when no longer needs Note: catheter replacement at scheduled intervals does NOT reduce infection rates

Enzyme that converts galactose to galactitol?

Aldose reductase

COPD in a heavy smoker may consist of both emphysema and chronic bronchitis and thus may presents with moth progressive exertion dyspnea (characteristic of emphysema) and frequent respiratory infections (chronic bronchitis). Expect PFT results?

All COPD yields a decreased FEV1/FVC ratio. Emphysema results in increased TLC and RV. Note: restrictive lung diseases can cause reduced TLC and RV and increased FEV1/FVC.

Type of bias that results from the way patients are ASSIGNED to the treatment and control groups. May occur when subjects are non randomly assigned to study groups of a clinical trial (example: physicians may preferentially enroll sicker patients into the experimental group)

Allocation bias Note: differs from selection bias which occurs when the studied sample does not represent the general population (because of nonrandom selection)

Activation of these receptors increases smooth muscle tone of the iris sphincter (causing mydriasis) and blood vessels (vasoconstriction)

Alpha 1 adrenergic agonists (Note: they can be used in patients with allergic rhinitis because their vasoconstriction action reduces nasal congestion, but vasoconstriction INHIBITS FLUSHING as opposed to antimuscarininc action of first generation antihistamines)

Type of medication commonly used for symptomatic BPH and often associated with orthostatic hypotension (drop in systolic >20 mmHg or diastolic >10 mmHg) resulting in lightheadedness and syncope due to cerebral hypo perfusion upon standing?

Alpha 1 adrenergic antagonists

Drug that causes a decrease in uterine contractions and pupillary dilation must be characterized as what type of adrenergic drug?

Alpha and beta adrenergic agonist Inhibition of uterine contractions (tocolysis) is a result of B2 adrenergic receptor stimulation. Alpha 1 receptor stimulation causes contraction of the ocular pupillary dilator muscle resulting in mydriasis.

Connective tissue disorder affecting type IV collagen that presents with hematuria, proteinuria, and HTN. Associated with hearing loss.

Alport syndrome (significant amount of type IV collagen in basement membrane of renal glomeruli)

Aspiration pneumonia & lung abscesses are associated with?

Altered consciousness (alcoholism, seizures, dementia), impaired swallowing (NG tube, dysphagia), and gingival or upper GI tract disease

31 year old man with HIV is diagnosed with pulmonary TB and started on 4 drug combination therapy. 3 weeks later he reports red urine and red staining of his contact lenses. Drug susceptibility test of his sputum isolates of M. tuberculosis shows resistance to several antimycobacterial agents. What explains the bacterial resistance to the drug responsible for his current symptoms?

Altered structure of enzymes involved in bacterial RNA synthesis The rifamycins block the action of the bacterial DNA dependent RNA polymerase, inhibiting transcription. Resistance is acquired by modification of the rifampin binding site on the bacterial DNA dependent RNA polymerase. Common side effects include hepatotoxicity, blood dycrasias, and harmless red-orange discoloration of body fluids. Note: before isonazid can inhibit mycolic acid synthesis, it must be converted to its active form by the mycobacterial catalase-peroxidase enzyme

Physicians should remain alert to patients with a low level of literacy as this can often impair communication and result in low quality medical care and poor adherence. Most appropriate course of action when dealing with such a patient?

Alternative learning methods (such as visual resources)

Process where the eons of a gene are reconnected in multiple ways during post-translational processing. Creates different mRNA sequences and subsequently different protein iso forms. Its a normal phenomenon in eukaryotes that increases the biodiversity of proteins encoded by the genome

Alternative splicing

Process by which a single gene can code for various unique proteins by selectively including or excluding different DNA coding regions into mature RNA

Alternative splicing Cancer cells may develop the ability to splice out a particular exon that codes for the transmembrane domain of the FAS receptor converting it to a soluble form not expressed on cell surface, allowing for evasion of apoptosis

55 year old man hospitalized with pancreatitis. During hospitalization, his condition deteriorates and he develops severe respiratory distress. There are crackles bilateral on PE and infiltrates bilaterally on CXR. Patient doesn't improve with mechanical ventilation and 100% O2 and dies a few days later. What lung autopsy finding is most likely?

Alveolar hyaline membranes Pancreatitis is a major risk factor for ARDS as it results in the release of a large amount of inflammatory cytokines and pancreatic enzymes, which leads to activation of neutrophils in the alveolar tissues. During the initial phase, interstitial and intraalveolar edema, inflammation, and fibrin deposition cause the alveoli to become lined with waxy hyaline membranes.

Elastase is derived from what cell?

Alveolar macrophages Alveolar fluid contains neutral proteases (elastases) that are derived from infiltrating neutrophils and alveolar macrophages. These proteases can cause destruction of terminal lung parenchyma (emphysema) when secreted in excess or if left uncheck by deficient antiprotenase activity

23 year old asymptomatic male is found to be homozygous for apolipoprotein E-4 allele. In the future, he is most likely to suffer from?

Alzheimer dementia Early onset familial Alzheimer disease (before age 60) is associated with three gene mutations: APP (chromosome 21), presenilin 1 on chromosome 14 and presenilin 2 on chromosome 1. Late onset familial Alzheimer disease is associated with apolipoprotein E4 genotype.

Inappropriate processing of amyloid precursor protein (located on chromosome 21) down the beta and gamma secretase pathway results in production of beta amyloid. Insufficient clearance of beta-amyloid results in formation of extracellular amyloid plaques which are neurotoxic and play key role in pathogenesis of what disease?

Alzheimer's dementia Note: processing via alpha and gamma secretase pathway does not result in amyloidogenic proteins

Cytoprotective free radical scavenger used to decrease nephrotoxicity associated with platinum containing and alkylating chemotherapeutic agents and to decrease xerostermia (dry mouth)

Amifostine

Patient with stable angina and long standing HTN. Labs reveal hyperkalemia with a potassium of 5.6. What drug in combination with lisinopriyl is most likely to account for patient's hyperkalemia?

Amiloride ACE inhibitors reduce vasoconstriction and aldosterone secretion by blocking conversion of angiotensin I to angiotensin II. Decreased aldosterone causes increased K+ retention which can lead to hyperkalemia (usually in patients with renal insufficiency and in those taking K+ sparing diuretics such as amiloride, triamterene, spironolactone or those taking K+ supplements)

Antifibrinolytic agent that inhibits plasminogen activators and, to a lesser degree, antiplasmin activity; helps achieve hemostasis when fibrinolysis is the cause of bleeding

Aminocaproic acid

Rate limiting enzyme in hepatic pathway of heme synthesis?

Aminolevulinate (ALA) synthase

Causes lengthening of the cardiac action potential which manifests as QT interval prolongation on ECG, but in contrast to other drugs, is associated with a very low risk of torsades de pointes

Amiodarone

65 year old male with new onset confusion, suprapubic discomfort, and lack of urine output. History of BPH, HTN, hyperlipidemia, refractory chronic insomnia, and T2DM complicated by neuropathy. On exam, patient is afebrile, confused, and only oriented to self. Suprapubic fullness is present and urinary catheter is inserted and immediately drains 1000mL of urine. What medication likely contributed to this patients condition?

Amitriptyline Tricyclic antidepressants have strong anticholinergic properties and are sometimes used for insomnia & pain management when other medications aren't effective. Potential side effects are confusion, constipation, and urinary retention. These medications should be used with caution in elderly patients.

46 year old with confusion and fever. She is disoriented, somnolent, and difficult to rouse. A friend who is with her reports a long psychiatric history & an attempted overdose on medication in the past. On PE< skin is flushed, oral mucosa dry, pupils dilated, and poorly responsive to light. Bowel sounds decreased. What drug could cause this presentation if taken in overdose?

Amitriptyline Anticholinergic toxicity is characterized be fever, cutaneous flushing, dry oral mucosa, dilated poorly reactive pupils, and confusion. TCAs, particularly amitriptyline, have strong anticholinergic adverse effects.

74 year old man hospitalized with urinary retention complicated by urosepsis. Patient requires endotracheal intubation and mechanical ventilation for 24 hours and then is successfully extubated. On third day of hospitalization, he develops fever and right jaw pain. PE shows firm swelling of pre auricular area on the right extending to angle of mandible. What serum marker is most helpful in confirming diagnosis?

Amyalse Acute bacterial parotitis occurs more commonly in elderly postoperative patients who are intubated or dehydrated. Staph aureus is most common bacterial etiology. Diagnosis can be confirmed with imaging and an elevated serum amylase level (with normal serum lipase level and no evidence of pancreatitis)

Produced by salivary glands & pancreas. Catalyzes hydrolysis of complex carbohydrates to simple sugars.

Amylase

Many patients with down syndrome develop easy onset alzheimer's dementia after age 35 due to have three copies of what gene?

Amyloid precursor protein (increases amyloid beta accumulation in the brain). Characteristic pathologic changes involve accumulation of intracellular neurofibrillary ranges and extracellular amyloid-beta plaques Note: Mutations in gene encoding for apolipoprotein E are associated with a higher risk of LATE onset alzheimer's disease as apolipoprotein E is involved in clearance of amyloid beta from the brain parenchyma

59 year old male with progressively worsening fatigue and chronic back pain. Back pain began three months ago and is worse with movement and positional changes. PE reveals mild tenderness over lower back. Labs reveal normocytic anemia and mild hypercalcemia. Bone marrow biopsy is performed which reveals abundant plasma cells. Patient at greatest risk of developing?

Amyloidosis AL amyloidosis is associated with MULTIPLE MYELOMA and other monoclonal plasma cell dycrasias. Amyloid protein is derived from monoclonal immunoglobulin light chains, and often forms deposits in the kidneys, heart, tongue, and nervous system

Caucasian male suffers from severe neurological disease dies of an overwhelming respiratory infection. Autopsy shows atrophic pre central gyrus and thin anterior roots the spinal cord. LM reveals severe loss of neurons in anterior horn of spinal cord & in the hypoglossal and ambiguous cranial nerve nuclei; corticospinal tracts stain only faintly, indicating demyelinization. Patient most likely suffered from?

Amyotrophic lateral sclerosis ALS causes both upper and lower motor neuron lesions. Loss of neurons in anterior horns in spinal cord (LMN lesion) causes muscle weakness and atrophy. Demyelination of lateral corticospinal tract (UMN lesion) leads to spasticity and hyperreflexia. Loss of neurons in CN V, IX, X, XII motor nuclei Copper-zinc superoxide dismutase (SOD1) gene is implicated. Treat with Riluzole which decreases glutamate release.

Dipalmitoyl phosphatidylcholine (lecithin) and phosphatidylglycerol are major components of surfactant. Fetal lung lecithin production increases sharply after 30 weeks gestation and phosphatidylglycerol production increases at 36 weeks gestation. Amniotic fluid sphingomyelin level should remain constant during the third trimester. A lecithin to sphingomyelin ratio of what indicates adequate surfactant production to avoid neonatal hyaline membrane disease?

An L/S ration greater than or equal to 2

IV fluids increase the intravascular and LV end diastolic volumes. The increase in preload stretches the myocardium and increases the end diastolic sarcomere length, leading to?

An increase in stroke volume and cardiac output by the Frank Starling mechanism

Most common cause and site of lung abscess?

Anaerobic bacteria from gingivodental sulcus. Dependent areas of the right lung (right bronchus is straighter than left).

Risk factors for lung abscess (symptoms: fever, night sweats, weight loss, cough productive of foul smelling sputum) include conditions that increase aspiration risk such as alcoholism, drug abuse, seizure disorders, stroke, and dementia. Most frequent cause of lung abscess?

Anaerobic bacteria normally found in the oral cavity such as Peptostreptococcus, Prevotella, Bacteroides, Fusobacterium species

Type I hypersensitivity reaction characterized by increased vascular permeability and multi system edema, leading to massive shifting of intravascular fluid to the extravascular compartment. Symptoms often begin within seconds to minutes after intravascular exposure to an inciting factor (insect stings, IV meds) but can take up to 2 hours to develop with orally ingested antigens. Results from widespread MAST CELL & BASOPHIL DEGRANULATION & resultant HISTAMINE & TRYPTASE release

Anaphylactic reaction (Note: tryptase is an enzyme that is relatively specific to mast cells & elevated serum levels of tryptase are often used to support a diagnosis of anaphylaxis)

Studying a target protein involved in signal transduction and cellular response to TSH. Protein is isolated and purified from thyroid follicular cells and contains multiple alpha helical regions. Each of these regions is composed of 20 amino acid residues consisting of valine, alanine, and isoleucine. This region of the protein likely forms what function?

Anchoring to cell membrane Integral membrane proteins contain transmembrane domains composed of alpha helices with hydrophobic amino acid residues (alanine, valine, leucine, isoleucine, phenylalanine, tryptophan, methionine, proline, glycine). These transmembrane domains help anchor the protein to the phospholipid bilayer of the cell membrane

Function of gp41

Anchors gp120 mediating fusion process between viruses and target cells

Use of ACE inhibitors or angiotensin II receptor blockers for HTN during pregnancy can result in fetal anuria, oligohydramnios, pulmonary hypoplasia, limb contractures & hypoplasia of skull bones because of a blockage of what substance that is necessary for normal renal development?

Angiotensin II

Increased renin, angiotensin I, & angiotensin II. Decreased aldosterone. No change in bradykinin. These are effects of what drug class?

Angiotensin II receptor blockers (block of angiotensin II receptors results in decreased aldosterone secretion & a fall in BP which increases renin, angiotensin I, and angiotensin II; ACE still intact so do not affect bradykinin degredation).

Patient with T2DM recently diagnosed with HTN stopped taking Lisinopril a week ago due to a dry, nagging cough. Best treatment for HTN?

Angiotensin recetor blocker ACE inhibitors and ARBs both reduce the risk of chronic kidney disease in patients with HTN and DM. ACE inhibitors raise bradykinin levels and cause nonproductive cough, which is not an effect of ARBs

Presents with painful monocular vision loss, headache, vomiting, conjunctival injection with a poorly reactive mid-dilated pupil

Angle closure glaucome (acute increase in IOP)

Radial subluxation (nursemaid's elbow) results from sudden traction on the outstretched and pronated arm of a child. Affected children are usually in little distress unless attempts are made to move the elbow. Ligament that is torn and displaced in this injury?

Annular ligament Note: reduction can be accomplished by fully supinating the child's forearm followed by fully flexing the elbow

Eating disorder characterized by LOW BODY WEIGHT, intense fear of becoming fat, & distorted body image. Medical complications such as bradycardia, hypotension, osteoporosis, amenorrhea, & cardiac atrophy are result of malnutrition & starvation

Anorexia nervosa (REMEMEBER: bulimia patients maintain their body weight at or above minimally normal level so no signs of malnutrition or starvation) Note: russell sign is calluses on dorm offhand from inducing vomiting

Marked menstrual cycle variability in a 14 year old girl who underwent menarche a year ago is most like caused by?

Anovulatory cycles (common in first several years after menarche and the last few years before menopause)

65 year old with severe bladder outlet obstruction. Ultrasound guided midline suprapubic cystotomy is planned. Besides the bladder wall, what structure is most likely to be penetrated by the trocar and cannula during the procedure?

Anterior abdominal aponeurosis, superficial fascia, transversalis fascia, and extraperitoneal fat The bladder is EXTRAPERITONEAL so the peritoneum will not be entered.

Last branch off the internal carotid artery before it trifurcates. Supplies posterior limb of internal capsule, optic that, lateral geniculate body, choroid plexus, uncus, hippocampus & amygdala

Anterior choroidal artery

Flattening of the deltoid muscle with acromial prominence after a shoulder injury in which the arm was externally rotated and abducted associated with deltoid muscle paralysis and loss of sensation over the lateral shoulder suggests what type of injury?

Anterior dislocation of the humerus

CREST syndrome (limited scleroderma) manifests with calcinosis, Raynaud's phenomenon, esophageal dysmotility, sclerodactyly, and telangiectasis. Antibody that that is found in patients with CREST syndrome?

Anti-centromere antibodies Note: Anti-DNA topoisomerase I (Scl-70) antibodies are highly specific for SYSTEMIC sclerosis

47 year old woman with symmetric poly arthritis (involving MCP & PIP joints) with prolonged morning stiffness and fatigue. Autoantibodies that are most specific for this condition?

Anti-citrullinated peptides (have the highest specificity for rheumatoid arthritis)

45 year old Caucasian male with 2 year history of abdominal discomfort, greasy stool, weight loss. Has been treated with joint pain with ibuprofen. Intestinal biopsy shows multiple macrophages loaded with PAS positive granules in the lamina propria. He should be treated with?

Antibiotics Caused by the gram positive actinomycete, Tropheryma whippelii, whipple disease is a rare systemic illness that involves the small intestine, joints, and CNS. Classic histologic findings include small intestine mucosa containing enlarged, foamy macrophages packed with rod shaped bacilli and PAS positive, diastase-resistant granules (which consist of lysosomes and partially digested bacteria). Most commonly seen in middle aged Caucasian males, presenting as malabsorption with diarrhea and weight loss. Arthropathy, polyarthritis, psychiatric, and cardiac abnormalities may also be observed.

Elevated eosinophils contribute to host defense against schistosomiasis parasitic infection through what mechanism?

Antibody dependent cell mediated cytotoxicity When stimulated by antibodies bound to a parasitic organism, eosinophils destroy the parasite via antibody dependent cell mediated cytotoxicity with enzymes from their cytoplasmic granules. Note: eosinophils also synthesize prostaglandins, leukotrienes, & cytokines that contribute to inflammation seen in late phase type 1 hypersensitivity and chronic allergic reactions but this would NOT be protective against schistosomiasis

Blurry vision, dry mouth, urinary retention, constipation are common side effects of what class of drugs?

Anticholinergic drugs

A radioimmunoassay uses specific antibodies and a fixed quantity of radiolabeled antigen to determine the amount of antigen present in an unknown sample. This is done by measuring the amount of radio labeled antigen displaced from the antibodies in the system Antibodies to X antigen are attached to assay plates and a fixed quantity of radiolabeled X antigen is added to the plates. Unlabed Y antigens are then added in increasing concentration. Graph shows: as Y antigen concentration increases, measured radioactivity remains the same. Interpretation?

Antigen X and antigen Y have no epitope in common If they shared common epitopes then as the Y antigen was added the measured radioactivity would decrease (Y is not radiolabeled and X is)

Classes of drugs most effective for motion sickness?

Antimuscarinic agents (scopolamine) and antihistamines with antimuscarinic action (meclizine, dimenhydrinate)

In addition to blocking histamine receptors, first generation antihistamines (chlorpheniramine, diphenhydramine) have antimuscarinic, anti alpha-adrenergic and anti-serotonergic properties. Side effects of these?

Antimuscarinic- effects on ocular ciliary muscles impair accommodation & cause blurred vision for close objects Anti-alpha adrenergic- peripheral vasodilation, hypotension, postural dizziness Anti-serotonin - appetite stimulation and weight gain

Superoxide dismutase, glutathione peroxidase, & catalase convert ROS to oxygen and water, neutralizing their capacity for cellular damage. They are what type of enzymes?

Antioxidant enzymes

HIV positive male for more than 5 years. Current blood samples demonstrate significant increase in pol gene mutations over the past 1 year. Likely due to?

Antiretroviral chemotherapy

What adverse effect is likely to persist after adding carbidopa to levodopa treatment?

Anxiety and agitation (can actually worsen because more dopamine becomes available to the brain)

Myxomatous changes (pathologic weakening of connective tissue) with pooling of proteoglycans in the media layer of large arteries are found in cystic medial degeneration, which predisposes to the development of what conditions?

Aortic dissections and aortic aneurysms Note: medial degeneration is frequently seen in younger individuals with Marfan syndrome (autosomal dominant defect in extracellular glycoprotein fibrillin-1) Note: Beta-aminopropionitrile, found in sweet peas, causes inhibition of the enzyme responsible for cross linking elastin and collagen fibers, lysl oxidase, & mimics myxomatous degeneration found in Marfan's

Traumatic aortic rupture is most often caused by the rapid deceleration that occurs in motor vehicle collisions. Most common site of injury?

Aortic isthmus (tethered by the ligamentum arteriosum and is relatively fixed and immobile compared to the adjacent descending aorta). Widened mediastinum may be seen on CXR.

Head 'pounding' accompanied by involuntary head bobbing on exertion and nocturnal palpitations are characteristic of what type of murmur?

Aortic regurgitation (associated with wide pulse pressure)

Most common complication of bicuspid aortic valve?

Aortic stenosis. Develops around age 50 in comparison to senile calcific stenosis of a normal aortic valve which generally becomes symptomatic over 65.

Frontal lobe injury results in executive dysfunction & personality changes secondary to impairment organizational, restraint, and motivational systems. Right sided lesions are more often associated with disinhibition and left sided lesions are more often associated with what?

Apathy and depression

Primary tuberculosis causes the formation of Gohn foci in the lower lungs. M. tuberculosis then spreads lymphatically to the hilarity lymph nodes forming a Gohn complex. Secondary (reactivation TB occurs in patients with prior TB infection that never cleared completely and is characterized by?

Apical cavitary lesions with caseous and liquefactive necrosis, hemoptysis, weight loss (secondary TB occurs most often in immunosuppressed patients)

Familial dysbetalipoproteinemia (type III hyperlipidemia) is caused by a mutation in what betalipoprotein?

ApoE ApoE is located on chylomicron remnants, IDL, & HDL and serves (along with Apo B48) to allow the chylomicron remnants to be recognized by the LDL receptor of hepatocytes and taken into liver for repackaging into VLDL.

34 year old IV drug user with fever, jaundice, and anorexia. Liver biopsy reveals mononuclear cell infiltrates, hepatocyte swelling, and acidophilic bodies. Formation of acidophilic bodies is a result of?

Apoptosis Fever, jaundice, and anorexia in an IV drug use is suggestive of viral hepatitis most likely due to hepatitis C infection. Acute viral hepatitis causes hepatocyte apoptosis and necrosis. Apoptotic hepatocytes shrink, undergo nuclear fragmentation, and become intensely eosinophilic. They may also be referred to as acidophilic bodies, Councilman bodies, & apoptotic bodies

Progressive development of spastic diplegia, abnormal movements, and growth delay in the setting of elevated arginine levels are features of what enzyme deficiency?

Arginase (Note: arginase is a urea cycle enzyme that produces urea and ornithine from arginine; patient's with deficiency in this enzyme have mild or no hyperammonemia & treatment includes arginine free, low protein diet)

43 year old with chest pressure that develops with exertion and sometimes with resting. Acetylcholine infusion administered during coronary angiography results in dilation of the epicardial coronary vessels. A reaction involving what amino acid is most likely responsible for the observed dilation?

Arginine Nitric oxide is synthesized from arginine by nitric oxide synthase. As a precursor of nitric oxide, arginine supplementation may play an adjunct role in the treatment of conditions that improve with vasodilation, such as stable angina.

23 year old woman gives birth to full term neonate with ambiguous genitalia & clitoromegaly. Labs reveal elevated testosterone and androstenedione. Mother experienced facial hair growth & voice deepening during pregnancy. Most likely deficient enzyme in newborn?

Aromatase Aromatase converts androgens to estrogens in the ovaries, testes, placenta, and other peripheral tissues. Placental aromatase deficiency, an autosomal recessive disorder, causes accumulation of androgens during pregnancy resulting in ambiguous external genitalia in female infants and maternal virilization. NOTE: 21 alpha hydroxylase deficiency results in ambiguous genitialia in female & decreased aldosterone synthesis (salt wasting and hypotension) but maternal virilization DOES NOT occur due to intact placental aromatase activity

Class of drugs that decrease the synthesis of estrogen from androgens suppressing estrogen levels and slowing progression of ER positive tumors in postmenopausal women

Aromatase inhibitors (anastrozole, letrozole, exemestane)

65 year old presents after missing a follow up appointment for CHF. Complains of disturbed color perception, anorexia, nausea, vomiting, and diarrhea that has gotten worse over past 2 weeks. HF has been controlled effectively. IF current symptoms are not treated what complication is most likely to develop?

Arrhythmia Digoxin toxicity presents with nonspecific GI (anorexia, nausea, vomiting) and neurologic (fatigue, confusion, weakness) symptoms. Changes in color vision are more specific but more rate. Life threatening ventricular arrhythmias are the most serious complication. Precipitating causes of toxicity include hypokalemia (increased digoxin binding), hypovolemia, and renal failure. Anti-digoxin antibody fragments are given with severe toxicity.

67 year old male with progressive dyspnea. Cannot tolerate moderate exertion and seep in half sitting position due to orthopnea. Has some swelling of ankles. What do you expect to be increased in this patient?

Arteriolar resistance Decreased cardiac output triggers a number of compensatory mechanisms. RAAS activation and increased sympathetic output raise arterial resistance (after load) and exacerbate heart failure by making it more difficult for the failing heart to pump blood to the tissues

Most common cause of intracranial hemorrhage in children and tends to be a single lesion?

Arteriovenous malformation

Rare normal variant that branches off either the right or left PCA & supplies the bilateral thamami and dorsal midbrain

Artery of Percheron

Patient with CXR that reveals pleural thickening and calcifications (pleural plaques) along the posterolateral middling regions and diaphragm along with pleural effusion. Patient likely has a history of exposure to what agent?

Asbestos (many patients are asymptomatic despite visible disease on imaging)

What portion of the nephron is impermeable to water regardless of the serum vasopressin levels?

Ascending limb of the loop of henle Reabsorption of electrolytes by the Na+/K+/2Cl- cotransporter occurs in the thick ascending limb and contributes to the formation of the corticomeduallary concentration gradient

Deficiency is characterized by microvascular bleeding, gingivitis, & impaired wound healing

Ascorbic acid (vitamin C) Note: Scurvy occurs in patients with abnormal eating patterns including the elderly, alcoholics and people who live alone. Ascorbic acid is a cofactor in hydroxylation of proline & lysine residues and is important in synthesis of collagen

Monomorphic fungus with only a mold form. Seen in tissue as septet hyphae with acute angle V shaped (dichotomous) branching

Aspergillus fumigatus

34 year old with recurrent transient pulmonary infiltrates. History of brachial asthma and has had several exacerbations over past few years. Her medications include albuterol as needed and medium dose inhaled glucocorticoids. Complete blood count shows eosinophilia. CT chest scan reveals proximal bronchiectasis. Patient's condition is most likely related to colonization of what organism?

Aspergillus fumigatus Allergic bronchopulmonary aspergillosis (ABPA) due to Aspergillus fumigatus may complicate asthma. ABPA can result in transient recurrent pulmonary infiltrates and eventual proximal bronchiectasis & occurs in 5-10% of corticosteroid dependent asthmatics.

43 year old man undergoes induction chemotherapy for myeloid leukemia. Three weeks later he is hospitalized with fever, cough, and generalized weakness. Crackles heard over right lung field. Hemoglobin is 9.5, leukocyte count is 900, and absolute neutrophil count is 100. CXR reveals dense infiltrate involving the right upper lung lobe. Broad spectrum antibiotics initiated. Bronchoscopy is performed and samples grow mold. Most likely isolate?

Aspergillus fumigatus (septate narrow hyphae that branch at 45 degree angle) Patients with profound and prolonged (several days) neutropenia are at especially high risk for viral and fungal infections. Most common fungal causes are Aspergillus and Candida. Respiratory infections from Candida are very uncommon & candidate blood stream infections are typically in patients with profound neutropenia who have an indwelling venous catheter. Invasive pulmonary aspergillosis presents with combination of fever, chest pain, cough, dyspnea, and hemoptysis. Febrile neutropenia is defined as a fever with an absolute neutrophil count less than 500.

NSAID that inhibits COX-1 and COX-2 enzymes via irreversible acetylation preventing conversion of arachidonic acid to prostaglandins, prostacyclin, and thromboxane?

Aspirin (acetylsalicylic acid) Other NSAIDS (diclofenac, ibuprofen, indomethacin) reversibly inhibit COX1 and COX2 COX-1 acetylation inhibits generation of TXA2 in platelets (anthrombotic effects). COX-2 acetylationblocks prostaglandin production in inflammatory cells resulting in anti-inflammatory, antipyretic, & analgesic effects

ADH acts primarily on the collecting ducts, increasing their permeability to water. In the absence of ADH, the tubular fluid is most concentrated in what location?

At the junction between the descending and ascending limbs of the loop of Henle Note: the most dilute region is the collecting ducts in this situation

54 year old man hospitalized after a planned abdominal surgery. Physician administers a new drug and patient develops flushing, diaphoresis, and nausea. BP is 100/70 & HR is 55. Pupils are constricted but reactive to light. Medication was most likely given for what condition?

Atonic bladder Cholinergic agonists are indicated in non-obstructive urinary retention & paralytic ileus (bethanechol), and glaucoma (carbachol & pilocarpine). Their side effects include nausea, vomiting, abdominal cramps, diarrhea, dyspnea, and increased secretions (sweating, lacrimation, salivation)

FIXED splitting of S2 occurs with what cardiac condition which causes increased SpO2 in the right atrium compared with the vena cava but identical right atrial and ventricular SpO2?

Atrial septal defect Note: VSD may cause WIDE splitting of S2

Left ventricular leads in biventricular pacemakers course through the coronary sinus when resides in what groove on the posterior aspect of the heart?

Atrioventricular groove Note: biventricular pacemaker leads do not pass through the pulmonary artery, Swan-Ganz catheter's do

67 year old man had dizziness and a syncopal episode while cleaning out garage. He woke up spontaneously a minute later without disorientation or confusion. ECG demonstrates bradycardia with regular rhythm and narrow QRS complexes. There is complete desynchronization between P waves and QRS complexes. Location most likely responsible for pacing this patient's ventricles?

Atrioventricular node Cardiac pacemaker impulse generation normally occurs in the SA node (60-100 bpm), which has the fastest firing rate of all the conductive cells. The cells in other areas of the conduction system (AV node, bundle of HIS, purkinje fibers) may serve as pacemakers if normal impulse condition is impaired. AV node becomes the pacemaker (45-55 bpm) in complete (3rd) degree AV block in which SA node impulses cause atrial contraction while impulses generated by AV node cause ventricular contraction. On ECG, atria and ventricles depolarize independently of each other (AV dissociation). QRS complexes are NARROW since ventricular depolarization proceeds normally. When electrical impulses are generated below the AV node & bundle of His, the HR can slow to as few as 20bpm & the ECG shows PROLONGED, abnormally shaped QRS complexes due to aberrant impulse conduction through the ventricles

Organophosphate (cholinesterase inhibitor) poisoning (increased salivation, miosis, bronchospasm, bradycardia) treatment?

Atropine (muscaranic antagonist) and pralidoxime (cholinesterase regenerator)

Physicians have responsibility to identify signs of potential drug seeking behavior and prescription drug misuse. This involves being alert to red flags (lost/stolen medication, multiple prescriptions from different providers, requesting medication by name, running out of medication, pain inconsistent with physical exam). Best initial approach in differentiating between drug seeking and inadequate pain management?

Attempt to clarify medication history by using prescription monitoring programs or other information sources

82 year old man with end stage lung cancer is not able to communicate and is admitted to hospital. Patient doesn't have a designated POA and doesn't have an advanced directive on file. High wife of many years is present and states that the patient has stated on several occasions not to resuscitate him if he were near death as he did not want to live 'attached to machines or brain dead, as he just wants to die in piece when the time comes'. Most appropriate course of action if the patient enters ventricular fibrillation?

Attend to the patient's comfort and allow his family to be present with him as he is dying Note: You would not place defibrillator pads and administer vasopressin. Do not resuscitate order includes BLS (mouth to mouth breathing, chest compressions) and ALS (intubation, mechanical ventilation, defibrillation, administering meds such as epinephrine or vasopressors).

Quick clinical tests including counting down from 100 by intervals of 3 or 7, reciting the months of the year in reverse order, and spelling 'world' backwards are components of the MMSE that asses what?

Attention and concentration

Child with lack of social engagement, speech delay, repetitive play, insistence on sameness are consistent with what disorder?

Autism spectrum disorder (can occur without or without language & intellectual impairment)

Acute intermittent porphyria

Autosomal dominant disease.PBG deaminase deficiency. Presents with abdominal pain, neurological manifestations, port-wine colored urine with ALA and PBG in urine.

Gaucher disease

Autosomal recessive lysosomal storage disorder characterized by B-glucocerebrosidase deficiency. Presents with pancytopenia, bone pain, and hepatosplenomegaly. Accumulation of glucocerebroside forms Gaucher cells (distended macrophages with 'wrinkled tissue paper appearance')

56 year old man with persistent left wrist pain. Fell onto outstretched hand. Mild swelling over dorsal of left hand with preserved ROM. Point tenderness over dorsolateral aspect of hand between tendons of extensor policies longis and extensor pollicis brevis. X-ray shows scaphoid fracture. Most at risk for developing what condition?

Avascular necrosis A fall onto an outstretched hand may cause fracture of the scaphoid bone. Exam shows tenderness in the anatomical snuff box. Scaphoid bone is vulnerable to avascular necrosis due to its tenuous blood supply (blood supply to the proximal pole of the schizoid proceeds in a retrograde manner and can be easily interrupted by a fracture). Dorsal scaphoid branch of radial artery supplies majority of scaphoid by entering near its distal pole.

African American woman with chronic progressive hip pain thats exacerbated by weight bearing. PE reveals restricted ROM with no swelling, erythema, or warmth in the hip area. History of sickle cell disease. Most likely cause of hip pain?

Avascular necrosis (osteonecrosis) Occurs due to impaired blood supply to a segment of bone & femoral head is most common location. Common causes are SCD, glucocorticoid therapy, vasculitis, & alcoholism

Maladaptive pattern of behavior characterized by social inhibition, feelings of inadequacy, and fear of embarrassment and rejection

Avoidant personality disorder

Characterized by avoidance of food intake due to dislike of the sensory experience involved in tasting or eating food and/or consequences of eating. Unlike anorexia & bulimia, there is no disturbance in body image

Avoidant/restrictive food disorder

Patient stetted on Allopurinol. Initiation of this drug will most likely increase the activity of what medication?

Azathioprine Xanthine oxidase catalyzes one of two major azathioprine inactivating pathways. Allopurinol competitively inhibits xanthine oxidase, which results in increased conversion of azathioprine to its active metabolite, 6-thioguanine

Macrolide antibiotic that binds to the 50S subunit of bacteria ribosomes, thereby inhibiting protein synthesis. Used for infections with Chlamydia, Mycoplasma, H. influenzae, & Moraxella catarrhalis.

Azithromycin

Vein that drains blood from the esophageal veins into superior vena cava?

Azygous vein

Patients who have undergone total gastrectomy require lifelong supplemental of what WATER soluble vitamin?

B12 (intrinsic factor secreted by gastric parietal cells is necessary for absorption of B12 in ileum)

Stimulation of what receptors causes bronchodilator, vasodilation in skeletal muscles, and uterine relaxation?

B2-adrenergic receptors

Stimulation of what receptors by catecholamines increases extracellular potassium movement into cells?

B2-receptors (nonselective B-blockers can impair potassium entry into cells and lead to minor elevations of plasma potassium)

Deficiencies of what vitamins are associated with hyperhomocysteinemia which is associated with atherosclerosis and thrombotic events?

B6, B12, and folate (B9)

Botulinum preformed neurotoxin produced by Clostridium botulinum prevents release of acetylcholine from presynaptic nerve terminals. This toxin impairs what type of neurotransmission?

BOTH peripheral muscarinic AND nicotinic neurotransmission causing both autonomic symptoms (fixed pupillary dilation, dry mouth) & skeletal muscle weakness (diplopia, dysphagia, dysphonia, respiratory depression). Effects at NMJ can be seen on EMG as decrease in muscle response following stimulation of motor nerve. High rate repetitive nerve stimulation improves deficit (rapid depolarization increases Ca2+, mobilizing Ach vesicles). CANNED foods create anaerobic environment ideal for this organism. Botulism causes descending paralysis starting from cranial nerves. Note: Lambert Eaton syndrome is similar but initial weakness involves the proximal extremities! TCA overdose produces muscarinic blockade with no effect on NMJ! Myasthenia graves does not present with muscarinic symptoms (dry mouth, mydriasis) and rapid nerve stimulation does not improve symptoms!

Antibiotic that prevents mucopeptide transfer into the growing cell wall, thereby inhibiting bacterial cell wall synthesis

Bacitracin

GABA-B receptor egoist effective as mono therapy for treatment of spasticity secondary to both brain and spinal cord disease, including MS

Baclofen Note: Tizanidine is an alpha 2 adrenergic agonist that is also effective and commonly used for this purpose

10 year old boy with SOB and palpitations for a day. Has had fever, poor appetite, and fatigue for 3 days. VS indicate tachypnea, tachycardia, and hypotension. New holosytolic murmur. Cardiac biopsy performed. What most likely precipitated patients current condition?

Bacterial infection Patient has pancarditis (most serious manifestation of ARF) & endocardial involvement resulting in valvular dysfunction (mitral valve regurgitation) is likely cause of his new murmur. Interstitial myocardial granulomas (aschoff bodies) are found in carditis due to acute rheumatic fever, which develops after untreated group A streptococcal pharyngeal infection. Aschoff bodies contain plump macrophages with abundant cytoplasm and central, slender ribbons of chromatin (Anitschkow cells). Over years, achoff bodies are replaced by fibrous scar tissue leading to mitral stenosis

Patient with low grade fever, sore throat, persistent cough, and CXR evidence of bilateral patchy infiltrate. Treatment for this atypical pneumonia?

Bacterial protein synthesis inhibitor such as a macrolide or tetracycline Note: Mycoplasma pneumoniae is the leading cause of atypical pneumonia. Can also be caused by Chlamydia pneumoniae. Legionella pneumophilia is another common cause but its associated with GI symptoms, more severe clinical illness & LOBAR infiltrate on CXR.

Associated with a grayish-white vaginal discharge with a 'fishy odor'. Etiology involves an overgrowth of Gardnerella vaginalis, an anaerobic gram variable rod. Clue cells (squamous epithelial cells covered with bacterial organisms) are seen on wet mount microscopy or cervical cytology

Bacterial vaginosis

22 year old male with fever, abdominal pain and vomiting. On exam patient appears acutely ill and there is RLQ tenderness with rebound as well as a palpable mass. CT of the abdomen demonstrates a periappendiceal fluid collection. Culture of this fluid would most likely isolate what organism?

Bacteroides fragillis Patient is suffering from perforated appendicitis which has evolved into an intraabdominal abscess. Intraabdominal infections are polymicrobial with B. fragilis and E. coli being the most prominent organisms isolated. Enterococci and streptococci are also sometimes isolated. Note. S. aeueus causes abscess formation of the SKIN not intraabdominal abscesses

Immunodeficiency resulting from a defect in expression of HLA class II antigens on the surfaces of antigen presenting cells. MHC Class II is needed to present foreign antigen to T cells to elicit a cell mediated AND humoral response (T cells must first activate B cells to mature and undergo class switching)

Bare lymphocyte syndrome

Squamous metaplasia of the respiratory epithelium is a reversible, adaptive response to chronic irritation such as smoking. Normal columnar epithelium mis replaced by squamous epithelium which is more resistant to irrigation but has reduced mucociliary clearance, increasing the risk of respiratory infections & squamous cell carcinoma. Condition with similar pathogenesis?

Barrett esophagus- esophageal squamous epithelium is replaced by columnar epithelium in response to chronic acid exposure

Autosomal recessive disorder of the renal tubule which leads to salt wasting due to a defective Na K 2Cl cotransporter. Presents clinically as hypokalemia, hypocholremia, metabolic alkalosis, and hyperreninemia with normal BP

Bartter syndrome

Patient with t(14, 18) chromosomal translocation will have what abnormality in gene expression?

Bcl2 overexpression Follicular lymphoma is non Hodgkin lymphoma of follicular B lymphocytes. Bcl-2 is protooncogene as it has anti-apoptotic effects

Drug class that increase the FREQUENCY of opening of the CNS GABA A receptor chloride channels and have anxiolytic, anticonvulsant, and muscle relaxant effects as well as sedative hypnotic effects

Benzodiazepines

35 year old man was hospitalized for a psychotic episode in which he heard voice of God and the devil and believed his family was plotting to kill him. He improved rapidly and was discharged on an unknown medication. Three weeks later, patient presents with generalized muscle stiffness and shaking of his right hand. He is alert and oriented with mild paranoia but no auditory hallucinations. Best treatment for patients current symptoms?

Benztropine Drug induced parkinsonism is a type of extrapyramidal symptom caused by medications that block D2 receptors. Management strategies include decreasing or discontinuing the offending medication and treatment with anticholinergic antiparkinsonian medication (trihexyphenidyl, benztropine)

Selection bias created by choosing hospitalized patients as the control group

Berkson's bias

May present with dyspnea and ill defined nodular or irregular opacities on CXR. Histology reveals noncaseaing epitheoid granulomas without obvious associated particles

Berylliosis

Asthma and COPD exacerbations, characterized by dyspnea, tachypnea, prolonged expiration, and bilateral wheezing, are the most frequent causes of pulsus paradoxus is the absence of significant pericardial disease. Best treatment to provide immediate relief?

Beta adrenergic agonists (cause bronchial smooth muscle relaxation via increased intracellular cAMP)

55 year old man with a history of T2DM with HbA1c level of 3.7%. Further evaluation with hemoglobin electrophoresis shows a hemoglobin A2 level of 7.5% (elevated). Most likely explanation for his falsely low HbA1c?

Beta thalassemia trait (HbA2 becomes elevated to compensate for beta global chain underproduction, but the resulting microcytic cells are prone to hemolysis & falsely low HbA1c levels are due to increased red cell turnover) Glycated hemoglobin (HbA1c) forms within circulating RBCs as hemoglobin A is exposed to glucose and HbA1c levels are a useful indicator of average glycemic control over the erythrocyte lifespan. HbA1c are affected by alterations in RBC survival. Condition that increase RBC turnover (hemolytic anemia such as beta thalassemia) can cause falsely low HbA1c levels. Nutrient deficients involving iron, folate, and B12 can reduce production of new red cells, leading to older erythrocyte preponderance and falsely elevated HbA1c levels.

Effect of beta blockers on RAAS?

Beta-adrenergic blocking drugs inhibit renin release by blocking beta-1 receptor mediated regulation of the RAAS. This reduces plasma renin activity with a resulting reduction in angiotensin I, angiotensin II, and aldosterone levels

Enzyme released by injured hepatocytes and bacteria that hydrolyzes bilirubin glucuronides to unconjugated bilirubin playing an important role in the formation of brown pigment gallstones.

Beta-glucuronidase

Prevent beta lactamase from inactivating penicillin class drugs which extends the spectrum of their activity

Beta-lactamase inhibitors (clavulanic acid, sulbactam, tazobactam)

23 year old woman presents with preterm, premature rupture of membranes at 30 weeks gestation. What medication has the greatest positive impact on fetal survival?

Betamethasone or dexamethasone Both maternal and fetal cortisol help to accelerate fetal lung maturation by stimulating surfactant production and betamethasone or dexamethasone are administered to pregnant women at risk of premature delivery to prevent neonatal respiratory distress syndrome

Patient with difficulty voiding after an elective hernia repair under spinal anesthesia. Bladder catheterization shows a post void residual of 300cc of urine (normal is less than 50cc). Patient would most likely benefit from what medication?

Bethanechol Postoperative urinary retention, with incomplete bladder emptying, is a common complication thought to indole decreased micturition reflex activity, decreased contractility of the bladder detrusor, and/or increased vesicle sphincter tone. May be treat with a muscarinic agonist (bethanechol) or an alpha 1 blocking drug.

Duodenal S cells secrete secretin in response to increasing H+ concentrations. Secretin increases secretion of what substance from the pancreas?

Bicarbonate Note: the chloride content of pancreatic secretions decreases in proportion to bicarbonate concentration increases

In females, the paramesonephric ducts fuse to form the fallopian tubes, uterus, cervix, and upper vagina. Disruptions of this process can lead to a variety of mullerian tract anomalies and renal anomalies are a common comorbidity. Incomplete lateral fusion of the paramesonephric ducts results in?

Bicornuate uterus (characterized by an indentation in the center of the fundus) Note: if a patient's hysterosalpingogram (contrast injection through cervix into uterus with concurrent pelvic X-ray) shows 2 unfused uterine horns with a central filling defect, it can be a bicornuate uterus or a longitudinal uterine septum (results from failed involution of the paramesonephric ducts) & MRI can distinguish these 2 as a bicornuate uterus has an abnormal contour to the uterine fungus and septate uterus has a normal uterine contour.

Young asymptomatic patient with a soft systolic ejection murmur at the right 2nd intercostal space radiating to the neck most likely has?

Bicuspid aortic valve

Characteristic abnormality seen in the brain of patients with Huntington disease?

Bilateral atrophy of the caudate nucleus and putamen (striatum)

45 year old woman with SLE being treated with multiple medications is found dead at home. Records show she had progressive truncal obesity, facial plethora, and proximal muscle weakness. Labs showed mild hyperglycemia. Autopsy found left coronary artery mainstream thrombus that caused SCD. Adrenal glands will most likely show?

Bilateral cortical atrophy Patient likely had iatrogenic cushing syndrome due to prolonged administration of glucocorticoids for management of SLE which would lead to bilateral adrenocortical atrophy. Cushing syndrome and lupus both increase risk of CAD.

34 year old male with easy bruising & occasional spontaneous bruising. Has also been experiencing generalized fatigue, poor appetite, and dull painful stiffness in lower back and joints at night. Has a history of Crohn disease that required a partial bowel resection one year ago. Treated with oral antibiotics for a perianal fistula three month ago. Mother died of colon cancer when she was 56. Most likely diagnosis?

Bile acid malabsorption Crohn disease with ill resection or extensive ill involvement can cause bile acid malabsorption, which may lead to impaired absorption of fat soluble vitamins (A, D, E, K). Vitamin K deficiency can result in impaired coagulation with easy bruising, large hematoma formation in deep tissues and joints (hemearthrosis) after minor trauma, and prolonged bleeding after surgery Note: autoimmune hepatitis is more common in women and is more often associated with ulcerative colitis

What lipid lowering agents, when used as mono therapy, increases triglyceride levels?

Bile acid resins (cholestyramine, cholestipol, colesvelam) They work by inhibiting the enterohepatic circulation of bile acids, leading to diversion of hepatic cholesterol to the synthesis of new bile acids, increased uptake of cholesterol from the circulation, and reduced blood LDL levels. However, they increase hepatic production of triglycerides (VLDL) & can cause hypertriglyceridemia

52 year old postmenopausal woman with a nonobstructing gallstone who prefers nonoperative management. Best medication to treat this patient?

Bile acid supplement Hydrophilic bile acids (ursodeoxycholic acid) improve cholesterol solubility by reducing the amount of cholesterol secreted into the bile and increasing biliary bile acid concentration.

Elevated cholesterol concentrations increase the likelihood of cholesterol precipitation and gallstone formation. High levels of what two substance increase cholesterol solubility and decrease the risk of gallstones?

Bile salts and phosphatidylcholine

Presents with jaundice, dark urine, and acholic (pale/clay colored) stools in the first 2 months of life due to conjugated hyperbilirubinemia. Biopsy reveals intrahepatic bile duct proliferation, portal tract edema & fibrosis.

Biliary atresia (progressive complete or partial obstruction of the extra hepatic bile ducts). Biliary tree is normal at birth undergoes destruction thought to be immune related or viral induced. Lab findings include elevated direct bilirubin and gamma-glutamyl transferase, consistent with cholestasis

MOA of polyene antifungals (Amphotericin B & Nystatin)

Bind ergosterol in fungal cell MEMBRANES creating pores and causing cell lysis

Function of T loop in tRNA?

Binding of tRNA to ribosomes

Patient diagnosed with cellulitis and started on appropriate treatment. Gram stain of exudate shows gram positive cocci in clusters. Organism most likely responsible for this patient's infection synthesizes a protein as part of its peptidoglycan cell wall that does what?

Binds the Fc portion of IgG Protein A is a virulence factor found in the peptidoglycan cell wall of S. aureus that binds to the Fc portion of IgG, leading to impaired complement activation, opsonization and phagocytosis

Post elective hip surgery patient is stated on a medication that is a low molecular weight fraction of a negatively charged chemical stored in mast cell granules. Mechanism of action of this medication?

Binds to antithrombin III. Note: by binding to antithrombin III, heparin induces conformational change of ATIII which inhibits Xa and neutralized thrombin while ATIII activated by LMWH acts predominantly on Xa, not thrombin

Bioavailability equation

Bioavailability (F) = (area under curve oral x dose IV)/(area under curve IV x dose oral)

Fraction of administered drug that reaches systemic circulation in chemically unchanged form

Bioavailability (Note: bioavailability of a drug administered non-intravenously is always less than 1)

B7 is a cofactor for what enzymes?

Biotin (B7) is a cofactor for all 4 carboxylase enzymes (pyruvate carboxylase, acetyl-CoA carboxylase, propionyl-CoA carboxylase, & 3-methylcrotonyl-CoA-carboxylase)

7 year old boy evaluated for fatigue, poor energy, SOB. Reports occasional pounding of the heat. PE reveals holosystolic murmur best heart at the lower sternal border. Echocardiography reveals apical displacement of the tricuspid valve leaflets, decreased RV volume, and atrialization of the right ventricle. Moderate/severe tricuspid regurgitation is also present. If patient's diagnosis is due to a side effect of a drug taken during pregnancy, his biological mother most likely had what condition?

Bipolar disorder Lithium is commonly used to treat bipolar disorder. Its use during pregnancy is associated with Ebstein's anomoly which is characterized by apical displacement of the tricuspid valve leaflets, decreased right ventricular volume, and atrialization of the RV.

The anticonvulsant valproate has mood stabilizing properties and is an effective maintenance treatment of what disorder?

Bipolar disorder Note: antidepressant monotherapy should be avoided in bipolar maintenance treatment due to the risk of mood destabilization

G6PD deficiency peripheral smear shows?

Bite cells & Heinz bodies (requires crystal violet stain)

Patients with multiple sclerosis often develop a spastic bladder a few weeks after developing an acute UMN lesion of the spinal cord. These patients present clinically with increased urinary frequency & urge incontinence. Urodynamic studies show?

Bladder hypertonia (bladder does not distend/relax properly due to loss of descending inhibitory control from the UMN). There is little or no residual urine after emptying as bladder contractility is normal but dispensability is poor

Round yeasts with single broad based buds and thick double refractive walls

Blastomyces

32 year old with low grade fever, cough, and generalized malaise. Otherwise healthy and no PMH. Recently came back from 'outdoor adventure' trip in Great Lakes. Microscope exam of a KOH prep of an exudate specimen reveals a large yeast with a SINGLE BUD. When incubated in a specific medium at 24 degrees C, a multicellular structure with branching tubular cells grow.

Blastomyces dermatitidis

Selective COX 2 inhibitors (celecoxib) relieve pain with a lower risk of what compared with nonselective NSAIDs?

Bleeding and gastric ulceration Note: COX2 inhibitors have been associated with an increased risk of cardiovascular event because they block prostacyclin production (has anticoagulant and vasodilator actions)

During an exercise stress test, what parameters are likely to be similar between systemic and pulmonary circulation?

Blood flow per minute The circulatory system is a continuous circuit and therefore the volume output of the LV must closely match the output of the RV. This balance is necessary to maintain continuous blood flow though the body and exists both at rest and during exercise

3 year old boy with constipation, abdominal pain, irritability, and pallor for the past three months. Mother reports they recently (4 months ago) moved to a new house owned by their family for many generations. Most appropriate diagnostic test for this patient?

Blood lead level (urine ALA, a heme synthesis substrate will also be elevated) Lead binds to sulfhydryl groups on proteins, replaces calcium in calcium dependent cellular functions and directly inhibits enzymes in synthesis.

Test that should be ordered before patient begins Canagliflozin therapy for diabetes mellitus?

Blood urea nitrogen and creatinine SGLT2 inhibitors (-gliflozin) are oral anti-diabetic agents that work by decreasing proximal tubular reabsorption of glucose, thereby promoting urinary glucose loss. It is recommended to monitor serum creatinine before and after initiating therapy with SGLT2 inhibitors. Side effects include UTI and genital mycotic infections due to glucosuria. Increased osmotic diuresis can cause symptomatic hypotension in patients (elderly and those on diuretics). Should be AVOIDED in patients with moderate to severe renal impairment due to lack of efficacy and increased risk of adverse effects.

Blood solubility of an anesthetic is indicated by?

Blood/gas partition coefficient. (Note: Drugs with a high blood gas partition coefficient are MORE soluble in the blood, have a SLOW rise in partial pressure, demonstrate SLOWER equilibration with brain, and have LONGER onset times)

Rare autosomal recessive condition caused by mutations in the BLM gene encoding helicase, an enzyme that unwinds the double helix during DNA replication. Patients typically present with growth retardation, facial anomalies, photosensitive skin rash, and immunodeficiency due to chromosomal instability and breakage

Bloom syndrome

Autosomal recessive disorder that leads to chromosomal instability. Characterized by small stature, infertility, predisposition to malignancy, and a class sun sensitive facial rash. Which enzyme affected?

Bloom syndrome Helicase

Disorder characterized by being excessively preoccupied with perceived defects or flaws in physical appearance

Body dysmorphic disorder

Major toxicities of Ganciclovir?

Bone marrow suppression (severe neutropenia, thrombocytopenia, leukopenia) & renal toxicity

Characterized by a persistent pattern of unstable relationships, mood lability, and impulsivity. May exhibit suicidal ideation or behavior in the context of an interpersonal crisis which with they feel rejected or abandoned

Borderline personality disorder

Bacillus anthracis edema factor is an adenylate cyclase that causes massive increases in intracellular cAMP, leading to neutrophil and macrophage dysfunction and tissue edema. This mechanism of action is similar to that of adenylate cyclase toxin, produced by what organism?

Bordetella pertussis

25 year old man with chronic cough for the past 3 weeks. PE reveals right sided face and arm swelling and engorgement of subcutaneous veins on the same side of the neck. What vein is most likely obstructed?

Brachiocephalic Brachiocephalic vein drains the ipsilateral jugular and subclavian veins. The bilateral brachiocephalic veins combine to form the SVC. Brachiocephalic vein obstruction causes symptoms similar to SVC syndrome, but only on one side of the body. Note: right brachiocephalic vein also drains the right lymphatic duct which drains lymph from the RUE, right face and neck, right hemithorax, and RUQ of abdomen.

Sotalol has both beta adrenergic blocking and class III antiarrythmic (K+ channel blocking) properties and is occasionally used in treatment of atrial fibrillation. Major side effects?

Bradycardia, proarrhythmia, and most commonly tornadoes de pointes due to QT interval prolongation

C1 inhibitor (C1INH) deficiency causes increased cleavage of C2 and C4 and results in inappropriate activation of complement cascade. Symptoms include facial swelling, life treating laryngeal edema, & GI manifestations. Increased levels of what substance in these patients is responsible for the angioedema?

Bradykinin (C1 inhibitor blocks kallikrein induced conversion of kininogen to bradykinin, a potent vasodilator associated with angioedema)

32 year old with pain and a mass in the right flank. Imaging reveals a large mass arising from the right kidney and smeller masses in the left kidney. Surgery is performed and histopathologic evaluation shows the mass is composed of fat, smooth muscle, and blood vessels. Patient is most likely to have what additional findings?

Brain hamaratomas and ash leaf skin patches Renal angiomyolipoma is a benign tumor composed of blood vessels, smooth muscle and fat. Bilateral renal angiomyolipoas are associated with tuberous sclerosis, an autosomal dominant condition characterized by cortical tubers and subependymal hamaratomas in the brain with consequent seizures and cognitive disability. Cardiac rhabdomyomas, facial angiofibromas, and leaf shaped patches of skin lacking pigment (ash leaf patches) can also occur

Maple syrup urine disease is an autosomal recessive disorder that presents with irritability, dystonia, poor feeding and a 'maple syrup' sent of urine within first few days of life. What amino acids should be restricted in diet?

Branched chain amino acids- leucine, isoleucine, valine Branched chain alpha ketoacid dehydrogenase complex deficiency which normally breaks down these AA for entry to TCA cycle. LEUCINE IS NEUROTOXIC.

Artery that supplies the scrotum, labia majora, and perineum

Branches of deep external pudendal artery

Histology of mucormycosis?

Broad ribbon-like nonseptate hyphae with right angle branching

Type of aphasia that results from damage to the inferior frontal gyrus of the dominant hemisphere. Patients are often frustrated as they understand language but cannot properly formulate the motor commands to write or form words (slow, fragmented speech). Aphasia may be associated with right upper limb and face weakness due to extension of the lesion into the primary motor cortex.

Broca's aphasia (motor, nonfluent)

Patient exposed to asbestos and is concerned about developing cancer. What malignancy is patient most likely to develop?

Bronchogenic carcinoma Patients with long history of asbestos exposure are at risk of developing asbestosis, pleural disease, and malignancies such as bronchogenic carcinoma and mesothelioma. Bronchogenic carcinoma is the most common malignancy in this population followed by mesothelioma. Smoking and asbestos exposure have a synergistic effect on the development of lung carcinoma.

Type of gallstones composed of calcium salts of uncongugated bilirubin that arise secondary to bacterial or helminthic infection of the biliary tract (E. coli, Ascaris lumbricoides, Clonorchis sinensis)

Brown pigment gallstones Note: Clonorchis sinensis is prevalent in East Asia

Legionella pneumophila commonly contaminates natural bodies of water, municipal water supplies, and water based cooling systems. The organism is inhaled in aerosolized water and establishes infection via the pulmonary route. Culture must be performed on?

Buffered charcoal yeast extract agar supplemented with L-cysteine and iron

Confronted by a patient who has failed previous treatment and feels hopeless that his condition will improve. Best initial approach to this patient?

Build the physician-patient relationship by empathizing with the patient's frustration and disappointment with past treatment failures, so that patient feels understood and will be more willing to work with you to develop a collaborative plan focused on realistic goals

First line treatment for major depressive disorder that does not cause sexual dysfunction?

Bupropion (norepinephrine dopamine reuptake inhibitor that does not effect serotonin) Note: contraindicated in patients with bulimia & anorexia nervosa & seizure disorders due to its lowering of seizure threshold!

First line antidepressant with amphetamine-like properties that works by inhibiting reuptake of norepinephrine & dopamine. Not associated with sexual side effects or weight gain but is associated with an increased seizure risk at high doses. Contraindicated in patients with seizure disorders, anorexia, and bulimia nervosa.

Buproprion

Nonbenzodiazepine anxiolytic used to treat generalized anxiety disorder. Has a slow onset of action (up to 2 weeks), lacks muscle relaxant or anticonvulsant properties, acts as a partial agonist of the 5HT1A receptor, and carries no risk of dependence.

Buspirone Note: it is not effective in treating panic disorder!

Histologically Burkitt lymphoma has a 'starry sky' appearance due to the presence of macrophages and apoptotic bodies in a sea of medium sized lymphocytes. The rates of mitosis and apoptosis in the cancerous tissue are high. Almost all causes of Burkitt lymphoma are associated with what translocation?

C-Myc gene on chromosome 8 usually onto the Ig heavy chain region of chromosome 14 [t(8,14)].

Recurrent infections by Neisseria species is a result of deficiency of what complement factors?

C5b-C9 which form the membrane attack complex

Reason for expansion in Huntington's disease?

CAG trinucleotide repeat expands because of looping of DNA and slippage of the DNA polymerase during gene replication

Specific RNA consisting of 90 nucleotides is purified and found to contain high amounts of chemically modified bases such as dihydrouridine, pseudouridine, and ribothymidine & its secondary structure arises from base pairing with the chain. Most likely composition of the 3' end of this molecule?

CCA tRNA is a small, noncoding form of RNA that has a CCA sequence at its 3'end that is used as a recognition sequence by proteins. The terminal 3' hydroxyl group of the CCA tail serves as the amino acid binding site

Patient diagnosed with somatostatinoma is found to have biliary stones. Suppression of what hormone is most likely responsible for the biliary stones?

CCK Somatostain secreted from pancreatic delta cells decreases the secretion of secretin, CCK, glucagon, gastrin, and insulin. Somatostatinoma presents with hyperglycemia or hypoglycemia, steatorrhea, and gallstones. Somatostatin is also secreted by the hypothalamus and inhibits the production of growth hormone from the pituitary gland.

Most important immune effector cells for control of M. tuberculosis when tuberculin skin testing is given and reveals significant skin induration around the injection site 48 hours later?

CD4 T lymphocytes & macrophages M. tuberculosis primarily replicates within the phagosome, leading to display of mycobacterial antigens on the MHC class II molecules. This results in activation of CD4 cells and subsequent control of the infection within macrophages. Note: neutrophils are crucial for most extracellular bacterial infections while macrophages are critical for intracellular infections

Candida antigen injected into skin produces 12mm of skin induration within 48 hours. What cells are directly involved in producing this response?

CD4+ T lymphocytes and macrophages Candidal antigen skin test assesses the activity of T cell mediated immunity though the recruitment of macrophages and CD4+ and CD8+ T lymphocytes in a type IV hypersensitivity reaction. Note: Anergy (failure to respond to Candida testing) is typical in patients with SCID

Enveloped virus that contains double stranded DNA genome and intranuclear and cytoplasmic inclusion (owl's eye) bodies common in transplant patients?

CMV

Most likely diagnosis associated with heterophile antibody negative mononucleosis like syndrome in immunocompetent patient?

CMV (pharyngitis and lymphadenopathy less common than with EBV)

Infectious esophagitis is common in patients with HIV. What type of esophagitis is characterized endoscopically by large shallow linear ulcerations (most commonly in distal esophagus) with intranuclear & cytoplasmic inclusions seen microscopically?

CMV esophagitis

Organ susceptibility to infarction after occlusion of a feeding artery is ranked from greatest to least? Why is that organ least vulnerable to infarction?

CNS, myocardium, kidney, spleen, and liver Liver has a dual blood supply (hepatic artery and portal vein) allowing it to tolerate arterial occlusion better than those with end arterial circulations (except in transplanted livers that undergo hepatic artery thrombosis, as the collateral blood supply is severed during transplantation) When the hepatic artery becomes occluded, the portal venous supple and retrograde arterial flow through accessory vessels (inferior phrenic, adrenal, intercostal arteries, etc.) is sufficient to sustain the liver parenchyma

Carbon monoxide binds to hemoglobin with much higher affinity than oxygen, thereby preventing oxygen binding to hemoglobin. It also reduces oxygen unloading from hemoglobin into tissues (shifts dissociation curve to left). Findings expected in arterial blood sample of a patient with lost consciousness due to CO poisoning?

CO increases caboxyhemoglobin concentrations but does not affect the partial pressure of oxygen and does not precipitate methemoglobinemia (formed when ferrous Fe2+ in heme is oxidized to ferric Fe3+).

Mechanically ventilated patient with diffuse cerebral edema has ventilator respiratory rate adjusted to receive a PaCO2 level of 26-30mmHg. What is the mechanism of this intervention?

CO2 is a potent vasodilator of cerebral vasculature. A drop in PaCO2 due to hyperventilation causes vasoconstriction. The resulting reduction in cerebral blood volume leads to decreased intracranial pressure.

Cardiac output is commonly calculated using CO=SR x HR equation, but it can also be determined with a pulmonary artery (Swan-Ganz) catheter by applying the Fick principle. What values are needed to calculate cardiac output using this principle?

CO= rate of O2 consumption/arteriovenous O2 content difference

MOA of Entacapone?

COMT inhibitor that increases the bioavailability of levodopa by inhibiting peripheral methylation. Note: used to treat Parkinson disease patients with motor fluctuations who are experiencing end of dose 'wearing-ff' periods with levodopa/carbidopa therapy

Agent applied to human cells that activates G protein dependent phospholipase C. What intracellular substance is like to increase after immediate exposure to this agent?

Ca2+

Patient with newly diagnosed celiac disease will have what lab abnormalities in serum Ca2+, phosphorus, and PTH?

Ca2+ - decreased Phosphorous- decreased PTH- increased The diarrhea caused by celiac disease can lead to vitamin D deficiency through malabsorption. Patients with vitamin D deficiency have decreased serum phosphorous, increased PTH (secondary hyperparathyroidism) and low Ca2+. They can also have symptoms such as bone pain and muscle weakness (osteomalacia due to decreased vitamin D). Note: hypoparathyroidism can be caused by severe hypomagnesemia which can be seen with prolonged diarrhea

After a ligand binds to G protein coupled receptor that activates phospholipase C, membrane phospholipids are broken down into DAG & IP3. Protein kinase C is then activated by what 2 substances?

Ca2+ and DAG (Ca2+ is released from ER under influence of IP3, thus with decreased IP3 there will be decreased activity of protein kinase C)

Acute lead poisoning can present with constipation, anemia, irritability, and confusion. Chelating agent for lead poisoning?

CaNa2EDTA (forms nonionizing salts to increase urinary lead excretion)

Immunosuppresants such as cyclosporine & tacrolimus work by inhibiting what essential protein in the activation of IL-2, which promotes the growth and differentiation of T cells?

Calcineurin (protein phosphatase that dephosphorylates NFAT, allowing NFAT to enter nucleus and bind IL-2 promotor)

Topical vitamin D analogs used to treat psoriasis that bind to and activate the vitamin D receptor, a nuclear transcription factor that causes inhibition of keratinocyte proliferation and stimulation of keratinocyte differentiation

Calcipotriene (calcipotriol) , calcitrol, tacalcitol

Produced by C cells of thyroid & can be used as tumor marker for medullary thyroid carcinoma

Calcitonin

Postoperative hypoparathyroidism with resultant hypocalcemia is a common complication of thyroidectomy due to inadvertent injury or removal of the parathyroid glands. Postoperative supplementation with oral calcium and vitamin D can be used to treat and prevent postoperative hypocalcemia. Medication of choice?

Calcitriol (active form of vitamin D)

Most common type of kidney stone that is visible on KUB (radiopaque)?

Calcium oxalate

Thiazide diuretics lower serum sodium, potassium, and magnesium levels, and increase what serum levels?

Calcium, uric acid, glucose, triglyceride, and cholesterol

8 year old boy with a 2 day history of fever, abdominal pain, and diarrhea. Patient's puppy had diarrhea one week ago. Stool is negative for ova and parasites. Most likely cause of patient's symptoms?

Campylobacter Compylobacter infection is a common cause of inflammatory gastroenteritis and can be acquired from domestic animals (cattle, chickens. sheep, dogs) or from contaminated food, such as undercooked poultry and underpasturized milk. Diarrhea is infmallatory and is accompanied by fever, abdominal pain, and tenesmus (feeling of incomplete defecation). Associated with Guillian Barre syndrome. Transmission is via fecal oral route.

Presence of central venous catheter and receipt of total parenteral nutrition are risk factors for what type of fungal infection?

Candidemia (Candidia in the bloodstreem) Candida display a morphology of branching pseudohyphae with blastoconidia

An arrangement in which a payor pays a fixed, predetermined fee to provide all the services required by a patient. Payment structure underlying health maintenance organization (HMO). Restricts patients to a limited panel of providers within the plan requiring referrals from a PCP prior to specialty consultations & denying payment for services that do not meet established guidelines

Capitation (payor may negotiate a captivated contract with an insurance company that then pays the providers, or a large medical group may negotiate directly with the payor)

22 year old with fever, headache, nausea, vomiting, and myalgia that began suddenly last night. This morning she was lethargic, confused, and difficult to arouse. She is exchange student who came to USA three months ago. OMH unknown. Temperature is 102.9. BP 100/60. PE shows nuchal rigidity and purpuric rash. What is the primary component of the vaccine that could have prevented patient's condition?

Capsular polysaccharide Antibodies against the polysaccharide capsule of N. meningitidis provide immunity against it. Quadrivalent meningococcal vaccines contain capsular polysaccharides from major serotypes (A, C, Y, W) of N. meningitidis. Serotype B vaccinations use recombinant proteins (due to molecular mimicry preventing polysaccharide vaccine --> difficulty in generating immune response without increasing risk of autoimmunity). Quadrivalent is part of routine vaccinations and serogroup B is recommended for specific high risk populations (asplenia, complement deficiency)

Trigeminal neuralgia presents with brief episodes of sudden and severe 'electric shock like' or 'stabbing' pain in the distribution of CN V (particularly V2 and V3). Treatment of choice?

Carbamazepine

Has been determined in healthy volunteers that the chloride content of erythrocytes is much lower in arterial blood than in venous blood. This is due to the action of what enzyme?

Carbonic anhydrase Carbonic anhydrase activity within erythrocytes forms bicarbonate from CO2 and water. Many of the bicarbonate ions diffuse out of the RBC into the plasma. To maintain the electrical neutrality chloride ions diffuse into the EBC to take their place. This process is called 'chloride shift' and it is the principal cause of high RBC chloride content in the venous blood

Cell attachment site for immunoglobulin molecule?

Carboxy terminal of the Fc portion of the heavy immunoglobulin chains (binds to Fc receptors on neutrophils and macrophages) Note: Antibody bound to antigen is able to signal for the phagocytosis of that antigen by a conformational change of the Fc region allowing binding to the Fc receptor on phagocytes leading to subsequent phagocytosis of the organism/antibody complex and destruction of the organism

Typically presents with episodic flushing, secretory diarrhea, and wheezing. Can lead to pathognomic plaque like deposits of fibrous tissue on the right sided endocardium, causing tricuspid regurgitation and right sided heart failure.

Carcinoid syndrome Carcinoids are well differentiated neuroendocrine tumors found most commonly in the distal small intestine and proximal colon with a strong propensity for metastasis to the liver. They secrete several products (histamine, serotonin, VIP) & in patients with liver metastasis these hormones are released directly into systemic circulation leading to carcinoid syndrome. Carcinoid heart disease is caused by excessive secretion of serotonin which stimulates fibroblast growth and fibrinogenesis. Endocardial fibrosis generally limited to right heart as vasoactive products are inactivated distally by pulmonary vascular endothelial monoamine oxidase Elevated 24 hour urinary 5-hydroxyindolacetic acid can confirm diagnosis

82 year old man with progressive exertion dyspnea, edema, ascites, elevated jugular venous pressure with rapid 'y' descent, prominent S4. Echocardiogram reveals LA enlargement, LVH, and NORMAL ejection fraction. Endomyocardial biopsy shows areas of myocardium infiltrated by an amorphous & acellular pink material. Most likely diagnosis?

Cardiac amyloidosis Patient has findings consistent with diastolic HF due to restrictive cardiomyopathy. Cardiac amyloidosis results from abnormal extracellular deposition of insoluble proteins such as monoclonal light changes (AL amyloidosis), mutated transthyretin (familial ATTR amyloidosis), or wild type transthyretin (senile systemic amyloidosis) in myocardial tissue. Restrictive cardiomyopathy can be caused by infiltrative disease (amyloidosis, sarcoidosis, hemochromatosis) and often results in diastolic HF due to ventricular hypertrophy with impaired ventricular filling.

Most common primary cardiac neoplasms that arise from the atria and grow into the atrial lumen. Microscopically are composed of scattered cells within a calculate mucopolysaccharide stroma

Cardiac myxoma

66 year old man with ST segment elevations in II, III, and aVF is diagnosed with an inferior wall MI. Lungs are clear to auscultation. CXR is normal. Emergent cardiac catheterization reveals complete occlusion of the proximal right coronary artery. He is persistently hypotensive in cardiac cath lab. What hemodynamic findings (CO, PCWP, CVP) are most likely to be observed in this patient?

Cardiac output: decreased PCWP: decreased Central venous pressure: increased The dominant right coronary artery perfuses both the inferior wall of the left ventricle and the majority of the right ventricle. Proximal occlusion can cause right ventricular MI, which presents with hypotension (reduced CO) and distended jugular veins (elevated central venous pressures). The lungs will be clear on auscultation and CXR (lack of pulmonary edema) unless concomitant left sided heart failure is also present.

Hypertrophic cardiomyopathy is characterized as asymmetric (septal) left ventricular hypertrophy that can result in sudden cardiac death. Autosomal dominant mutations affecting what genes are responsible for the majority of cases?

Cardiac sarcomere genes (cardiac beta myosin heavy chain and myosin binding protein C)

Patient with pulsus paradoxus (fall in systolic BP greater than or equal to 10 on inspiration), hypotension, muffled heart sounds jugular venous dissension, and tachycardia after being involved in an MVC. Most likely cause of his condition?

Cardiac tamponade Note: lung exam is normal in cardiac tamponade which can help distinguish it from tension pneumothorax

34 year old with recent viral infection presenting with hypotension, tachycardia, jugular venous dissension with clear lungs, muffled heart sounds, and pulsus paradoxus (manifesting as a loss of palpable pulse during inspiration) is consistent with?

Cardiac tamponade Note: most likely due to the recent viral illness causing viral pericarditis with significant pericardial fluid accumulation

Pulsus paradoxus (greater than 10mmHg drop in systolic BP during inspiration) is detected by inflating a blood pressure cuff above systolic pressure and gradually deflating it. The difference between the systolic pressure at which Korotokoff sounds first become audible during expiration and the pressure at which they are head throughout all phases of respiration quantifies pulsus paradoxus. Pulsus prardoxus is most commonly seen in patients with what conditions?

Cardiac tamponade (most common), severe asthma, COPD, and constrictive pericarditis Note: in these conditions that impair expansion into the pericardial space, the increased RV volume occurring with inspiration leads to bowing of the inter ventricular septum toward the LV, leading to a decrease in LV end diastolic volume and stroke volume, with resultant decrease in systolic pressure during inspiration

Assists with long chain fatty acid transport in the mitochondria. Deficiencies in this substance lead to fatty acid oxidation defects in cardiac & skeletal muscle

Carnitine

Descriptive study that tracks patients with a known condition (particular exposure, risk factor, or disease) to document the natural history or response to treatment. Cannot quantify statistical significance.

Case series

Large epitheloid macrophages with pale pink granular cytoplasm & surface CD14 at peripherary are characteristic of what?

Caseating granulomas of TB

Foreign antigen recognition at the cell surface by cytotoxic T lymphocytes stimulates a response that results in rapid cell death. What substance acts as an effector of this response?

Caspase Apoptosis can occur through either the intrinsic (mitochondria mediated) pathway or the extrinsic (receptor initiated) pathway. Both pathways converge in the activation of caspases which are proteolytic enzymes that cleave cellular proteins

4 year old with severe and recurrent infections in lungs and skin. Neutrophils fail to turn blue following exposure to nitroblue tetrazolium. What substance is likely produced by the microorganisms responsible for his infections?

Catalase (catalase positive organisms can destroy the hydrogen peroxide produced by their own metabolic activity) Chronic granulomatous disease is an X-linked recessive disorder resulting from deficiency of NADPH oxidase, the enzyme responsible for formation of reactive oxygen species in phagosomes. Neutrophils unable to kill catalase producing organisms resulting in recurrent bacterial and fungal infections that frequently involve lungs, skin, & lymph nodes

Location of the trochlear nucleus?

Caudal midbrain Nerve exits the dorsal midbrain just below the inferior colliculus (below the red nucleus). Trochlear nerve is the only nerve to decussate before innervating its target (superior oblique muscle)

Effect of chronic kidney disease on Ca2+, PTH, and vitamin D?

Causes hyperphosphatemia (binds serum Ca2+) and low 1,25 dihydroxyvitamin D (decreases intestinal Ca2+ absorption and Ca2+ release from bone). The resulting hypocalcemia stimulates release of PTH, causing secondary hyperparathyroidism

Consist of dilated vascular spaces with thin walled endothelial cells. Present as soft, blue, compressible masses up to a few centimeters in size. May appear on the skin (usually based in the dermis), mucosa, deep tissues, and viscera.

Cavernous hemangioma Note: when in the brain and viscera they are associated with VHL disease Note: superficial hemangioma appear during first weeks of life, grow rapidly and then frequently regress spontaneously by late childhood

48 year old man with high grade fever, progressive headache, double vision. Has been having purulent nasal drainage and frontal headache for past several days. PE shows ptosis, mydriasis, mild proptosis, and loss of coral reflex of right eye. Visual acuity is normal bilaterally but patient cannot move right eye in any direction. Decreased sensation of upper face. What structure most likely involved in his presentation?

Cavernous sinus Infection of the medial face, sinuses (ethmoidal, sphenoidal), or teeth may spread through the valveless facial venous system into the cavernous sinus, resulting in cavernous sinus thrombosis. Patient present with headache, fever, propotosis, and ipsilateral defects in CNs III, IV, VI, V1 and V2

Third generation parenteral cephalosporin with poor anaerobic activity. Used primarily for the treatment of community acquired pneumonia and meningitis. Active against H. influenzae, Klebsiella pneumoniae, E.coli, S. pneumoniae, S. pyogenes, Neisseria, and Serratia species

Ceftriaxone

Selective COX-2 inhibitor that proves anti-inflammatory benefits without interfering with physiological functions of COX-1. Has no significant effect on platelet fiction & associated with lower incidence of GI bleeding, however, it is associated with increased incidence of thrombosis

Celecoxib

Serology testing with elevated IgA anti-endomysial and anti-tissue transglutaminase antibodies is suggestive of?

Celiac disease (diagnosis confirmed by endoscopic biopsy)

Presents between 6 and 24 months with abdominal pain, diarrhea, vomiting, and weight loss. Duodenal biopsy reveals crypt hyperplasia, villous atrophy, and intraepithelial lymphocyte infiltration. Most likely what disease & what is treatment?

Celiac disease (gluten-sensitive enteropathy). Treatment with gluten free diet.

Imaging studies reveal mass in right hepatic lobe & patient is scheduled to undergo percutaneous embolization of artery supplying the tumor. Contrast material administration into what structure would enhance the artery prior to embolization?

Celiac trunk Celiac trunk is first main branch of abdominal aorta that provides oxygenated blood to sleep, stomach, liver, abdominal esophagus, and parts of the duodenum and pancreas. Proper hepatic artery (off of common hepatic artery, which branches directly off celiac trunk) is a distal branch of the celiac trunk and supplies arterial blood to liver

Patient with bone pain, fatigue, anemia, kidney disease, and hypercalcemia is diagnosed with multiple myeloma and is treated with an infusion of a boronic acid containing dipeptide that has high affinity for the proteasome catalytic site. Treatment results in?

Cell apoptosis. As a result of increased immunoglobulin protein production, plasma cells are particularly susceptible to the effects of proteasome inhibitors, such as bortezomib (boronic acid containing dipeptide). Proteosomes break down proteins into their component building blocks for new proteins. Inhibition of proteosome results in accumulation of toxic intracellular proteins (increasing apoptosis). Proteosomes also balance pro an anti apoptotic proteins and their inhibition increases proapoptotic proteins.

Patient with inflammatory acne. Exocrine glands involved in this condition release their secretions by what mechanism?

Cell lysis releases entire contents of the cytoplasm & cell membrane (sebaceous glands are the primary glands involved in acne and are of the holocrine type). Meibomian glands of the eye also release contents via this mechanism. Merocrine glands (salivary, eccrine sweat, apocrine sweat) secrete watery, secretory product via exocytosis with no loss of cell membrane Apocrine glands (mammary) release membrane bound vesicles containing secretory product

54 year old with fever, chills, and malaise over the past day. Recently diagnosed with Hodgkins disease. Temperature is 103, BP102/61, pulse is 114. Patient appears ill. Blood cultures grow motile, gram positive rods that produce a very narrow zone of beta hemolysis on blood agar. Bacteria grow well at refrigerated temperature. What process is most important for eliminating these bacteria from the body?

Cell mediated immunity Listeria monocytogenes is a facultative, intracellular, gram positive rod that produces a narrow zone of beta hemolysis on blood agar, shows tumbling motility at room temperature and can grow at refrigerated temperatures. Intracellular pathogens such as Listeria are largely eliminated by the cell mediated immune response (T cells, macrophages, cytokines). Patients with impaired cell mediated immunity are at risk for invasive Listeria infections (particularly people at extremes of age or those who are immunosuppressed or pregnant).

62 year old woman with longstanding T2DM experiences progressive renal decline over 4-5 years despite optimal BP control. She's started on interim hemodialysis and eventually undergoes transplantation of decreased donor renal allograft. 3 weeks after surgery she develops generalized malaise. PMH of diabetic retinopathy & osteoporosis and currently on mycophenolate and tacrolimus. Elevated BUN and creatinine on labs. out likely cause of her current condition?

Cell mediated immunity Acute renal allograft rejection can be antibody or cell mediated process. Acute cellular rejection is associated with diffuse lymphocytic infiltration of the renal vasculature (endothelitis), tubules, and interstitium.

Amphotericin B can cause a decline in renal function bu binding to?

Cell membrane cholesterol Amphotericin B binds the ergosterol of fungal cell membranes to exert anti fungal effects. However, it also binds cholesterol to some degree causing toxicity to human tissues. The most important adverse effects are nephrotoxicity, hypokalemia, and hypomagnesia Note: Azoles- bind cytochrome P450 enzymes Griseofulvin (used for dermatophytes)- bind microtubule proteins

65 year old with sudden onset right calf and foot pain. Immediate angiography is ordered that reveals an obstructive thrombus in the right common femoral artery. Thrombus extraction is followed by a rapid surge of creatine kinase level, which is best explained by?

Cell membrane damage Reperfusion injury is thought to occur secondary to oxygen free radical generation, activation of complement pathway, mitochondrial damage, and inflammation. When cells within the heart, brain, or skeletal muscle are injured, the enzyme creatine kinase leaks across the damaged cell membrane and into the circulation

Type of diabetes insipidus with increase of urine osmolarity after administering ADH?

Central DI Note: Persistent central DI is caused by damage to hypothalamus whereas transient central DI is caused by damage to posterior pituitary

Most common cause of lobar/cortical parenchymal hemorrhage, particularly in older adults?

Cerebral amyloid angiopathy

75 year old man with problems with vision and right sided hemisensory loss that started an hour ago. Head CT reveals multiple, small lobar hemorrhages of varying ages in the occipital and parietal areas with a medium size acute bleed in the left parietooccipital lobe. Two years ago, patient developed sudden right arm weakness and neuroimaging at the time demonstrated a small left frontal lobe hemorrhage. No head trauma. Not on anticoagulants. Patient most likely suffered from?

Cerebral amyloid angiopathy (most common cause of spontaneous lobar hemorrhage, particularly in the elderly) Due to beta amyloid deposition in the walls of small to medium sized cerebral arteries, resulting in vessel wall weakening and predisposition to rupture. Most common sites of hemorrhage include occipital (resulting in homonymous hemianopsia) and parietal lobes (resulting in contralateral hemisensory loss). Frontal lobe hemorrhage is less common but can result in contralateral hemiparesis.

HPV infection of basal cervical cells can progress to premalignant and cancerous lesions. Cervical intraepithelial neoplasia refers to atypical squares cells and is classified as low grade if it extends less than 1/3 of the epithelium or high grade if extends beyond 1/3 of the epithelium. Invasion of the base membrane signifies?

Cervical carcinoma

Presents as deep, painful ulcers with ragged borders that as associated with grey exudate and inguinal lymphadenopathy

Chancroid (due to Haemophilus ducreyi, a gram negative bacteruim transmitted through sexual contact) Diagnosis is established by gram stain and culture of the organism from scraping of the ulcer base (you 'do cry' from H. ducreyi). Organism appears as curved gram negative rods often in a clumping pattern on gram stain

Presents with purulent ulcers with 'ragged' borders and suppurative lymphadenopathy. Gram stain may occasionally reveal gram negative rods in a 'school of fish chain'

Chancroid (due to Heaemophilus ducreyi)

Deep intraparenchymal hemorrhage is most commonly caused by hypertensive vasculopathy of the small penetrating branches of the cerebral arteries and is called?

Charcot-Couchard aneurysm rupture Note: saccular aneurysms typically cause subarachnoid hemorrhage

Autosomal recessive disorder of neutrophil phagosome lysosome fusion that results in immunodeficiency (recurrent pyogenic infections), partial albinism, & neurologic defects (nystagmus, peripheral and cranial neuropathies) most commonly diagnosed in childhood.

Chediak-Higashi syndrome Note: immunodeficiency results from defect in neutrophil function & causes abnormal giant lysosomal inclusions

Wilson disease (hepatolenticular degeneration) is an autosomal recessive mutation of the ATP7B gene that results in excess copper leading to toxic accumulation in the liver, basal ganglia, and cornea. Treatment?

Chelation therapy with D-penicillamine to remove excess loosely bound serum copper

What does aspiration of gastric acid cause?

Chemical pneumonitis. Typically occurs within hours of aspiration event & resolves without antibiotics

Small, red cutaneous papule common in aging adults (usually appear 3rd or 4th decade of life). Do not regress spontaneously and typically increase in number with age. Light microscopy shows proliferation of capillaries and post capillary venules in the papillary dermis

Cherry hemangioma (most common benign vascular tumors in adults and are also known as cherry angiomas or senile hemangioma)

29 year old woman with fever and skin rash. Returned from Brazil 10 days ago. Symptoms started 5 ays ago with headache, retro-orbital pain, high fever, and joint & muscle pains. Reports an episode of epistaxis. Noticed skin rash all over her body today. Exam reveals diffuse maculopapular rash and scattered petichiae. Labs show thrombocytopenia. What disease has similar mode of transmission to that causing the patient's symptoms? Differential diagnosis? How to differentiate them?

Chikungunya Fever Patient presents with classic Dengue fever: acute febrile illness with headache, retro-orbital pain, and joint and muscle pain. Other findings include hemorrhage (epistaxis, petechiae, puprura, melana), thrombocytopenia, leukopenia, and hemoconcentration. Ades aegypti mosquitos can transmit the viruses that cause dengue fever and chikungunya. Chikungunya is a febrile illness with flulike symptoms, prominent polyarthralgias, and diffuse macular rash

Officials of a public health department report a higher than normal prevalence of AML among children age 5-12 in their community. Some households in the community are exposed to chemical waste from a nearby factory and worry exposure to this waste is responsible for increased prevalence of AML. Case control study is designed to evaluate the officials' claim that exposure to chemical waste is associated with AML in childhood. Which population should be selected as the control group?

Children who do not have AML, REGARDLESS of exposure status to a chemical waste Selection of control subjects in case control studies is intended to provide an accurate estimation of exposure frequency among the non diseased general population. Cases and controls should be selected based on disease status, not exposure status

45 year old with groin pain and swelling. Noted a sore on his penis a month ago but didn't seek medical attention because it was painless and disappeared in a week. Began experiencing painful swelling in his inguinal region with inflammation of the overlying skin and eventual formation of several draining ulcers several days ago. Has mild fever and fatigue that began at the same time as his groin symptoms. Cell scrapings from lesions show cytoplasmic inclusion bodies. Most likely cause of patient's condition?

Chlamydia trachomatis Chlamydia trachomatis serotypes L1 through L3 cause lymphogranuloma venereum, a sexually transmitted disease characterized initially by painless ulcers with later progression to painful inguinal lymphadenopathy ('boboes') and ulceration. Chlamydial inclusion bodies are seen in the host cell cytoplasm. Note: Klebsiella granulomatis also presents with a painless genital papule that eventually ulcerates, but lymphadenopathy is not present

Drug that suppresses bacterial protein synthesis by binding to the ribosomal 50S subunit and inhibiting the peptidyl transferase enzyme. Hematologic side effects include dose related anemia, leukopenia, and/or thrombocytopenia that are reversible by withdrawing the medication. Also causes dose independent aplastic anemia (irreversible) which is which is usually severe and fatal without bone marrow transplant

Chloramphenicol

Characterized by congenital obstruction of the posterior nasal passages. Infants with bilateral choanal atresia can present with upper airway obstruction and cyanosis with feeding, but do not have excessive drooling (in contrast to TE fistula with esophageal atresia). Inability to pass a nasogastric tube through nares is diagnostic

Choanal atresia

Risk of what cancer is increased with fibrotic diseases of the bile ducts, such as primary sclerosing cholangitis, congenital fibropolycystic diseases, & chronic infection with Opisthorchis sinensis (liver fluke)?

Cholangiocarcinoma

Yellowish eyelid papule or plaque containing lipid laden macrophages is most likely xanthelasma which can occur in association with primary or secondary hyperlipidemia. Common cause of xanthelasma?

Cholestatic conditions such as primary biliary cirrhosis leading to hypercholesterolemia Note: ADPKD causes mild persistent proteinuria that can result in dyslipidemia, but its a less common cause of xanthelasma than biliary cirrhosis

Aromatase favors the formation of what type of gallstones?

Cholesterol Aromatase catalyzes conversion of androgens to estrogen which increases hepatic cholesterol uptake. Estrogens up regulate HMG-CoA reductase which increases cholesterol synthesis

73 year old with unstable angina undergoes coronary angiography via the femoral approach. Stent is placed in RCA and he is discharged safely. Returns two days later with blue discoloration of his right toe and reports pain and mild tingling in the affected toe. PMH of hyperlipidemia and CAD. On PE right toe is cyanotic and their is lived reticular affecting the right thigh. Bilateral LE pulses are palpable. Serum creatinine is 2.8 and preoperatively it was 1.0. What histopathologic finding would likely be seen on biopsy of patient's kidney?

Cholesterol clefts in arterial lumen Signs of embolism (Blue toe, lived reticularis with normal peripheral pulses) following an invasive procedure is suggestive of atheroembolic disease in which cholesterol containing debris gets dislodged from larger arteries and lodges in smaller vessels, resulting in ischemia. Invasive vascular procedures can be complicated by atheroembolic renal disease which may involve the kidneys (most commonly- oliguria, azotemia), GI tract, CNS, and skin. Light microscopy shows a partially or completely obstructed arterial lumen with needle shaped cholesterol clefts within the atheromatous thrombus.

Patient with myasthenia graves presents with muscle weakness & edrophonium administration (Tension test) produces no improvement in symptoms. Diagnosis and treatment?

Cholinergic crisis (too much Ach) & discontinue AChE inhibitors temporarily NOTE: Myasthenic crisis also presents with exacerbations of muscle weakness but edrophonium administration IMPROVES symptoms. Increase dose of AChE inhibitors in this case.

What nerve is responsible for taste to anterior 2/3 of tongue?

Chorda tympani branch of facial nerve

Characteristic of posterior uveitis & typically presents with painless vision loss and floaters. Ophthalmic exam may show inflammation/leukocytes in vitreous humor.

Chorodial inflamamtion Note: uveitis is often associated with systemic inflammatory disorders (IBD, ankylosing spondylitis)

Modified postganglionic sympathetic neurons derived from neural crest that receive sympathetic input (neuroendocrine cells)

Chromaffin cells (have deeply basophilic cytoplasm & derived from neural crest)

Patients with both sporadic and hereditary (associated with VHL disease) renal cell carcinomas are found to have deletions or mutations involving the VHL gene on what chromosome?

Chromosome 3p

Some patients with non small cell lung carcinoma harbor a chromosomal rearrangement that creates a fusion gene between EML4 and ALK resulting in a constitutively active tyrosine kinase that causes malignancy. This is similar to the pathophysiology of CML in which there is a translocation of what two chromosomes leading to BCR-ABL fusion protein with constitutive tyrosine kinase activity

Chromosomes 9 and 22 Note: the kinase activity of BCR-ABL fusion protein is the target of protein kinase inhibitor, Imatinib

32 year old woman with worsening SOB. Normal PaO2 (partial pressure of oxygen dissolved in blood), normal %O2 saturation of hemoglobin, decreased O2 content. Most likely cause?

Chronic blood loss (low hemoglobin concentration)

Symptoms & treatment of Giardia lamblia?

Chronic diarrhea, malabsorption, flatulance. Treated with Metronidozole. Most common enteric parasite in US and Canada. Common in campers/hikers who drink contaminated water

44 year old woman complaining of indigestion. Patient develops discomfort in her RUQ and nausea after fatty meals, which subside spontaneously in several hours. Immigrated to USA 10 years ago from Nepal. Ultrasound revels gallstones and she undergoes cholecystectomy. Stones have low cholesterol and appear small, dark, and speculated. Most likely condition that predisposed this patient to her gallstone formation?

Chronic hemolysis Black pigment stones arise from conditions that increase the about of unconjugated bilirubin in bile, which promotes calcium bilirubinate precipitation. This may occur in the setting of chronic hemolysis (sickle cell anemia, B-thalasemia, hereditary spherocytosis) and increased enterohepatic cycling of bilirubin (ileal disease) Note: obesity/metabolic syndrome, multiparty, OCP use, and rapid weight loss are significant risk factors for CHOLESTEROL gallstones

4 year old dies in hospital from infection. Autopsy reveals bone deformities and hepatosplenomegaly. Clumps of erythroid precursor cells are found in liver & spleen. Presence of precursor cells is related to what condition?

Chronic hemolysis Presence of erythroid precursors in liver & spleen is indicative of extramedullary hematopoiesis, characterized by EPO stimulated hyperplastic marrow cell invasion of extramedulalry organs. Extramedullary hematopoiesis is most frequently caused by severe chronic hemolytic anemias (such as B-thalassemia)

36 year old man with chronic mild dyspnea and fatigue. Reports his mind seems 'foggy' all the time and he can't think clearly. Lives a sedentary lifestyle and smokes half a pack of cigarettes a day for 5 years. Lung auscultation is unremarkable and CXR is normal. BMI is 31. ABG show partial pressure of oxygen is 60 mmHg and partial pressure of CO2 is 54 mmHg. His estimated alveolar to arterial oxygen gradient is 10 mmHg. What best explains this patients lab findings?

Chronic hypoventilation Obesity hypoventilation syndrome is characterized by chronic fatigue, dyspnea, difficulty concentrating, and evidence of hypoventilation (partial pressure of PaCO2 > 45 mmHg while awake). Its one of the important causes of hypoxemia with a NORMAL (5-15 mmHg) alveolar to arteriolar oxygen gradient. Hypoxemia in the setting of a normal A-a gradient indicates both the alveolar and the arterial partial pressure of O2 are low which can occur with alveolar hypoventilation or inspiration of air at high altitude.

Most common form of hypothyroidism. Histopathology shows intense lymphocytic infiltrate with germinal centers. Residual follicles may be surrounded by Hurthle cells (large oxyphilic cells filled with granular cytoplasm)

Chronic lymphocytic (Hashimoto) thyroiditis

Cytokine independent activation of signal transducers and activators of transcription (STAT) proteins (JAK-STAT pathway) is associated with what disorders?

Chronic myeloproliferative disorders (usually JAK2 mutation)

Manifests months to years after transplantation & presents with worsening HTN and a slowly progressive rise in serum creatinine. Mediated by a chronic, indirect immune response against donor alloantigens & results in obliterate intimal thickening, tubular atrophy, and interstitial fibrosis

Chronic renal allograft rejection

Pathologic condition that leads to abdominal aortic aneurysm?

Chronic transmural inflammation AAA is associated with risk factors (age >60, smoking, HTN, male sex, family history) that lead to chronic transmural inflammation and extracellular matrix degradation within the wall of the aorta. This leads to weakening and progressive expansion of the aortic wall, resulting in aneurysm formation, typically below the renal arteries

42 year old woman presents with nonproductive cough and worsening of SOB with exertion. Has a history of primary pulmonary HTN and underwent lung transplant 8 months ago. Denies missing any doses of transplant medications. PFT demonstrates an FEV 1 67% of her best post transplant FEV 1. Lug biopsy shows areas of total fibrotic obstruction in the terminal bronchioles. Condition is most likely caused by?

Chronic transplant rejection Cough, dyspnea, obstructive findings on spirometry, and fibrotic destruction of small airways is likely chronic transplant rejection which is a major problem in lung transplant recipients (occurs in half of all patients within 5 years of transplant). It affects small airways causing bronchiolitis obliterates. Characterized by lymphocytic inflammation, fibrosis, and ultimately destruction of the bronchioles. Note: acute rejection occurs within the first 6 months & is frequently asymptomatic and discovered on surveillance biopsies as perivascular and interstitial mononuclear cell infiltrates

Patient diagnosed with varicella-zoster infection. Viral strains isolated from the patient are found to lack viral phosphorylating enzymes. What drug will be effective in treating his infection?

Cidofovir (absence of viral kinase in a herpesvirus strain confers resistance to nucleoside analog antiviral drugs) Acyclovir, valacyclovir, famciclovir, and ganciclovir are all nucleoside analogues that require both herpes viral and cellular kinases for conversion to their active nucleoside triphosphate form. Cidofovir is nucleoside monophosphate (nucleotides) that requires only cellular kinases for activation.

Imipenem binds to penicillin binding proteins, thereby inhibiting the final step of peptidoglycan synthesis in bacterial cell walls. What drug can be used in conjunction with imipenem because it prevents renal metabolism of imipenem by competitively inhibiting dehydropeptidase on the renal tubule brush border?

Cilastatin

Respiratory epithelium changes in composition as the airways continue distally from the trachea to the alveolar ducts. What feature is the last to disappear?

Cilia (presence of cilia beyond the most distal mucus producing cells prevents bronchiolar mucus accumulation and airflow obstruction) Bronchi have ciliated pseudostratified columnar epithelium with mucin secreting goblet cells and submucosal mucoserous glands. The airway epithelium gradually changes to ciliated simple cuboidal by the level of the terminal bronchioles. Bronchioles lack glands and cartilage, and the number of goblet cells decreases distally, ending before the terminal bronchioles. Ciliated epithelium persists up to the respiratory bronchioles.

Phosphodiesterase inhibitor that is occasionally used in patients with symptomatic peripheral vascular disease (claudication)

Cilostazol

Second generation quinolone that acts by interfering with bacterial DNA gyrate (topoisomerase II). Active against most gram negative rods & bacilli, including Leigonella and Psudomonas aeruginosa. Not effective against anaerobes

Ciprofloxacin

78 year old with 2 days of nausea, vomiting, abdominal pain, and headache. Has a history of COPD for which she takes theophylline. She was recently hospitalized for a stroke which was complicated by a UTI and delirium. She has a fine tremor on exam and is agitated and restless. Serum theophylline levels are markedly elevated. Treatment with what agent most likely precipitated her current symptoms?

Ciprofloxacin (treated with Ciprofloxacin for UTI which caused theophylline toxicity) Theophylline is an adenosine receptor antagonist and indirect adrenergic agent (phosphodiesterase inhibitor that increases cAMP levels similar to B-adrenergic agonists causing bronchodilation) with a narrow therapeutic index. Its predominantly metabolized by the hepatic cytochrome oxidases. Inhibition of these enzymes by concurrent illness (infection, fever) or drugs (cimetidine, ciprofloxacin, macrolides, verapamil) can raise serum concentration and cause toxicity (tremor, insomnia, GI disturbance, hypotension, tachycardia, seizures, cardiac arrhythmias)

7 year old with vaccinations up to date. Sustains injury and anaerobic cultures of wound aspirate grow Clostridium tetani. Patient never develops tetanus because?

Circulating antibodies neutralize bacterial products C. tetani produces the protein exotoxin tetanospasmin that blocks release of inhibitor neurotransmitters from inhibitory motor interneurons in the CNS. Tetanus is prevented by immunization with toxoid that triggers the production of antitoxin antibodies (active immunity)

Block sodium channels and inhibit phase 0 depolarization of the action potential, slowing condition. Have moderate K+ channel blocking activity, leading to a slow rate of depolarization and prolongation of the action potential duration in the cardiac myocytes

Class IA antiarrhythmics (quinidine, procainamide, disopyramide)

Torsades de pointes refers to polymorphic ventricular tachycardia that occurs in the setting of congenital or acquired prolonged QT interval. Torsades is most commonly precipitated by medications that prolong the QT interval such as?

Class IA antiarrythmics (quinidine, procainamide, disopyrimide), Class III (solatol, ilbutilide, dofetilide, amiodarone), antipsychotics (haloperidol) & antibiotics (macrolides, fluroquinalones)

Antiarrhythmics that predominantly block K+ channels and inhibit the outward K+ currents during phase 3 of the cardiac action potential, thereby prolonging depolarization and total action potential duration

Class III antiarrhythmics (amiodarone, sotalol, dofetillide)

E. Coli sepsis, vomiting, lethargy, renal dysfunction, cataracts, and jaundice in a neonate who recently started breastfeeding are clinical features classic of?

Classic Galactosemia. Build up of galactose-1-phosphate is responsible for liver and renal dysfunction. Galacitol accumulation in the lens results in cataracts. (stop breastfeeding and switch to soy based formula!)

Involves a neutral stimulus being repeatedly paired with a non-neutral stimulus that elicits a reflexive, unconditioned response

Classical conditioning (over time the firmly neutral stimulus can evoke a conditioned response by itself in absence of non-neutral stimulus)

Results when the maxillary prominence fails to fuse with the intermaxillary segment during the 5th-6th week of embryonic development

Cleft lip

Occurs when the palatine shelves fail to fuse with one another or with the primary palate

Cleft palate

Active against anaerobic gram negative rods and aerobic gram positive cocci. Most significant side effect is pseudomembranous colitis

Clindamycin

64 year old man with exertional chest pain for the past 6 months. History PAD, T2DM, lifelong 1PPD smoker. Undergoes treadmill exercise stress testing and develops substernal chest pain on moderate exertion accompanied by ECG changes that resolve immediately upon rest. Refuses invasive cardiac testing. Started on low dose ASA therapy for secondary prevention of CVD but experiences SOB and wheezing with the ASA. Best alternative therapy?

Clipidogrel (irreversibly blocks the P2Y12 component of ADP receptors on the platelet surface and prevents platelet aggregation & is as effective as ASA in preventing cardiovascular events in patients with coronary heart disease- such as this patient with stable angina)

Heath care providers working on a team should convey what type of communication in which team members repeat back the information received to ensure that the correct information has been conveyed?

Closed loop communication Receiver repeats the message back to the sender & then the sender will confirm the message by saying 'yes' if it is accurate. Closed loop communication is highly effective and reduces risk of medical errors

A machine was found to be inadequately sterilizing equipment due to a malfunction limiting the temperature to 212 F during the 15 minute cycle. Patients who were operated on by these surgical instruments most likely developed an infection from what bacteria?

Clostridium perfringens Spore forming bacteria can survive boiling temperatures. Bacillus and Clostridium species are common pathogenic spore forming bacteria. Autoclave machines use heated stem at temperature of 273.2 F to rapidly destroy bacteria, fungi, viruses and spores

Drug of choice for treatment resistance schizophrenia?

Clozapine (treatment requires monitoring absolute neutrophil count due to risk of agranulocytosis)

Nonciliated cells found predominantly in the terminal portions of the bronchioles that release a secretory protein (protects against airway inflammation and oxidative stress) and surfactant components (prevent bronchiolar collapse)

Club cells

Type of headaches that cause rapid onset severe, episodic unilateral perioribital & temporal pain associated with lacrimation, nasal congestion, and ptosis. Classically occur at the same time each day and are more common in males. Commonly occur a few hours after going to sleep. Last about 30 minutes to 2 hours. Often patients have a pain free interval of about a year between each series of attacks

Cluster headaches

Brachial-femoral pulse delay and blood pressure discrepancy between upper and lower extremities are common manifestations of?

Coarctation of aorta (most commonly located just distal to left subclavian artery)

Decreased femoral to brachial blood pressure ratio is found in what cardiac condition?

Coarctation of the aorta

22 year old Caucasian male with severe headaches and vomiting. Soon after, he slips into a coma and dies. Autopsy shows a ruptured cerebral aneurysm with extensive intracranial hemorrhage. This patient's condition is most likely associated with?

Coarctation of the aorta Patient's with adult type coarctation of the aorta commonly die of hypertension associated complications, including LV failure, ruptured dissecting aortic aneurysm, and intracranial hemorrhage. These patients are at increased risk for ruptured intracranial aneurysms because of the increased incidence of congenital berry aneurysms in the circle of willis as well as aortic arch hypertension

28 year old man with depression, suicidal ideation, fatigue, hypersomnia, hyrerphagia, and vivid dreams. Recently kicked out of moms house due to stealing money to support drug addiction. PE shows old injuries from previous PVC but no other abnormalities. Most likely due to withdrawal of what substance?

Cocaine Cocaine withdrawal has minor physical symptoms, in contrast to opiates, alcohol, and benzo withdrawal. Following a binge, patients experience a period of acute, intense symptoms of severe depression with suicidal ideation, pronounced lassitude, and drug craving ('the crash').

Intoxicated patient with nasal septal perforation, agitation, tachycardia, HTN, light responsive mydriasis due to increased sympathetic activity. MOA of this drug?

Cocaine acts as indirect sympathomimetic by inhibiting presynaptic reuptake of NE, DA, 5HT. Potent vasoconstrictor that can cause myocardial ischemia and atrophy of nasal mucosa/septum

Causes disseminated mycosis in immunocompromised. Visualized as large thick walled spherules containing endospores. Prevalent in southwest USA

Coccidioides immitis

Patient treated with folic acid for anemia experiences moderate increase in hemoglobin level. She returns complaining of bilateral foot numbers and difficulty walking, which is most likely related to a deficiency of?

Colbamin (B12) Deficiency of B12 is associated with both megaloblastic anemia and neurologic dysfunction while folate deficiency is only associated with megaloblastic anemia. Moderate improvement of hemoglobin level often occurs when a deficiency in B12 is treated with folate or vice versa. Treatment of B12 deficiency with folate alone can actually worsen neurologic dysfunction which is likely what happened in this patients case

Gout patients with contraindications to NSAIDS (peptic ulcer disease & renal impairment) are treated with what drug that impairs neutrophil migration and phagocytosis by interfering with microtubule formation?

Colchicine

Causes water diarrhea (>5-10 episodes daily). Commonly seen in middle aged women and not associated with bleeding, ulcerations, inflammation, or abdominal pain

Collagenous colitis

Patient has CT pulmonary angiogram which shows right lower lobe pulmonary artery occlusion consistent with PE but no evidence of pulmonary infarction. What is the likely reason for the lack of lung tissue necrosis in this patient?

Collateral circulation The lungs are supplied by dual circulation from both the pulmonary and bronchial arteries. This collateral circulation can help protect against lung infarction due to pulmonary artery occlusion as the bronchial circulation can continue to provide blood to the lung parenchyma. Pulmonary arteries are major suppliers of blood & the bronchial arteries supply nutrients, remove waste, and provide collateral flow. When a clot occludes the pulmonary system, the bronchial system continues to supply nutrients and can even backfill the pulmonary capillaries to partial maintain gas exchange. Distal PE in small arteries are more likely to cause infarction as they can occlude areas distal to the anastomosis. When pulmonary infarction occurs its typically hemorrhagic due to low density of lung tissue and dual blood supply. Note: intrapulmonary shunting occurs when an area of the lung is adequately perfused but poorly ventilated; in PE there is redistribution of blood away from the segments affected by the clot & areas distal to the clot receive adequate ventilation and poor perfusion (dead space ventilation)

34 year old male being treated for acute leukemia develops oliguria. Renal biopsy reveals multiple uric acid crystals obstructing the renal tubular lumen. Principal site of uric acid precipitation would be?

Collecting ducts due to low urine pH Tumor lysis syndrome occurs when tumors with a high cell turnover are treated with chemotherapy. The lysis of tumor cells causes intracellular ions such as potassium and phosphorus, & uric acid (metabolite of tumor nucleic acid) to be released into serum. Uric ace dis soluble at physiologic pH, but can precipitate in the normally acidic environment of the distal tubules and collecting ducts. The prevention of tumor lysis syndrome includes urine alkalization an dehydration as a high urine flow and high pH along the nephron prevent crystallization and precipitation of uric acid.

Isolated mitral stenosis elevates left atrial diastolic pressure and can therefore cause elevated pulmonary capillary wedge pressure, pulmonary HTN, decreased pulmonary vascular compliance, right ventricular dilation, and functional tricuspid regurgitation. Diastolic pressure in the LV is usually near normal or even secreted with severe mitral stenosis. Increased LV diastolic pressure in a patient with mitral stenosis would indicate what?

Combined disease involving both the mitral and aortic valves

Injury to what nerve is common and typically results from trauma to the leg near the head of the fibula. Results in unable to evert, numbness over dorm of foot and lateral shin, 'foot drop', and 'steppage gait'.

Common peroneal nerve

Patient with weakness on foot dorsiflexion ('foot drop') and eversion, as well as toe extension. Sensory loss over the lateral leg and dorsolateral foot. Most likely injured nerve?

Common peroneal nerve (susceptible to injury at the lateral neck of the fibula caused by compression or fracture). Deep peroneal nerve innervates anterior compartment muscles responsible for ankle dorsiflexion (tibialis anterior) and toe extension (extensor digitorum longus, extensor hallucis longus). Superficial peroneal nerve injury causes impaired foot eversion (peroneus longus and braves) and sensory loss over the lateral leg and dorsolateral foot.

Heterogenous condition caused by B and T cell dysfunction characterized by hypogammaglobulinemia. Presents with recurrent infections in adults & children (AFTER NEONATAL period). Cell mediated immunity is not as impaired as in SCID and thymic aplasia is unlikely

Common variable immunodeficiency

Presents with global ventricular dilation & increased ICP without significant blockage to CSF flow. Usually results from impaired CSF absorption by arachnoid villi due to inflammation and fibrosis caused by hemorrhage or infection and rarely results from increased production of CSF caused by papillomas of choroid plexus. More often found in infants and children

Communicating hydrocephalus

Randomized clinical trial

Compares two or more treatments; Subjects randomly assigned to medication or placebo and then followed for outcome

Patient with headache, dizziness, nausea for past several hours. Husband also complains of headache. Has been brining wood in fireplace to warm house after losing electricity. Substance responsible for condition most likely impairs hemoglobin function through what mechanism?

Competitive binding to heme. CO binds heme iron in hemoglobin with affinity much greater than O2 generating carboxyhemoglobin. Remaining binding sites on carboxyhemoglobin have increased affinity for O2 causing O2 dissociation curve to shift to left, impeding O2 delivery to tissues. CO also binds cardiac myoglobin with high affinity disrupting hearts ability to use O2 and decreasing cardiac output. CO binds cytochrome oxidase inhibiting aerobic metabolism & exacerbating tissue hypoxia as well.

Effect of competitive inhibitors on Km and Vmax?

Competitive inhibitors compete with substrate for active binding site on enzymes. Additional substrate is required to achieve the same rate of reaction, increasing the measured value of the Michaelis constant (Km). Competitive inhibitors don't affect enzyme function and therefore maximal velocity (Vmax) is unchanged in their presence

Patient involved in MVC receives blood transfusion and shortly after develops fever, hypotension, chest and back pain, and hemoglobinuria. Most likely cause of these findings?

Complement mediated cell lysis Acute hemolytic transfusion reaction is an antibody mediated (type II) hypersensitivity reaction caused by pre-existing ABO antibodies that bind antigens on transfused donor erythrocytes. Subsequent complement activation results in erythrocyte lysis, vasodilation, and symptoms of shock.

Pregnant women >20 weeks gestation can experience hypotension and syncope while in the supine position. This is due to?

Compression of the IVC by the gravid uterus which reduces venous return and cardiac output

54 year old with difficulty hearing. The tuning fork is placed on her left mastoid process until the sound is no longer audible. The tuning fork is then placed near her left auditory meatus and she reports hearing no sound. When the handle of the vibrating fork is placed on the middle of her forehead she hears the vibration more strongly in her left ear, then her right. Patient is most likely experiencing what type of hearing loss?

Conductive loss in left ear In conductive hearing loss, bone conduction will be greater than air conduction (abnormal Rinne test) and the Weber test will lateralize to the affected ear. In sensorineural hearing loss, air condition will be greater than bone condition (normal Rinne test) and the Weber test will lateralize to the unaffected ear

Matching is used in case control studies in order to control what?

Confounding (when a perceived association between an exposure and an outcome is actually explained by a confounding variable associated with both the exposure and outcome) Matching variables should always be the potential confounders of the study (age, race, etc.). Cases and controls are then selected based on the matching variables so that both groups have similar distribution in accordance with the variables

Formed when the processus vaginalis, an outpouching of the peritoneum, fails to obliterate, leaving a path to allow bowel contents into the inguinal canal. Present as a mass in the groin and are more common in preterm males. More prominent with increased abdominal pressure

Congenital inguinal hernia

Hydrocephalus, intracranial calcifications, and chorioretinitis (cotton white/yellow scars on retina) are classic triad of?

Congenital toxoplasmosis. Expecting mothers should avoid cat feces to prevent exposure to Toxoplasma!

Difference between pneumococcal conjugate and polysaccharide vaccines?

Conjugate vaccine induces a more robust immune response through B & T cell activation Pneumococcal conjugate vaccines are strongly immunogenic in infancy due to both B and T cell recruitment. The provide nighter, longer lasting antibody titers relative to the pneumococcal polysaccharide vaccines. Polysaccharide vaccine is poorly immunogenic in infants due to their relatively immature humoral antibody response

Form of one way DNA transfer performed by bacteria carrying a plasmid with the fertility (F) factor. Donating bacterium produces a sex pills which then forms direct connection with receiving bacterium. Donating bacterium then transfers single DNA strand containing F factor to recipient and then complementary DNA strands are synthesized

Conjugation

Gap junctions facilitate communication and coordination between cells & play an important role in labor contractions. What are the proteins that assemble into gap junctions & their density increases in the uterus before delivery in response to rising estrogen levels?

Connexins (allow passage of ions between myometrial cells)

Most common persistent opioid side effect?

Constipation (miosis also persists)

Calcification and thickening of the pericardium are common features of what condition on CT?

Constrictive pericarditis. Clinical findings include slowly progressive dyspnea, peripheral edema, and ascites. Possible causes are radiation therapy to chest, cardiac surgery, and TB.

Out with physician colleague who is consuming several alcoholic drinks. Later that evening the colleague gives medication orders over the phone and appears grossly intoxicated with slurred speech. When confronted, colleague insists he is fine, not drunk, and his on call duties end in 30 minutes anyway. Most appropriate course of action?

Contact colleague's on call supervisor to report the problem now Physicians are ethically and legally obligated to report impaired colleagues in a timely manner to protect patient safety and can assist the impaired physician in receiving appropriate evaluation and treatment In a non-emergent situation contact the designated hospital committee (physician health program). If this is not possible then contact the state licensing board

45 year old sustains traumatic injury at work and presents with deep laceration in the pal of his right hand. Three weeks later, actin containing fibroblasts and increased metalloproteinase activity are detected at the site of injury. These findings are most likely related to what complication?

Contracture During wound healing, excessive matrix metalloproteinase activity and myfibroblast accumulation in the wound margins can result in contracture. Contractors produce deformities of the wound and surrounding tissues more often on the palms, soles, anterior thorax, or serious burn sites

Acute stroke in dorsolateral thalamus presents with?

Contralateral hemisensory loss (weeks to months later can develop excruciating pain on side of deficits- thalamic pain syndrome)

Most common finding with posterior cerebral artery stroke?

Contralateral homonymous hemianopia, often with macular sparing (macula supplied by MCA)

Anterior cerebral arteries supply the medial portions of the 2 hemispheres (frontal and parietal lobes). Occlusion can cause?

Contralateral motor and sensory deficits of the lower extremities, behavioral changes, and urinary incontinence

Injury to Meyer's loop in temporal loop results in?

Contralateral superior quadrantanopia

Patient involved in MVC with retroperitoneal hematoma on CT. Most likely injury?

Contusion of body of pancreas Pancreatic injury is frequent cause of retroperitoneal hematomas that can occur following severe blunt or penetrating abdominal trauma such as malpositioned seat belts or steering wheels during MVC

Disorder characterized by neurological symptoms that are incompatible with a neurological disease

Conversion disorder (functional neurological symptom disorder) Symptoms include weakness, paralysis, gait disturbance, blindness, diplopia, aphonia, anesthesia, and seizures

MOA of flucytosine

Converted to 5-FU in fungal call and interferes with fungal RNA and protein synthesis

Function of chemokine receptor CCR5

Coreceptor that enables HIV virus to enter cells. Deletion of both CCR5 alleles results in resistance to HIV infection and deletion of one allele results in delayed manifestations of HIV disease

Kawasaki disease is a vasculitis of medium sized arteries that affects young children (usually less than 5). Most commonly presents in patients of Asian ethnicity. Diagnosis is based on presence of fever for greater than or equal to five days plus 4 of the following findings: bilateral nonexudative conjunctival injection, cervical lymphadenopathy, mucositis (erythema of palatine mucosa, fissured erythematous lips, strawberry tongue), extremity changes (edema of hands & feet, erythema of palms & soles, desquamatoun of fingertips), and rash (erythematous on extremities that spreads to trunk). Most serious complication?

Coronary artery aneurysm

Blood oxygen content of the aorta and what other vessel at rest would have the greatest difference?

Coronary sinus (cardiac venous blood drains into the RA via the coronary sinus) Myocardial oxygen extraction exceeds that of any other tissue or organ; therefore the cardiac venous blood in the coronary sinus is the most deoxygenated blood in the body. Due to the high degree of oxygen extraction, increases in myocardial oxygen demand can only be met by an increase in coronary blood flow (adenosine and NO are the most important vasodilators responsible for increasing coronary flow)

69 year old woman with Alzheimers. Has not eat or drunk anything in 24 hours. Normal blood glucose level. What hormone contributes to normal glucose level by binding to an intracellular receptor?

Cortisol (binds to recptors in the cytoplasm, complex translocates to nucleus & binds hormone responsive DNA elements altering gene transcription to enhance hepatic glucose production & limit peripheral glucose utilization) NE, E, and glucagon all exert their metabolic effects via membrane bound G protein coupled receptors that activate AC and increase cAMP. Growth hormone receptor is membrane bound that activates JAK-STAT pathway.

Intracellular polyphosphate granules are characteristic of what organism that can be visualized on microscopy after growth on Loeffler medium and staining with methylene blue?

Corynebacterium diphtheriae

Intracellular polyphosphate granules are characteristic of what bacteria?

Corynebacterium diphtheriae Granules within the cytoplasm are evident with methylene blue staining

Produces chest pain that usually occurs after repetitive activity & is characterized by pain that is reproducible with palpation and worsened with movement or changes in position

Costosternal syndrome (costochondritis)

How does invasive pulmonary aspergillosis present?

Cough, hemoptysis, pleuritic chest pain, and fever in a SEVERELY IMMUNOSUPPRESSED & NEUTROPENIC patient

Patient with HTN, hematuria, and moderate proteinuria. Renal biopsy shows linear deposits of immunoglobulin along the GBM, characteristic of anti-GBM disease. What would light microscopy findings reveal?

Crescent formation Ant-GBM antibodies react with type IV collagen causing rapidly progressive glomerulonephritis with glomerular crescent formation on LM. Immunofluorescence showing linear deposits of IgG and C3 along the GBM is characteristic

Detection of elevated amount of 14-3-3 protein in the CSF is helpful in diagnosing what disease?

Creutzfelt-Jakob disease (neurodegenerative disease caused by prions)

74 year old man with several months of choking spells, dysphagia, and cough & recurrent epidsodes of pneumonia. Has foul smelling breath but oropharyngeal and neck exams are normal. Barium swallow study reveals abnormality in the upper esophagus. Mechanism of his symptoms?

Cricopharyngeal motor dysfunction Diminished relaxation of cricopharyngal muscles during swallowing results in increased intraluminal pressure in the oropharynx. Can eventually cause the mucosa to herniate through a zone of muscle weakness in the posterior hypo pharynx, forming a false diverticulum, Zenker diverticulum. Presents in elderly (commonly males) with oropharyngeal dysphagia, halitosis, regurgitation, and recurrent aspiration

Autosomal recessive disorder of bilirubin metabolism caused by genetic lack of UGT enzyme needed to catalyze bile glucuronidation?

Crigler-Najjar syndrome (results in accumulation of unconjugated bilirubin in infants causing kernicterus and death)

Typically presents with the insidious onset of abdominal pain, diarrhea, and constitutional symptoms (weight loss, fever). Patient's are prone to developing fistulas/abscesses as the lesions infect the entire thickness of the bowel wall. Perianal disease (skin tags, fissures) is also common.

Crohn disease

24 year old woman with recurrent bloody diarrhea and right sided abdominal pain. Has had these symptoms intermittently for several years. Colonoscopy shows evidence of inflammation and dysplasia & patient subsequently undergoes a colectomy of the involved region. Findings shown in image. The intestinal wall was thickened overall. Most likely diagnosis?

Crohn disease Crohn disease is an inflammatory bowel disease characterized by patchy inflammation that can occur throughout the entire GI tract. Gross pathology demonstrates skip lesions, cobblestoning of the mucosa, transmural bowel wall thickening, and creeping fat. Transmural inflammation leads to abscesses, strictures, and fistulas.

32 year old man with abdominal pain and periodic diarrhea is found to have heme positive stools. Stool cultures show no growth of pathogenic organisms. Colonoscopy with biopsy of colonic mucosa is performed. LM shows noncaseating granulomas. Patient most likely suffers from what disorder?

Crohn disease Noncaseating granulomas are a hallmark of Crohn disease and can help distinguish it from UC, which doesn't form granulomas. Histopathology in Crohn disease also demonstrates transmural inflammation, distortion of the normal architecture, and Paneth cell metaplasia

Linear ulcerations with adjacent normal mucosa (cobblestone appearance) in the small intestine is characteristic of what disease?

Crohn's Disease

Mechanism that triggers degranulation & vasoactive substance (histamine, tryptase) release by mast cells & basophils initiating an allergic response?

Cross linking of multiple membrane bound IgE antibodies by a multivalent antigen results in high-affinity IgE receptor AGGREGATION

8 year old boy with rapid and irregular movements of hands for 1 week. Has been making unintentional 'funny faces' and has had trouble controlling the volume of his voice. Temp is 102. On exam, he moves his hands frequently and erratically and has a new systolic murmur. Several circular, faintly erythematous lesions are noted on his abdomen. Mechanism for his condition?

Cross reactivity of antibodies against bacterial and host antigens Acute rheumatic fever is an autoimmune reaction following untreated group A step pharyngitis. Major manifestations are arthritis, pancarditis, syndenham chorea, erythema marginatum, and subcutaneous nodules. Anti-group A strep antibodies (anti-M protein & anti-N-acetyl-beta-D-glucosamine) cross react and attack cardiac myosin and CNS lisoganglioside antigens

Exposure and outcome are measured simultaneously at a particular point in time ('snap shot' study).

Cross sectional Example: a group of people are RANDOMLY selected to evaluate the relationship between HTN and a common mutation in the structure of a sodium channel protein. Prevalence of HTN was determined and then based on the analysis of the results, it was concluded that the sodium channel structure mutation is associated with HTN

Subjects are randomly allocated to a sequence of 2 or more treatments given consecutively. A washout (no treatment) period is often added between treatment intervals to limit the confounding effect of the prior treatment.

Crossover study Note: differentiate from cross-SECTIONAL study which is a prevalence study that is characterized by the simultaneous measurement of exposure and outcome; a snapshot study design that frequently uses surveys

35 year old man with kidney transplant one year ago presents with pleuritic chest pain and cough. CXR reveals infiltrate in RLL. Started on broad spectrum antibiotics but his condition continues to worsen. Bronchoscopy with bronchoalveolar lavage is performed. Mucicarmine staining reveals a red capsule. Most like cause of his condition?

Cryptococcus Neoformans Only pathogenic fungus that has a polysaccharide capsule. Capsule appears red on mucicarmine stain and as a clear unstained zone with India ink. Seen in tissue as round cells with narrow based buds.

Encapsulated yeast that causes meningoencephalitis and pulmonary cryptocossosis primarily in immunocompromised, particularly those with HIV. Found EXTRACELLULARLY as its large polysaccharide capsule inhibits phagocytosis by macrophages

Cryptococcus neoformans

Yeast with thick polysaccharide capsule found in soil contained by bird droppings (pigeons) that is inhaled into the alveoli of the lungs where it usually establishes an asymptomatic infection. Can spread through blood to CNS in immunocompromised

Cryptococcus neoformans (diagnosis confirmed by India ink stain revealing round/oval yeast with thick capsule & most patients also undergo antigen testing using latex agglutination)

Major toxicities of acyclovir?

Crystal nephropathy & neurotoxicity that manifests as delirium and/or tremor

Number of new cases of a disease over a specific period divided by the total population at risk at the BEGINNING of the study (the proportion of at risk individuals who contract the disease over the specific period)

Cumulative incidence Note: it does not account of deaths in the period under the study because the patients who died were at some point in the intervening period, still considered part of baseline population at risk for the disease

Stress related mucosal disease is usually caused by local ischemia in the setting of severe psychologic stress (shock, extensive burns, sepsis, severe trauma). Ulcers arising in the setting of severe trauma/burns are called Curling ulcers and ulcers arising from intracranial injury are caused by direct vagal stimulation resulting in Act release and hypersecretion of gastric acid, are prone to perforation, and are called?

Cushing ulcers

Small cell lung carcinoma is associated with what paraneoplastic syndromes?

Cushing's (ACTH secretion) and SIADH. Note: Squamous cell carcinomas can secrete PTHrP causing hypercalcemia

What do superficial inguinal nodes drain? Deep inguinal nodes drain?

Cutaneous lymph from umbilicus down including the external genitalia & anus below dentate line (EXCEPTION: glans penis & posterior calf drain to deep inguinal nodes)

50 year old man with severe dizziness and confusion. He states he had an epidote of chest pain and took several tablets of nitroglycerin. Current medications include AsA for MI prevention, an occasional acetaminophen for headaches, and occasionally tadalafil for erectile dysfunction. BP is 50/20 & HR is 120. What cellular change is most likely responsible for his symptoms?

Cyclic GMP accumulation Using nitrates together with phosphodiesterase (PDE) inhibitors used for erectile dysfunction and pulmonary hypertension causes a profound system hypotension because they both increase intracellular cGMP which causes vascular smooth muscle relaxation. Their use together is absolutely CONTRAINDICATED

Alkylating agent that is cell cycle NONSPECIFIC with side effect of bone marrow suppression, alopecia, and hemorrhagic cystitis?

Cyclophosphamide

Drug used to treat psoriasis that inhibits NFAT (nuclear factor of activated T cells) from entering the nucleus and modulating transcription activity, impairing the production and release of IL-2 and inhibiting IL-2 induced activation of T cells

Cyclosporine

12 year old boy with intermittent substernal chest pain that occurs after heavy activity. Labs show increased serum methionine level. Which amino acid is most likely essential in this patient?

Cysteine Homocystinuria is a condition that leads to hypercoagulability & thromboembolic occlusion. Since homocysteine is prothrombic, individuals with complete cystathionine synthase deficiency can develop premature acute coronary syndrome. Other features are ectopia lentis & intellectual disability.

Comprised of lymphatic cysts lined by thin epithelium. These benign tumors are often present at birth and are most commonly located on the posterior neck and lateral chest wall. Frequently found in neonates with Turner syndrome and Down syndrome

Cystic hygroma

Must ingest EGGS from stool of tapeworm carrier to develop what?

Cysticercosis (ingestion of LARVAE cysts leads to taeniasis aka tapeworm infestation)

Patient with kidney stone, dark urine, feeling feverish. Father had similar symptoms. Most likely type of kidney stone?

Cystine Stones caused by high levels of cystine in urine as a result of a genetic defect in the kidney transporter for cystine. Cystinuria is homozygous recessive condition caused by mutation in SLC7A9 gene

Metabolize endogenous toxins & drugs and are capable of producing ROS (which can contribute to hepatotoxicity seen with certain drugs)

Cytochrome P450 enzymes

Final component of the electron transport chain that functions to convert molecular oxygen to water while establishing the proton gradient necessary for the synthesis of ATP

Cytochrome oxidase

Home to the enzymes necessary for glycolysis, fatty acid synthesis, and pentose phosphate pathway?

Cytosol Note: Transketolase is an enzyme of the pentose phosphate pathway that uses thiamine as cofactor to shuttle 2 carbon fragments between sugar molecules & its found in cytosol

Portion of tRNA that contains numerous dihydrouridine residues & facilitates correct tRNA recognition by the proper aminoacyl tRNA synthetase

D loop

Polysaccharides must be degraded to monosaccharides by pancreatic brush border amylases before they can be absorbed. Monosaccharides can be absorbed directly. Name of the monosaccharide whose absorption is not affected by exocrine pancreatic insufficiency and can be used to differentiate between pancreatic versus mucosal causes of malabsorption?

D-xylose

Receptors located in the walls of renal, splanchnic, and mesenteric blood vessels. Stimulation of these receptors causes vasodilation & increased renal and splanchnic blood flow

D1-adrenergic (dopaminergic)

First generation high potency antipsychotics (haloperidol, fluphenazine) strongly block what receptors leading to extrapyramidal symptoms (dystonic reactions, akathisia, parkinsonism)?

D2 receptors (nigrostriatal pathway) Treat with medications with M1 receptor antagonist properties (benztropine, diphenhydramine) to help reestablish the dopaminergic-cholinergic balance and relieve dystonia

Mutations in dystrophin gene are responsible for what X-linked familial cardiac condition?

DIALATED cardiomyopathy Note: dystrophin mutations are also found in Duchenne and Becker muscular dystrophy which are both associated with cardiac involvement

8 year old boy with unsteady gait, numerous superficial blanching nests of distended capillaries on sun exposed areas of skin, and recurrent sinopulmonary infections. Genetic defect in this child plays a role in?

DNA break repair Cerebellar taxis, telangiectasis and increased risk of sinopulmonary infections are the triad of ataxia telangiectasia. Exhibits autosomal recessive inheritance & defect is in ATM gene which plays a role in DNA break repair. Immune deficiency primarily manifests as IgA deficiency and predisposes to infections of upper and lower airways. Predisposes to hematologic malignancy.

Exposure to ionizing radiation, including therapeutic and palliative radiation therapy, induces DNA damage through what two mechanisms?

DNA double strand breaks and formation of oxygen free radicals

Southern blotting can be used to detect DNA mutations. 4 steps of southern blotting?

DNA extraction from individual's cells, restriction endonuclease digestion of sample DNA, get electrophoresis to separate various sizes of DNA fragments, gene identification with labeled DNA probe

Only prokaryotic DNA polymerase to have 5' to 3' exonuclease activity (which is used to remove RNA primer synthesized by RNA primase)

DNA polymerase I NOTE: All 3 prokaryotic DNA polymerases can remove mismatched nucleotides via their 3' to 5' exonuclease activity

Specific prokaryotic enzyme responsible for removing short fragments of RNA that are based paired to the DNA template?

DNA polymerase I During bacterial DNA replication, DNA polymerase I functions to remove RNA primers via 5' to 3' exonuclease activity and replace them with DNA via 5' to 3' polymerase activity. DNA polymerase I is the only bacterial DNA polymerase that possesses 5' to 3' exonuclease activity Note: primase synthesizes the primer, it doesn't remove it!

New onset genital vesicular rash with positive Tzanck smear in a previously asymptomatic patient is suggestive of primary genital herpes simplex virus due to HSV-2. What can reduce reoccurrence of reactivation of HSV-2?

Daily treatment with oral valacyclovir, acyclovir or famciclovir

Developmental anomaly characterized by hypoplasia/abscence of the cerebellar vermis and cystic dilation of the 4th ventricle with posterior fossa enlargement. Patients often present during infancy with developmental delay and progressive skull enlargement. Other features may include cerebellar dysfunction and noncommunicating hydrocephalus (due to atresia of the foramina of luschka and magendie)

Dandy-Walker malformation Note: Arnold chiari malformation is characterized by downward displacement of the cerebellar vermis and tonsils below the foramen magnum. Patients also frequently have meningiomyelocele Note: encephalocele is a rare neural tube defect characterized by protrusion of the brain and menengies through an abnormal opening in the skill

28 year old at initial OB visit. No concerns. Irregular periods that last 3-4 days. 16 weeks based on LMP. Second trimester maternal serum quadruple screen is performed and shows decreased alpha fetoprotein level? Reason for decreased AFP?

Dating error Maternal serum quadruple screen is performed to assess risk of congenital defects. Accurate dating is critical for determine whether levels of alpha fetoprotein and other analyses are abnormal for gestational age. Irregular menses results in risk of inaccurate pregnancy dating Note: decreased ESTRIOL levels are associated with placental abnormalities and fetal growth restriction- risk increased with tobacco use

Parameter that accounts for the difference between minute ventilation and alveolar ventilation?

Dead space volume Minute ventilation (L/min)= tidal volume (L) x breaths/min Alveolar ventilation (L/min)= (tidal volume-dead space volume) x breaths/min

6 month old with hepatomegaly, hypotonia, and height and weight below the 10th percentile. Labs show hypoglycemia and ketoacidosis. Liver biopsy shows hepatic fibrosis without fat accumulation. Further analysis reveals abundant quantities of multi branched polysaccharide with abnormally short outer chains within the cytosol of the hepatocytes. Most likely deficient enzyme?

Debranching enzyme Debranching enzyme deficiency (cori disease) leads to the accumulation of glycogen with abnormally SHORT outer chains (limit dextrin) due to the inability to degrade alpha 1,6-glycosidic branch points. Patients present with hypoglycemia, ketoacidosis, hepatomegaly, MUSCLE WEAKNESS, and HYPOTONIA

What would uterine curettage specimen show in an ectopic pregnancy?

Decidualized endometrium only, consistent with dilated coiled endometrial glands and vascularized edematous stroma (occurs in luteal phase of endometrial cycle under influence of progesterone to prepare for implantation); embryonic and trophoblastic tissue will be absent from uterus

PFT findings associated with obesity related restrictive lung disease?

Decrease FEV1, FVC, ERV, & TLC. Normal RV Morbid, central obesity can cause a pattern of extrinsic restrictive pulmonary function tests. The most common indicator of obesity related disease is a reduction in expiratory reserve volume and functional residual capacity, but forced expiatory volume in 1 second, forced vital capacity, and total lung capacity are also typically decreased. Obesity alters the ability of the lung and chest wall to stretch in response to increased lung pressure. To compensate for decreased compliance, obese patients typically have increased respiratory rates with reduced tidal volumes (rapid-shallow breathing)

Affect of myocardial infarction on a cardiac function curve (cardiac output & venous return)?

Decrease both slope and maximal height of cardiac output. Venous return stays the same (preload is not affected). MI causes a sharp decrease in cardiac output due to loss of function of a zone of myocardium Note: chronic anemia would increase CO in an effort to meet metabolic demands of tissues & venous return would slightly increase due to decreased blood viscosity Note: anaphylaxis causes widespread venous dilation with increased capillary permeability and 3rd spacing fluids results in a high drop in venous return and an increase in CO in an attempt to maintain BP

Patient with recurrent episodes of recurrent chest pain that occur during physical activity. Given a sublingual tablet and reports rapid relief of pain. This drug improves symptoms by cause what hemodynamic change?

Decrease in LV end diastolic volume Sublingual nitroglycerin is used for rapid symptom relief in patients with stable angina. The primary anti-ischemic effect of nitrates is mediated by ventilation with a decrease in LV end diastolic volume & wall stress, resulting in decreased myocardial oxygen demand and relief of angina symptoms.

Cardiorespiratory response to exercise includes increased HR, CO, and RR in order to balance increased total tissue oxygen consumption and CO2 production. These coordinated adaptations result in relatively constant arterial blood gas values whereas venous oxygen is decreased and venous CO2 is increased. What effect does high altitude & panic attack have on PaCO2?

Decreased PaCO2 due to hyperventilation.

Laboratory findings of primary hypothyroidism?

Decreased T4 levels and increased TSH. T3 is primarily produced by conversion of T4 in peripheral tissues and serum levels widely fluctuate due to its short half life and are often normal range in patients with hypothyroidism Note: T3 levels correlate with clinical status in HYPERthyroidism more closely than in hypothyroidism and are often elevated

Most common adverse affects of psychostimulant (methylphenidate & amphetamines) medications used to treat ADHD?

Decreased appetite, weight loss, and insomnia (they are usually mild and can be managed without stopping the medication)

Drug is developed that is a pure adrenergic agonist without any significant antagonist properties. When infused it increases systolic and diastolic pressures and a delayed (reflexive) decrease in heart rate is noted. Administration of this drug to healthy individuals would most likely result in?

Decreased atrioventricular node condition velocity Alpha adrenergic agonists increases systolic and diastolic blood pressure by stimulating alpha 1 adrenoreceptors on the vascular walls causing vasoconstriction. The elevated systemic blood pressure then causes a reflexive increase in vagal tone, resulting in decreased HR and slow atrioventricular node condition. Examples of selective alpha 1 adrenergic agonists with this effect are phenylephrine and methoxamine.

Absence of normal enteral stimulation in patients receiving total parenteral nutrition leads to biliary stasis and increased risk of gallstones due to what?

Decreased cholecystokinin release Note: resection of ileum can also increase risk of gallstones due to disruption of normal enterohepatic circulation of bile acids

Disseminated intravascular coagulation is a common complication of gram negative bacterial sepsis, acute pancreatitis, and burn injury. In gram negative sepsis, DIC results from activation of the coagulation cascade by bacterial endotoxins causing the formation of micro emboli. Peripheral smear shows schistocytes and thrombocytopenia. Labs show?

Decreased fibirinogen levels and prolonged PT and PTT

32 year old man with mild abdominal discomfort and anorexia admits to consuming large amounts of alcohol realty. He undergoes liver biopsy which reveals triglyceride accumulation within the hepatoceullular cytoplasm. Most likely cause of his condition?

Decreased free fatty acid oxidation (secondary to excess NADH production by the 2 major alcohol metabolism enzymes, alcohol dehydrogenase, and aldehyde dehydrogenase) Other contributing factors include impaired lipoprotein assembly and secretion & increase in peripheral fat catabolism.

Hemolytic uremic syndrome, caused by infection with shiga toxin producing organisms, manifests with acute kidney injury, microangiopathic hemolytic anemia, and thrombocytopenia. Characteristic laboratory abnormalities include?

Decreased hemoglobin, haptoglobin, and platelet count & increased bleeding time, lactate dehydrogenase, indirect bilirubin, BUN, and creatinine Note: their is no DIC associated with HUS!

63 year old man with dyspnea, bilateral crackles, and presence of an S3 heart sound. Had a recent MI. Pulse ox is 89%. What most likely account for his dyspnea?

Decreased lung compliance Symptoms suggest left heart failure. In LV failure fluid accumulation in the lung interstitium results in decreased compliance

87 year old patient with fever, leukocytosis, hypotension, fever, and tachycardia. Most likely cause of increased anion gap?

Decreased oxidative phosphorylation lactic acidosis in septic shock results from tissue hypoxia, which impairs oxidative phosphorylation leading to a build up of NADH and causes shunting of pyruvate to lactate following glycolysis. Hepatic hypo perfusion also contributes to the build up of lactic acid as the liver is the primary site of lactate clearance (via conversion back to glucose)

Anemia in a woman of childbearing age is most commonly caused by iron deficiency secondary to menstrual blood loss. Iron deficiency is associated with what laboratory levels?

Decreased serum ferritin, increased total iron binding capacity (transferrin), and microcytic hypochromic RBCs

42 year old man complains of fatigue and exertion dyspnea. Over the last 2 weeks his symptoms have progressively worsened and he can no longer walk across the room without become SOB. He dies in hospital despite aggressive treatment. Autopsy is performed and cross section of heart reveals dilated left ventricle. Patient's symptoms were most likely cause by what mechanism?

Decreased ventricular contraction force Dilation of the LV cavity commonly occurs in response to systolic dysfunction (ischemic heart disease, dilated cardiomyopathy) or certain types of valvular disease (AR, MR). Chronic volume overload causes progressive eccentric hypertrophy that eventually leads to reduced ventricular contractility and decompensated HF.

64 year old male with 2 months of episodic facial pain. Describes sudden onset, severe, electric shock pain over his right cheek and ear lasting several seconds. Repots shaving and washing his face can trigger the pain and its becoming more frequent. Patient started on single drug therapy. 3 months later his symptoms are improved but he has pancytopenia on lab work. Most likely mechanism of action of the drug used to treat this patient?

Decreases Na+ current on multiple levels First line treatment for trigeminal neuralgia is carbamazepine, a neuoepileptic medication that inhibits neuronal high frequency firing by reducing the ability of sodium channels to recover from inactivation. Can cause bone marrow suppression so CBC must be monitored periodically Note: Carbamazepine can also cause SIADH leading to hyponatremia

Effects of elevated prolactin on GnRH, LH, FSH?

Decreases all 3 hormones. Large prolactin secreting pituitary adenomas can cause bitemporal hemianopsia by compressing the optic chaiasm as well.

What effect would a ureteral constriction have o nGFR and FF?

Decreases both GFR and FF (FF=GFR/RPF)

24 year old fell and now has extensive bruising over lateral right arm and total inability to extend the right wrist. X-ray reveals a mid shaft fracture of the right humerus. What artery is most likely to be injured in this patient?

Deep brachial artery Patient is exhibiting signs of radial nerve injury (wrist drop). Radial nerve originates from C5-T1 and enervates most of the forearm extensors at the elbow (triceps) and most of the hand extensors at the wrist. The deep brachial artery (profunda brachii) and radial nerve run together along the posterior aspect of the humerus & mid shaft fractures of the humerus risk injury to these two structures Note: supracondylar fractures of the humerus may injure the brachial artery & fractures of the surgical neck of the humerus may damage axillary nerve and posterior circumflex artery

Lymph from glans penis and penile urethra drain into what nodes?

Deep inguinal nodes

29 year old with right leg injury after a cycling accident. Undergoes closed reduction and immobilization of tibia and fibular shaft fractures. Begins complaining of severe increasing pain several hours later and PE shows increased tension within the anterior compartment. What structure is most likely to be compromised by this acute complication?

Deep peroneal nerve Acute compartment syndrome is caused by increased pressure within the fascial compartments of the limbs, leading to impaired perfusion. ACS can cause severe pain, myonecrosis, and nerve injury. Anterior compartment of the leg, which contains the deep peroneal nerve, is the most common site of ACS.

Underlying biochemical feature of megablastosis?

Defect in DNA synthesis

15 year old with difficulty with movement. Had to stop playing basketball due to worsening symptoms. On exam, significant kyphoscoliosis and high plantar arch are noted. His brother suffered from a neurologic disorder and died of heart failure at age 35. Patient most likely has what neuropathic findings?

Degeneration of spinocerebellar tracts Friedrich ataxia is an autosomal recessive disorder caused by a mutation in the frataxin (FXN) gene. AN increased number of trinucleotide repeats (GAA) is present in the mutated gene, causing decreased FXN expression. FXN gene codes for an essential mitochondrial protein involved in the assembly of iron surfer enzymes. Mutation leads to decreased mitochondrial energy production and increased oxidative stress, causing degeneration of neural tracts and peripheral nerves. Its characterized by cerebellar ataxia (spinocerebellar tract degeneration), spastic muscle weakness (lateral corticospinal tract degeneration) and loss of position/vibration sensation (dorsal column/dorsal root ganglia degeneration), kyphoscoliosis, and hypertrophic cardiomyopathy. Foot abnormalities and diabetes mellitus are also common.

34 year old with progressive low back pain for several years. Patient reports stiffness and pain are worst in morning and gradually improve throughout the day. Has been taking ibuprofen with some relief. Labs show a positive HLA-B27 antigen. X-ray reveals fusion of sacroiliac joints. Most helpful way to monitor disease progression in this patient?

Degree of chest expansion Ankylosing spondylitis is characterized by stiffness and fusion of axial joints (anklysosis) and inflammation at the site of insertion of tendons into bone (enthesitis). Involvement of the thoracic spine and costovertebral and costersternal junctions can limit chest wall expansion leading to hypoventilation. Chest expansion should be monitored regularly. Can also cause ascending aortitis which can lead to dilation of the aortic ring and aortic insufficient. Anterior uveitits develops in some and presents with pain, blurred vision, photophobia, and conjunctival erythema.

55 year old male found to have a solitary mass in sigmoid colon on colonoscopy. Biopsy is consistent with colon cancer. What features would carry worst prognosis in this patient?

Degree of tumor penetration Prognosis of colorectal adenocarcinoma is directly related to the STAGE of the tumor not the grade! If tumor is confined o the mucosa (stage A) patient has 90%+ change of five year survival. If it involves the muscular layer, 5 year survival rate is 70-80%. Lymph node involvement (stage C) and distant metastasis (stage D) have a poor prognosis. Tumor cell differentiation, degree of aneuploidy, and number of mitotic figures all determine the GRADE of the tumor.

Reversible, acute onset confusional state characterized by fluctuating level of consciousness with deficients in attention, memory, & executive function.

Delirium (most often seen in elderly patients with medical illness)

Replication defective RNA virus only capable of causing infection when encapsulated with HBsAg

Delta agent/hepatitis delta virus

32 year old woman with left eye pain that is accentuated by ocular movements. Also has periodic dimming of vision of the same eye. Patient reports several similar episodes over the last month, most often after a hot shower or intense workout. A year ago she had an episode of numbness and tingling in her left arm that spontaneously resolved. Smokes 1/2 PPD. Opthalmologic exam shows swelling of the optic disc on the left. Neurologic exam shows mild intention tremor and broad based gait. What is the most likely cause of this patient's condition

Demyelinating plaques Multiple sclerosis is a chronic immune mediated disease characterized by episodic, progressive neurologic deficits involving multiple anatomic regions within the CNS (oligodendrocyte destruction). Demyelinating plaques are commonly seen as hyperintense lesions on T2 weighted MRI of the brain and spinal cord & oligoclonal IgG bands on CSF analysis are typical. Common manifestations include optic neuritis (painful eye movements), intranuclear ophthalmoplegia (impaired adduction on lateral gaze), sensory deficits (arm numbness and tingling, cerebellar dysfunction (intention tremor, broad based gait), electrical sensations in the limbs or back with neck flexion, and fatigue which may worsen with heat exposure due to slowing of neuronal condition in unmyelinated nerves. Risk factors: female, caucasian, HLA-DRB1, low vitamin D, smoking, location (USA, Europe), cold climate

Transmitted by the Aedes mosquito and are single stranded RNA viruses with 4 different serotypes. Primary (first) infection can be asymptomatic or cause a self limited disease in most adults. Symptomatic disease can cause high grade fever, headache, retro-orbital pain, bleeding (epistaxis, petechiae/purpura), diffuse macular rash, muscle and joint pain ('break bone fever'), leukopenia, thrombocytopenia, and elevated live enzymes

Dengue virus Primary infection leads to lifelong immunity against the same serotype, but individuals can be infected with a different serotype. Secondary infection with a DIFFERENT viral serotype can cause a more severe illness, possible due to antibody dependent enhancement of infection, enhanced immune complex formation, and/or accelerated (NOT blunted) T lymphocyte response. Dengue hemorrhagic fever, which can be a serious manifestation of secondary infection, is due to increased capillary permeabiltiy & can be manifested by marked thrombocytopenia, prolonged fever, respiratory/circulatory failure, shock, and spontaneous bleeding.

Acute cardiac transplant rejection occurs weeks following transplantation and is primarily a cell mediated process. On histopathologic analysis of end-myocardial biopsy what would be seen?

Dense mononuclear lymphocytic infiltrate with cardiac myocyte damage Treatment with immunosuppressive drugs is aimed primarily at preventing this form of rejection

Patient with photosensitive rash in classic 'butterfly' distribution at the molar region of the face associated with inflammatory arthritis and ulcer at the hard palate has clinical features typical for new onset SLE. SLE is characterized by loss of immune self tolerance with production of autoantibodies against nuclear antigens. Binding of autoantibodies to self antigens leads to?

Deposition of immune complexes in tissues and consumption of complement due to complement activation Lab abnormalities: anemia, leukopenia, thrombocytopenia, positive ANA, anti dsDNA, anti-Smith, low complement levels , increased immune complexes

Rare condition that presents with erythematous, pruritic papules, vesicles, and bullae that appear bilaterally and symmetrically on the extensor surfaces, elbows, knees, upper back, and buttocks. Strongly associated with celiac disease.

Dermatitis herptiformis

Enzyme that catalyzes conversion of cholesterol to pregneonolone (first step in synthesis of steroid horomones)

Desmolase

52 year old with 2 month history of oral lesions that cause pain with chewing and swallowing. On exam, there is erosions of the buccal and gingival mucosa and several flaccid bullae with erosions scattered over her trunk. Blisters spread laterally with pressure and traction on seemingly uninvolved skin produces blistering. Autoantibodies directed against what structure is most likely responsible for this process?

Desmosomes Pemphigus vulgaris is an autoimmune bulls disease characterized by autoantibodies directed against desmosomal proteins (desmoglein). It presents with painful flaccid bullae and erosions affecting the skin and mucosal membranes. The bullae spread laterally with pressure (Asboe-Hansen sign) and new blisters may form with gentle rubbing (Nikolsky sign). Biopsy of a lesion will show detached keratinocytes (acantholysis), retained keratinocytes along the BM and eosinophilic inflammatory infiltrate. Note: autoantibodies against hemidesmosomes causes bullous pemphigoid in which the bullae are usually tense and remain intact as the entire epidermis separate from the dermis (no intraepithelial cleavage) & mucosal involvement is less common & Nikolsky sign is absent

When is B-hCG detectable in serum? urine?

Detectable in serum about 8 days after fertilization & detectable in urine 14 days after fertilization (produced by synctiotrophoblast after implantation which occurs about 6-7days after fertilization)

Carcinoembryonic antigen (CEA) levels are increased in colon cancer and in other malignancies and certain benign diseases. CEA cannot diagnose colon cancer, but is helpful for?

Detecting residual disease and reoccurance Note: elevated initial CEA levels are associated with a worse prognosis

Bias that refers to the face that a risk factor itself may lead to extensive diagnostic investigation and increase the probability that that a disease is identified (example: patients who smoke may undergo increased imaging surveillance due to their smoking status, which would detect more cases of cancer in general)

Detection bias

19 year old hispanic woman with abdominal pain and nausea. She struggles to describe the pain and point to lower abdomen. Shakes head no when asked if she could be pregnant or is sexually active. Affirmatively says 'now' when asked about her LMP. Immigrated to USA several months ago. PE shows moderate abdominal tenderness, no rebound or glaring, and VS stable. Patient discharged with diagnosis of food poisoning. Later that day she is taken by ambulance to ED after experiencing severe abdominal pain followed by LOC & she is found to have ruptured ectopic pregnancy. What intervention could have prevented the delay in diagnosis.

Determining whether the patient needs language assistance Physicians must be able to identify patients with limited English proficiency and ensure that professional interpreters are made available in high risk clinical situations

Pneumococcal meningiti is associated with massive CSF inflammatory cytokine release in response to bacterial cell wall components. Treatment with what prior to antibiotic therapy has been shown to reduce inflammation and decrease the risk of adverse outcomes (seizures, focal neurologic deficits) and death?

Dexamethasone Note: other forms of bacterial meningitis do not seem to benefit from glucocorticoid treatment

Iron chelating agent that can prevent anthracycline-induced cardiotoxicity

Dexrazoaxne

Results from maldevelopment of the third (inferior parathyroid and thymus) and fourth (superior parathyroid) pharyngeal/branchial pouches. Subsequent parathyroid and thymic hypoplasia results in hypocalcemia and T cell deficiency

DiGeorge syndrome (22q11.2 microdeletion)

Individual with a point mutation affecting the gene responsible for neurophysin synthesis is most likely to suffer from what condition?

Diabetes insipidus Neurophysins are carrier proteins for oxytocin and ADH. Oxytocin and ADH are carried by unique neurophysins from their site of production in the cell bodies of the paraventricular and supraoptic nuclei to their site of release in the axon terminals of the posterior pituitary. Point mutations in neurophysin II underlie most cases of herreditary hypothalamic diabetes insidious, a disorder resulting from insufficient ADH release into the systemic circulation

55 year old woman with increased swelling around ankles and face that has progressively worsening over last 2 months. PE reveals 2+ pitting edema in LE, trace edema in UE, and periorbital edema. Serum creatinine is 2mg/dL and albumin level is 2.8g/dL. UA shows 3+ proteinuria and no hematuria or casts. Kidney biopsy is performed and reveals Kimmelsteil-Wilson nodules (nodular glomerusclerosis). Most likely explanation for patient's biopsy findings?

Diabetes mellitus Diabetic nephropathy often results in progressive proteinuria with the eventual development of nephrotic symptoms and renal failure. Hyaline arteriolosclerosis and nodular glomerulosclerosis (Kimmelstein-Wilson nodules) are seen on renal biopsy. Presence of Kimmelstein-Wilson nodules indicates irreversible glomerular damage and predicts rapid decline in kidney function

Interscalene nerve block anesthetizes the brachial plexus as it passes through the scalene triangle. It is used to provide anesthesia for the shoulder and upper arm. Nearly all patients develop transient ipsilateral paralysis of what due to involvement of the phrenic nerve (C3-C5) roots as they pass through the interscalene sheath?

Diaphragm

Obstetrical ultrasound reveals twins, a male and a female. Type of twin placentation that is most likely?

Dichorionic & diamnionic Dizygotic twins (can have 2 different sexes) result from fertilization of 2 oocytes by 2 different sperm and always have 2 amnions and 2 chorions (chorions and placentas may be fused depending on the proximity of the implantation site). In contrast, monozygotic twins (genetically identical, same sex)arise from fertilization of a single oocyte. Monozygotic twinning may occur at different stages of embryogenesis which affects the organization of the fetal membranes: dichorionic/diamnionic (days 0-4) monochorionic/diamnionic (days 4-8) MOST COMMON monochorionic/monoamnionic (days 8-12) monochorionic/monoamnionic conjoined twins (>13 days)

Characterized by periodic, simultaneous, and non-peristaltic contractions of the esophagus due to impaired inhibitory innervation within the esophageal myenteric plexus. Patients typically present with liquid/solid dysphagia and chest pain due to inefficient propulsion of food into the stomach

Diffuse esophageal spasm

Analysis of alveolar gases in a particular lung segment reveals the following: Alveolar PO2: 104 Pulmonary venous PO2: 70 Pulmonary venous CO2: 40 Best explanation for these respiratory gas findings?

Diffusion impairment Gas exchange between the alveoli and pulmonary capillary blood depends on both perfusion and diffusion. The exchange of O2 and CO2 in a normal individual at rest is perfusion limited, so alveolar and capillary partial pressure are equal (104). Situations in which O2 exchange becomes diffusion limited (emphysema, pulmonary fibrosis) cause a large gradient between alveolar and capillary PO2. PCO2 is less affected due to the greater diffusing capacity of CO2.

Trimethoprim (in bacteria), methotrexate (in human cells), and pyrimethamine (in some protozoa) all inhibit what enzyme?

Dihydrofolate reductase Trimethoprim restricts bacterial growth through this process & works well in conjunction with sulfonamide which inhibits an earlier step in the bacterial folic acid pathway producing a sequential blockade Sulfamethoxazole is a structural analog of paraaminobenzoic acid (PABA) that competes with PABA to inhibit dihydropteroate synthetase, one of the enzymes needed for dihydrofolic acid synthesis (an intermediate step in the formation of THF)

Patient with ECG revealing bradycardia and PR interval prolongation. What CCB medication class would be most effective for lowering BP without worsening ECG abnormalities?

Dihydropyridines CCB (Nifedipine, amlodipine, felodipine)- primarily affect arterial smooth muscle causing vasodilation with little or no effect on cardiac condition or contractility. Can cause reflex tachycardia in response to peripheral vasodilation and are therefore useful in hypertensive patients with resting bradycardia. Nondihydropyridines (verapimil, diltiazem) affect the myocardium and can cause bradycardia and slowed AV conduction.

34 year old male undergoes doxorubicin containing chemotherapy for non-hodgkin lymphoma. Several weeks lateral develops exertion dyspnea & has difficulty sleeping flat at night causing him to use multiple pillows to fall asleep. Condition most likely responsible for his symptoms?

Dilated cardiomyopathy Anthracycline chemotherapeutic agents (doxorubicin, daunorubicin) cause cardiotoxicity mainly through the formation of anthracycline-topoisomerase II DNA cleavage complexes that affect healthy cardiomyocytes. The cardiotoxicity is dependent on the cumulative dose of anthracycline received and it manifests as dilated cardiomyopathy. Anthracyclines can also form iron containing complexes that produce DNA damaging free radicals, which also likely plays a role in cardiotoxicity. ADMINISTER DEXRAZOXANE to prevent cardiomyopathy (both free radical formation and inhibits formation of anthracycline-topoisomerase II complexes in healthy cardiomyocytes)

Acute arsenic poisoning impairs cellular respiration via binding to sulfhydryl groups, inhibition of pyruvate dehydrogenase and disruption of gluconeogenesis and glutathione metabolism. Presents with abdominal pain, vomiting, diarrhea, hypotension, QTC prolongation, and garlic odor on breath. Insecticides and contaminated water are common sources of arsenic. Chelating agent of choice?

Dimercaperol (sulfhydryl group of dimercaperol combines with sulfhydryl group of arsenic and displaces arsenic ions from sulfhydryl groups of enzymes involved in cellular respiration)

5 year old boy recently moved to US from Nepal with difficulty breathing, low grade fever, gray exudate, enlarged cervical nodes, partial soft tissue paralysis. MOA of exotoxin?

Diphtheria toxin is an AB exotoxin that ribosylates and inactivates elongation factor-2. This inhibits protein synthesis and leads to cell death

Patient presents with DVT and is started on a medication that prolongs activated partial thromboplastin time and prothrombin time in a dose dependent manner but has no effect on thrombin time. What agent was administered?

Direct factor Xa inhibitor (apixaban, rivaroxaban) Thrombin time is prolonged with medications that directly or indirectly inhibit thrombin. Because factor Xa is located at the junction of intrinsic and extrinsic pathways, direct factor Xa inhibitors prolong both PTT and PT with no effect on TT. Direct thrombin inhibitors (dabigatran) prevent the formation of thrombin (factor IIa) resulting in prolongation of PTT, PT, and TT. Note: unfractionated heparin binds to antithrombin and causes inactivation of several coagulation factors, most significantly thrombin and factor Xa & as a result PTT and thrombin time will be prolonged (PT reagent contains heparin neutralizers that minimize the prolongation of PT).

Type of hernias that occur most commonly in older men and are caused by weakness of the transversalis fascia in Hesselbach's triangle. They are medial to the inferior epigastric vessels, protrude only through the external inguinal ring, and are covered by the external spermatic fascia

Direct hernia

Parkinson patient is started on Pramipexole. Mechanism of action of Pramipexole?

Directly stimulates dopamine receptors Dopamine agonists (bromocriptine, pramipexole, ropinirole) have a long half life and can delay the need to start Levodopa in Parkinson's patients, thereby postponing the development of motor fluctuations until later in the disease course. Note: bromocriptine also treated hyperprolactinemia

Advanced Alzheimer's patient presents to hospital with daughter. He receives no medication for 2 days due to physician error. There are no consequences to this error other than extension of the patient's hospital stay by 2 days. Most appropriate course of action?

Disclose medication error and provide an apology (in this cause to daughter as patient's altered mental status prevent him from being able to understand what occurred) in a timely fashion REGARDLESS of whether harm has occurred

Agranulocytsis is a serious complication of antithyroid drugs (methimazole, propothiouracil) that results from drug induced granulocyte destruction and consequent neutropenia. Initial manifestations often include fever, sore throat, and oral ulcerations. Treatment for suspected agranulocytosis?

Discontinue drug immediately (agranulocytosis is reversible) & get a complete blood count with differential help confirm the diagnosis

Payment arrangement in which an insurer pays a provider for each individual service provided at a pre-arranged, discounted rate

Discounted fee for service

Defense mechanism in which emotions are transferred from the person or object causing the negative emotions to a more neutral, personally acceptable person or object

Displacement

Immature defense mechanism that involves shifting of emotions to a safer alternate object or person that represents the original in some regard

Displacement

Clostridium difficile toxins exert their effects by?

Disrupting actin cytoskeletal structure and intracellular signaling (inactivate Rho regulatory proteins). Results in disruption of intercellular cell junctions and increased paracellular intestinal fluid secretion. Toxin A causes more intestinal inflammation. Toxin B is more cytotoxic

Mechanism responsible for the antiseptic properties of isopropanol?

Disruption of lipid structure in cell membranes causing them to be leaky and by denaturing cellular proteins Note: alcohols are bactericidal, tuberculocidal, fungicidal, and virucidal, but they do not destroy bacterial spores

A tubular fluid sample with an osmolarity of 110 mOsm/L is obtained from a healthy animal after 12 hours of water deprivation. Assuming the physiology of this animal mirrors normal human physiology what portion of the tube was most likely sampled?

Distal tubule In an animal deprived of water for 12 hours, ADH levels will be high. Thus, the distal tubule will contain the most dilute urine while the collecting ducts will contain the most concentrated urine in the nephron.

65 year old woman with T2DM and HTN presents for routine follow up. Reports occasional numbness in her feet. Takes ibuprofen for CBP and hydrochlorothiazine, metformin, atorvostatin, and insulin deter. UA shows albuminuria. Lisinopril is initiated for early diabetic nephropathy. Next day patient returns complaining of lightheadedness and near syncope. BP is 102/66 supine and 80/45 standing. Most likely factor contributing to her current symptoms?

Diuretic therapy ACE inhibitors can cause significant first dose hypotension in patients with volume depletion (from diuretic use) or heart failure. ACE inhibitors cause abrupt removal of vasoconstrictive effects of angiotensin II, resulting in decreased vascular tone and drop in BP. To reduce this risk of first dose hypotension, ACE inhibitor therapy should be initiated at low doses.

Risk is the probability of developing a disease over a certain period of time. How do you calculate this probability?

Divide the number of affected subjects by the total number of subjects in the corresponding exposure group

Case fatality rate is a measure of the severity of a disease or condition. it is defined as the proportion of reported cases of a specific disease or condition that are fatal within the population affected by the disease or condition over a specific time and can be calculated by?

Dividing the number of fatal cases of a disease or condition by the total number of people with that disease or condition

75 year old female who has been experiencing worsening forgetfulness. No symptoms of depression. An extensive work up reveals diffuse cortical atrophy with no other abnormalities. Patient would most likely benefit form treatment with?

Donepizil A chronically progressive pre-senile dementia with cortical atrophy but no other radiological or laboratory abnormalities permits a clinical diagnosis of Alzheimer's disease, which is a diagnosis of exclusion. Current therapies include cholinesterase inhibitors (Donepezil), antioxidants (vitamin E) and NMDA receptor antagonists (memantine).

Very intoxicated man with pancreatitis is refusing medical treatment and wants to be discharged. Most appropriate course of action?

Dont allow the patient to leave the ED and reassess when sober Patients who are temporarily incapacitated shouldn't be allowed to make important health care decisions. Acutely intoxicated patients should be reassessed when sober.

Restless leg syndrome is characterized by an uncomfortable sensation in the legs accompanied by an urge to move them. It is worse at rest and when falling asleep. Preferred medication?

Dopamine agonists (Pramipexole, Ropinirole) Ergot—Bromocriptine. Non-ergot (preferred)—pramipexole, ropinirole

Location of facial nucleus?

Dorsolateral aspect of the caudal pons Facial nerve exits at the ventrolateral pontomedullary junction (below the middle cerebellar peduncles) Note: SPARING of the forehead muscles indicates that facial weakness is a result of a central lesion affecting the corticobulbar tract & NOT due to direct damage to the facial nucleus/nerve

Antibiotic effective against Lyme disease and many tick borne rickettsial diseases

Doxycycline

2 antineoplastic drugs inhibit intracellular thymidylate formation. Chemotheraputic effect of drug X can be overcome by N5-formyl-tetrahydrofolate (folinic acid, leucovorin) supplementation, but that of drug Y is not affect. What drugs are these?

Drug X- Methotrexate Drug Y- Fluorouracil Leucovorin enhances effects of 5-FU & can 'rescue' cells from toxicity of methotrexate as it can bypass the step inhibited by methotrexate

Autosomal recessive disorder characterized by absence of biliary transport protein MRP2 used in hepatocellular excretion of bilirubin glucuronides into bile canaliculi.

Dubin-Johnson syndrome (asymptomatic patients with dark liver)

Occurs when the duodenal lumen fails to recanalize after endothelial proliferation during the 8th-10th week of gestation

Duodenal astresia

Down syndrome is associated with decreased maternal serum alpha fetoprotein and increased nuchal translucency. Most common GI complication in these patients?

Duodenal astresia Note: down syndrome also associated with imperforate anus, hirschsprung disease, TE fistula & celiac disease

Celiac disease is caused by an immune mediated reaction to gluten and can present with bloating, diarrhea, and flatulance, as well as short stature and weight loss. Extraintestinal symptoms include iron deficiency anemia and dermatitis herpetaformis. Screening tests show an elevated level of IgA against tissue transglutaminase. What is confirmatory test?

Duodenal biopsy showing villus flattening and intraepithelial lymphocyte infiltration

Ulcer in what location is very unlikely to be malignant?

Duodenum Duodenal ulcers are not associated with an increased risk of carcinoma in the same location. In contrast, ulcers located in the esophagus, stomach, and colon may be malignant and biopsy is required

42 year old woman with several months of episodes of SOB and cough. Lung exam and CXR are norma. Spirometry shows FEV1 is 71% of predicted value. Sputum analysis shows predominant eosinophils. Patients symptoms are most likely related to exposure to what?

Dust mites Intermittent respiratory symptoms in a patients with a normal CXR, sputum eosinophils, and reduced FEV1 (less than 80) suggest a diagnosis of asthma. Common asthma triggers include exercise, cold air, respiratory infection, and exposure to inhaled allergens (dust mites, cockroaches, pet dander, mold, pollen)

36 year old male complaining of periodic involuntary deviation of head to the right, accompanied by muscle pain in neck. Spells start spontaneously, last 30 minutes and can sometimes be suppressed by placing hand on the chin. Condition is best characterized as?

Dystonia Dystonia is a syndrome of prolonged, repetitive muscle contractions. May be the result of impaired function of the basal ganglia. Cervical dystonia (spasmodic torticollis), blepharospasm (involuntary, faceable closure of eyelids), and writers cramp are the most common types of focal dystonia Note: hemiballism is flinging of the limbs (arm and/or leg) on one side of the body and occurs due to contralateral injury in or near the subthalamic nucleus

Lactose fermenting, indole positive gram negative rods that are an important cause of UTI in women

E. coli Note: Indole positivity (ability to convert tryptophan to indole) of E. coli distinguishes it from Enterobacter cloacae, another gram negative lactose fermenting rod that is a another common cause of UTI in women

24 year old woman with persistent white plaques on her tongue and recurrent episodes of genital itching and vaginal discharge. Unintentional weight loss and diarrhea recently. On PE she has generalized lymphadenopathy and white plaques in her oropharynx that can be easily scraped off with a tongue depressor. Vaginal wet mount with KOH shows budding yeast with pseudohyphae. Patient is at risk of developing a malignant B lymphocyte proliferation associated with?

EBV Patient has recurrent vaginal Candida infections, weight loss, diarrhea, generalized lymphadenopathy, and thrush therefore she likely has HIV. Patients with HIV often experience reactivation of EBV infections with a resulting increased incidence of EBV induced lymphoproliferative disorders (such as Burkitt lymphoma and AIDS related primary CNS lymphoma).

Exposure to monoclonal anti-CD21 antibody would prevent cell infection with which virus?

EBV (EBV envelope glycoprotein gp350 binds CD21 on B cells & nasopharyngeal epithelial cells) Note: parvovirus B19 attaches to human erythroid cells via blood group P antigen

G6PD deficiency anemia is characterized by?

EPISODES of hemolytic anemia precipitated by oxidative stress (drugs, infections, etc.) Heinz bodies (denaturated Hb)

Elderly woman with jaw claudication, an episode of amaurosis fugax (transient monocular visual loss), and achy pain in the shoulders and hip girdles (polymyalgia rheumatica) most likely has giant cell (temporal) arteritis. Best initial test for GCA?

ESR and CRP ESR and CRP have very high sensitivity for GCA. Patients with suspected GCA who have elevated ESR or CRP should be referred for a temporal artery biopsy so confirm diagnosis

Frequently indicate a deep hemorrhage (hematoma) due to bony fracture, ligamentous rupture, or muscular injury. Do not blanch under pressure as RBCs are not contained in vasculature. Often pass through evolution of color change (blue or red to brown, green, and yellow) which is useful in estimating age of injury

Ecchymosis

Parasympathetic system uses both preganglionic and postganglionic cholinergic neurons. Sympathetic nervous system typically regulates visceral function via 2 neuron signal transmission involving cholinergic preganglionic neurons and adrenergic postganglionic neuron. Two exceptions to this rule are?

Eccrine sweat glands and the adrenal medulla (both innervated by cholinergic neurons)

Cutaneous necrotic disease with strong association with Pseudomonas aeruginosa bacteremia

Ecthyma gangrenosum

Perivascular invasion and release of tissue destructive exotoxins causing vascular destruction and insufficient blood flow to patches of skin that become edematous and necrose is MOA of what?

Ecthyma gangrenosum (associated with Pseudomonas aeruginosa becteremia in septic, neutropenic, hospitalized patient)

Germ layer that gives rise to surface ectoderm, neural tube, and neural crest?

Ectoderm

Prescription was written for 2.0 mg but patient mistakenly receives 20 mg. Most effective intervention to reduce this type of error?

Educate physicians to avoid use of trailing zeros Note: avoiding the use of unsafe abbreviations in medication orders can also help reduce the indigence of medication errors

Patient reports she has been doubling her dose of peroxitine when she is having a bad day and occasionally skips doses on days when she feels better. Most appropriate action by her physician?

Educate the patient about the risks of irregular dosing Patients' misunderstanding of medication use can result in medication errors, including potential overdose, toxicity, and withdrawal. Physicians must assess the patient's understanding and provide targeted education to address misconceptions.

Present with the effect of the main exposure on the outcome is modified by the presence of another variable. NOT a bias.

Effect modification Note: with effect modulation the different strata will have different measure of association. In contrast, with confounding, stratification usually reveals no significant difference between the strata

Measure of the maximum ceiling of activity [Emax] of a drug with respect to a particular pharmacologic end point. Refers to the intrinsic ability of a drug to elicit an effect

Efficacy

Heritable connective tissue disease associated with abnormal COLLAGEN formation. Usually manifests clinically as hyper mobile joints, hyperelastic skin, and fragile tissue susceptible to bruising, wounding, and hemarthrosis

Ehlers-Danlos syndrome (commonly due to deficiencies in lysyl hydroxylase & procollagen peptidase enzymes)

Mucosal cyanosis and fingernail clubbing may be observed in cyanotic congenital heart disease or as late fears of ASDs or VSDs which is referred to as what?

Eisenmenger syndrome (blood flow across the VSD in Eisenmenger syndrome is right to left due to elevated pulmonary vascular resistance resulting in decreased SpO2 in the left ventricle)

Minimal change disease is the most common cause of nephrotic syndrome in children. Classic manifestations include proteinuria (resulting in frothy urine), hypoalbumineria, hyperlipidemia, and edema that are usually reversible with corticosteroids. Often occurs after an inciting event such as URI, immunization, or insect bite. The principal lesion is a diffuse foot process effacement that can be seen on?

Electron microscopy Note: light and immunofluresence microscopy are typically normal!

50 year old female alcoholic with melana & malaise. BP is 92/65. PE reveals epigastric tenderness and internal and external hemorrhoids. Tenderness form T5-T8. Lab result that would be indicative of hypotension?

Elevated BUN:creatinine ratio Patient has GI bleed which you should suspect is due to ruptured esophageal varices (tenderness from T5-T8, alcoholic, & hemorrhoids indicating portal HTN). GI bleed can elevate BUN: creatinine ratio & volume depletion (indicated by her low BP) will decrease blood flow to kidneys and further elevate BUN by decreasing filtration. Other lab values expected in alcoholic hepatitis are leukocytosis with bandemia, thrombocytopenia (decreased TPO produced by liver & platelet sequestration in spleen)

45 year old male with history of MI and CAD presents with worsening claudication when walking upstairs. PE reveals yellowing of palmar creases. Familial dysbetalipoproteinemia is suspected. Lipid panel would show?

Elevated chylomicron remnants, VLDL, IDL, and LDL with normal HDL

46 year old woman with painful rash involving groin and legs that has been worsening over last 2 weeks. Diagnosed with DM 6 months ago and has occasional loose stools. PE shows coalescing erythematous plaques with crusting and scaling at the borders and central areas of iron induration. Biopsy reveals superficial necrolysis. Most likely laboratory abnormality?

Elevated glucagon Glucagonoma presents with hyperglycemia (often as newly diagnosed DM) and necrolytic migratory erythema (blistering erythematous plaques with central clearing) affecting the groin, face, and extremities. Diagnosis is made by detecting elevated glucagon levels.

Pernicious anemia is an autoimmune disease characterized by CD4+ cell mediated destruction of parietal cells. As parietal cells are destroyed their ability to secrete HCl decreases, resulting in?

Elevated intraluminal pH that leads to up regulation of gastrin secretion. Patients also develop vitamin B12 deficiency as a result of decreased intrinsic factor secretion

In spherocytosis, mean corpuscular hemoglobin concentration (MCHC) is increased due to mild dehydration of the red blood cell. Markers of hemolysis are often evident and include?

Elevated lactate dehydrogenase, reticulocytosis, and decreased haptoglobin

Cause of hypocalcemia after massive transfusion over 24 hours?

Elevated plasma levels of citrate (product added to stored blood) which chelates calcium (and Mg2+) reducing plasma levels and causing paresthesias

Reflux esophagitis commonly occurs due to GERD and is characterized histologically by?

Elongation of the papillae, basal cell hypertrophy, and intraepithelial eosinophils Note: repeated epithelial injury by reflux esophagitis can lead to Barrett esophagus, which is characterized by intestinal metaplasia with goblet cells (premalignant condition that increases risk of esophageal adenocarcinoma)

Providing informal treatment to friends is ethically problematic and should generally be limited to?

Emergency situations in which no other physician is available

62 year old with SOB that has progressively worsened over the last few months. 50 pack year smoking history. Decreased breath sounds and scattered wheezes bilaterally. CXR shows hyper inflated lungs and flattened diaphragm. What would you expect on PFT?

Emphysema causes destruction of the inter alveolar walls and decreased lung elastic recoil, leading to increased RV, increased FRC, and increased TLC. COPD presents with wheezing due to air being forced through narrow, congested airways. Diminished breath sounds due to overall decrease in airflow. Barrel shaped chest due to compensatory breathing at higher lung volumes to help keep airways open.

Diabetic peripheral neuropathy is associated with poor glycemic control and/or long duration of disease and is caused by?

Endoneural arteriole hyalinization (due to nonenzymatic glycosylation of proteins) leading to nerve ischemia and osmotic damage to axons and Schwann cells (due to sorbital accumulation in peripheral nerves).

63 year old man diagnosed with renal cell carcinoma and started on high dose IL-2. 4 weeks later there is significant reduction in his tumor burden. Mechanism responsible for the IL-2 (aldesleukin) mediated regression of his malignancy?

Enhanced activity of natural killer cells IL-2 is produced by helper T cells and stimulates the growth of CD4+ and CD8+ T cells and B cells. IL-2 also activates NK cells and monocytes. The increased activity of T cells and NK cells is thought to be responsible for IL-2's anti-cancer effect on metastatic melanoma and RCC

5 year old with recurrent generalized tonic clonic seizures over past 24 hours. Patient has been having high fever and flulike syndrome for the past 3 days. Has a temperature of 103 on exam, appears lethargic, and does not follow simple commands consistently. During exam he suddenly develops sustained, generalized tonic clonic convulsions without fully regaining consciousness between episodes. MOA of the most appropriate initial therapy for his seizures?

Enhanced postsynaptic chloride influx Patient has status epilepticus likely due to complex febrile seizures or infectious meningitis/encephalitis. IV benzodiazepines (lorazepam) are the initial drug of choice for status epilepticus. They work by enhancing the effect of GABA at the GABA-A receptor leading to increased chloride influx and suppression of action potential firing. Note: Phenytoin blocks presynaptic voltage gated Na+ channels and can be used to prevent recurrence of status epileptics, but it has a slow onset of action so a rapid acting benzo should be used first

Catalyzes conversion of 2-phosphoglycerate to phosphenoenolpyruvate

Enolase

Patients experiencing intimate partner violence should be assessed for safety and approached in a supportive, nonjudgemental, open-ended manner. The physician's priority should be?

Ensuring the patient's safety through the use of referrals and developing an emergency safety plan

Recombinant form of human TNF receptor that binds to TNF-alpha. Used to treat moderate to severe plaque type psoriasis

Enteracept

"Stacked brick' intestinal adhesion is characteristic of what organism that adheres to human jejunal, ideal, and colonic mucosa in an aggregative, stacked brick pattern, and does not invade

Enteroaggregative E. Coli (EAEC)

Abetalipoproteinemia is an inherited inability to synthesize apolipoprotein B, an important component of chylomicrons and VLDL. Findings on biopsy?

Enterocytes with normal architecture and clear or foamy cytoplasm (lipids absorbed by the small intestine cannot be transported into the blood and accumulate in the intestinal epithelium) A betalipoproteinemia is most commonly caused my autosomal recessive loss of function mutation in the microsomal triglyceride transfer protein (MTP). MTP is a chaperone protein necessary for proper folding of apoB and participates in transfer of lipids to newly formed chylomicrons and VLDL particles. Manifests during 1st year of life with symptoms of malabsorption (abdominal dissension, foul smelling stool). Labs show low plasma triglyceride and cholesterol levels and chylomicrons, VLDL, and apoB are entirely absent from the blood. Poor lipid absorption causes deficiency of fat soluble vitamins (particular vitamin E) and essential fatty acids. This results in RBCs with abnormal membranes and thorny projections called acanthocytes as well as neurologic abnormalities (ataxia, retinitis pigmentosa)

Enzyme that activates trypsin from its inactive precursor trypsinogen

Enteropeptidase (duodenal brush border enzyme)

In absence of normal intestinal microbial flora (as may be the case after a course of antibiotics), Clostridium difficile can overgrow and produce what two toxins?

Enterotoxin (toxin A) leading to diarrhea & cytotoxin (toxin B) causing pseudomembrane formation Clinical disease from C. difficile overgrowth can range from transient diarrhea to severe pseudomembranous colitis

6 year old boy with 2 day history of fever and headaches. Vomited once this afternoon. All vaccinations are UTD. Temperature is 102. Exam shows mild pharyngeal erythema and neck stiffness. CSF shows normal glucose, elevated protein, and pleocytosis with lymphocyte predominance. Most likely infectious agent to have caused this patients illness?

Enterovirus (coxsackievirus, echovirus, poliovirus) Fever, meningeal signs (headache, neck stiffness), and CSF fluid that shows lymphocytic pleocytosis, a modestly elevated protein level and normal glucose are suggestive of aseptic meningitis. Enteroviruses are the most common cause of aseptic meningitis! Aseptic meningitis is typically les severe and focal neuralgic signs, seizures, and alterations in mental status are absent.

Mutations in what gene enable escape of HIV from host neutralizing antibodies?

Env gene

Viral particles isolated from nasal exudate of 10 year old are shown to lose their infectivity once exposed to ether. These viral particles are therefore most likely?

Enveloped Ether and other organic solvents dissolve the lipid bilayer that makes up the outer viral envelops. Loss of infectivity after ether exposure is characteristic feature of enveloped viruses & non enveloped viruses are generally resistant to action of ether

53 year old man presents for follow up after acute MI. Used to smoke 2 PPD but quit after MI. Father has HTN and mother has T2DM. Obese on exam. What lipid lowering agent would be most effective for preventing future cardiovascular events in this patient?

Enzyme inhibitor Although low HDL concentration is associated with an increased cardiovascular risk, the use of medications to raise HDL levels does not improve cardiovascular outcomes. HMG-CoA reductase inhibitors (statins) lower total cholesterol and LDL levels. Statins are most effective lipid lowering drugs for primary and secondary prevention of CV events, regardless of baseline lipid levels.

Small to medium sized vessel vasculitis characterized by late onset asthma, rhinosinusitis, and eosinophilia. Mononeuritis multiplex (asymmetric multifocal neuropathy) due to involvement of the epineural vessels of peripheral nerves is common. Associated with antibodies against neutrophil myeloperoxidase (p-ANCA)

Eosinophilic granulomatosis with polyangitis (Churg-Strauss)

Type of xanthomas that contain fat rather than cholesterol. Arise in large clusters, most commonly on the shoulders or buttocks & manifest as red yellow papule. Characteristic of hyperlipidemias that result in elevated triglycerides including Types I, IV, V.

Eruptive xanthomas

Characteristic lesion in Borrelia burgdorferi infection (Lyme disease) is?

Erythema migrans which begins as an erythematous macule at the site of an Ixodes tick bite (7-14 days after the bite). Erythema spreads outward, often leaving a zone of central cleaning ('bulls eye rash')

Pityriaasis versicolor (tinea versicolor) is a superficial skin infection caused by Malassezia species. It causes?

Erythematous, hyper or hypopigmented macules and patches. Melassezia forms spores and hyphae, producing characteristic 'spaghetti and meatball' appearance on KOH preparation LM.

Newborn is admitted to NICU with jaundice, hepatomegaly, an generalized edema. Labs reveal low hemoglobin and positive direct Coombs test. Peripheral blood smear show many nucleated erythrocytes. Newborn has significant respiratory distress due to pleural effusions and ascites and dies soon after birth. Autopsy show areas of extra medullar hematopoiesis in many tissues. Most likely cause of this patients condition?

Erythrocyte opsonization by maternal antibodies Hemolytic disease of newborn (erythroblastosis fetalis) most commonly occurs from maternal sensitization to Rh antigens during a prior pregnancy with Rh+ fetus. In subsequent Rh+ pregnancies, maternal anti-Rh IgG antibodies cross placenta & cause severe autoimmune hemolytic anemia in fetus and life threatening hydrops fetalis

Tissue that cannot use ketone bodies as a significant reserve of energy during starvation?

Erythrocytes (Ketones are used as energy source in mitochondria of peripheral tissues; erythrocytes lack mitochondria) Note: liver is also unable to utilize ketones for energy as it lacks succinylcholine CoA-acetoacetate CoA transferase (thiophorase) which is required to convert acetoacetate to acetoacetyl CoA

Triad of low hemoglobin, thrombocytopenia, and absent hemopoietic cells in the bone marrow is consistent with aplastic anemia. What blood parameter is expected to be elevated in aplastic anemia?

Erythropoietin

49 year old woman with 10 months of irregular vaginal bleeding. LMP was 3 years ago. Ultrasound shows thick endometrium and solid left adnexal mass. Ovarian mass is yellow and firm. Pathology reports small cuboidal cells in sheets with gland like structures containing acidophilic material. Cells are arranged in a microfollicular pattern around a pink, eosinophilic center. What is most likely secreted by this tumor?

Estrogen Granulosa cell tumors are sex cord stroll tumors of the ovary that secrete estrogen and can cause endometrial hyperplasia. Call-exner bodies (arranged in a microfollicular or rosette pattern 7 have coffee bean nuclei) are seen on microscopy. On gross pathology, the tumor appears yellow due to the lipid content in theca cells. Can cause early sexual development in young patients.

Pathogenetic components that contribute to gallstone formation in pregnancy & with OCP use?

Estrogen increases cholesterol synthesis by up regulating hepatic HMG-CoA reductase activity, which causes bile to become supersaturated with cholesterol & progesterone reduces bile acid secretion and slows gallbladder emptying

PCOS is characterized by elevated LH & estrogen levels, excess androgen production, & insulin resistance. Clinical features include obesity, menstrual irregularities, hirtuism, enlarged ovaries, and increased risk of DM and endometrial hyperplasia. Best way to treat patient with PCOS who desires fertility but weight loss has not restored ovulation?

Estrogen receptor modulation with Clomiphene (SERM that decreases negative feedback inhibition on the hypothalamus by circulating estrogen, thereby increasing gonadotropin production) Note: patients who don't want to become pregnant can be started on estrogen-progestin OCPs to minimize endometrial proliferation, reduce androgenic symptoms, and prevent unwanted pregnancy; spironolactone is an androgen receptor antagonist used that is used in those not responsive to OCPs who don't with to become pregnant (risk of fetal abnormalities)

SERMs (tamoxifen, raloxifine) are competitive inhibitors of estrogen binding to estrogen receptors. Adverse effect that is unique to tamoxifen?

Estrogenic effect on the uterus & can cause endometrial hyperplasia and cancer

Absence seizures a a type of generalized epilepsy characterized by brief staring spells with momentary loss of awareness followed by an abrupt return to full consciousness. Commonly presents in children ages 4-10. A classic 3 Hz spike wave is seen on EEG. Best treatment?

Ethosuximide (inhibits T-type Ca2+ channels in thalamic neurons) Valproate can also be used if first line therapy is not tolerated or ineffective

Refers to adverse reactions that occur due to known pharmacologic actions of the drug but at a lower dose than expected (due to altered metabolism or increased end organ susceptibility).

Exaggerated drug sensitivity Example: tinnitus after a single dose of ASA which occurs more commonly with chronic ASA use

Anterior cruciate ligament can be damaged by sudden decelerations and pivots on an extended knee during sports activity. PE will show?

Excessive anterior motion of the tibia with respect to the femur (positive Lachman, anterior drawer)

Hyperthyroidism causes a hyperadrenergic state characterized by HTN, palpitations/tachycardia, sweating, heat intolerance, tremor, & hyperreflexia. Beta blockers can relieve these symptoms but cannot relieve what symptom of Graves disease?

Exophthalmos (due to an immune mediated increase in orbital soft tissue mass and does not improve with beta blockers, but do typically respond to glucocorticoids as immune mediated)

6 month old with poor feeding & irritability. PE revels diffuse skin erythema & epidermic easily comes off with gentle pressure (nikolsky's sign). Most likely cause?

Exotoxin-mediated skin damage Staphylococcal scalded skin syndrome occurs in infants & children due to production of EXOtoxin exfoliation (acts as protease and cleaves desmoglein in desmosomes). Causes widespread epidermal sloughing especially with gentle pressure.

Most effective treatment for specific phobia?

Exposure based BEHAVIORAL therapy in which patients are systematically confronted with their feared objects or situations Note: psychodynamic psychotherapy focuses on developing insight into unresolved conflicts that began in childhood (this is NOT used for specific phobias)

Retrospective cohort study

Exposure status is determined retrospectively and then patients are tracked from that point of time, typically using medical records

3 muscles used when sitting up from supine position?

External abdominal obliques, rectus abdominis, hip flexors (mainly iliopsoas)

71 year old man with several episodes of bright red blood per rectum. Abdominal angiogram shows active bleeding from sigmoid colon. Catheter embolization is planned via femoral artery. Arterial catheter is likely to proceed in what order?

External iliac, common iliac, abdominal aorta, inferior mesenteric The handout encompasses the distal 1/3 of the transverse colon, descending colon, sigmoid colon, and rectum. These structures receive their main arterial blood supply from the inferior mesenteric artery

20 year old woman with lethargy, abdominal pain, and nausea. Has had polyuria and excessive thirst for one day. Complains of dysuria and chills as well. PE shows tachycardia and dry MM. Labs reveal decreased Na+, hyperglycemia, low bicarbonate, & high anion gap. What would you expect her intracellular and extracellular K+ levels to be?

Extracellular- increased Intracellular- decreased DKA is often precipitated by infections, including UTIs. Most patients with DKA have a normal/increased serum K+ level despite a total potassium body deficit. Loss of intracellular free water caused by increased plasma osmolality leads to extracellular movement of K+ secondary to increasing intracellular K+ concentration. Lack of insulin causes extracellular shifting as well as insulin normal promotes intracellular uptake of K+. Replacement of potassium is a crucial step of management of patients with DKA, despite a normal or elevated serum K+!

Reduces intestinal absorption of cholesterol & as a result the amount of dietary cholesterol reaching the liver decreases. To compensate, liver increases LDL receptor expression which draws cholesterol out of the circulation

Ezetimibe

Filtration fraction is the fraction of plasma flowing through the glomeruli that is filtered across the glomerular capillaries into bowman space. Equation?

FF=GFR/RPF RPF = RBF x (1-hematocrit)

Effect of SF-1/NR5A1 mutation (resulting in selective impairment in sertoli cell function) on testosterone, LH, FSH, inhibin levels?

FSH- increased Inhibin-decreased Testosterone- normal LH- normal Sertoli cells produce inhibin in response to FSH from anterior pituitary which feeds back to suppress FSH production in pituitary. Sertoli cells facilitate spermatogenesis within seminiferous tubules so fertility would be impaired. Leydig cells unaffected so LH & testosterone normal.

Disease characterized by alpha galactosidase A deficiency that causes accumulation of sphingolipid globotriaosylceramide. Earliest manifestations are episodic neuropathic pain, hypohydrosis, and angiokeratomas. Glomerular (proteinuria, RF), cardiac (LVH), and cerebrovascular (TIA, stroke) complications develop in adulthood

Fabry disease

Malignant tumors of the parotid gland can compress what nerve leading to what condition?

Facial nerve and its branches leading to ipsilateral facial droop Facial nerve exits the sylomastoid foramen and courses through the substance of the parotid gland where is decides into five terminal branches that innervate the muscles of facial expression

Transport of glucose into the cells of most tissues occurs by means of?

Facilitated diffusion (doesn't require energy & uses a carrier protein). Glucose moves from areas of high concentration to areas of low concentration with the help of transmembrane glucose transport proteins (GLUT). These carrier proteins are stereoselective and have preference for D-glucose

22 year old sexually active woman with fever, dermatitis, and arthralgias. Gram negative bacteria isolated from patient's blood produces an enzyme that splits the IgA molecule at the hinge region. Most important role of this bacterial enzyme in the course of the infection?

Facilitates mucosal adherence of bacteria IgA protease is produced by N. meningitidis, N. gonorrhoeae, S. pneumoniae, and H. influenzae. It cleaves secretory IgA at its hinge region, rendering it ineffective. Secretory IgA exists on mucosal surfaces and in secretions & acts to bind and inhibit the action of pills as well as other cell surface antigens that normally mediate mucosal adherence and subsequent penetration.

Autosomal recessive condition caused by mutations in genes responsible for the repair of inter strand DNA crosslinks. Most common inherited cause of aplastic anemia and presents with short stature, absent thumbs, and increased malignancy risk

Faconi anemia

Disorder characterized by deliberately feigning symptoms due to desire to assume sick role

Factitious disorder

12 year old boy experiences excessive bleeding following a tooth extraction. PMH includes episodes of painful joint swelling from minor trauma. Normal bleeding time and aPTT, but prolonged PT. Most likely has a deficiency in what factor?

Factor VII Normal bleeding time indicates adequate platelet hemostatic function. Normal aPTT indicates intact intrinsic coagulation system. Prolonged PT in the setting of a normal aPTT indicates a defect in the extrinsic coagulation system at a step that isn't shared with the intrinsic system Note: coagulopathies present with deep tissue bleeding into joints, muscles, and subcutaneous tissue and platelet defects present with mucocutaneous bleeding (epistaxis, petechiae)

Failure of PT to correct with Vitamin K supplementation indicates what?

Factor VII deficiency (has the shortest half life of all coagulation factors and is often due to underlying liver disease)

6 hour old infant with excessive secretions and choking/cyanosis whenever attempting to breastfeed. Cardiac, respiratory, and abdominal examination are otherwise normal. Most likely cause of patient's condition?

Failure of primitive foregut to separate from the airway Tracheoesophageal fistula with esophageal atresia results from failure of the primitive foregut to appropriately divide into separate trachea and esophageal structures. Diagnosis can be confirmed by x-ray after inability to pass nasogastric tube into the stomach. X-ray also shows a stomach bubble which results from air flow from the trachea through the fistula to the distal esophagus. Presents with drooling, choking, cough, and cyanosis with feeds due to reflux of breast milk and aspiration into the trachea Prenatal US may reveal polyhydramnois due to inability of fetus to swallow amniotic fluid.

An echocardiogram showing an aorta lying anterior to the pulmonary artery is diagnostic of transposition of the great arteries. This life threatening cyanotic condition results from?

Failure of the fetal aorticopulmonary septum to spiral normally during septation of the trucus arteriosus

Attaches the liver to the anterior abdominal body wall. Derivative of the embryonic ventral mesentery and contains the round ligament, the remnant of the fetal umbilical vein

Falciform ligament

Type II error?

Falsely conclude that there is no difference when a difference really does exist (a larger sample size = increased power = ability to detect a difference when one truly exists) TYPE 1 error: falsely conclude that there is a difference (no difference really exists)

Incapacitated patient without advanced directive designating a proxy who will likely need intubation. Course of action?

Family member can act as surrogate decision maker. If no family then person who cares about and knows patient's wishes can act as substitute. In emergency though, physician can treat incapacitated patient without consent.

Benign tumors of the esophagus (most commonly leiomyomas) are rare and slow growing. On light microscopy, leiomyomas consist of?

Fasicles of spindle cells with variable amounts of fibrosis

Type 1 diabetes mellitus typically presents subacutely with polyuria, polydipsia, and polyphasic accompanied by fatigue and weight loss. The diagnosis can be confirmed with?

Fasting blood glucose (greater than or equal to 126) or hemoglobin A1c (greater than or equal to 6.5%) measurement Note: oral glucose tolerance test is the preferred screening method for gestational diabetes and cystic fibrosis related diabetes, and it can also be used when the results from other tests are unequivocal. but its not preferred for general use because its more expensive and less convienient

What should be strongly suspected in a patient with severe long bone and/or pelvic fractures who develops acute onset neurologic abnormalities, hypoxemia, and a petechial rash?

Fat embolism syndrome (occlusion of the pulmonary microvessels by fat globules is an early histologic finding of this syndrome)

Preceded by localized trauma to breast and typically presents as an irregular breast mass without nipple discharge. Histology shows liquefactive necrosis of adipocytes with hemorrhage

Fat necrosis

Binding site for C1 complement component on immunoglobulin molecule?

Fc region of heavy immunoglobulin chain in region near hinge point

Newborn with abnormal sexual differentiation. Karyotype shows 46 XY genotype. Biopsy of gonadal tissue shows lack of sertoli cells but normally functioning leydig cells. What phenotype most likely present?

Female and male internal reproductive organs & male external genitalia SRY gene on Y chromosome codes for testes determining factor which differentiates the primitive gonads into testes. Sertoli cells product anti-mullerian hormone which causes regression of the mullerin (paramesonephric) ducts and suppresses female internal reproductive organ development. Leydig cells produce testosterone differentiating wolffian ducts into internal male reproductive organs. DHT is required for differentiation of external male genitalia.

65 year old female with nausea, vomiting, and abdominal pain that began 6 hours ago. There is a tender bulge below the inguinal ligament, just lateral to the pubic tubercle & the overlying skin is erythematous. What structure is likely immediately lateral to the bulge?

Femoral vein Femoral hernias are inferior to the inguinal ligament, lateral to the pubic tubercle & lacunar ligament, and medial to the femoral artery & vein. They can present with groin discomfort or manifest with a bulge on the upper thigh. More common in women and tend to occur on right side. Incarceration and strangulation are common complications of femoral hernias.

Amniocentesis is performed in pregnant patients and the phospholipid content of the amniotic fluid is determined in order to check for?

Fetal lung maturity Phospholipids, including dipalmitoyl phosphatidylcholine, are a major component of pulmonary surfactant. The amniotic fluid lecithin (phosphatidylcholine) to sphingomyelin ratio (L/S ratio) is measured in order to assess fetal lung maturity. The fetal lungs are considered mature when they are producing enough surfactant to yield an L/S ratio greater than 2

Maternal hyperglycemia causes increased transplacental glucose delivery to the infant, fetal hyperglycemia and ultimately beta cell hyperplasia. The resulting hyperinsulinemia caused by insulin hyperplasia is associated with what symptoms in the newborn after delivery?

Fetal macrosomia and hypoglycemia Affected infants require frequent feedings to parental glucose to prevent hypoglycemia and subsequent neurologic injury. Supportive care is all that is required in most cases as the beta cell hyperplasia will resolve over time

43 year old male recently returned from South America is evaluated for medical illness. NO PMH. Liver biopsy is performed and LM demonstrates spotty hepatocyte necrosis and inflammatory cell infiltration. Most likely initial clinical presentation of this patient?

Fever, anorexia, dark colored urine Acute hepatitis A is a self limited infection that typically presents with prodromal symptoms (fever, malaise, anorexia, nausea/vomiting RUQ pain) followed by signs of cholestasis (jaundice, pruritic, dark colored urine, clay colored stool). Acute viral hepatitis is characterized histologically with the presences of 'spotty necrosis' with ballooning degeneration (hepatocyte swelling with wipsy/clear cytoplasm), Councilman bodies (eosinophilic apoptotic hepatocyte) and mononuclear cell infiltrates.

Lung abscess clinical presentation?

Fever, cough with copious production of greenish foul smelling sputum, malaise, weight loss, clubbing, leukocytosis lasting a few weeks. Cavitary lesion with air fluid levels on CXR.

Patient with preexisting gallbladder disease is found to have elevated LDL and triglyceride levels. What drug class should be avoided in treatment?

Fibrates (gemfibrizol) Fibrates are effective in patient's with hypertriglyceridemia due to their ability to up regulate lipoprotein lipase resulting in increased oxidation of fatty acids. But, they reduce cholesterol solubility and promote gallstone formation by reducing bile acid synthesis. They inhibit the rate limiting enzyme inbuilt acid synthesis, cholesterol 7-alpha-hydroxylase. Note: patients on vibrates should be monitored for myopathy, a risk significantly increased when vibrates are combined with statins

Best medication for lowering triglyceride levels?

Fibrates (lower triglyceride levels by activating peroxisome proliferator activated receptor alpha, which leads to decreased hepatic VLDL production and increases lipoprotein lipase activity) Note: fish oil supplements containing high concentrations of omega 3 fatty acids lower triglycerides by decreasing production of VLDL and apolipoprotein B

Viridans streptococci are normal inhabitants of the oral cavity and are a cause of transient bacteremia after dental procedures in healthy and disease individuals. These gram positive organisms produce extracellular polysaccharides (dextrans) using sucrose as a substrate. What is the adherence site for these bacteria?

Fibrin-platelet aggregates In patients with preexisting valvular lesions, viridians streptococci can adhere to fibrin platelet aggregates and establish infection that leads to endocarditis

34 year old woman with mass in right breast. No PMH. 2cm nodule noted on exam and patient is referred for biopsy. Histologic evaluation of the specimen reveals a cellular/myxoid storm that encircles and sometimes compresses epithelium-lined glandular and cystic spaces. Most likely diagnosis?

Fibroadenoma Fibroadenoma is most common benign tumor of the breast arising in young women 15-35 years old.Characterized by nodules that are well demarcated, painless, mobile, spherical and can be 1 to 10 cm in size. Can be multiple or bilateral. Often increase in size during pregnancy, lactation, or with estrogen therapy, and regress after menopause.

Small, firm, and mobile breast masses that occur due to proliferation of breast stroma and ducts with stromal proliferation compressing the ducts to slits

Fibroadenomas

Diffuse small cysts with or without metaplasia & frequently causes cyclic breast pain without nipple discharge

Fibrocystic changes of breast

Occurs most commonly in women age 20-55 and presents with diffuse musculoskeletal pain, fatigue, & neuropsychiatric disturbances. Characterized by abnormal central processing of painful stimuli. Although initially painful, aerobic exercise helps to improve pain and functioning in these paitents

Fibromyalgia

Adhesion of cells to the extracellular matrix involves integrin mediated binding to what substances?

Fibronectin, collagen, and laminin Note: differential expression of integrin subtypes affects adhesion properties of individual cells, and has been found t correlate with malignant behavior in a number of tumors

54 year old man dies from profuse upper GI hemorrhage. Patient's condition likely resulted from what process in the liver?

Fibrosis & nodular parenchymal regeneration Cirrhosis is characterized by diffuse hepatic fibrosis with replacement of the normal lobular architecture by fibrous lined parenchymal nodules. Portal HTN from increased resistance to hepatic blood flow typically develops in advanced cirrhosis leading to development of gastroesophageal varices which can rupture & cause death. Chronic viral hepatitis (B and C), alcohol, hemochromatosis, and nonalcoholic fatty liver disease are the most common causes of cirrhosis in the USA.

25 year old woman with CREST syndrome complaining of severe heart burn that is resistant to over the counter antacids. Most likely cause of patients heartburn?

Fibrous replacement of the muscularis in the lower esophagus Systemic sclerosis may results in esophageal dysmotility and incompetence of the lower esophageal sphincter due to atrophy and fibrous replacement of the esophageal muscularis. This can cause gastroesophageal reflux with an increased risk of Barrett's esophagus and stricture formation

Patient diagnosed with C. difficile colitis and placed on macrocyclic antibiotic that inhibits the sigma subunit of RNA polymerase. What is this antibiotic?

Fidaxomicin Fidaxomicin is a macrocyclic antibiotic (related to macrocodes) that inhibits the sigma subunit of RNA polymerase, leading to protein synthesis impairment and cell death (bactericidal against C. difficile). It is administered orally and has minimal systemic absorption, resulting in high fecal concentrations. Has a narrow spectrum of activity with a lesser effect on normal colonic flora than Vancomycin (the other antibiotic used to treat C. Difficile). Vancomycin inhibits cell wall synthesis and is bacterioSTATIC against C. Difficile.

Examination of stillborn shows several dysmorphic features including closely set eyes and a midline mass consistent with proboscis. Fetal autopsy reveals fused cerebral hemispheres with an absent forebrain fissure and and a single intracranial ventricle. Most likely mechanism for these findings?

Field defect Developmental field defect describes multiple malformations that occur secondary to an embryonic disturbance in an adjoining group of cells. Holoproencephaly is a developmental field defect characterized by a spectrum of fetal anomalies due to incomplete division of the forebrain (prosencephalon) into 2 hemispheres like caused by a combination of genetic (trisonomy 13, SHH gene mutations) & environmental (maternal alcohol use) factors.

Drug that is a 5 alpha reductase inhibitor that suppresses peripheral conversion of testosterone to DHT. Used for treatment of BPH & androgenetic alopecia

Finasteride

28 year old male with hair loss around the temples that began 3 years ago. Several family members became almost completely bald by late 30s. PE shows moderate thinning of hair at the anterior scalp, temporal region, and vertex. What drug can be used to treat this patient's condition?

Finasteride (5 alpha reductase inhibitor) Androgenetic alopecia shows polygenic inheritance of hair loss at the anterior scalp and vertex, with DHT being the primary pathogenic factor. 5 alpha reductase inhibitors decreased the conversion of T to DHT and are effective for treating the condition Note: intralesional glucocorticoids are used to treat alopecia ACRETA which is an autoimmune disorder that causes rapid onset patchy or diffuse hair loss

14 year old boy presents with mother for 3 month check up for diabetes. Patient admits that he does not take his lunchtime dose of insulin. What would most likely increase this patient's adherence with insulin administration?

Finding peers who take insulin consistently at school Adolescents have low treatment adherence rates related to issues with autonomy, rebellion against authority figures, and lack of understanding of potential risks. Peer behavior has a strong influence on adherence at this age due to adolescents' desire to fit in with their social groups.

Most appropriate initial action when when abuse of prescription opioids is suspected?

First engage patients in a nonjudgmental, collaborative discussion to UNDERSTAND the reasons for misuse 'I can understand that the pain medication is not helping; let's discuss how you are using it'

Patient prescribed diazepam to decrease muscle spasticity of the involved extremities. What other drug class should he be advised to avoid?

First generation H1-histamine receptor antagonists (diphenhydramine, chlorpheniramine, promethazine, hydroxyzine), alcohol, barbiturates, neuropleptics as they can cause significant sedition when used with other medications that can cause CNS depression such as benzodiazepines

Type of kinetics in which a constant fraction of a drug is eliminated per unit time based on plasma concentration

First order kinetics

Patient with abdominal discomfort, bloating, flatulance, diarrhea undergoes histopathlogic evaluation of hyperemic mucosa seen during GI endoscopy which reveals hyperplasia of branched, tubular submucosal glands containing alkaline secretions. What area is the site of biopsy?

First part of the duodenum Gastric acid is neutralized by bicarbonate from the submucosal glands of the duodenum (Brunner glands) & from pancreatic duct secretion. Chronic overproduction of gastric acid can lead to hyperplasia of the submucosal glands of the duodenum

Inflammatory bowel disease, especially ulcerative pancolitis, is associated with a significant risk for colorectal carcinoma. Compared with sporadic colorectal cancer, colitis associated colorectal cancer is more likely to occur at a younger age, is typically more aggressive with a higher histopathologic grade, have a higher number of mutinous or signet ring cells, are associated with p53 mutations occurring early in the course & APC mutations occurring much later, frequently multifocal and evolves from what typee of lesions?

Flat nonpolypoid lesions (vs. polypoid in sporadic) Monitor regularly via colonoscopy with random biopsies!

Median nerve courses between the humeral and ulnar heads of the pronator teres muscle and then runs between what two muscles before crossing the wrist within the carpal tunnel?

Flexor digitorum superficialis & flexor digitorum profundus Proximal median nerve lesions may result in sensory loss over the first 3 digits and impairment of thumb flexion/opposition, flexion of 2nd/3rd digits and wrist abduction/flexion

Patient started on pioglitazone therapy to reduce insulin resistance. Most likely to experience what adverse effects?

Fluid retention, weight gain, edema (can exacerbate underlying CHF) Note: Thiazolidinediones increase insulin sensitivity rather than insulin secretion and are therefore associated with a very low risk of hypoglycemia

7 year old boy with mental developmental delay but advanced social and verbal skills. Has 'elfin' face and appears quite pleasant. Hypercalcemia & cardiovascular problems. Test that would allow you to directly visualize the chromosomal area of the genetic disorder you are suspecting is?

Fluorescent in situ hybridization Patient has Williams syndrome which is a congenital disease involving microdeletion of long arm on chromosome 7. Karyotype and FISH both directly visualize chromosomes but karyotype identifies gross chromosome anomalies (duplication or deletion NOT microdeletion) while FISH uses a fluorescent DNA probe to bind specific gene sites (probe against microdeleted region)

Phentyoin, anti epileptic drug used to treat both focal and generalized seizures, can result in deficiency of what substance?

Folate, resulting in megaloblastic anemia Note: phenytoin can also cause peripheral neuropathy with chronic therapy

Gastric bypass surgery can cause small intestinal bacterial overgrowth due to excessive bacterial proliferation in the blind ended gastroduodenal segment. Small intestinal bacterial overgrowth is associated with nausea, boating, abdominal discomfort and results in deficiency of most vitamins (B12, A, D, E) and iron but increased production of what 2 substances?

Folic acid & Vitamin K

Vitamin that is essential cofactor in nucleic acid synthesis

Folic acid (B9)

Patient recently began taking Methotrexate for psosiasis, but has been accidentally taking the medication daily instead of 3 times a week. Has aphthous ulcers and pancytopenia. Next best step in management?

Folinic acid Folinic acid (leucovorin) can reverse the toxicity of methotrexate in noncancerous cells in the GI mucosa and bone marrow if administered at the appropriate time. Leucovorin serves as a reduced form of folic acid that doesn't require the action of dihydrofolate reductase

42 year old previously healthy woman with fever and sore throat. PE shows tonsillar exudate and contender cervical lymph node that measures 3.5cm in diameter. Oral antibiotic therapy started and one week later patients reports symptoms have resolved. Lymph node has decreased slightly in size. Throughout several follow up visits throughout the following year, patient remains asymptomatic & size of lymph node fluctuates but doesn't disappear completely. Biopsy is performed. Most likely to be seen on biopsy?

Follicular lymphoma Follicular lymphoma is the most common indolent non-Hodgkin lymphoma in adults. It is of B-cell origin and presents with painless waxing and waning lymphadenopathy. Cytogenetic change t(14, 18) is characteristic and results in over expression of bel-2 oncogene

Disinfectants that function by alkylating and cross linking DNA and proteins. They are most commonly used for sterilization of hospital instruments that cannot withstand autoclave temperatures

Formaldehyde and glutaraldehyde

Patient with recurrent CMV viremia despite treatment with ganciclovir. Genotype analysis is performed and demonstrates ganciclovir-resistant CMV. Patient is started on an alternative IV antiviral agent and subsequently develops hypocalcemia and hypomagnesemia. What agent is most likely cause of these side effects?

Foscarnet (analog of pyrophosphate that can chelate calcium and promote nephrotoxic renal magnesium wasting leading to hypocalcemia and hypomagnesemia which can then lead to seizures)

42 year old with a history of HIV with abdominal pain. CT reveals colitis and colonoscopy reveals areas of erosions and ulcerations. Pathology demonstrates cells with viral intranuclear and intracytoplasmic inclusions. Antiviral therapy is initiated to treat the colitis with an IV agent that does not require intracellular activation and is known to bind with viral encoded enzymes such as DNA polymerase, RNA polymerase, and reverse transcriptase. Most likely agent patient received?

Foscarnet (used to treated ganciclovir resistant CMV infections)

Occurs with the deletion/addition of a number of bases not divisible by 3 in the coding region of a gene. Alter the reading frame of the genetic code, dramatically changing the protein structure & often resulting in formation of a premature stop codon

Frameshift mutation

3 year old boy being evaluated for respiratory infections. Mutation in an exon of a gene that codes for a transmembrane chloride channel is identified. Abnormal mRNA is isolated and cDNA is synthesized. Amplified cDNA samples from the patient and his healthy sibling are analyzed using gel electrophoresis and compared to DNA fragments of known size to determine base pair length. Patient's DNA size is 101 base pairs and his siblings is 129 base pairs. Most likely mutation responsible for patient's condition?

Frameshift mutation Deletion or addition of a number of bases not divisible by 3 in the coding region of a gene causes a frameshift mutation, which alter the reading frame of the genetic code resulting in the formation of nonfunctional proteins

Substance that provides the fastest reversal of warfarin's effects?

Fresh frozen plasma Both vitamin K and FFP are used for reversing warfarin induced anticoagulation. FFP rapidly reverses warfarin's effects whereas vitamin K requires time for clotting factor re-synthesis

24 year old female with bloody emesis. Was diagnosed with factitious disorder recently. Patient admits to having ingested rat poisoning several days ago. Immediate treatment of this patient should include?

Fresh frozen plasma Most available rodenticides contain brodifacoum ('superwarfarin'), a long acting 4-hydroxycoumarin derivative. The anticoagulant effect is generally seen about 48 hours following ingestion, which represents the time required for deflation of the coagulation factors. A patient who has ingested a quantity of rodenticide sufficient to cause coagulopathy and abnormal bleeding (similar to warfarin toxicity) requires immediate treatment with FFP in addition to Vitamin K. Note: cryoprecipitate contains factor VIII, factor XIII, vWF, and fibrinogen Note: protamine sulfate is used to treat heparin overdose

Drugs used for rapid reperfusion in patient with acute MI?

Fribrinolytic drugs (tPA, reteplase, tenecteplase, streptokinase)

Sorbitol is normally metabolized into what substance before being excreted from most cells?

Fructose This process is slow. In long standing hyperglycemia, sorbitol accumulates in tissues with lower sorbitol dehydrogenase activity (retina, lens, kidney, peripheral nerves) and increases osmotic/oxidative stress

Fructose and sucrose restriction is the treatment for the deficiency of what enzyme that results in vomiting & hypoglycemia in infants after fruit or juice is introduced?

Fructose 1-phosphate aldolase (aldolase B) deficiency

Substance that is rapidly metabolized because it bypasses phosphofructokinase, the major rate limiting enzyme of glycolysis

Fructose-1-P (dietary fructose phosphorylated by fructokinase to form fructose-1-phosphate)

FADH2 is produced by FAD during the conversion of succinate to what substance in the TCA cycle by the enzyme succinate dehydrogenase?

Fumarate

Patient with severe dyspnea, orthopnea, and fatigue who suffered an MI 6 months ago and has not been compliant with medication sine then. Auscultation reveals crackles at the lung bases, S3 gallop, and II/IV holosystolic murmur over the APEX. After initial treatment with diuretics & vasodilators the patients condition improves and there are no appreciable gallops or murmurs. Murmur hears at time of presentation was most likely due to?

Functional mitral regurgitation Acute hemodynamic changes can produce functional heart murmurs, in the absence of a fixed valve lesion. Dilation of the LV in response to increased preload can result in functional regurgitation which can be eliminated with preload reduction and reduced by afterload reduction

Point on spirometry tracing at which pulmonary vascular resistance is the lowest?

Functional residual capacity (middle of a spirometry graph) Increased lung volumes increased PVR due to the longitudinal stretching of the alveolar capillaries by the expanding alveoli. Decreased lung volumes also increase PVR due to decreased radial traction from adjacent tissues on the large extra-alveolar vessels

72 year old with difficulty hearing. She was admitted one week ago for dyspnea, orthopnea, and bilateral leg swelling which has been slowly improving with treatment. Hearing was normal prior to hospitalization. PMH of HTN, HF, and CKD. Exam shows bilateral moderate sensorineural hearing loss. Medication most likely to have contributed to patient's hearing impairment?

Furosemide Ototoxicity secondary to loop diuretics usually occurs with higher dosages, preexisting chronic renal disease, rapid IV administration, or when used in combo with other ototoxic agents (ahminoglycosides, salicylate, cisplatin). hearing impairment is usually reversible but can be permanent.

Bleomycin and Doxorubicin are specific to what cell cycle phase?

G2

Neurotransmitter formed from glutamate decarboxylation, a reaction catalyzed by glutamate decarboxylase

GABA

Loss of what neurotransmitter is the most characteristic biochemical feature of Huntington disease?

GABA (loss of GABA-containing neurons)

Janus tyrosine kinase (JAK) is a cytoplasmic protein activated by ligand binding to non-G protein coupled transmembrane receptors that lack intrinsic tyrosine kinase activity. Ligands for these receptors are?

GH, erythropoietin, cytokines (interferon) Note: JAKs activate STATs which enter the nucleus to promote gene transcription Note: ligands such as insulin and EDGF act through transmembrane receptors that HAVE intrinsic tyrosine kinase domains

Common complication of Rotavirus?

GI obstruction due to small bowel intussusception

Glucose transport protein that is an insulin sensitive transporter expressed in skeletal muscle cells and adipocytes. In the absence of insulin, it is requested in the cytoplasm but as insulin concentrations rise, the receptors translocate to the plasma membrane facilitating glucose transport into the cell

GLUT-4 GLUT 1 ,2,3,5 are always presents on the plasma membrane regardless of insulin levels and constitutively transport glucose (insulin dependent)

Under normal conditions, KRAS is only active when bound to what substance?

GTP Regulation of the Ras-MAPK signal transduction pathway requires a balance between active (GTP bound) and inactive (GDP bound) Ras proteins. RAS gene mutations, which result in constitutively activated Das proteins, are implicated in the development of malignant tumors (colorectal and pancreatic)

Splice site mutations may result in inappropriate removal of exons and retention of introns, leading to the formation of dysfunctional proteins. Splicing is performed by splicodsomes which remove introns containing what sequences at the 5' and 3' splice sites?

GU at the 5' splice site & AG at the 3' splice site

Lenticular accumulation of galacitol in the lenses of patients with galactosemia can cause osmotic damage and development of cataracts. Cataracts may be the only manifestation of what enzyme deficiency?

Galactokinase

Galactosemia with cataract formation only (absence of renal and liver dysfunction) is due to deficiency in what enzyme?

Galactokinase (phosphorylates galactose). Galactose-1-phosphate does not build up so no renal or liver dysfunction. Galacitol does build up in lens resulting in cataracts.

Classic galactosemia results from deficiency in?

Galactose-1-phosphate uridyl transferase

Galactosemia presents in neonates with jaundice, vomiting, poor feeding, lethargy, hypoglycemia, galactose-1-phosphate accumulation. Galactose & lactose are excluded from the diet in patients with this disease. What is absent enzyme?

Galactose-1-phosphate uridyltransferase

68 year old woman comes to office for preventative visit. No complaints, feels well. Firm mass palpated on RUQ. CT abdomen reveals calcified gallbladder. Most at risk for?

Gallbladder adenocarcinoma. Porcelain gallbladder is a potential complication of chronic cholecystitis and is often found in association with multiple gallstones. It is due to dystrophic intramural deposition of calcium salts in the setting of chronic inflammation. Associated with increased risk of adenocarcinoma of gallbladder

64 year old man with 2 days of dysuria, urinary frequency, and urgency. Recently underwent cystoscopy for evaluation of hematuria. Temperature is 100.8. Suprapubic tenderness on exam. No CVA tenderness. Midstream urine culture grows gram positive cocci in chains. The organism responsible for this patient's condition is most likely to demonstrate?

Gamma hemolysis This patient who recently underwent a cystoscopy has a UTI caused by Enterococcus, aerobic gram positive cocci in pairs and chains that when grown on blood agar reveals no hemolysis (gamma hemolysis). Other characteristics include pyrrolidonyl arylamidase (PYR) positivity and ability to grow in bile and 6.5% NaCl. Unable to convert nitrates to nitrites (urinalysis nitrite will be negative). Very important cause of UTIs are they are highly resistant to antibiotics. Note: ability to decompose hydrogen peroxide is suggestive of the presence of catalase (staph) & possession of cytochrome oxidase indicates oxidase positive.

Osteocytes remain connected to each other by?

Gap junctions Osteocytes have long intracanalicular processes that extend through the ossified bone matrix. Cytoplasmic processes send signals to and exchange nutrients and waste products with the osteocytes within neighboring lamellae via gap junctions. Osteocytes can sense mechanical stresses & send signals to modulate the activity of surface osteoblasts, helping to regulate bony remodeling

Upper GI endoscopy reveals small ulcer with clean base in duodenal bulb. Biopsy of what area will most likely demonstrate the infectious agent responsible for this patient's current condition?

Gastric antrum H. pylori is a common cause of peptic ulcers. Duodenal ulcers are associated with heady colonization in the gastric antrum, whereas colonization in the gastric corpus (body) is associated with gastric ulcers. Colonization of H. pylori in the gastric antrum is associated with decreased somatostatin formation and increased gastrin secretion which stimulate the parietal cells to produce excess acid. Results in increased acid load emptying into the proximal duodenum leading to duodenal ulcer formation (presents with upper abdominal pain that improves with eating). Gastric ulcers are due to direct mucosal damage (have normal or reduced acid levels) & worsen with eating

Most common ectopic tissue found in Meckel's diverticulum?

Gastric epithelium leading to ulceration and subsequent bleeding

43 year old with several month history of fatigue, rash, flushing, and abdominal cramps. Rash is worse with rubbing or scratching, and has diffuse itching after hot showers. Frequently feels dizzy and lightheaded after prolonged standing and had an episode of syncope while working in hot sun. Skin biopsy shows large clusters of mast cells that are positive for KIT (CD 117). Most likely additional finding?

Gastric hypersecretion Systemic mastocytosis is characterized by the abnormal proliferation of mast cells and increased histamine release from degranulation of mast cells. Histamine causes hypersecretion of gastric acid by parietal cells in the stomach as well as a variety of other symptoms (hypotension, flushing, pruritus). Excess acid inactivates pancreatic and intestinal enzymes causing diarrhea and other intestinal symptoms. Mast cell proliferation is associated with mutations in KIT receptor tyrosine kinase. Clonal mast cell proliferation occurs in bone marrow, skin, and other organs.

64 year old woman with sudden onset right arm weakness and difficulty speaking which completely resolved in 20 minutes. History of HTN and hypercholesterolemia for which she takes amlodipine and rosuvastatin. Carotid doppler reveals mild left common carotid artery stenosis. If patient is started on an additional medication indicated for her condition what adverse effect is most likely to occur?

Gastrointestinal bleeding Patient had TIA. In addition to BP control and statin therapy, low dose aspirin is used to prevent ischemic stroke in patients with TIA. Gastrointestinal mucosal injury and bleeding are most common side effects of ASA which are due primarily to COX-1 inhibition that results in impaired prostaglandin dependent GI mucosal defense and decreased platelet aggregation

3 year old boy with intellectual disability and speech delay. Does not imitate his parents activities, have a social smile, or show interest in other children. Verbal comprehension is poor. Lab testing shows 226 CGG trinucleotide repeats in a gene located on the X chromosome. Most likely cause of this patient's clinical condition?

Gene methylation Fragile X syndrome is caused by an increased number of CGG trinucleotide repeats on the fragile X mental retardation 1 (FMR1) gene on the long arm of the X chromosome leading to hypermethylation and inactivation of FMR1. Most common cause of inherited intellectual disability. Physical findings include macroorchidism, long and narrow face, prominent forehead & chin. Neuropsychiatric disorders include developmental delay, ADHD, autism spectrum disorder.

Involves excessive, chronic worry over multiple issues lasting more than 6 months. Treated with SSRI/SNRI and cognitive behavioral therapy

Generalized anxiety disorder

Term that refers to a point mutation that results in replacement of a purine nucleotide for another purine or pyrimidine for pyrimidine

Genetic transition (transversion is purine to pyrimidine or vice versa)

1 year old boy with repeated infections over past 4 months. Also has persistent diarrhea and recent stool antigen detection was positive for Giardia. Immunoglobulin panel demonstrates low serum levels of all immunoglobulin types. Has an uncle who passed away from pneumonia as a teen. Intradermal injection of Candida antigens results in large indurated nodule within 48 hours (intact T lymphocyte function). Lymph nodes lack what structure?

Germinal centers X-linked agammaglobulinemia results from mutation in bruton tyrosine kinase gene that causes failure of bone marrow pre-B cells to develop into mature B lymphocytes. Germinal centers and primary lymphoid follies don't form due to absence of B cells. T cell numbers and function remain intact

The primary immune response to a new antigen initially results in plasma cells that only produce IgM. Isotype switching later occurs in what are that requires interaction of the CD40 receptor on b cells with the CD40 ligand (CD 154) expressed by activated T cells?

Germinal centers of lymph nodes Note: IgG is the main serum immunoglobulin of the secondary response Note: recombination of the V, D, J regions of heavy chains and V and J regions of light chains occurs via DNA rearrangement during B cell maturation within the bone marrow. Affinity maturation then occurs later on in the germinal centers through the process of somatic hypermutation

4 day old premature infant in NICU with decreased level of consciousness & hypotonia. PE reveals lethargic infant with weak, high pitched cry, prominent scalp veins, & tense fontanels. Cranial US shows blood in lateral ventricles. Most likely source of bleeding?

Germinal matrix Neonatal intraventriuclar hemorrhage usually occurs in the fragile germinal matrix and increases in frequency with decreasing age & brith weight. Its a common complication of prematurity that can lead to long term neurodevelopment impairment.

Syndrome characterized by agraphia (inability to write), acalculia (inability to carry out mathematical calculations), finger agnosia (inability to identify individual fingers on the hand), and left right disorientation

Gerstmann syndrome (results from damage to the angular gyrus of the dominant parietal lobe- supplied by MCA)

Polyhydramnois (excessive accumulation of amniotic fluid) presents with increased abdominal circumference out of proportion to gestational age. Etiology is decreased fetal swallowing or increased fetal urination. Fetal anomalies associated with impaired swallowing include?

Gestational obstruction (duodenal, esophageal, intestinal atresia) and anencephaly Note: causes of increased fetal urination include high cardiac output due to anemia or twin twin transfusion syndrome. Maternal diabetes and multiple gestations can cause mild polyhydramnios.

Stool with protozoan flagellate with ellipsoid cysts, smooth well-defined walls, and 2+ nuclei is consistent with?

Giardia lamblia

Autosomal recessive disorder that is caused by a deficient or defective glycoprotein GP IIb/IIIa on platelet surfaces and that typically presents in childhood with mucocutaneous bleeding

Glanzzmann thrombasthenia

Patient presents with inferior wall MI. Therapy is initiated which includes a medication to treat her bradycardia. BP and pulse normalize but she then complains of severe right sided eye pain. Most likely cause of her eye pain?

Glaucoma MI in inferior leads is typically due to block of the right coronary artery which supplies the SA and AV nodes, leading to bradycardia. Atropine blocks vagal influence on SA & AV nodes and increases heart rate in these patients. Common side effect of Atropine is increased intraocular pressure (can precipitate acute closed angle glaucoma in susceptible individuals) which presents with unilateral severe eye pain & visual disturbances.

62 year old man with new onset generalized tonic clonic seizure. Wife reports over the past several weeks he has been having episodic headaches, nausea, and progressive weakness. Headaches are worse at night and wake him up from sleep. Admitted for further evaluation and dies a few days later due to aspiration pneumonia and septic shock. Brain secretion obtained during postmortem exam is shown. Most likely had what condition?

Glioblastoma Most common primary cerebral neoplasm of adults that is typically located within the cerebral hemispheres and may cross the midline ('butterfly glioma'). Highly malignant and grossly contain areas of necrosis and hemorrhage

64 year old man with new onset headache is diagnosed with intracranial mass after abnormal MRI of brain. Mass is biopsied and reveals an area of necrosis & hemorrhage surrounded by columns of tumor cells (pseudopalisading necrosis). Capillaries are seen at the periphery. Diagnosis?

Glioblastoma multiforme (most common brain neoplasm in adults)

Arrangement in which an insurer pays a provider a single payment to cover all the expenses associated with an incident of care

Global payment (most commonly done for elective surgeries in which the global payment covers the surge as well as any ore and post op visits needed)

Crescent formation on LM is diagnostic of rapidly progressive glomerulonephritis. Crescents consist of?

Glomerular parietal cells, monocytes, macrophages, and abundant FIBRIN. Crescents eventually become sclerotic, disrupting glomerular function & causing irreversible renal injury

Although usually self limiting, patients with Henoch-Schonlein purpura should be observed carefully because of what possible, serious complication?

Glomerulonephritis & even end stage renal disease

Nerve responsible for sensation and taste to posterior 1/3 of tongue?

Glossopharyngeal nerve (CN IX)

Bradycardia, hypotension, hypoglycemia are seen with beta blocker toxicity. Specific antidote for beta blocker toxicity as it increases intracellular cAMP and cardiac contractiliy?

Glucagon

Normal blood glucose levels are maintained by opposing effects of insulin and glucagon. Effects of glucagon and insulin?

Glucagon stimulates hepatic glycogenolysis and gluconeogenesis. Insulin increases peripheral glucose uptake and inhibits lipolysis and ketoacid formation, as well as suppresses glucagon release

Avoiding use of what medication in patient with RA can help prevent osteoporosis/fragility fractures (occur in absence of trauma)?

Glucocorticoids

Patient with a history of hypothyroidism (puts her at increased risk for other autoimmune encocrinopathies, such as autoimmune adrenalitis) presents with intractable vomiting and abdominal pain. Has a several week history of anorexia and weight loss. Generalized hyper pigmentation is present and is most conspicuous over the face, neck, and dorsal surface of hands. Temperature is 99.7. Blood pressure is 80/40. 1L bolus is rapidly infused and high flow IV fluids are continues. Patient also needs immediate treatment with what medication?

Glucocorticoids (dexamethasone) Patient likely has chronic primary adrenal insufficiency. Patients with adrenal insufficiency are not able to increase glucocorticoid production in response to acute stress (illness, surgery). Adrenal crisis is characterized by severe hypotension, abdominal pain, vomiting, weakness, and fever. In addition to aggressive fluid resuscitation, treatment requires immediate glucocorticoid supplementation.

24 year old woman with gestational diabetes mellitus during 1st pregnancy. Mother and sister had high blood sugar during pregnancy. If her gestational hyperglycemia is genetically predisposed, she likely has decreased activity of what enzyme?

Glucokinase Glucokinase has a lower glucose affinity than other hexokinases allowing it to function as a glucose sensor in pancreatic beta cells by controlling the rate of glucose entry into the glycolytic pathway based on blood glucose levels. Mutations in glucokinase gene are a cause of maturity onset diabetes of the young characterized by mild non progressive hyperglycemia (which worsens with pregnancy induced insulin resistance). Less ATP produced & therefore less insulin released.

20 year old man in ED for weakness, malaise, dark urine. Was treated for a bacterial skin infection several days ago. PE reveals scleral icterus. Lab results show anemia with elevated reticulocytes. Abnormal erythrocytes on peripheral smear. Most likely deficient enzyme?

Glucose 6 phosphate dehydrogenase (rate limiting step of PPP) G6PD is common X-linked recessive disorder resulting in episodes of hemolytic anemia during times of increased oxidative stress (use of antimalarials/sulfonamides, infections)

56 year old man with progressive visual impairment. PMH includes poorly controlled DM. PE shows bilateral clouding of the lens. What metabolic conversion is most likely contributing to his current condition?

Glucose to sorbitol In hyperglycemic states, aldose reductase converts glucose to sorbitol at a rate faster than sorbitol can be metabolized. Sorbitol accumulates in certain cells such as lens cells, causing an influx of water and resulting in osmotic cellular injury. Depletion of NADPH by aldose reductase also increases oxidative stress, which accelerates the development of cataracts and diabetic microvascular complications (neuropathy, retinopathy)

Enzyme that catalyzes initial and rate limiting step of the pentose phosphate pathway

Glucose-6-phosphate dehydrogenase

African american male develops symptoms of anemia (malaise, pallor, jaundice, hemoglobinemia, hemoglobinuria-dark red urine) after starting TMP-SMX/Bactrim for prostatitis. Best explanation for this?

Glucose-6-phosphate dehydrogenase deficiency X-linked disorder of pentose phosphate pathway. Amount of NADPH produced in RBCs is low which impaired glutathione mediated inactivation of free radicals. Hemolytic episodes are induced by infections, medications, and other oxidants

Patient with history of HTN and depression is found obtunded in her apartment. She is hypotensive and bradycardic likely due to beta blocker overdose. Drug of choice for beta blocker overdose?

Glugacon It increases HR and contractility independent of adrenergic receptors by activating G protein coupled receptors on cardiac myocytes causing activation of adenylate cyclase and raising intracellular cAMP. Results in calcium release from intracellular stores and increased SA node firing

Major amino acid in the blood that transports excess ammonia from peripheral tissues to kidney

Glutamate

Enzyme that catalyzes the reaction of glutathione disulfide to glutathione using NADPH

Glutathione reductase

Most abundant amino acid in collagen?

Glycine (occupies every 3rd amino acid position in each alpha chain)

Release of what neurotransmitter is most likely to be directly to be impaired in C. tetani infection?

Glycine and GABA Tetanospasmin is a neuro-exotoxin released by Clostridium tetani. The toxin blocks the release of glycine and GABA from the spinal inhibitory neurons that regulate lower motor neurons. These disinhibited motor neurons cause increased activation of muscles, leading to spasms and hyperreflexia

46 year old Caucasian male being worked up for abdominal discomfort, loose stool, and recent weight loss. Intestinal biopsy performed and PAS reaction is used on the sample. The intense pink discoloration most likely indicates the presence of?

Glycoprotein The glycoprotein in the cell walls of actinomycete Tropheryma whippelii colors magneta with PAS and is diastase resistant making this stain excellent choice in evaluating for Whipple disease.

Agents useful for ovarian suppression of estrogen production in premenopausal women with ER positive breast cancer?

GnRH analogs (goserelin) which suppress LH and FSH release

What structure is responsible for sudden muscle relaxation?

Golgi tendon organ Muscle spindle system is a feedback system that monitors and maintains muscle length, while the golgi tendon system is a feedback system that monitors and maintains muscle force. Golgi tendon organs are exquisitely sensitive to increases in muscle tension but are relatively insensitive to passive stretch. When the muscle exert too much force, golgi tendon organ inhibit contraction of the muscle, causing sudden muscle relaxation & therefore prevent damage to the MSK system

Occurs with increased frequency in patients with activating mutations involving PHOSPHORIBOSYL PYROPHOSPHATE SYNTHETASE (PRPP) due to increased production and degradation of purines

Gout PRPP is the enzyme responsible for the production of activated ribose necessary for de novo synthesis of purine & pyrimidine nucleotides. Activating mutations in PRPP will cause increased production of purines and as a result more purines will undergo degradation, increasing risk of gout

Man with HCV undergoes liver transplant & is discharged on immunosuppressants. One week later he develops nausea, vomiting, abdominal pain, bloody diarrhea & maculopapular rash over neck, back, extremities that extends to palms and soles. Endoscopy reveals multiple ulcerations in intestinal mucosa. Most like cause of current condition?

Graft T cell sensitization against host MHC antigens GVHD can occur following transplants of organs rich in lymphocytes (liver). T lymphocytes in donor organ become sensitized against MHC antigens of recipient and attack host's tissues. Skin, liver, and GI tract are most effected. In contrast to GVHD, acute & chronic graft rejections lead to graft failure WITHOUT significant involvement of other organ systems.

Neutropenia causes increased susceptibility to what type of infections?

Gram-negative organisms (such as P. aeruginosa)

Poststreptococcal glomerulonephritis occurs most frequently in children and presents with nephritic syndrome (oliguric renal failure, HTN, red cell casts, cola colored urine, mild proteinuria, & periorbital edema) following recent skin infection. Immunofluorescence microscopy shows what?

Granular deposits of IgG, IgM, and C3 in the mesangium & basement membranes (labs show decreased C3 & elevated titers of streptococcal antibodies- anti-DNAse B, anti-hyaluronidase, antistreptolysin O). Note: electron microscopy shows subepithelial humps representing antigen-antibody complexes at the epithelial surfaces

60 year old main sustains MI and is eventually discharged with conservative management. 12 days after his MI he dies suddenly. LM would most likely show what change in the myocardium of the inferior wall?

Granulation tissue with neovascularization

Nasal mucosal ulcerations (saddle nose) & glomerulonephritis (oliguria) associated with c-ANCA is associated with what disease?

Granulomatosis with polyangitis (Wegener's)

Disease in which thyroid follicular epithelium is tall & crowded with hyperactive reabsorption causing scalloping around the edges of the colloid.

Graves disease

Diagnosis of schizophrenia requires?

Greater than or equal to 2 of the following 5 symptoms: delusions, ,hallucinations, disorganized speech, grossly disorganized behavior, and negative symptoms. One of these must be delusions, hallucinations or disorganized speech. Total illness duration but be 6 months (including prodromal and residual period) with greater than or equal to one month of active symptoms

Criteria for major depressive disorder?

Greater than or equal to a 2 week period with five or more of the following: depressed mood, loss of interest, sleep disturbance, appetite disturbance, loss of energy, psychomotor agitation or retardation, impaired concentration, guild, and suicidal thoughts

Most likely diagnosis in patient with progressive, symmetric, ascending lower extremity weakness, depressed DTRs, & paresthesias with recent diarrheal illness?

Guillain-Barre syndrome (likely preceded by Campylobacter jejuni)

Acute, demyelinating polyneuropathy thought to be caused by molecular mimicry. Antibodies formed against infectious agent cross react with myelin and Schwann cells, leading to immune mediated demyelination. Light microscopy shows segmental demyelination and endometrial inflammatory infiltrate composed of lymphocytes and macrophages

Guillian-Barre syndrome

Hyperestrinism in liver cirrhosis likely arises due to increases in androstenedione production, androgen aromatization, and sex hormone binding globulin concentration (preferentially binds testosterone). Impaired estrogen metabolism by the liver may also be a contributing factor. A decreased free testosterone/estrogen ratio leads to?

Gynecomastia, testicular atrophy decreased body hair and SPIDER ANGIOMATA (due to estrogen's effects on arteriolar dilation). Number and size of spider angiomatas correlates with severity of liver disease.

Region on a sarcomere that contains only thick filaments and no thin filaments?

H band H band is part of the A band (which is on either side of the M line) where thick filaments have no overlapping thin (actin) filaments.

PROXIMAL duodenal ulcers are most commonly caused by?

H. pylori infection of NSAID use (ulcers found beyond duodenal bulb are suggestive of Zollinger-Ellison Syndrome)

Presence of what serologic marker in a pregnant woman with HBV is most likely to increase the risk of vertical transmission of the virus?

HBeAg (marker of viral replication and increased infectivity) Note: newborns of all mothers with active HBV are passively immunized at birth with hepatitis b immune globulin followed by active immunization with recombinant HBV vaccine

HPV infection, especially with strains 16 and 18, is the strongest risk factor for the development of cervical dysplasia and invasive cervical carcinoma. Coinfection with what virus allows HPV infection to persist and enhances expression of HPV oncogenes, increasing the risk for cervical dysplasia/cancer?

HIV

Oral thrush in an otherwise healthy person suggests possibility of what?

HIV infection

Digoxin toxicity typically presents with cardiac arrhythmia and nonspecific GI, neurological, and visual symptoms. Another sign is caused by inhibition of Na+/K+ ATPase pumps which is?

HYPERKALEMIA

Small gram negative rod that causes chancroid, a STI characterized by PAINFUL genital ulcers with an erythematous base. Lesions typically begin as papule or pustules and evolve into an ulcer. More than one lesion often present

Haemophilus ducreyi

Inhaled anesthetic hepatotoxicity is most frequently associated with which inhaled anesthetic?

Halothane (causes hypersensitivity reaction to drug that causes immune mediated attack against hepatocytes)

Most common benign lung tumors composed of disorganized cartilage, fibrous, and adipose tissue that present as asymptomatic peripherally located 'coin lesions' in patients 50-60 years old?

Hamaratomas

MOA of bisphosphonates (aldendronate, risedronate)?

Have a chemical structure similar to pyrophosphate and attach to hydroxyapatite binding sites on bony surfaces to inhibit bone respiration by mature osteoclasts

What is an appropriate question to ask in taking a sexual history in order to use neutral, open, and nonjudgemental approach?

Have your sexual partners been men, women or both?

Tendency of study subjects to change their behavior as a result of their awareness that they are being studied

Hawthorne effect (observer effect)

Location of caudate?

Head of the caudate lies in the inferolateral wall of the anterior horn of the lateral ventricle. Its separated from the globes pallidus and putamen by the internal capsule. Caudate is atrophied in Huntington disease. There is decreased metabolic activity of the stratum (composed of the putamen and caudate) in Huntington disease. Note: Posterior limb (MOST IMPORTANT) of internal capsule separates the globes pallidus and putamen from the thalamus and carries corticospinal motor and somatic sensory fibers as well as visual and auditory and sensory fibers. The genu/knee of the internal capsule lies between the anterior and posterior branches and carries corticobulbar fibers. Anterior limb of internal capsule separate caudate from the globus pallidus and putamen and carries a portion of the thalamocortical fibers. Insular cortex (insula) plays a role in the limbic system (emotion) as well as in the coordination of some autonomic functions, particular cardiac system

Main adverse effect seen with nitrate therapy?

Headaches, cutaneous flushing, reflex tachycardia, lightheadedness, & hypotension due to systemic vasodilation Frequently used for symptomatic relief with chronic stable angina. Anti-ishemic effect is mediated by systemic vasodilation (predominantly veins) with a decreased in LV end diastolic volume & wall stress resulting in decreased O2 demand, relieving angina symptoms. Avoid use in patients with hypertrophic cardiomyopathy, RV infarction, and those on phosphodiesterase inhibitors (increases risk of severe hypotension).

Insurance plan with low monthly premiums, low copayments & deductibles, and low total cost for the patient. Reduces utilization by confining patients to a limited panel of providers, requiring referral from a PCP prior to specialist consultations, & denying payment for services that don't meet established guidelines

Health maintenance organization (HMO)

The process of enabling people to increase control over their health and its determinants, and thereby improve their health

Health promotion (falls under primary prevention- preventing a disease process from getting established; examples: regular exercise, losing weight, abstaining from smoking, improving diet)

Chronic non-atrophic gastritis affecting the gastric ANTRUM is usually the result of?

Helicobacter pylori infection. Upper GI endoscopy reveals diffuse erythema of the astral mucosa. Biopsy reveals inflammatory cell infiltrate involving the superficial mucosal layers. Associated with increased risk of peptic ulcer & involvement of the gastric body in longstanding infection is associated with an increased risk of gastric adenocarcinoma and MALT lymphoma Note: pernicious anemia occurs in autoimmune ATROPHIC gastritis which affects the BODY of the stomach

Enzyme responsible for covering heme to biliverdin, pigment that causes the greenish color to develop in bruises several days after injury

Heme oxygenase

Nondominant lesions of parietal lobe results in?

Hemi-neglect & constructional apraxia Note: (dominant lesions result in Gerstmann syndrome)

Brown sequard syndrome characterized by ipsilateral spastic paralysis (due to corticospinal tract injury), ipsilateral loss of tactile/vibratory/position sensation (dorsal columns), and contralateral loss of pain and temperature sensation (spinothalamic tract) 1-2 levels below the level of injury results from?

Hemisection of the spinal cord

Skin pigmentation, hepatomegaly (with eventual progression to cirrhosis), impaired glucose homeostasis, cardiac dysfunction, atypical arthritis, and hypogonadism in middle aged adult is presentation of what disease?

Hemochromatosis

Speed of hemoglobin movement during gel electrophoresis?

Hemoglobin A > S > C

Deficiency of glucose-6-phosphate dehydrogenase results in?

Hemolytic anemia due to inability to generate NADPH in oxidative portion of pentose phosphate pathway

Classic manifestations of hemophilia include hemarthrosis, delayed and prolonged bleeding following minor trauma or surgical procedures, and intramuscular hemorrhage. MOA of desmopressin in treatment of a mild form of that condition?

Hemophilia A: desmopressin increases circulating level of factor VIII

Paroxysmal nocturnal hemoglobinuria is due to a PIGA gene defect (synthesizes GPI anchor protein that attaches CD55 and CD59) that leads to uncontrolled complement mediated hemolysis. Classic triad includes hemolytic anemia (hemoglobinuria), pancytopenia, and thrombosis at atypical sites (hepatic, portal, cerebral veins). Most likely pathologic renal finding associated with this condition?

Hemosiderosis (chronic hemolysis can cause iron deposition in the kidney)

Children between 3 and 11 years old with recent infection, palpable skin lesions, with or without colicky abdominal pain and arthralgia is characteristic of what disease?

Henoch-Schonlein purpura

IgA mediated type III hypersensitivity reaction in children that generally follows infection. Deposition of circulating IgA containing immune complexes in small vessels results in systemic vasculitis. Common manifestations include palpable lower extremity purpura, abdominal pain, arthralgia, hematuria

Henoch-Schonlein purpura

Small vessel leukocytoclastic angiitis associated with IgA and C3 deposition is typical of what disease?

Henoch-Schonlein purpura

57 year old with high grade fever, chills, and RLQ abdominal pain. Imaging reveals a fluid filled cavity in right lobe of the liver. Most likely diagnosis?

Hepatic abscess S. aureus can cause hepatic abscesses via hematogenous seeding of liver. Enteric bacteria (E. coli, Klebsiella, enterococci) can cause hepatic abscesses by ascending the biliary tract (ascending cholangitis), portal vein pyemia, or direct invasion fro man adjacent area (cholecystitis). In underdeveloped areas, hepatic abscesses have a high indicence and are usually caused by parasitic infections (Entomeba histolytia, echinococcal)

Main mechanism of excess copper removal in healthy human body is?

Hepatic excretion into bile Absorbed copper is used to form ceruloplasmin, which accounts for 90-95% of circulating copper. Senescent ceruloplasmin and the unabsorbed copper are secreted into bile and excreted in stool which is the main route of copper elimination. Renal tubular secretion accounts for 5-15% of daily copper excretion

Hepcidin is a polypeptide that acute as the central regulator of iron hemostasis. It is synthesized and secreted by?

Hepatic parenchymal cells High iron levels and inflammatory conditions increase the synthesis of hepcidin while hypoxia and increased erythropoiesis act to lower hepcidin levels. Low hepcidin levels increase intestinal absorption and stimulate the release of iron by macrophages

22 year old with fever and joint pain found to have atypical lymphocytes on blood smear. Condition is caused by enveloped virus containing partially double stranded circular DNA. An enzyme packed in its vision has RNA dependent DNA polymerase. Most likely what virus?

Hepatitis B virus Replication of HBV genome occurs within a newly synthesized capsid through the action of reverse transcriptase on an RNA template. The mature capsid contains partially double stranded circular DNA and reverse transcriptase

Man dies from progressive liver disease/hepatocellular carcinoma. Changed & unchanged hepatocytes demonstrate foreign DNA fragments integrated into their genome. Most likely organism?

Hepatitis B virus. HBV integrates its DNA into host genome increasing resk of hepatocellular carcinoma. HCV (RNA virus) lacks reverse transcriptase and doesn't integrate into host genome

64 year old with fever, anorexia, nausea. Recently immigrated to USA. Several weeks ago was found to have a positive PPD test. No symptoms. Diagnosed with latent TB. Was started on isoniazid with pyridoxine therapy supplementation one month ago. Temp is 100.9. On PE there is mild abdominal discomfort. Most likely cause of this patients symptoms?

Hepatocyte damage Isoniazid can be directly hepatotoxic, causing acute, mild hepatic dysfunction in 10-20% of patients and frank hepatitis (fever, anorexia, nausea, sometimes progressing to hepatic failure) in a small percentage of patients.

Autosomal recessive disorder of sphingomyelinase deficiency common in Ashkenazi Jewish population results in what classic presentation?

Hepatosplenomegaly, neurologic regression, and 'cherry red' spot on macula Note: lipid laden foam cells accumulate in the lifer and spleen and the nervous system resulting in symptoms; death usually by age 3

C1 inhibitor deficiency is associated with what condition?

Hereditary angioedema (C1 inhibitor degrades C1 and prevents excessive complement activation and inflammation)

Multiple inhibitory mechanisms exist to prevent premature activation of trypsinogen before it reaches the duodenal lumen, including cleavage inactivation of trypsin by trypsin itself and production of trypsin inhibitors (SPINK1). Gene mutations that render trypsin insensitive to cleavage inactivation cause?

Hereditary pancreatitis Note: patient with mutations affecting the serum protease inhibitor, alpha-1-antitrypsin, can have emphysema & liver cirrhosis but there is no association between cirrhosis and trypsinogen gene mutations!

Autosomal dominant defect in RBC structural proteins (spectrin, ankyrin, protein 4.1) characterized by increased erythrocyte osmotic facility and MCHC increased above 36 g/dL.

Hereditary spherocytosis

Causes multiple, bilateral, painful, ulcerating lesions. Initial infections frequently associated with systemic symptoms such as fever, headache, myalgia and tender lymphadenopathy

Herpes simplex virus

Potentially fatal complication from primary infection or reactivation of latent disease. Nervous system invasion leads to edema and NECROSIS OF TEMPORAL LOBE which can result in aphasia and personality changes in addition to the classic features if encephalitis (headache, fever, altered mental status, seizure)

Herpes simplex virus type 1 encephalitis

4 year old boy with 3 days of fever, irritability, and oral lesions. Refuses to eat or drink and has had lack of urination. PE demonstrates painful ulcers on tongue and gingiva, swollen gums, and cervical lymphadenopathy. Microscopic exam of an oral base scraping reveals multinucleate giant cells. Most likely diagnosis?

Herpetic gingivostomatitis caused by primary infection with HSV 1 (peak age for primary infection is 6 months to 5 years old) Note: in contrast to primary infection with HSV1, reactivation of latent herpes infection in the trigeminal ganglia results in MILD perioral vesicles

Mature B lymphocytes are exposed to a specific virus in vitro and acquire the ability to proliferate indefinitely while maintaining the capacity to secrete immunoglobulins. What test is used to diagnose an in vivo infection with this virus?

Heterophile antibodies EBC commonly infects B cells stimulating the to proliferate continuously ('transformation' or 'immortalization'). EBV is an oncogenic virus that promotes polyclonal B cell proliferation and heterophile antibody production. Heterophile antibody test is sensitive and highly specific for EBV infection

Term that describes the presence of different mitochondrial genomes within a single cell

Heteroplasmy

Mitochondrial diseases are characterized by exclusively maternal inheritance. The variable severity of these diseases is explained by the random distribution of normal and mutated mitochondria between daughter cells during mitosis & as a result some cells may have completely healthy mitochondria, while other cells contain mitochondria affected by genetic mutation. term for this is called?

Heteroplasmy 3 mitochondrial diseases: leber hereditary optic neuropathy, myoclonic epilepsy with ragged red fibers, mitochondrial encephalomyopathy with lactic acidosis and stroke like episodes

Patient with lactic acidosis and ragged skeletal muscle fibers histologically suggests mitochondrial myopathy. Variable clinical expressions in affected family members can occur due to?

Heteroplasmy (coexistence of distinct versions of mitochondrial genomes in an individual cell)

34 year old man with an LDL level of 310 and a normal serum triglyceride level. His father suffered an MI at age 39 and his parental grandfather died of heart attack at age 40. Most likely diagnosis?

Heterozygous familial hypercholesterolemia (familial dyslipiedmia type II)- autosomal dominant LDL receptor defect that causes high LDL levels and increases resk of premature atherosclerosis

Patient found to excrete large amounts of fructose in urine despite maintaining moderate fructose intake. Has hereditary defect in fructose metabolism but is asymptomatic and has no other medical problems. He is most like to remain able to metabolize fructose due to compensatory activity of what enzyme?

Hexokinase Essential fructosuria is a benign disorder of fructose metabolism caused by fructokinase deficiency. Some of dietary fructose is converted by hexokinase to fructose 6 phosphate which can enter glycolysis (this pathway is not significant in normal individuals)

7 month old boy recently immigrated from Isreal. Met developmental milestones in first 4 months of life but can no longer roll over by himself or smile. Difficulty with vision and hearing. Cherry red spot on macula. PE reveals NORMAL sized abdominal organs. Most likely deficient enzyme?

Hexosaminidase A (Tay-Sachs disease) Leads to accumulation of GM2 ganglioside within CNS. Lose hearing and vision early followed by paralysis. Note: Neimann-Pick also presents with cherry red spot but is associated with HEPATOSPLENOMEGALY, LAD, and bone marrow suppression.

Fructose 2,6-biphosphate activates PFK-1 increasing glycolysis and inhibits fructose 1,6-biphosphatase decreasing gluconeogenesis. Fructose 2,6 biphosphte concentration is regulated by a bifunctional enzyme complex: PFK-2 increases it in response to insulin and fructose 2,6-biphosphatase decrease it in response to glucagon. Increase fructose 2,6 biphosphate would up regulate fatty acid synthesis, conversion of fructose 6 phosphate to fructose 1,6 biphosphate, glucose to glycogen, and NAD+ to NADH, however, it would decrease conversion of alanine and other substrates to glucose because?

High concentrations of fructose 2,6 biphosphate inhibit gluconeogenesis Note: glycogen formation is stimulated by increased levels of insulin and glucose 6 phosphate. Because elevated insulin levels also increase fructose 2,6 biphosphate formation, the rise of fructose 2,6 biphosphate levels in hepatocytes is typically concurrent with increased glycogen synthesis

Characteristics of a drug that make it more susceptible to hepatic metabolism and clearance?

High lipophilicity/high volume of distribution/good penetration into CNS Drugs that are more lipophilic are preferentially processed by the liver into more polar compounds for easier elimination in the bile and urine. Kidney is the primary site of elimination of most drugs, but highly lipophilic drugs tend to be poorly eliminated in the kidney as they rapidly cross tubular cell membranes after filtration to reenter the tissues.

Quality of HIV-1 that allows for evasion of host humoral and cellular immune responses & development of resistance to anti-retroviral drugs?

High mutability

Most likely reason for the recurrence of infection of Neisseria gonorrhoeae several months after initial infection that initially improved with Ceftriaxone and azithromycin?

High variability of microbial surface antigens (porins, Opa proteins, lipooligosaccharide) prevents the formation of protective immunity and leads to susceptibility to repeat infection

58 year old with a BMI of 26.9. Fasting plasma glucose found to be 160mg/dL, although he has not been previously diagnosed with DM. Has elevated triglyceride levels and HDL is low. What additional finding would be suggestive of increased insulin resistance?

High waist circumference Visceral obesity as measured by waist circumference or waist to hip ratio is an important predictor of insulin resistance. High LDL levels DO NOT increase with insulin resistance, only triglyceride increases.

Redistribution of fat from the extremities to the trunk is a common adverse effect of what medication type?

Highly active antiretroviral therapy Lipoatrophy occurs in face, extremities, and buttocks (most common with NRTIs stavudine & zidovudine & protease inhibitors). Central fat accumulation in the trunk & viscera can occur with any HAART regimen.

A new amino glycoside antibiotic is developed that is believed to be particularly effective against Pseudomonas. The volume of distribution of the drug is determined to be 4.5L. New drug is mostly to have what property?

Highly charged Volume of distribution refers to a hypothetical volume of fluid into which the administered amount of drug would need to be uniformly distributed to produced the observed plasma concentration. Vd= amount of drug given (mg)/plasma concentration of drug (mg/L) Drugs that are avidly bound in the tissues have the highest volume of distribution, often much higher than total body water volume, because they accumulate readily within cells maintaining low plasma concentrations (bottom portion of equation is low). Characteristics of a drug such as high molecular weight, high plasma protein binding, high charge, and hydrophilicity tend to trap the drug in the plasma compartment (as opposed to being distributed into the interstitial fluid & tissues) resulting in a low volume of distribution (3-5L) If it can go into extracellular fluid is would have a volume of distribution of about 14L and drugs with the highest volume of distribution that can cross cell membranes reach intracellular compartment have a Vd of 41L

First area of the brain that is damaged during global cerebral ischemia?

Hippocampus Note: cells most susceptible to ischemia are the pyramidal cells of the hippocampus and neocortex & purkinje cells of the cerebellum

What is needed to confirm diagnosis of Mucormycosis and what is treatment?

Histologic examination and surgical debridement with anti fungal therapy

'Beads' are composed of DNA wrapped around a core of proteins. What protein is found outside this core and helps promote chromatin compaction?

Histone H1 Nucleosomes are composed of DNA wrapped around a core of 8 histone proteins (2 molecules each of H2A, H2B, H3, H4). H1 histone is located outside of this histone core and helps package nucleosomes into more compact structures by binding and linking the DNA between adjacent nucleosomes

Mechanism responsible for transcriptional repression of various genes in patients with Huntingtin Disease?

Histone deacetylation (silences genes necessary for neuronal survival)

Can survive intracellularly within macrophages. Causes disseminated mycosis in immunocompromised patients. Clinical features include systemic symptoms (fever, weight loss), painful oral ulcers, lymphadenopathy, hepatosplenomegaly

Histoplasma capsulatum (image shows small ovoid yeast bodies within macrophage). CXR of disseminated histoplasmosis- diffuse pulmonary infiltrates with hilar adenopathy, cavitary lesions in upper lung lobes, calcified nodes, fibrotic scarring

28 year old previously healthy man with episodic fevers, night sweats, and weight loss for several months. Immigrated from Kenya. Cervical lymphadenopathy present on exam. Lymph node biopsy shows cells with abundant cytoplasm, bilobed or doubled nuclei, and inclusion-like eosinophilic nuclei. Most likely diagnosis?

Hodgkin lymphoma (presence of reed-sternberg cells on lymph node biopsy are diagnostic of classic Hodgkin lymphoma)

Increased blood oxygen saturation between 2 right sided chambers or vessels indicates presence of left to right shunt. If abnormal oxygen increase occurs between the right atrium and right ventricle, a ventricular septal defect is likely present? Small VSDs produce what type of murmur?

Holosystolic murmur heard loudest over the lower left sternal border

Presents between ages 3 and 10 with a dislocated lens (ectopia lentis- downward). Associated with marfanoid habitus (elongated limbs, arachnodactyly, scoliosis) & intellectual disability. Patients are at a high risk of THROMBOEMBOLIC OCClUSION of both large & small vessels (especially heart, brain & kidneys) which is a major cause of morbidity & mortality.

Homocystinuria

Alkaptonuria is an autosomal recessive disorder characterized by clinical features of black urine color when exposed to air, a blue-black pigmentation on the face, and ochronotic arthropathy. Lack of what enzyme blocks the metabolism of tyrosine to fumurate leading to an accumulation of homogentisic acid?

Homogentisic acid deoxygenase

38 year old man with pain in multiple joints. Has a five year history of lumbar pain and a 2 year history of bilateral knee pain. Works in construction and pain is worse after a long day on his feet. PE shows blue-black spots on his sclerae and diffuse darkening of the auricular helices. Most likely cause of this patient's arthritis?

Homogentisic acid deoxygenase deficiency Alkaptonuria is an autosomal recessive disorder caused by a deficiency of homogentisic acid deoxygenase, an enzyme involved in tyrosine metabolism. Excess homogentisic acid causes diffuse blue black deposits in connective tissues. Adults have sclerae and ear cartilage hyperpigmentation along with osteoarthropathy of the spine and large joints. Urine of these patients turns black when exposed to air due to oxidation of homogentisic acid.

29 year old woman with right hand clumsiness. Injured right upper extremity after falling off bike 6 months ago. Has since has pins and needles sensation in right hand associated with mild weakness. PE shows decreased sensation over 5th digit and flattened hypothenar eminence. Triceps reflexes are 2+ bilaterally. Nerve affected in this patient is commonly injury at what location?

Hook of the hamate Ulnar nerve can be injured at the medial epicondyle of the humerus or in Guyon's canal near the hook of hamate and pisiform bone in the wrist. Patients have sensory loss over medial one and a half digits and hypothenar eminence and weakness on wrist flexion/adduction, finger abduction/adduction, and flexion of the 4th and 5th digits. Hypothenar eminence may flatten due to muscle atrophy. Weakness in the lumbricals can produce ulnar claw deformity during finger extension

Mycobacterium tuberculosis triggers CD4 T lymphocytes to release interferon gamma which leads to macrophage activation (improves intracellular killing ability) and differentiation into epitheliod histiocytes. Epithelioid histiocytes and what other cells are the key components of granuloma formation?

Horseshore shaped, multinucleated Langhans giant cells (fused, activated macrophages)

Pseudomas aeruginosa infections are common in what type of patients?

Hospitalized, neutropenic, indwelling catheters, burn

44 year old man with PCKD undergoes renal transplant. One week later he develops low grade fever, body aches, and decreased urine output. Abdominal tenderness over the graft is present on palpation. Serum creatinine is 2.2mg/dL but was only 1.2mg/dL two days ago. Doppler studies reveal adequate graft perfusion. Biopsy of graft demonstrates dense interstitial infiltration by mononuclear cells. Most likely cause of this patient's condition?

Host T cell sensitization against graft of MHC antigens Acute cellular rejection most often occurs within weeks of transplantation due to sensitization of host T lymphocytes against donor MHC (HLA) antigens causing a MONONUCLEAR (lymphocytic) infiltrate on histopathology and graft destruction. Prevention is attempted with calcineurin inhibitors (cyclosporine, tacrolimus) and treatment includes systemic corticosteroids. Note: acute humoral (B-cell) rejection causes necrotizing vasculitis with NEUTROPHILIC infiltrate

29 year old man with increased sweating, heat intolerance, insomnia, and unintentional weight loss over past 4 weeks. Noticed right testis feels bigger, but has no pain. Thyroid is mildly enlarged on exam. Testicular exam reveals an enlarged, contender right testicle. Labs reveal elevated T4 and T3. Scrotal US shows hypoechoic mass within right testicle. Elevated levels of what marker would most likely explain his symptoms?

Human chorionic gonadotropin Patient likely has a testicular germ cell tumor producing large quantities of hCG which has a structure similar to TSH. Patients with testicular germ cell tumors or gestational trophoblastic disease can develop very high levels of hCG which can stimulate TSH receptors and cause paraneoplastic hyperthyroidism.

Gene that codes for a P-glycoprotein, a transmembrane ATP dependent efflux pump protein that has a broad specificity for hydrophobic compounds. This protein can both reduce the influx of drugs into the cytosol and can increase the efflux from the cytosol, preventing the action of chemotherapeutic agents

Human multi drug resistance gene (MDR1)

34 year old man with HIV with several months of pain and itching in the perirectal area. Reports intermittent rectal bleeding and often sees bright red blood on the tissue after wiping. On exam, a single hard mass with superficial ulceration is noted in anal canal. No palpable regional lymphadenopathy. Pathogen most likely responsible for patient's current anal pathology?

Human papillomavirus HPV 16 and 18 are strongly associated with anal and cervical squamous cell carcinoma. HIV infection increases prevalence of HPV infection and the risk of anal carcinoma; risk is further increased in men who have sex with men

Peptide hormone secreted by synctiotrophoblasts that increases maternal insulin resistance during 2nd and 3rd trimesters, leading to a rise in serum glucose that helps provide adequate nutrition to the growing fetus?

Human placental lactogen Note: gestational diabetes occurs when the compensatory rise in maternal insulin secretion is inadequate to prevent serum glucose levels from reaching excessively high levels

Presents with normal ICP & increased dilation of ventricles secondary to cortical atrophy. Occurs most often in elderly patients with dementia

Hydrocephalus ex vacuo >< Normal pressure hydrocephalus, which is due to enlarged ventricles in the elderly

Type of hydrocephalus that can result from neurodegenerative diseases (AIDS dementia) due to significant cortical atrophy which allows the ventricles to expand while maintaining normal pressure

Hydrocephalus ex-vacuo

40 year old with involuntary movements, ataxia, tremor that began 2 weeks ago and have acutely worsened over the last two days. PMH of bipolar disorder for which she takes Lithium. Was started on new medication 2 months ago for newly diagnosed HTN. Neuro exam reveals resting tremor with difficulty with balance. ECG shows NSR with nonspecific T wave abnormalities. Most likely agent being used to treat her HTN?

Hydrochlorothiazide Chronic lithium toxicity (confusion, ataxia, neuromuscular excitability) can be precipitated by volume depletion and drug interactions with thiazide diuretics, ACE inhibitors, and NSAIDs. Lithiumm is similar to sodium in its properties and is almost exclusively excreted by the kidneys. Like Na+ its filtered and reabsorbed mostly in the PCT and any caused of decreased GFR can cause proximal sodium/lithium reabsorption and lead to increased lithium retention. Thiazide diuretics limit Na+ reabsorption in the DT, causing mild volume depletion that stimulates PT reabsorption, which can lead to lithium toxicity over time.

BPH leads to progressive bladder outlet obstruction. Its characterized by a combination of epithelial and stroll hyperplasia, predominantly in the periurethral and transition zones. On palpation the prostate has a rubbery consistency (in contrast to prostate CA which is nodular and firm). Over time increased urinary pressures associated with BPH can lead to what effects in the kidney?

Hydronephrosis and renal parenchymal atrophy with scarring (can progress to chronic kidney disease)

Cyanide positioning presents with condition, flushing or a 'cherry red' skin color, abdominal pain and vomiting. Antidote?

Hydroxycolbamin (B12 precursor) - binds to cyanide forming cyanocolbalmin which can be easily excreted in urine

Neuroepileptic malignant syndrome is an adverse reaction to antipsychotic medication that is characterized by severe 'lead-pipe' rigidity, hyperthermia, autonomic instability, and mental status change. Symptoms overlapp with those of serotonin syndrome but patients with NMS do not exhibit what symptoms that are characteristic of serotonin syndrome?

Hyeprreflexia and clonus

45 year old with 2 month history of left arm clumsiness and weakness. On PE there is reduced muscle strength in the left upper extremity. MRI reveals lesion involving the corticospinal tract. What additional signs is most likely to be seen in this patient?

Hyperactive deep tendon reflexes Upper motor neuron damage leads to spastic paralysis, hyperreflexia, and upping plantar reflex (babinski sign) due to loss of descending inhibition over lower motor neurons in the anterior horn. Conversely, lower motor neuron lesions cause flaccid paralysis, hypotonia, hyporeflexia, muscle atrophy, and fasciculations

This form of rejection occurs immediately upon initial perfusion of the transplanted organ and is often diagnosed intraoperatively due to mottling and cyanosis of organ

Hyperacute rejection (type II hypersensitivity reaction mediated by preformed anti-donor IgG antibodies in the recipient that attack transplanted organ)

Effects of hyperammonemia on the glutamate-glutamine cycle

Hyperammonemia in advanced liver failure occurs as direct result of the cirrhotic liver's inability to metabolize nitrogenous waste products. Ammonia crosses BBB & causes excess glutamine to accumulate in astrocytes (astrocyte swelling and impaired glutamine release), decreasing amount of glutamine available for conversion to glutamate in neurons, resulting in disruption of excitatory neurotransmission

Thiazide diuretics work by blocking Na+-Cl- symporters in the DCT, causing enhanced Na, Cl, and water excretion. Since only a small amount of filtered Na+ reaches the distal tubules, thiazides are not as efficacious as loop diuretics. Unlike loop diuretics, thiazides can cause?

Hypercalcemia

34 year old with progressive weight gain over past year. Labs reveal elevated cortisol thats not suppressed with low dose dexamthasone. 24 hour urine free cortisol concentration and plasma ACTH are elevated. Most likely histopathologic finding on patient's adrenal glands?

Hyperplasia of the zone fasciculata and reticularis ACTH is the major trophic hormone of the zona fasiculata and reticularis whereas the zone glomerulosa is primarily regulated by angiotensin II. Excess production of ACTH causes increased cortisol synthesis within the zone fasiculata (Cushing's manifestations) and increased androgen production within the zona reticularis (irregular menstruation, hirtuism in women)

53 year old patient with a positive Babinski sign (extension/dorsiflexion of toes following firm stroking of the lateral plantar foot from the heel to the ball of the foot) is like to exhibit what other symptoms?

Hyperreflexia, muscle paralysis/weakness, & spasticity (pyramidal signs indicative of an UMN lesion) Note: its normal for infants up to 12 months to have positive Babinski sign due to incomplete myelination of the corticospinal tracts

How does allergic bronchopulmonary aspergillosis present?

Hypersensitivity reaction with wheezing and migratory pulmonary infiltrates in patients with asthma. Increased titers of IgE and antibodies to Aspergillus are diagnostic.

Major risk factor for aortic dissection?

Hypertension In longstanding HTN there is medial hypertrophy of the aortic vasa vasorum and consequently reduced blood flow to the aortic media. This can cause medial degeneration with loss of smooth muscle cells, leading to aortic enlargement and increased wall stiffness. Both of these changes exacerbate aortic wall stress which is already increased due to the HTN itself.

62 year old man with difficulty seeing. Had had blurry vision for the past 2 weeks. A day ago, he notice a shadow develop suddenly across the visual field of his right eye. Has had no recent headaches, double vision, vertigo, light sensitivity, or nausea. PEERL on exam. Visual field testing by confrontation reveals patchy loss of vision. A flame shaped retinal hemorrhage in the right eye is noted on fundoycopic exam. Most likely cause of his visual disturbance?

Hypertension Severe HTN in retinal precapillary arterioles causes endothelial disruption, leakage of plasma into the arteriolar wall, and fibrinous necrosis. The necrotic vessels can then bleed into the nerve fiber layer which can be seen on exam as dot or flame shaped hemorrhages. Hypertensive retinal hemorrhage typically causes painless, unilateral visual disturbances, ranging from mild obscuration without loss of visual acuity to permanent blindness. Other findings of hypertensive retinopathy include thickening of the arteriolar walls ('copper or silver wiring'), compression of the associated veins (arteriovenous nicking), and small white foci of retinal ischemia ('cotton wool spots').

Erythropoiesis stimulating agents (erythropoietin & darbepoetin alpha) can substantially improve anemia symptoms, avoiding the need for blood transfusions in chronic kidney disease & dialysis patients. Complications of erythropoiesis stimulating agents?

Hypertension & thromboembolic events

Single most important risk factor for the development of intimal tears leading to aortic dissection?

Hypertension. HTN, smoking, DM, hypercholesterolemia are all major risk factors for atherosclerosis, which predisposes more to aortic ANEURYSM formation than aortic DISSECTION

Patient with sudden right sided weakness. Has a history of HTN but does not follow up routinely with physician. Takes no meds. Decreased strength 3/5 on the right. Initial CT reveals no abnormalities. Four weeks later repeat imagine reveals 9mm fluid filled cavitary lesion in the left internal capsule. Most likely caused by?

Hypertensive arteriolar sclerosis (hardening/thinkening of the vessel wall predisposes to thrombotic vessel occlusion) In acute setting, CT imaging may not reveal the expected hypo density of ischemic stroke due to the small infarct size, but after several weeks the necrosis turns into cavitary spaces filled with CSF and surrounded by scar tissue called lacunas Note: Charcot-Bouchard aneurysms are caused by chronic HTN and involve the same penetrating arterioles as lacunar stoke, but the aneurysm rupture leads to intraparenchymal hemorrhage within deep brain structures which would appear acutely as a relatively large intracerebral hyperdensity on CT scan (patient's initial normal CT rules this out)

Bifid carotid pulse with brisk upstroke ('spike and dome') is characteristic of what condition associated with dynamic left ventricular outflow tract obstruction during systole?

Hypertrophic cardiomyopathy (SpO2 in the cardiac chambers remains normal)

Type of anemia associated with iron deficiency?

Hypochromic, microcytic

Type of anemia associated with thalassemias?

Hypochromic, microcytic

Type of anemia associated with B6 (pyridoxine) deficiency?

Hypochromic, microcytic, sideroblastic

Iron deficiency anemia

Hypochromic, microcytic; blood loss (especially occult GI blood loss) must be ruled out!

Most patients who have narcolepsy with cataplexy demonstrate undetectable levels of what compound in their CSF?

Hypocretin-1 Hypocretin-1 (orexin-A) and hypocretin-2 (orexin-B) are neuropeptides produced in the lateral hypothalamus that function to promote wakefulness and inhibit REM sleep related phenomena.

57 year old man with unrefreshing sleep, loud snoring, and frequently gasps for breath. PMH of HTN. BMI is 30. PE reveals a bulky tongue and crowded, narrow oropharynx. Electrical stimulation of what nerve may improve the pathophysiologic cause of this patient's symptoms?

Hypoglossal Neuromuscular weakness of the oropharynx is involved in the pathophysiology of obstructive sleep apnea. Electrical stimulation of the hypoglossal nerve causes the tongue to move forward slightly, increases the diameter of the oropharyngeal airway and decreases the frequency of apnea events.

Nerve responsible for motor innervation to tongue?

Hypoglossal nerve (CNXII) ** Note: exception is the palatoglossus muscle which is enervated by Vagus nerve (CN X)

Most important complication during the recovery phase of acute tubular necrosis?

Hypokalemia (due to high volume, hypotonic urine)

1 month old boy being evaluated for cystic fibrosis, as he was born with a 508 mutation affecting an epithelial transmembrane protein. Confirmatory tests are pending. PE shows a somnolent boy with a sunken fontanelle and dry mucous membranes. What electrolyte disturbance is responsible for his current symptoms?

Hyponatremia (due to excessive salt wasting from his sweat) Patients with CF produce eccrine swear with higher than normal concentrations of sodium and chloride. Exposure to high temperature or exercise can lead to hyponatremia and hypochloremia due to excessive sodium chloride loss through sweat and salt supplementation is recommended

A collection of tan fat containing tissue is found around the kidneys and adrenal glands of a newborn during surgery. Surgical removal of this tissue would contribute to?

Hypothermia Brown adipose tissue is found in newborns and in hibernating mammals. Brown adipose cells contain several intracytoplasmic fat droplets and many more mitochondria than white adipose cells. They function to produce heat by uncoupling oxidative phosphorylation with the protein thermogenin.

Combination of acute onset dyspnea, calf swelling, and history of prolonged immobility is strongly suggestive of pulmonary embolism. What would arterial blood gas values show in setting of PE?

Hypoxemia and respiratory alkalosis (hypocapnia from hyperventilation in attempt to improve blood oxygenation) Note: bicarbonate levels remain normal in the acute phase, but metabolic compensation with renal bicarbonate loss occurs in about 48 hours

Patient from Southeast Asia with cough, night sweats, weight loss, and an upper lobe lesion on CXR. Tissue microscopy shows epitheliod histiocytes and multinucleate Langhans giant cells (predominant cells found in granulomas). Most important substance driving the development of this patient's observed microscopic lesion?

IFN-gamma T helper subtype 1 cells release IFN-gamma leading to the activation of macrophages, a process critical for the control of M. tuberculosis infection. Activated macrophages form mature phagolysosomes that destroy phagocytksed mycobacteria and can differentiate into epithelia and Langhans giant cells to wall off extracellular mycobacteria within caveating granulomas. IFN-gamma, IL-12, and TNF-alpha are the critical cytokines in the formation and maintenance of granulomas. TNF-alpha helps recruit additional monocytes and macrophages. Infected macrophages secrete IL-12 which induces activated T helper cells to differentiate into TH1 subtype.

Cytokine that attenuates the immune response related to inflammatory bowel disease through inhibition of Th1 cytokines, reduction of MHC II expression, and suppression of activated macrophages & dendritic cells

IL-10

Cytokine that regulates the activation and differentiation of T cells to aid in tumor cell destruction that is FDA approved for treatment of renal cell carcinoma and melanoma

IL-2

2 day old newborn develops lethargy and respiratory distress. Blood cultures grow beta-hemolytic gram positive cocci in chains that are bacitracin resistant. What measure could have prevented this patient's condition most effectively?

INTRAPARTUM ampicillin Universal prenatal screening for group B strep colonization by vaginal-rectal culture at 35-37 weeks gestation is recommended to identify colonized women who require intrapartum antibiotics, with penicillin or ampicillin to prevent neonatal Group B streptococci sepsis, pneumonia, and meningitis

Most promising means of achieving pregnancy in woman with Turner syndrome?

IVF Note: spontaneous pregnancy occasionally occurs but risk of spontaneous abortion, Down syndrome, & Turner syndrome are all increased in such cases

Babesiosis should be considered in patients with febrile illness who reside in geographic areas where they are exposed o the IXODES tick. How is diagnosis can be established?

Identification of intraerythrocytic organisms on peripheral blood smear Note: Malaria is transmitted by the ANOPHELES mosquito; it also causes intraerythrocytic ring inclusions & has a typical incubation period of weeks (pay close attention to travel history)

Inferior vena cava is formed by the union of the right and left common iliac veins at the level of L4-L5. Renal arteries and vein lie at the level of L1. IVC returns venous blood to the heart from the lower extremities, portal system, and abdominal and pelvic viscera.

Identify IVC on CT

Classic PE findings in mitral stenosis include a loud first heart sound (S1) when the mitral valve closes, an early diastolic high frequency opening snap after the second heart sound (S2) which is caused by the sudden opening of the mitral valve leaflets when the LV pressure falls below the left atrial pressure at the beginning of diastole & a low pitched diastolic rumble (due to elevated pressure gradient between LA and LV during diastole). These findings are best heart at the cardiac apex using the bell of the stethoscope with the patient laying on the left side held in expiration. As MS becomes more severe the opening snap occurs earlier after S2

Identify high frequency sound opening snap heard on on tracing

7 year old boy with fatigue, easy bruising, and frequent epistaxis for 2 weeks. Takes no medications and has no PMH. PE reveals pallor and multiple ecchymosis. No lymphadenopathy or hepatosplenomegaly. Labs reveal pancytopenia. Bone marrow biopsy reveals an abundance of fat cells & scattered clusters of morphologically normal white blood cells. Most likely diagnosis?

Idiopathic aplastic anemia Aplastic anemia is most commonly due to a toxic effect (chemotherapy, gasoline, tobacco smoke, carbamazepine, hepatitis, EBV) or an autoimmune response causing apoptosis of pluripotent stem cells (pancytopenia). Bone marrow biopsy shows hypocellularity with an abundance of fat cells. Splenomegaly is notably absent due to a lack of available hematopoietic progenitor cells and therefore a lack of extramedullay hematopoiesis.

Condition that typically presents in young obese women with daily headache, bilaterally symmetric papilledema, and transient visual disturbances related to impaired cerebral venous outflow and elevated ICP. Symptoms tend to worsen during valsalva (bending down or coughing) as ICP increases

Idiopathic intracranial hypertension (pseudotumor cerebri)

Increased intracranial pressure compresses the optics nerves resulting in impaired axoplasmic flow and optic disc edema in what condition?

Idiopathic intracranial hypertension (psueudotumor cerebri)

An unpredictable drug reaction due to genetic difference or complex metabolic interactions in certain patients (ex: primaquine in patients with G6PD deficiency can lead to oxidative cellular injury & non immune hemolytic anemia).

Idiosyncratic drug reaction

Frequently presents as recurrent, self limited painless hematuria within 5 days of a URI (synpharyngitic hematuria). Kidney biopsy will show mesangial IgA deposits on immunoflurence.

IgA nephropathy (Berger disease) When accompanied by extra renal symptoms (abdominal pain, arthralgia, purpuric skin lesions) syndrome is called Henoch-Schonlein purpura Note: PSGN is seen 1-3 weeks after streptococcal pharyngitis and is not recurrent

A number of medications including opioids, radiocontrast agents and some antibiotics (ex: vancomycin) can result in what type of adverse reaction producing symptoms of diffuse itching and pain, bronchospasm, and localized swelling (urticaria)?

IgE INDEPENDENT mast cell degranulation (via activation of protein kinase A and PI3 kinase)

Anti-Rh immune globulin consists of anti-D antibodies that opsonize Rh+ fetal erythrocytes, promoting clearance by maternal reticuloendothelial macrophages and preventing maternal Rh sensitization. Routinely administered to Rh-negtive women at 28 weeks gestation and immediately postpartum. What immunoglobulin class do the antibodies belong to?

IgG (does not cause significant transplacental fetal hemolysis because the quantity administered is very small compared to typical immunologic reaction!)

Opsonization occurs when host proteins such as immunoglobulins or complement bind to foreign cells such as bacteria and coat the surface, enhancing phagocytosis. Most important opsonins (coating proteins) are?

IgG and C3b

68 year old woman with history of recent episode of acute calculous cholecystitis that was managed non-operatively. PE shows distended tympanic abdomen with high pitched bowel sounds. Abdominal x-ray reveals air in gallbladder in biliary tree. Gallstone likely lodged where?

Ileum Gallstone ileus results from passage of large gallstone through cholecystenteric fistula into the small bowel where it causes obstruction at ileum (narrowest portion of intestine). Present with symptoms of SBO and X-ray revels gas in gallbladder and biliary tree (pneumobilia).

Somatic symptom disorder involving excessive fears of having a serious physical disease

Illness anxiety disorder Note: usually due to misinterpretation of bodily symptoms or normal functions Example: woman with gas pain becoming preoccupied with having colon cancer

Potent inhibitor of the BCR/ABL protein tyrosine kinase used to treat CML

Imatinib (inhibits cellular proliferation of BCR/ABL expressing cells without inducing apoptosis)

On flow cytometric analysis of a sample of fetal thymus, a certain population of cells is identified that is positive for both CD4 and CD8 cell surface antigens. These cells are best characterized as?

Immature cortical T lymphocytes Immature T lymphocytes express both CD4 and CD8 cell surface antigens in addition to a complete TCR or a pro-TCR. These lymphocytes exist in the thymus cortex where they undergo positive selection and in the thymic medulla where they undergo negative selection

9 year old by with red colored urine, periorbital edema, and pedal edema. Recently treated for impetigo. What immune component is responsible for the damage caused to his kidneys?

Immune complexes (glomerulonephritis of PSGN is mediated by a type III hypersensitivity reaction) PSGN presents with edema, hematuria and an antecedent history of streptococcal infection (impetigo, cellulitis, pharyngitis). Infection must be caused by a nephritogenic strain of group A beta hemolytic streptococcus.

Unpredictable immune mediated response to medications

Immunologic drug reactions (drug allergies)

66 year old man with metastatic prostate cancer is started on Flutamide which led to significant pain relief and a decrease in the size of the primary tumor. Mechanism of action of Flutamide?

Impaired androgen receptor interaction Flutamide is a non steroid anti-androgen that acts as a competitive inhibitor of testosterone receptors. Its used in combination with long actin GnRH agonists (leuprolide) for the treatment of prostate cancer

Reason Crohn disease is associated with oxalate kidney stones?

Impaired bile acid absorption in the terminal ileum leads to loss of bile acids in the feces with subsequent fat malabsorption. Lipids then bind calcium ions, and the resulting soap complex is excreted. Free oxalate (normally bound to calcium to form an unabsorable complex to be excreted) is absorbed and forms urinary calculi (enteric oxaluria).

Homeless man found unresponsive on sidewalk. Temp is 101. Patient is ill appearing, thin, disheveled with poor dentition. Few white patches are noted on the oral mucosa. Lung auscultation reveals bilateral crackles. CXR reveals bilateral interstitial infiltrates & a silver stained bronchoalveolar lavage specimen is identified. What most likely predisposed this patient to his current lung condition?

Impaired cell mediated immunity Pneumocystis pneumonia is an atypical fungal infection seen primarily in those with impaired cell mediated immunity (advanced AIDS). Manifestations include slowly worsening pulmonary symptoms, hypoxia, bilateral infiltrates on CXR. The cystic organism can be visualized using silver stain of respiratory secretions. Note: patient has oral pharyngeal candidiasis which is also often seen in those with impaired cell mediated immunity

5 week old boy brought for evaluation after having tonic clonic movements of left upper and lower extremity that lasted 3 minutes. Newborn screen was positive for cystic fibrosis. On exam infant is postictal, but no longer seizing. Head CT shows right sided intracranial hemorrhage. What is most likely cause of infants presentation?

Impaired gamma carboxylation Vitamin K is a necessary cofactor for gamma-glutamyl carboxylase, an enzyme that carboxylates coagulation factors II, VII, IX and X. Newborns who do not receive prophylactic supplementation are at risk for bleeding complications. Patients with CF are also at risk for vitamin K deficiency due to poor absorption of fat soluble vitamins

Type 2 diabetes mellitus is due to insulin resistance and relative insulin deficiency. Insulin resistance in adipose cells hinders the antilipolytic effects of insulin, leading to lipolysis and release of free fatty acids. In turn, chronically elevated free fatty acid levels contribute to insulin resistance by?

Impairing insulin dependent glucose uptake and increasing hepatic gluconeogenesis Note: C-peptide is increased with increased production production of insulin, however, c-peptide does NOT cause insulin resistance

What does enteropeptidase deficiency result in?

Impairment of protein and fat absorption leading to diarrhea, failure to thrive, and hypoproteinemia (it's required to activate trypsin)

Young boy recently immigrated to US presents with sore throat & fever. Mild tonsillar erythema with exudates on PE. Clumped, gram positive bacteria with polar granules that stain deeply with aniline dyes on microscopic exam. Pathogenicity of organism responsible for condition?

Impairment of protein synthesis. Patient has corynebacterium diphtheriae which inhibits host cell protein synthesis by catalyzing the ADP-ribosylation of host cell elongation factor-2

Caused by incomplete degeneration of the central portion of the fibrous tissue band connecting the walls of the vagina. Adolescent patients typically present with primary amenorrhea, normal secondary sex characteristics, and cyclic abdominal or pelvic pain due to accumulation of menstrual blood in the vagina and uterus (hematocolpos)

Imperforate hymen Note: pressure from the collecting blood can also cause back pain and difficulties with defecation

First order kinetics vs. zero order kinetics?

In first order kinetics, a constant fraction (or proportion) of a drug is metabolized per unit of time so the amount metabolized changes based on the serum concentration. In zero order kinetics, a constant amount of drug is metabolized per unit of time, independent of serum levels

Acute nausea following administration of systemic chemotherapy results from stimulation of the chemoreceptor trigger zone (CTZ) which lies where?

In the area prostrema of the dorsal MEDULLA near 4th ventricle

Corresponds to the number of new cases of a disease in a certain population at risk over a given time period

Incidence (vs. prevalence which is the total number of cases in the population over a given period).

Autosomal recessive lysosomal storage disorder that occurs due to defects in protein targeting, a process by which proteins are transported to their appropriate intra or extracellular location. Patients with this disorder typically presents with failure to thrive, cognitive defects in the 1st year of life, coarse facial features, & corneal clouding. Typically fatal in childhood

Inclusion cell (I-cell) disease Note: Defective phosphotransferase enzyme that usually adds mannose resides onto proteins in order to allow them to reach the lysosome causes extracellular secretion of these proteins and accumulation of cellular debris in the lysosome forming inclusion bodies)

A boy born with down syndrome is examined in the nursery shortly after birth. Has a large, reproducible midline abdominal protrusion covered by skin that is more pronounced when he cries. Umbilical stump is at the center of the protrusion. Most likely cause of this abdominal finding?

Incomplete closure of the umbilical ring (allowing protrusion of bowel through the abdominal musculature) Normally the umbilical ring, or the congenital fascial opening for the umbilical cord, closes and forms the lines alba, a midline band of fibrous tissue. Most umbilical hernias are reducible, asymptomatic, and resolve spontaneously in first few years of life. Can occur in isolation or in associated with other conditions, such as down syndrome.

37 year old woman with sudden onset right arm weakness & difficulty speaking. On OCPs. MRI reveals acute infarction in the left frontal lobe. Cardiac exam normal. During echocardiogram, agitated normal saline is injected into a peripheral vein and bubbles are seen passing into the left side of heart. Most likely cause of the observed finding?

Incomplete fusion of atrial septum premium & secundum Cryptogenic stroke is associated with atrial septal abnormalities like patent foramen coal and ASD. At birth decreased pulmonary vascular resistance lowers RA pressure and raises LA pressure pushing the septum primum against the septum secundum, closing the foramen oval. Fibrosis and and tissue remodeling use flap closed but incomplete fusion is common (25% of normal adults). Although the foramen oval usually remains functionally closed, transient increases of RA pressure above LA pressure can produce a right to left shunt leading to a paradoxical embolism of venous clots into arterial circulation. Particularly concerning in patients on OCPs due to increased risk of VTE. Note: ASD (caused by aplasia of the septum primum or sesundum during development) is much less common than PFO and characterized by fixed splitting of S2.

Antiviral drugs currently recommended for the treatment of primary genital herpes include the nucleoside analogs (acyclovir). MOA of these drugs?

Incorporated into newly replicating viral DNA and ultimately terminate viral DNA chain synthesis

MOA of fibrinolytic drugs?

Increase clot lysis by increasing formation of plasmin from plasminogen.

Result of administration of metyrapone in patient with intact HPA axis?

Increase in ACTH, 11-deoxycortisol, urinary 17-hydroxycorticosteroid levels. Decrease in cortisol synthesis via inhibition of 11-B-hydroxylase. Failure of this to occur implies primary or secondary adrenal insufficiency

Dobutamine is a beta adrenergic agonist with predominant activity on B1 receptors, weaker activity on B2 receptors, and minimal activity on alpha-1 receptors. It is used for management of refractory HF associated with severe left ventricular systolic dysfunction and cariogenic shock. Effects of Dobutamine?

Increase in heart rate (decreased in diastolic filling time), increase in cardiac contractility, decreased systemic vascular resistance, decreased pulmonary capillary wedge pressure, increase in myocardial oxygen demand

44 year old male with history of IV drug abuse develops severe aortic regurgitation as a sequela to prior infection with infective endocarditis. What change is most responsible for maintaining cardiac output in the setting of aortic regurgitation?

Increase in left ventricular stroke volume In chronic aortic regurgitation, persistent LV volume overload triggers ECCENTRIC hypertrophy (ventricular wall lengthening due to addition of myocardial contractile fibrils organized in series) which causes a compensatory increase in stroke volume to maintain cardiac output. This compensatory mechanism allows for a relatively long asymptomatic period in most patients, but LV dysfunction eventually occurs leading to heart failure Note: CONCENTRIC hypertrophy occurs due to conditions that created ventricular PRESSURE overload (HTN, aortic stenosis) & involves ventricular wall thickening due to addition of myocardial contractile fibrils in parallel

People traveling to elevations >2500 m (8,000ft) are at risk of developing high altitude sickness which is characterized by headache, fatigue, nausea, dizziness and sleep disturbances. Hypobaric hypoxia is the primary cause which leads to?

Increase in respiratory rate with the development of respiratory alkalosis (compensated by renal excretion of bicarbonate within 24-48 hours. ABG levels: pH: 7.46 pCO2: 29 mmHg pO2: 75 mmHg

Stimulant medications are first line treatment for ADHD. What is their MOA?

Increase the availability of norepinephrine and dopamine in the prefrontal cortex (increase release and block reuptake)

ANP and BNP are released from the atria and ventricles respectively, in response to myocardial wall stretch due to intravascular volume expansion. These endogenous hormones promote?

Increased GFR, natriuresis, and diuresis

2 day old boy brought to ICU from nursery with tachypnea and hypoxia. He was born at 33 weeks and his mom didn't receive prenatal care. Sample of patient's lung tissue is obtained after he dies. Analysis of several large, cuboidal alveolar cells under electron microscopy reveals decreased number of granules containing parallel stacks of membrane lamellae. What pathologic process is most likely to result from the abnormal autopsy findings.

Increased alveolar tendency to collapse The lamellar bodies of type II pneumnocytes store and release pulmonary surfactant into the fluid layer lining the inner surfaces of alveoli. The major function of surfactant is to reduce surface tension in this fluid layer, and so a surfactant deficiency can cause alveolar atelectasis as seen in neonatal respiratory distress syndrome

56 year old smoker with chronic cough. Has been hospitalized several times in the past for respiratory infections requiring parenteral antibiotics. Has smoked cigarettes for the past 30 years. O2 sat is 90% at rest and decreases to 84% with moderate exertion. Lab value you would expect to find in this patient?

Increased erythropoetin production COPD can cause hypoxia sufficient to stimulate erythropoietin production by the cortical cells of the kidney

Minimal change disease is the most common cause of nephrotic syndrome in children that typically presents suddenly after a URI. Classic features are heavy proteinuria, hypoalbuminemia (<3 g/dL), generalized edema, and hyperlipidemia. Pathogenesis of minimal change disease?

Increased glomerular capillary permeability causes massive protein loss in the urine. Hypoalbuminemia reduces plasma oncotic pressure, which causes a fluid shift into the interstitial space, resulting in edema. Low oncotic pressure also triggers increased lipoprotein production in the liver (hyperlipidemia)

Multiple myeloma is associated with with anemia, bone pain (due to radiolucent lytic lesions in the spine), hypercalcemia, renal insufficiency (increased creatinine), and elevated total protein. Diagnosis is confirmed by serum protein electrophoresis showing monoclonal increase in immune globulins. Explanation of lab values seen in MM?

Increased osteoclastic activity (induced by tumor cells) in MM leads to elevated serum Ca2+ and reduced PTH. This decreases renal Ca2+ reabsorption, causing hypercalcuria. Hypercalcemia and light chain cast nephropathy cause progressive renal failure, leading to reduced 1,25-dihydroxyvitamin D synthesis

A left shift of the hemoglobin oxygen dissociation curve indicates increased hemoglobin O2 affinity and can be caused by?

Increased pH, decreased 2,3-biphosphoclycerate, and decreased temperature. A leftward shift of the oxygen dissociation curve means that O2 is relatively less available to tissues

82 year old man with fatigue and palpitations. Pulse is 130 and rhythm is irregularly irregular. He is treated with Digoxin. How does digoxin lower heart rate?

Increased parasympathetic tone (leads to inhibition of AV nodal conduction) Digoxin is used for ventricular rate control in atrial fibrillation as it decreases AV nodal conduction by increasing parasympathetic vagal tone. Digoxin is also used for HF due to its positive inotropic effect. These effects are accomplished via inhibition of Na+/K+ ATPase pump

Man with SOB and persistent nonproductive cough. Crackles bilaterally and drumstick shaped fingers. CXR shows diffuse reticular opacities. PFTs show decreased FVC, increased FEV1/FVC ration, & expiratory flow rates that are higher than normal when corrected for lung volume. Supernormal expiratory flow rates are due to?

Increased radial traction on airway walls. Patient has interstitial lung disease. Progressive pulmonary fibrosis --> thickening and stiffening of pulmonary interstitium --> increased lung elastic recoil & airway widening due to increased outward pulling (radial traction) by surrounding fibrotic tissue --> decrease in airflow resistance & supernormal expiratory flow rates

10 year old boy who sweats a lot and is much taller than other children. PE shows long extremities. Both parents are average height and have normal limbs. TSH is normal. Most likely cause of patients condition?

Increased release if IGF-1 from the liver Hypersecretion of GH before fusion of the epiphyseal growth plates causes gigantism, a condition characterized by rapid linear growth. GH is secreted by AP and acts to promote bone, cartilage, and soft tissue growth by stimulating release of IGF-1 from liver.

Heart failure occurs there is impaired ventricular filling or ejection of blood causing reduced cardiac output and excessive ventricular filling pressures. Reduced cardiac output is initially detected as drop in blood pressure which triggers compensatory neurohumeral stimulation directed at maintaining BP and tissue perfusion. This compensatory response is mediated through what?

Increased sympathetic nervous system activity, stimulation of RAAS and release of ADH. However these adaptations lead to adverse consequences. Increased after load from excessive vasoconstriction further impedes cardiac output and leads to excess fluid retention due to poor renal perfusion. RAAS also increases fluid retention. Deleterious cardiac remodeling occurs due to hemodynamic stress and neurohumeral stimulation

53 year old with progressive exertion dyspnea. Smoked 2 packs of cigarettes per day for the past 35 years. PE reveals increased AP diameter of chest. Auscultation reveals decreased breath sounds and scattered wheezes throughout. Extremities are unremarkable. Echocardiogram shows moderate dilation of the right ventricle and increased central venous pressure. Absence of peripheral edema is best explained by what compensatory mechanism?

Increased tissue lymphatic drainage Right heart failure increases central venous pressure which leads to a rise in capillary hydrostatic pressure, net plasma filtration, and interstitial fluid pressure. As the interstitial fluid pressure increases so does lymphatic drainage, which can compensate for moderate central venous pressure elevations and prevent the development of clinically apparent edema. Large central venous pressure elevations can overwhelm lymphatic reabsorptive capacity leading to development of overt edema Note: Patients with car pulmonate have elevated levels of aldosterone in response to decreased CO and renal hypo perfusion. Although the increase ANP/BNP levels in these patients tend to inhibit the RAAS, there is net sodium and water retention which exacerbates edema formation

Presence of acute phase reactants has what effect on ESR?

Increases ESR (causes erythrocytes to form a stacked, rouleaux formation)

Selective vasoconstriction of the efferent arteriole (up to a certain extent) increases hydrostatic pressure in the golumerular capullaries having what effect on filtration fraction and glomerular filtration rate?

Increases GFR and FF FF = GFR/RPF (constricting efferent arteriole increases GFR and decreases RPF, increasing FF) Note: as efferent arteriolar constriction continues to increase, the glomerular filtration rate begins to decrease due to a low mediated rise in the oncotic pressure in the glomerular capillaries; FF always increases with increasing efferent arteriole constriction

The degree of overlap between the healthy and diseased population curves limits the maximum combined sensitivity and specificity of a quantitative test. The degree to which sensitivity or specificity is affected depends on the chosen cutoff value. How does decreased overlap between healthy and diseased curves affect the sensitivity and specificity of the test?

Increases both sensitivity and specificity

What is the mechanism during squatting that relieves a child with tetralogy of ballot's symptoms?

Increases systemic vascular resistance In patients with TOF, squatting during a Tet spell increases systemic vascular resistance and decreases right to left shunting, thereby increasing the pulmonary blood flow and improving oxygen status

Positive predictive value represents the probability of truly having a disease given a positive test result. Relationship to prevalence?

Increases with increasing prevalence and decreases with decreasing prevalence PPV = a/(a+b)

Most colonic polyps are non neoplastic (hyperplastic, inflammatory, juvenile, submucosal) and don't increase the risk of colon adenocarcinoma. Neoplastic polyps include serrated and adenomatous (villous & tubular). Villous are most likely to undergo malignant transformation than tubular adenomas. Most important characteristic that correlates with malignancy risk?

Increasing polyp size (villous histology & high grade dysplasia are additional risk factors) Note: Hamartomatous polyps consist of disorganized mucosal glands, smooth muscle, and connective tissue. Sporadic juvenile polyps are solitary hamartomatous lesions that are not associated with increased risk of colon cancer. Multiple hamartomatous polyps frequently occur in Peutz-Jeghers and juvenile polyposis syndrome both of which confer increased risk of malignancy

5 month old boy brought in for right sided scrotal enlargement that has been present since birth and increases when he cries or strains to pass a bowel movement. On US there is found ro be a fluid collection around the right testis. The specific embryologic defect giving rise to the patient's condition can also lead to what?

Indirect inguinal hernia Communicating hydroceles and indirect inguinal hernias are caused by an incomplete obliteration of the processus vaginalis. The resultant connection between the scrotum and abdominal cavity can allow for fluid leakage (hydrocele) or the passage of abdominal contents (indirect inguinal hernia). Both can present as asymptomatic scrotal mass that increases in size during valsalva maneuvers.

Symptoms include constipation, mild weakness, lethargy, and poor feeding and are frequently due to honey consumption

Infant botulism Note: infant botulism results from consuming C. botulinum SPORES, but adult botulism results from consuming PREFORMED TOXIN (usually from canned foods)

Common precipitating factors of G6PD deficiency?

Infections, drugs (dapsone, antimalarials, sulfonamide antibiotics such as TMP-SMX), diabetic ketoacidosis, favism (fave beans)

Chronic vascular inflammation and scarring associated with rheumatic heart disease presdisposes to an increased risk of?

Infective endocarditis (valvular vegetations with destruction of underlying cardiac tissue) Note: vegetations are caused by bacterial colonization and growth on a sterile FIBRIN-platlet nidus that forms on the damaged/disrupted endothelial surface of the valvular apparatus

If rectus abdominis muscle is incised laterally, what structure is at greatest risk of injury?

Inferior epigastric artery Horizoantal transection of the rectus abdominis muscle must be performed with great cation as the inferior epigastric arteries enter the muscle at the level of the arcuate line. The inferior epigastric arteries below the arcuate line are susceptible to injury (hematoma, etc.) due to lack of supporting posterior rectus sheath (there is only anterior rectus sheath below the arcuate line)

Two branches of the external iliac artery?

Inferior epigastric artery- takes off immediately proximal to the inguinal ligament & supplies blood to the lower anterior abdominal wall Deep circumflex iliac artery- branches more laterally & supplies blood to lower abdominal wall

External iliac vein receives drainage from what two veins?

Inferior epigastric vein (supplies anterior abdominal wall) & deep circumflex vein (supplies iliac crest)

Anatomic landmark to differentiate between direct and indirect inguinal hernias?

Inferior epigastric vessels Indirect inguinal hernias are located lateral to the inferior epigastric vessels. They can continue into the scrotum and are felt by deep palpation of thee external inguinal ring with the tip of the finger. Direct inguinal hernias are located medial to the inferior epigastric vessels, do not protrude into the scrotum, and are best felt with the pulp of the finger

What structure prevents the proper ascent of a horseshoe kidney?

Inferior mesenteric artery In horseshoe kidney, the kidneys are fused at the poles. Isthmus of the horseshoe kidney lies anterior to aorta and posterior to IMA.

Where does lymph from the upper 1/3 of rectum drain?

Inferior mesenteric nodes

Maxillary devision of trigeminal nerve (V2), infraorbital vessels, and branches from sphenopalatine ganglion pass through what fissure?

Inferior orbital fissure

32 year old woman complains of heaviness in her eyelids at end of day. Chest imaging shows anterior mediastinal mass. Organ from which mass most likely originated shares embryologic origin with what structure?

Inferior parathyroid glands Patient has myasthenia graves which is associated with thymoma/thymic hyperplasia. Both the thumbs and inferior parathyroid glands are derived from the third pharyngeal pouch.

Venous components of external hemorrhoids (painful) drain into what vein?

Inferior rectal vein and then into the internal pudendal vein which communicates with the internal iliac vein

Passes through the right side of the central tendon of the diaphragm at the level of T8 & a penetrating would to the back to the immediate right of the vertebral bodies could strike this structure

Inferior vena cava

29 year old 8 days after uncomplicated vaginal delivery presents with fever, abdominal pain and is diagnosed with right ovarian vein thrombosis. If she remains untreated, she is at risk for extension of the thrombus into what vein?

Inferior vena cava Venous stasis and hypercoagulability from pregnancy in addition to endothelial damage from delivery can cause ovarian vein thrombosis in the puerperium. Ovarian vein thrombosis is an example of septic pelvic thrombophlebitis, a complication that can occur after vaginal delivery or c-section. Symptoms include fever and localized abdominal or flank pain. Usually thought to be uterine infection, but then fever persists despite antibiotics and a CT confirms presence of a thrombus in ovarian vein. Most thrombosis is right sided and can extend into the IVC but left ovarian vein thrombosis can extend into left renal vein

The most highly oxygenated blood in the fetus is carried by the umbilical vein which empties directly into what structure?

Inferior vena cava (via the ductus venosus) Remember ligamentum teres is the remnant of the umbilical vein in the adult)

Most likely mechanism for developing gallstones in Crohn's disease patients?

Inflammation of the terminal ileum causes decreased bile acid reabsorption. Terminal ileum is a typical location of Crohn's disease. When the mucosa of the terminal ileum is inflamed, bile acids are lost in the feces and a lesser amount of bile acid is present in the bile, causing ratio of cholesterol to bile acids to increase. Supersaturation of the bile with cholesterol leads to formation of gallstones.

Lactose intolerance presents with abdominal distension, cramping, flatulance, and diarrhea. Secondary lactase deficiency can occur after?

Inflammatory (celiac disease) or infectious (giardiasis) processes that damage the microvilli of the small intestines Note: primary lactose intolerance is due to geneticallyy regulated reduction of lactase production; increased incidence with age

Monoclonal antibody to TNF-alpha used to treat RA, ankylosing spondylitis, and fistulizing Crohn's disease

Infliximab

Fractures to the orbital floor commonly result from direct frontal trauma to the orbit. Can result in paresthesia of the upper cheek, upper lip, and upper gingiva if what nerve is damaged that runs along the orbital floor in a groove in the maxilla before exiting the skull just inferior to the orbit?

Infraorbital nerve (continuation of the maxillary nerve) Note: inferior rectus muscle can also become entrapped limiting vertical gaze

Shoulder abduction normal, but external rotation against resistance is weak & painful. Most likely injured muscle?

Infraspinatous Rotator cuff muscles: Supraspinatous- abduction Infraspinatous- external rotation Teres minor- helps with adduction & external rotation Subscapularis- helps with adduction & internal rotation ** see pic on phone for rotator cuff muscle table

Patient with complete tear of quadriceps tendon. Lidocaine injection at what site is most likely to provide anesthesia for surgical fixation?

Inguinal crease (femoral nerve block) at lateral border of femoral artery- will anesthetize the skin and muscles of the anterior thigh, femur, and knee

Fever, anorexia, nausea, myalgia, arthralgia, and rash in a patient who has recently undergone surgery outside US. PE reveals tender hepatomegaly and jaundice. Labs reveal markedly elevated aminotransferase levels and prolonged prothrombin time, leukocytosis, and eosinophila. Most likely diagnosis?

Inhaled anesthetic hepatotoxicity Note: albumin levels normal because it has a long half life of 20 days but prothrombin time is prolonged because of deficiency of factor VII which has a short half life

Vinca alkaloids (Vincristine & Vinblastine) MOA?

Inhibit microtubule formation- M phase specific!

MOA of triazoles (-azoles)

Inhibit synthesis of ergosterol

MOA of echinocandins (-fungins)

Inhibit synthesis of glucan (component of fungal cell WALL- NOT MEMBRANE)

Shiga like toxins (vero cytoxins) produced by EHEC are nearly identical to the Shiga toxin produced by Shigella dysenteriae. MOA?

Inhibit the 60S ribosomal subunit in human cells, blocking protein synthesis by preventing binding of tRNA

Triptans are an abortive migraine medication that are serotonin 5-HT1B/5HT-1D agonists that directly counter the pathophysiologic mechanism of migraine headaches by?

Inhibiting the release of vasoactive peptides (substance P & calcitonin gene related peptide), promoting vasoconstriction, and blocking pain pathways in the brainstem

34 year old kidney transplant patient treated with cyclosporine presents with nausea and anorexia. BP 160/96. Creatinine is 3.4 and cyclosporine level is markedly increased. A month ago he had normal BP, creatinine, and cyclosporine levels. Further evaluation reveals he has been drinking increased amount of grapefruit juice lately to improve his healthy. Mechanism most likely responsible for his current condition?

Inhibition of CYP450 enzymes in the gut wall Calcineurin inhibitor nephrotoxicity with resultant impairment of renal function is the most significant adverse effect of cyclosporine. Calcineurin inhibitors cause dose dependent renal vasoconstriction and tubular cell damage, which can precipitate acute renal failure. CYP3A is responsible for cyclosporine metabolism in the small intestine and liver. Grapefruit juice inhibits this enzyme and increases the nephrotoxicity of cyclosporine by raising circulating drug levels (pharmokinetic interaction).

35 year old man with history of PUD is diagnosed with acute gouty arthritis. After making diagnosis, physician prescribes an appropriate medication. Shortly after starting the medication, patient develops nausea, vomiting, and diarrhea. Most likely mechanism of action of drug prescribed to patient?

Inhibition of microtubule formation Colchicine is a second line agent for treating acute gouty arthritis. It inhibits tubular polymerization and microtubule formation in leukocytes, reducing neutrophil chemotaxis and emigration to sites inflamed by tissue deposition of monosodium rate crystals. GI mucosal function is also impaired by microtubule disruption leading to diarrhea and less commonly nausea, vomiting, and abdominal pain Note: NSAIDs are mainstay of treatment for acute gouty arthritis, but Colchicine is used in patients with mild to moderate renal failure, PUD, or other contraindications to NSAIDs. Probenecid (decreases PT reabsorption of uric acid) & allopurinol (converts xanthine to uric acid) are contraindicate in ACUTE gouty arthritis

Initial and maintenance treatments for Crytococcal meningitis?

Initial- Amphotericin B and flucytosine Maintenance- flucanazole

Cutaneous, strawberry type capillary hemangioma are common, benign, congenital tumors which are composed of unencapsulated aggregates of closely pared, thin walled capillaries. What is their course over time?

Initially grow in proportion to the growth of the child, before eventually regressing (in most cases the tumor will regress completely by age 7)

Pudendal nerve is derived from S2-S4 nerve roots and provides sensory innervation to the perineum and genitals as well as innervation to sphincter urethrae & external anal sphincter. How is pudendal nerve block performed?

Injecting anesthesia intravaginally medially in very close proximity to the ischial spine (posterolateral to vaginal sidewall) through the sacrospinous ligament (runs medially and posteriorly from ischial spine to sacrum)

Hypocalcemia can cause muscle cramps, perioral paresthesias, hypotension, and neuromuscular hyperexcitability. Common cause of primary hypoparathyroidism and hypocalcemia?

Injury to the parathyroid glands during thyroid surgery

40 year old female with progressive muscle weakness & myotonia. History of cataracts & DMT2 Takes an unusually long time to release grip. This condition is caused by?

Instability during maternal meiosis Patient has myotonic dystrophy type I which is a CTG repeat expansion disorder. CTG repeat alters RNS binding protein activity resulting in abnormal function of skeletal muscle chloride channel, insulin receptor, & cardiac troponin T.

Acanthosis nigricans presents with thickening and hyper pigmentation of skin in the flexural areas. Lesions have a classic 'velvety' texture and are commonly seen with skin tags (acrochordons) in the same area. Acanthosis nigrans commonly associated with?

Insulin resistant states (DM, acromegaly, obesity) and gastrointestinal malignancies (most commonly gastric adenocarcinoma)

Hypoglycemia is characterized by tremor, diaphoresis, or confusion associated with low blood glucose level and resolution of symptoms when blood glucose level is corrected. Hypoglycemia with an elevated insulin level and low c-peptide level suggests exogenous insulin secretion, whereas elevated c-peptide sugests ?

Insulin secretagogue use or an insulin secreting tumor (differentiate by testing the urine or blood for hypoglycemic agents)

Uptake of glucose by skeletal muscle is mediated by GLUT 4, which is translocated to the cell membrane in response to insulin and muscle contraction. Hypoglycemia can be precipitated by exercise in patients with insulin treated diabetes because?

Insulin will continue to be released from the injection site despite falling glucose levels. Strenuous exercise may cause changes in the skin perfusion that can lead to increased insulin absorption (especially if insulin is injected into an exercising limb). In normal individuals, hypoglycemia doesn't occur because a drop in blood glucose stops insulin release and counter regulatory hormones (glucagon) will increase endogenous production via glycogenolysis and gluconeogenesis

Catheter based radiofrequency ablation of the atrioventricular node with placement of a permanent ventricular is planned for patient with atrial fibrillation. Application of the radiofreuency at what location is most likely to achieve ablation?

Interatrial septum near the opening of the coronary sinus The atrioventricular node is located on the endocardial surface of the RA near the insertion of the septal leaflet of the tricuspid valve and the orifice of the coronary sinus

MOA of Ribavirin?

Interferes with the duplication of viral genetic material (induces lethal hypermutation, inhibits RNA polymerase and inosine monophosphate dehydrogenase depleting GTP, causes defective 5' cap formation on viral mRNA transcripts, and modulates more effective immune response)

3 month old with fever, irritability, and vomiting for 2 days. Blood cultures grow M. tuberculosis. One of her brothers died of disseminated mycobacterial infection during infancy. Impairment of what protective mechanism most likely contributes tooth's patients infection?

Interferon signaling Inherited defects involving the interferon gamma signaling pathway result in disseminated mycobacterial disease in infancy or early childhood. Patients require lifelong treatment with antimycobacterial agents.

Acquired skin disease involving the upper natal cleft of the buttocks

Intergluteal pilonidal disease

65 year old patient with limited active abduction of the shoulder joints bilaterally, headaches, visual and muscular symptoms, an enlarged temporal artery, and an elevated ESR. Most important mediator of this patient's current condition?

Interleukin-6 Cell mediated immunity is primary mechanism underlying giant cell arteritis. Production of cytokines, in particular IL-6 is an important driver of this process and closely correlates with the severity of symptoms. Inflammatory infiltrate in affected vessels is composed of lymphocytes (predominately CD4 positive T cells) and macrophages and frequently contains multinucleated giant cells. Note: monoclonal antibody against IL-6, Tocilizumab, is effective in treating GCA.

Retinal artery occlusion is a cause of acute, painless, monocular vision loss. Its usually caused by thromboembolic complications of atherosclosceosis traveling by what path?

Internal carotid arter --> opthalmic artery --> retinal artery

Hemorrhoids that originate above the dentate line are?

Internal hemorrhoids (external hemorrhoids originate below the dentate line)

Bilateral ligation of what arteries is most likely to control postpartum bleeding and preserve fertility (preventing need for hysterectomy)?

Internal iliac (uterine arteries branch off of internal iliac arteries) Postpartum hemorrhage is an obstetrical emergency. Bilateral ligation of the internal iliac artery can decrease uterine blood flow and control postpartum hemorrhage that is unresponsive to medical management (uterine massage, uterotonic medications). Risk factors for postpartum hemorrhage include prolonged labor and twin gestation that lead to uterine atony (boggy uterus unable to contract effectively after placental delivery). Uterus has a collateral blood flow from the ovarian arteries thats sufficient to maintain uterine function after internal iliac ligation.

A surgeon can most likely palpate the right ureter immediately anterior to what structure within the true pelvis?

Internal iliac artery Ureters pass posterior to the ovarian (gonadal) vessels within the retroperitoneum and anterior to the common/external iliac arteries to reach the true pelvis. Within the true pelvis, the ureters lie anterior to the internal iliac artery and medial to the ovarian vessels. The uterine artery crosses over the anterior surface of the ureter ('water under bridge')

Lymph from prostate drains into what nodes?

Internal iliac nodes

Where does lymph from prostate drain?

Internal iliac nodes

58 year old with large ulcerative mass in middle third of rectum extending to rectosigmoid junction. Surgical resection of sectosigmoid colon is performed. Frozen section reveals metastasis in pararectal lymph nodes. What other lymph node group should be sampled for further assessment of metastatic disease?

Internal iliac nodes Note: lymphatic drainage of the rectum proximal to the anal dentate line occurs via the inferior mesenteric and internal iliac lymph nodes. Areas distal to dentate line drain primarily to the inguinal nodes.

Clinicians have a legal and ethical obligation to report elder abuse, neglect, and exploitation. If there is reason to suspect abuse or next then the next step is?

Interview the patient ALONE to avoid intimidation by possible abusers Reporting would be warranted as the next step if the caregiver refused to allow the physician to speak privately with the patient or if elder abuse was suspected after interviewing the patient alone

What is strongest contributor in down regulating gastric secretion after a meal?

Intestinal influences. Cephalic and gastric phases stimulate gastric acid secretion while intestinal influences reduce gastric acid secretion by inhibiting gastrin stimulated histamine release from ECLs via release of peptide YY, somatostatin, & prostaglandins

Aortic dissection classically presents with severe retrosternal chest pain that radiates to the back and is typically triggered by what event?

Intimal tearing Develops when overwhelming hemodynamic stress leads to tearing of the aortic intimacy with blood subsequently dissecting through the aortic media. The resulting intramural hematoma can extend both proximally and distally and can compress major arterial branches and impair blood flow. Note: Dissections involving any portion of the ascending aorta are classified as type A and those confined to the descending aorta are classified as type B

45 year old female with sclerodactyly (skin thinking and tightening over the fingers due to collagen deposition) and raynaud phenomenon (cold induced vasospasm) has CREST syndrome, a localized variant of systemic sclerosis. Accentuated second heart sound is heard over upper LSB and abdomen is soft with mild hepatomegaly. Bilateral LE pitting edema noted. She is complaining of progressive exertion dyspnea and fatigue. Cause of her dyspnea/fatigue?

Intimal thickening of pulmonary arterioles Accentuated 2nd heart sound indicated elevated pulmonary artery pressure which has caused car pulmonale (LE edema and hepatomegaly). Pulmonary HTN is a common complication of systemic sclerosis. Collagen deposition is triggered by increased proliferation and accumulation of monoclonal T cell that secrete cytokines such as TGF-B. Microvascular injury of pulmonary arterioles leads to narrowing of lumen and increased pulmonary circulation pressure and eventually right heart failure.

Transient myocardial ischemia causes myocardial cells to increase in size. This is due in part to?

Intracellular Ca2+ accumulation Ion pump failure due to ATP deficiency during cardiac ischemia causes intracellular accumulation of Na+ and Ca2+. The increased intracellular solute concentration draws free water into the cell, causing the cellular and mitochondrial swelling that is observed histologically

Biochemical target of action of pioglitazone?

Intracellular nuclear receptor Thiazolidinediones bind to peroxisome proliferator activated receptor gamma, an intracellular nuclear receptor that acts as a transcriptional regular of many genes involved in glucose and lipid metabolism

45 year old patient with new onset seizure. Noncontrast head CT and brain MRI are consistent with a diagnosis of cavernous hemangioma. Patient is at greatest risk of developing what complication?

Intracerebral hemorrhage Cavernous hemangioma are vascular malformations that occur most commonly within the brain parenchyma above the cerebellar tentorium. They consist of clusters of dilated, thin walled capillaries with little or no intervening nervous tissue & carry an increased risk of ICH and seizure

Most common cause of unilateral, alarming, bloody or serosanguineous (blood tinged) nipple discharge usually without associated breast mass or skin changes?

Intraductal papilloma

Caused by proliferation of papillary cells in a duct or cyst wall with a fibrovascular core and may contain foci of atypic or ductal carcinoma in situ

Intraductal papilloma Note: bloody discharge results from twisting of vascular stalk of papilloma in the duct

Severe hypoglycemia with loss of consciousness is typically treated with intravenous dextrose in a medical setting and with what in a nonmedical setting?

Intramuscular glucagon

Overall increase in staphylococcal bloodstream infections is most likely related to increased use of?

Intravascular devices Indwelling central catheters predispose to bacteremia and sepsis and should be monitored regularly for signs and symptoms of infection. Catheter related infections should be suspected in hospitalized patients with new onset fever or bacteremia

Duchenne muscular dystrophy presents with progressive proximal muscle weakness in young boys due to increased number of muscle fiber degeneration. It is caused by frameshift mutations (most common) or nonsense mutations in the dystrophin gene that lead to the formation of a truncated, defective protein. Result of nonsense mutation?

Introduction of premature stop codon (UAA, UAG, UGA) in the coding sequence of mRNA

Substance is identified that specifically and completely inhibits glucose transport in proximal renal tubules. If a heathy volunteer is given this substance, glucose clearance will best approximate clearance of what substance?

Inulin (inhibition of sodium coupled carrier mediated transport of glucose by the proximal tubule would cause all filtered glucose to be excreted and the glucose clearance to approach the value of GFR which is estimated by calculating the clearance of inulin). Note: OAH is freely filtered and secreted and is used to calculate renal plasma flow

What value can be used to estimate GFR since its neither secreted nor resorbed by the renal tubules?

Inulin clearance

Patient discharged home to live with daughter who has supervised his meds in the past after acute exacerbation of CHF. A week later he is re hospitalized with recurrent CHF. Patient states that he thinks he took his meds appropriately but pill count shows he has not taken majority of his prescribed drugs. Daughter says she organizes pills daily and explains which meds he has to take before leaving work. What would have likely prevented this outcome?

Involvement of a social worker in discharge planning (social worker assesses whether patient has adequate family or caregiver support at home)

Anticholinergic agent and derivative of atropine that blocks the action of acetylcholine (released by vagues nerve) at muscarinic receptors, preventing bronchocontriction and reducing the parasympathetic stimulation of tracheobronchial submucosal glands in the lungs

Ipratropium Note: ipratropium would have no improvement in subjects that lack a functioning vagal nerve

Aneurysm of the internal carotid artery can laterally impinge upon the optic chaism causing what visual field defect?

Ipsilateral nasal hemianopia (damages uncrossed optic nerve fibers from the temporal portion of the retina)

Aneurysmal compression of CN III produces mydriasis (due to superficial parasympathetic fiber damage) with diplopia, ptosis, and 'down and out' deviation of the ipsilateral eye (due to somatic efferent fire injury). CN III courses between the posterior cerebral and superior cerebellar arteries as it exits the midbrain in the interpeduncular space and is particularly susceptible to injury from aneurysms located where?

Ipsilateral posterior communicating artery aneurysms

52 year old man with abdominal pain & watery diarrhea that have been progressive over the past month. Says he is depressed and has difficulty remembering things. 20 year history of alcohol abuse. Pigmented scaley skin rash is present in molar distribution of face, neck, and back of hands. Rash has been present for several months and worsens with exposure to sunlight. Its determined symptoms are secondary to lack of specific nutrient. What 3 enzymes in TCA cycle would be decreased as a result?

Isocitrate dehydrogenase, alpha ketoglutarate dehydrogenase complex, malate dehydrogenase. Niacin (B3) deficiency results in diarrhea, dementia, dermatitis (pellagra). Niacin is precursor for NAD and NADP which are important cofactors for dehydrogenase and reductase enzymes

31 year old man with a 2 week history of fever, night sweats and productive cough. Reports 11 pound weight loss. CXR shows cavitary lesion on the right. Sputum culture is obtained. When exposed to drug A, isolates of the pathogen growing in culture quickly become less resistant to decoloration with an acid-alcohol agent and stop proliferating. What is drug A?

Isoniazid Mycolic acids in the cell wall cause mycobacteria to be acid fast and they retain the carbofuchsin dye and resist declaration by an acid-alcohol decolorizing agent. Isoniazid is an antimycobacterial agent that specifically inhibits the synthesis of mycelia acids, which are essential components of the unique mycobacterial peptidoglycan cell wall. Without mycolic acids, the mycobacteria lose their acid-fastness and become unable to synthesize new cell walls or multiply.

B1 and B2 adrenergic agonist that causes increased myocardial contractility and decreased systemic vascular resistance?

Isoproterenol

Silicosis is associated with an increased risk of tuberculosis because?

It impairs the macrophage effector arm of cell mediated immunity. Macrophage phagolysosomes may be disrupted by internalized silica particles, causing release of the particles and viable mycobacteria & thereby inhibiting immune system's ability to control TB infection

Paraaminohippuric acid (PAH) is freely filtered from the blood in the glomerular capillaries to the tubular fluid in Bowman's space and its also secreted by the cells lining the proximal tubule of the nephron. Since its both filtered and secreted, its clearance can be used to estimate RPF. The secretion of PAH can be SATURATED at high blood concentrations because?

It is secreted from the blood into the tubular fluid by the cells of the proximal tubule by a CARRIER PROTEIN MEDIATED PROCESS Note: filtration of PAH is not a process that can be saturated (and therefore neither can excretion), only the secretion can be saturated.

Babesiosis and lyme disease are both transmitted by what tick and occur in similar geographic regions (confection is common)?

Ixodes tick Note: babesiosis causes acute febrile illness, thrombocytopenia, hemolytic anemia (indirect hyperbilirubinemia, elevated LDH, low haptoglobin), abnormal LFTs, and ring shaped and maltese cross intraerythrocytic inclusions. Splenectomy places patients at increased risk for severe babesiosis which can manifest with an acute respiratory distress syndrome (dyspnea, coarse crackles, bilateral infiltrates on CXR)

14 year old boy with delayed puberty and inability to distinguish smells (anosmia). Most likely diagnosis results from?

Kallman syndrome results from a failure of GnRH secreting neurons to migrate from their origin in the olfactory placode (outside the CNS) to their normal anatomic location in the hypothalamus. Usually a mutation in the KAL-1 gene or FGFR-1 gene.

HIV patient complaining of intermittent bloody diarrhea and abdominal pain. Endoscopy findings of reddish/violet flat maculopapular lesions to raised hemorrhagic nodules or polypoid masses. Biopsy shows spindle cells, neovascularization, and extravasated RBCs. Most likely diagnosis?

Kaposi's sarcoma

Form of primary ciliary dyskinesia characterized by the triad of situs inversus (point of maximum impulse right 5th intercostal space), chronic sinusitis, and bronchiectasis. Occurs due to mutations that impair the structure or function of cilia (such as failure of the dynein arms to develop properly). Infertility of men (impaired sperm motility) and women (immobility of fallopian tube cilia) is another frequent manifestation.

Kartagener syndrome

63 year old man with fever, anorexia, and discomfort associated with breathing. Radiographic studies of chest show multiple round lesions in both lung fields. Labs reveal a hematocrit of 56% (elevated). Biopsy reveals rounded and polygonal cells with abundant clear cytoplasm. Most likely primary location of this patient's metastatic disease?

Kidney Clear cell carcinoma is the most common subtype of RCC and is composed of large, rounded or polygonal cells with clear cytoplasm. Classic triad of flank pain, palpable mass, and hematuria occurs in a MINORITY of patients. Non-specific symptoms (fever, malaise, anorexia, weight loss) and paraneoplastic syndromes ( erythrocytosis & polycythemia due to EPO secretion, hypercalcemia due to PTHrP secretion) are more common. Often detected incidentally at an advanced stage and the lung is the most common site of metastasis

Microtubule associated ATP powered motor protein that facilitates anterograde transport of neurotransmitter containing secretory vesicles down axons to synaptic terminals

Kinesin Note: without kinesin secretory vesicles in nerve terminals would be absent

Protein that is involved in the transport process of herpes labialis HSV-1 reoccurrence?

Kinesin Kinesin is a microtubule associated motor protein that functions in the anterograde transport of materials and organelles within cells. Reactivation of latent HSV requires anterograde transport of viral particles from neuronal cell bodies in the sensory ganglia to the skin and oral mucosa. Note: dynein participates in retrograde transport. It is important in establishing the latent phase following primary HSV infection by transporting viral particles to the neural sensory ganglia.

Encapsulated, lactose-fermenting (pink on MacConkey agar), gram negative bacillus that appears mucoid in culture & has a thick capsule that is seen as a clear zone on gram stain. Causes pneumonia in individuals with impaired host defenses, especially alcoholics. Characterized by tissue necrosis and early abscess formation with production of thick, mucoid, blood tinged sputum (currant jelly sputum)

Klebsiella pneumoniae Most cases arise due to colonization of the oropharynx followed by micro aspiration of upper airway secretions. Because aspiration usually occurs during sleep (supine), the posterior segments of the upper lobes (particularly right) are classically affected.

Bilateral damage to the amagydala results in what syndrome characterized by hyperplasia, hyperorality, and hypersexuality?

Kluver-Bucy syndrome

34 year old woman with PCOS presents for treatment of infertility. Menses are irregular and occur ever 2-3 months consistent with chronic an ovulation. Clomiphene therapy fails. She receives ovulation induction therapy with a short course of menotropins followed by a single injection of hCG. Use of hCG therapy mimics what physiologic event?

LH surge Anovulation is a common cause of infertility. Menotropin (human menopausal gonadotropin) is a treatment option that acts like FSH and triggers formation of a dominant ovarian follicle. Ovulation is then induced by hCG administration which mimics LH surge

Klinefelter syndrome (47, XXY) is characterized by long lower extremities, small, firm testes, and azoospermia. Expected hormone levels?

LH- increased FSH- increased Testosterone- decreased Sperm count- none Inhibit-decreased (due to sertoli cell damage) Estrogen- high (increased aromatase activity stimulated by the gonadotropin excess) Progressive destruction and hyalinization of the seminiferous tubules causes testes to be small, round and firm. Seminiferous tubules and leading cells are damaged resulting in decreased testosterone and inhibit and increased FSH and LH due to loss of feedback inhibition

Traveler's diarrhea is most frequently related to enterotoxigenic E. coli that produces heat labile (LT, choleragen-like) & heat stable (ST) enterotoxins. Both cause water and electrolyte loss and watery diarrhea. MOA of LT and ST?

LT activates adenylate cyclase leading to increased intracellular cAMP and ST activates guanylate cyclase leading to increased intracellular cGMP

Diagnostic findings consistent with diastolic heart failure?

LVEDP- increased LVEDV- normal LVEF- normal (>50%) Diastolic heart failure is caused by decreased ventricular compliance and is characterized by normal left ventricular ejection fracture, normal LV end diastolic volume, and ELEVATED left ventricle filling pressures. Conditions that can decrease LV compliance: ischemia (impaired myocardial relaxation, increased stiffness (amyloid deposition), LV hypertrophy (HTN) Note: systolic HF is caused by a primary decrease in myocardial contractility (rather than compliance). Its characterized by a reduced EF (<50%), increased LV volume, and elevated LV pressure.

Inflammation of the vestibular nerve that causes acute onset vertigo, nausea, and vomiting. Usually occurs in a single episode following a viral syndrome

Labrynthitis

Interferon gamma release assays test for latent TB infection by measuring the amount of IFN-gamma released by T lymphocytes when exposed to antigens unique to M. tuberculosis. Advantage over tuberculin skin test?

Lacks cross reactivity to BCG vaccine & follow up visit not required

Increased breath hydrogen content, reduced stool pH, and elevated stool osmolality in a patient with flatulance, crampy abdominal pain, and water diarrhea after diary product consumption suggest what condition?

Lactose intolerance

Neuromuscular disorder presenting with proximal muscle weakness (gait alteration, difficulty rising from a chair, difficulty climbing stairs), cranial nerve involvement (diplopia, dysarthria, dysphagia, ptosis) and autonomic symptoms (dry mouth, impotence). Approximately half of all patients with this disorder also have malignancy (classically small cell lung cancer)

Lambert Eaton Myasthenic Syndrome

Digital clubbing is often associated with prolonged hypoxia. Examples of diseases it can be found in?

Large cell lung cancer, tuberculosis, cystic fibrosis, and suppurative lung diseases such as empyema, bronchiectasis, and chronic lung abscesses

Presents with inspiratory stridor during infancy due to the collapse of supraglottic structures during inspiration. Stridor is classically worse in the supine position and improves with upright positioning.

Laryngomalacia Laryngomalacia is a congenital softening of the tissues of the larynx (voice box) above the vocal cords. This is the most common cause of noisy breathing in infancy. The laryngeal structure is malformed and floppy, causing the tissues to fall over the airway opening and partially block it.

Sickle cell disease is characterized by repeated splenic infarctions that ultimately result in splenic atrophy and fibrosis, a process that is typically complete by?

Late childhood/adolescence Patient's with sickle cell disease or other hemolytic anemias have increased folic acid requirements due to increased erythrocyte turnover and are thus prone to developing relative folic acid deficiency and megaloblastic anemia (MCV > 110). Note: extramedullary hematopoiesis can occur in SCD and lead to an ELEVATED reticulocyte count and MCV as the reticulocytes produced are larger than those released by the bone marrow. Look at reticulocyte count to differentiate extramedullary hematopoiesis from increased folic acid requirement as a low reticulocyte index suggests an inadequate response to anemia, lack of erythropoiesis, and likely folic acid deficiency

All diuretics except for K+ sparing class cause K+ loss. They do this by increasing the volume delivered to the collecting duct, where aldosterone then attempts to reclaim the additional volume at the expense of K+. K+ sparing diuretics act on what portion of the nephron to antagonize the effects of aldosterone?

Late distal tubule and cortical collecting duct

Antioxidant use for more than five years associated with statistically significant decreased risk of stroke while use for less than five years associated with a nonstatistically significant risk reduction. What explains this discrepancy?

Latent period. Time elapsed from initial exposure or pathogenic insult to clinical manifestation of disease is known as latent prior for noncommunicable diseases (cancer, heart disease). This concept can be extended to risk factors/reducers. In this case more than five years of continuous antioxidant use was required to reveal their protective effect on stroke

Nerve that travels under the inguinal ligament. Provides sensory innervation to the anterolateral thigh. Entrapment (neuralgia parasthetica) most commonly occurs in obsession or pregnant persons or in those who wear garments too tight around the hip

Lateral femoral cutaneous nerve

Drain via the lateral track and communicate with the popliteal and inguinal nodes & are more likely to cause lymphadenopathy in both the POPLITEAL & INGUINAL areas

Lateral lower extremity lesions

Leads to contralateral loss of pain and temperature sensations along with ipsilateral paralysis of CN V, IX, X, XI.

Lateral medullary (wallenberg) syndrome Note: medial medullary syndrome causes contralateral spastic paralysis, contralateral sensory loss of vibration and proprioception, and ipsilateral placid paralysis of the tongue (CN XII)

59 year old with diplopia and a drooping right eyelid. PE shows a dilated right pupil that is nonreactive to both light and accommodation. Theres is vertical and horizontal diplopia. When asked to stare straight ahead, the right eye is directed inferiorly and laterally with respect to the left eye. MRI reveals aneurysm involving the right posterior communicating artery. What muscle is most likely to remain functionally intact in this patient?

Lateral rectus Lesions involving the oculomotor nerve cause ptosis, a downward and laterally deviated eye, impaired pupillary constriction and accommodation and diagonal diplopia. Most dreaded cause of CN III palsy is an enlarging intracranial aneurysm which can lead to rupture. Aneurysms causing CN III palsy most often involve the PCA.

Route of CSF flow?

Lateral ventricles --> interventricular foramina of Monro --> third ventricle --> cerebral aqueduct of Sylvius --> fourth ventricle --> lateral foramina of Luschka OR single middle foramen of Magendie --> subarachnoid space

Muscle innervated by thoracodorsal nerve. Most powerful adductor of the arm & assists with extension & medial rotation

Latissimis dorsi

Large thoracolumbar muscle that originates from the iliac crest and lumbar fascia to the spinous processes of T7-12 and lower ribs and inserts at the bicipital groove of the humerus. Innervated by thoracodorsal nerve (fibers originating from C6-C8). Primary functions include extension, adduction, and medial rotation of humerus. Vulnerable to injury from external trauma due to broad area and exposed location

Latissimus dorsi

Phenomenon in which gametogenesis results in separation of paired chromosomes so that offspring only inherit half of each parent's genetic compsition

Law of segregation

Difference between the initial detection of the disease and a specific outcome or measure endpoint

Lead time Note: lead time bias can occur when a test detects or diagnoses the disease at an earlier stage than another test without impacting the natural history of the disease (a study comparing disease survival times may erroneously conclude using the earlier detection test prolongs survival when the increased survival time is solely due to earlier detection of disease.

64 year old man with worsening cough and shortness of breath. Temperature is 100.4. CXR reveals infiltrate in the right lower lobe consistent with lobar pneumonia. Cause of pulmonary findings in this patient?

Leakage of protein rich fluid in the alveolar airspaces 4 stages of inflammatory response in lobar pneumonia: 1) congestion (day 0-2): neutrophils respond to bacteria and release cytokines that increase permeability of the pulmonary capillary endothelium leading to abundant proteinaceous fluid and erythrocytes in alveolar space 2) red hepatization (day 2-4): proteinaceous fluid transforms into fibrin strands, resulting in a confluent exudate of fibrin, neutrophils, erythrocytes (liver like aka red, firm, airless) 3) gray hepatization (day 4-7): red ell disintegration with increased leukocyte infiltration & neutrophils replaced by macrophages that began repair 4) resolution (> 7 days): macrophages secrete digestive enzymes that liquify fibrinous exudate which is reabsorbed, expectorated or phagocytksed by macrophages; lung parenchyma regains normal appearance around 3 weeks

8 year old boy presents due to school refusal. He hasn't gone to school in a few days saying he has a stomachache, but when it comes to soccer practice, he has no physical problems. Patient states that 'kids laugh at home when the teacher said she couldn't read his homework'. Mom says he is doing well in reading and math but when it comes to writing practice he fidgets a lot. He stares out the window and last week threw his writing book on the floor and began to cry. Because he doesn't pay attention his writing is unclear and disorganized and he is way behind the other kids in his class mom says. Exam is normal. Most likely explanation for this patient's school refusal?

Learning disorder Learning disorders are characterized by difficulties with key academic skills (reading, writing, math) resulting in performance well below expectations for age level. children commonly display symptoms of anxiety, inattention, or hyperactivity when under stress. If a child is showing these symptoms, learning disorder should be ruled out. Note: ADHD is characterized by a pattern of inattention and/or hyperactivity-impulsivity across at least TWO settings & there is no indication this patient struggles with inattention or hyperactivity when asked to read or do math or in the home setting.

Patient presents to discuss insomnia. What treatment recommendation for insomnia is an example of stimulus control therapy?

Leaving the bedroom if unable to fall asleep within 20 minutes Non-pharmacologic treatments for insomnia include sleep hygiene, stimulus control, relaxation, sleep restriction, and cognitive behavioral therapy. Stimulus control focuses on eliminating stimulating bedroom activities and getting into bed only when sleepy

Main toxin produced by Clostridium perfringens?

Lecithinase (aka alpha toxin) Functions to degrade lecithin, a component of cellular phospholipid membranes, leading to membrane destruction, cell death, and widespread necrosis and hemolysis

Patient with persistent atrial fibrillation experiences a cerebral thromboembolic event. Most likely source of thrombus?

Left atrial appendage

Enlargement of what chamber of the heart is likely associated with dysphagia, palpitations, and dyspnea on exertion in a patient with history of rheumatic heart disease?

Left atrium (RHD affects the mitral valve- stenosis/regurgitation) Note: posterior surface of the heart is mostly formed by the let atrium which lies directly over the esophagus so dilation and posteriorly displaced left atrium can compress esophagus (and recurrent laryngeal nerve in rare cases)

Portion of the heart that makes up the majority of the heart's posterior surface and resides adjacent to the esophagus?

Left atrium (enlargement of the LA, by atrial fibrillation of mitral stenosis for example, can compress esophagus and cause dysphagia) Note: TEE probe is closest to the left atrium and the left atrium, atrial septum & mitral valve are particularly well visualized using it If probe placed posteriorly, DESCENDING thoracic aorta would be clearly visualized (allows for detection of dissection or aneurysm)

66 year old man with sudden onset chest pain and dyspnea. ECG reveals ST elevation MI and emergency cardiac catheterization is performed. LEFT dominant circulation is noted with a normal left main coronary artery. A stenotic region is identified in one of the other coronary vessels. During PCI of the lesion, a small thrombus detaches and moves forward causing obstruction of the artery supplying the AV node. Atherosclerosis of what artery most likely cause his MI?

Left circumflex artery Coronary dominance is determined by the coronary artery supplying the posterior descending artery, which also supplied blood to the AV node via the AV nodal artery. The PDA originates from the RCA in 70% of patients (right dominant), both the RCA and LCX in 20% (codominant) and the LCX in 10% (left dominant circulation). AV NODAL ARTERY ARISEs FROM DOMINANT CORONARY ARTEY

Artery that branches off the inferior mesenteric artery to supply the transverse and descending colon?

Left colic artery

55 year old male with progressive headaches for the past 2 months. Headaches are throbbing, often associated with nausea, and worsen whenever he coughs or bears down during a bowel movement. PMH includes migraines without aura. MRI reveals a cystic mass in the left cerebellar hemisphere. What would you expect to find on clinical exam?

Left dysdiadochokinesia (impaired rapid alternating movements), limb dysmetria (overshoot/undershoot during targeted movement), and intention tremor (tremor during targeted movement). The cerebellar hemispheres are responsible for motor planning and coordination of the ipsilateral extremities via their connections with the lateral descending motor systems.

52 year old alcoholic with hematemesis that began an hour ago. Exam of abdomen reveals a palpable spleen and moderate ascites. Endoscopy reveals bleeding esophageal varicies. Patients current condition most likely resulted from chronic shunting of blood through what vein?

Left gastric

Patient sustained stab wound. PE shows a literally directed anterior chest wall stab found at the 5th intercostal space at the left MCL. Structure most likely to have been injured in this patient?

Left lung The left ventricle forms the apex of the heart and can reach as far as the 5th intercostal space at the left MCL. All other chambers of the heart lie medial to the left MCL. The lungs overlap much of the anterior surface of the heart.

Patient with right arm parasthesias for a few hours develops parasthesias on the entire right side of his body & then subsequently right sided conclusions followed by a generalized tonic clonic seizure. Where did seizure originate?

Left primary somatosensory cortex (postcentral gyrus) Primary somatosensory cortex processes all somatic sensory modalities (touch, temperature/pain, vibration/proprioception) of the contralateral body & a partial (focal) seizure in this region can result in contralateral sensory disturbance (paresthesias)

Forms apex & left border of heart on frontal CXR. A stab wound in the 4th intercostal space in the midclavicular line could strike this chamber, but only after passing through the bulk of the left lung

Left ventricle

Prolonged, burning substernal pain and ST segment elevation in leads I and v3-v6 is strongly suggestive of anterolateral left ventricle infarction. Common consequences of left ventricle infarction are?

Left ventricle failure, cariogenic acute pulmonary edema, pulmonary venous hypertension (congestion), & transudate of plasma into the lung interstitium and alveoli

65 year old man with longstanding HTN and T2DM presents for routine check up. Ex-smoker with 15 pack year smoking history. Cardiac auscultation findings over apex are given (in audio- extra low frequency heart sound at the end of diastole just before S1 known as S4) and are best explained by?

Left ventricle hypertrophy (likely due to stiff LV which occurs following prolonged HTN) Note: S4 heart sound is due to decreased LV compliance and is often associated with restrictive cardiomyopathy and LVH

Light microscopic exam of lungs of a 65 year old with dyspnea reveals macrophages containing golden cytoplasmic granules that turn dark blue with Prussian blue staining. Condition most likely associated with these findings?

Left ventricular dysfunction Presence of hemosiderin-laden macrophages in pulmonary alveoli indicates chronic elevation off pulmonary cagily hydrostatic pressures, most commonly the result of left sided heart failure (rise in pressure leads to extravasation of RBCs which become phagocytosed by macrophages

Disease of young children that results in isolated idiopathic osteonecrosis of the hip

Legg-Calve-Perthes disease

23 year old woman diagnosed with multiple sclerosis. What neuronal property would decrease as a direct result of demyelination?

Length (space) constant Speed of conduction down an axon depends on two constants: length and time constants. Demyelination decreases neuronal signaling velocity by decreasing the length constant and increasing the time constant (velocity = length/time). Length constant is a measure of how far along an axon electrical impulse can propagate without requiring active regeneration by ion channels. Time constant is a measure of the time it takes for the membrane potential to respond to a change in membrane permeability (sodium channel activation).

Protein hormone produced by adipocytes in proportion to the quantity of fat stored. Acts on the arcuate nucleus of the hypothalamus to inhibit production of neuropeptide Y (NPY = incr appetite) and stimulate production of alpha-MSH (increasing satiety)

Leptin Mutations in leptin gene or receptor result in hyperplasia and profound obesity. In humans these mutations don't exist and its thought that the sustained elevation in leptin levels from the enlarged fat stores results in leptin desensitization (leptin resistance similar to insulin resistance)

Patients who experience major depressive and hypomanic episodes are diagnosed with bipolar II disorder. In contrast to manic episodes (required for bipolar I disorder) hypomanic episodes are...?

Less severe, DO NOT INVOLE PSYCHOSIS, and cause a lesser degree of functional impairment. Patient's exhibit noticeable change in behavior but are often very productive despite requiring less sleep, are often able to work, and are rarely hospitalized.

In order to encircle the cardiac part of the stomach, a gastric band must pass through what structure?

Lesser omentum Lesser omentum is a double layer of peritoneum that extends from the liver to the lesser curvature of the stomach and the beginning of the duodenum. Divided into the hepatogastric and hepatoduodenal ligaments. Between the 2 layers of the lesser omentum lie the hepatic artery, common bile duct, portal vein, hepatic plexus, and lymphatics as well as the left and right gastric arteries.

34 year old woman with a 5 day history of nasal congestion and cough mentions she always received antibiotics for similar symptoms in the past and is requesting antibiotics today. No fever. Exam reveals pharyngeal erythema but is otherwise normal. Most appropriate response?

Let me explain why antibiotics are not the best choice and offer options to relieve your symptoms Empiric therapy for patients with uncomplicated URI is contraindicated. Patients requesting antibiotic prescriptions should be educated in a nonjudgemental manner about the lack of efficacy and adverse effects of antibiotic therapy.

Gonadotropin-releasing hormone agonist that causes a transient increase in pituitary LH secretion leading to a rise in testosterone levels. However, continuous infusion of this drug suppresses LH release and leads to decrease in testosterone production

Leuprolide (DHT levels will follow testosterone since this drug has no effect on 5-alpha reductase)

Muscle innervated by C3 & C4 cervical nerves and dorsal scapular nerve. Acts to elevate the scapula & raise the medial border (inferior rotation of the glenoid)

Levator scapulae

Drug that is effective in suppressing ventricular tachyarrhythmias induced by rapidly depolarizing and ISCHEMIC myocardium because it binds to inactivated sodium channels and rapidly dissociates

Lidocaine (class IB antiarrhythmic drug)

Congenital long QT syndrome is mot often caused by genetic mutations in K+ channel protein that contributes to the outward rectifying K+ channel. A decrease in outward K+ current leads to prolongation of action potential duration and QT interval which predisposes to the development of?

Life threatening ventricular arrhythmias (torsades de pointes) that can cause palpitations, syncope, seizures, or sudden cardiac death

Spinal stenosis is an abnormal narrowing of the spinal canal occurring most commonly in the lumbar region. Compression of nerve roots result in LE pain, numbness/parasthesias, weakness. Symptoms are posture dependent as extension of the lumbar spine (standing, walking upright) further narrows the spinal canal whereas lumbar flexion (walking uphill, leaning on a stroller/shopping cart) relieves the pain. Thickening of what ligament contributes to the symptoms of spinal stenosis?

Ligamentum flavum Most common cause of spinal stenosis is degenerative arthritis of the spine which results in narrowing of the spinal canal due to intervertebral disc degeneration & herniation, ligamentum flavum hypertrophy (due to disproportionate load on the posterior aspect of spinal column), and osteophyte formation affecting the facet joints. These changes compress nerve roots and lead to neurologic symptoms.

Evaluation of a patient's access to guns is a key part of suicide risk assessment. Interventions to reduce suicide risk involve controlling factors that can be changed and include?

Limiting access to lethal methods (GUNS), decreasing stress, increasing psychosocial support, treating psychiatric illness and substance abuse, and managing pain

Term used to describe a pair of alleles inherited together in the same gamete (haplotype) more often or less often than would be expected given random pairing

Linkage disequilibirum (often occurs when the genes are in close physical proximity on same chromosome)

59 year old female with fever, skin flushing, and altered level of consciousness. BP is 50/20. HR is 120. If blood cultures are positive for E. coli, what bacterial factor is most likely responsible for her condition?

Lipid A Gram negative sepsis is caused by the release of LPS by bacterial cells during division of by bacteriolysis. LPS IS NOT ACTIVELY SECRETED BY BACTERIA. LIPID A IS THE TOXIC COMPONENT OF LPS which causes activation of macrophages leading to the widespread release of IL-1 and TNF-alpha which cause the signs and symptoms of septic shock: fever, hypertension, diarrhea, oliguria, vascular compromise, DIC K1 capsular antigen: E. coli strains that cause neonatal meningitis O antigen: cell wall outer membrane polysaccharide antigen used to classify gram negative bacteria H antigen: heat labile protein of enterobacteriaceae

64 year old male presents with difficulty walking. Reports experiencing muscle cramps in his right thigh after walking about one block on level ground. Cramps quickly subside with rest. Also recently noticed decreased sexual performance. PMH of MI and carotid endarterectomy. Most likely cause of patients symptoms?

Lipid filled intimal plaque Intermittent muscle pain reproducibly caused by exercise and relived by a brief period of rest defines claudication. Claudication is almost always the result of atherosclerosis of larger, named arteries. The obstruction of blood flow in these arteries is the result of fixed stenotic lesions produced by atheromas, which are lipid filled intimal plaques that bulge into the arterial lumen. The stenoses prevent sufficient increases in blood flow to exercising muscles, resulting in ischemic muscle pain. Aortoiliac atherosclerosis can diminish blood flow to the internal pudendal branches of the internal iliac artery, making it difficult to sustain an erection

An insoluble pigment composed of lipid polymers and protein-complexed phospholipids & is considered a sign of 'wear and tear' or aging. Yellow, brown finely granular perinuclear pigment that is the product of free radical injury and lipid peroxidation. Commonly seen in the heart and liver of aging or cachectic, malnourished patients

Lipofuscin

Patient dies of MI. On autopsy there are two 5-6 mm cavities in the deep structures of her brain filled with clear fluid. Process most likely responsible for brain findings?

Lipohyalinosis or microatheroma formation with small vessel occlusion Lacunar infarctions are small (less than 15mm) cavity infarctions that result from small vessel occlusion (via lipohyalinosis or microatheroma formation) in the penetrating vessels (lenticulostriate arteries) supplying the deep brain structures. Liquefactive necrosis and the formation of a small fluid filled cavity are the result of the occlusion. Uncontrolled HTN and DM are major risk factors.

Neisseria meningitides can cause rapid onset sepsis and circulatory collapse in perviously healthy young individuals. What virulence factor in the pathogens outer membrane is the major underlying cause of the disease severity?

Lipooligosaccharide (endotoxin that binds to TLR4 on monocytic & dendritic cells & triggers release of inflammatory cytokines leading to endothelial damage, capillary leakage, and hemorrhagic necrosis) NOTE: the capsular polysaccharide of N. meningitides is not the direct cause of meningioccocal shock, it just helps the bacteria resist phagocytosis & complement activation

Lethal tissue ischemia causes coagulative necrosis (digestive enzymes denature before they can significantly disrupt tissue integrity) in most tissues, including the myocardium,. The exception is ischemic cell death in the CNS which causes what type of necrosis?

Liquefactive

59 year old man being treated for hyperhcolesterolemia, DM, and HTN comes for follow up 3 weeks after starting a new medication. Blood work shows increase in K+ from 4.8 to 5.2 and a creatinine elevation from 1.2 to 1.6. Administration of what drug is most likely responsive for his change in renal function?

Lisinopril (or other ACE inhibitor) Common side effects of ACE inhibitors include decreased GFR, hyperkalemia, and cough. Angioedema is a rare, life threatening side effect. ACE inhibitors prevent the efferent arteriole from constricting more than the afferent arteriole, decreasing GFR & it is expected for GFR to decrease in all patients initially (not a concern unless its more than 30% decrease).

Term used to describe congenital absence of gyri

Lissencephaly

53 year old man with difficulty breathing and increasing fatigue. SOB at night and has been sleeping in a recliner to help relieve dyspnea. PMH includes HTN and hyperlipidemia. Suffered MI two months ago and before then smoked 1PPD for 25 years. Low frequency diastolic heart sound heart shortly after second heart sound when patients lies in the left lateral decubitus position is heard on cardiac exam. Cardiac imaging shows hyperkinesis of LV free wall. Most likely to accentuate his abnormal auscultation finding?

Listening at the end of expiration (makes sound more audible by decreasing lung volume and bringing heart closer to chest wall) The third heart sound (S3) is a low frequency heart sound occurring during early diastole after S2. Left ventricular gallops (S3 and/or S4) are best heart with the bell of the stethoscope over the cardiac apex while the patient is in the left lateral decubitus position at end expiration

Relatively common food borne illness associated with outbreaks after the consumption of contained food, particularly processed meats and dairy products. Healthy patients may develop gastroenteritis; patients with impaired cell mediated immunity are at risk for invasive infection (sepsis, meningoencephalitis) and infection in pregnancy is associated with adverse fetal outcomes (fetal death, premature birth)

Listeria

Two drugs that thyroid function should be checked when using?

Lithium & Amiodarone

Patient being treated for bipolar disorder complains of excessive thirst and urination. Mechanism?

Lithium induced diabetes insipidus is the result of lithium's antagonizing effect on the action of vasopressin on principal cells within the collecting duct system

Verapamil affects cardiac contractility and is useful in controlling systemic hypertension, but has little effect on skeletal muscle. What property of skeletal muscle is responsible for its resistance to Verapamil?

Little dependence on extracellular Ca2+ influx Contraction initiation in cardiac and smooth muscle cells is dependent on extracellular Ca2+ influx through L-type Ca2+ channels, which can be prevented with CCB (Verapamil). Skeletal muscle is resistant to Ca2+ channel blockers as Ca2+ release by the sarcoplasmic reticulum is triggered by a MECHANICAL interaction between L-type and RyR Ca2+ channels (vs. calcium induced calcium release in cardiac and smooth muscle cells)

Infant born to a 27 year old immigrant is evaluated for hearing loss. PE demonstrates white pupils and a continuous machinery murmur over the left infraclavicular area. What preconception intervention would have prevented infants condition?

Live attenuated vaccine Congenital rubella syndrome is predominantly characterized by neonatal defects in the head (microcephaly, mental retardation), eyes (cataracts), ears (deafness), and heart/cardiovascular system (PDA, peripheral pulmonic stenosis). To decrease incidence of this syndrome, CDC recommends vaccination of children 12-15 months and again at 4-6 years old and nonpregnant females of childbearing age who lack serum antibody against rubella with live, attenuated rubella virus vaccine

Metabolic changes in what organ is most responsible for a patient's recovery from hypoglycemia when given glucagon?

Liver Glucagon increases serum glucose by increasing hepatic glycogenolysis and gluconeogenesis. Glucagon also stimulates insulin secretion from the pancreas (so glucose can be taken up by insulin sensitive tissues). Unlike epinephrine, glucagon does NOT have a significant effect on glucose homeostasis in the skeletal muscle, adipose tissue, and renal cortex

47 year old patient who has been on systemic glucocorticoids for most of the past year. What tissue is most likely to have the largest increase in protein synthesis in response to her long term therapy?

Liver Glucocorticoids are predominantly catabolic, causing muscle weakness, skin thinning, impaired wound healing, osteoporosis, and immunosuppression. However, they increase hepatic gluconeogenesis and glycogenesis. This, along with the peripheral antagonism of insulin, contributes to the development of hyperglycemia

Prevents in early childhood with mild hypoglycemia, ketosis, and hepatomegaly. Does not affect skeletal muscles and shows an excess of NORMALLY structured glycogen

Liver glycogen phosphorylase deficiency (Hers disease)

What test should be obtained before starting a patient on statin therapy?

Liver transaminase levels Repeat levels if symptoms of hepatic injury occur

44 year old man hospitalized for cholecystitis is being evaluated for anxiety and agitation. Underwent cholecystectomy without complications. Two days after admission developed anxiety, tremulousness, severe agitation, and verbal aggression. Has been drinking 8 beers daily for last several years. Exam shows tremors bilaterally. Most appropriate pharmacotherapy?

Long acting benzodiazepine (diazepam, chlordiazepoxide) Benzos substitute for the action of alcohol on the GABA receptors are are indicated for treatment of alcohol withdrawal. Note: benzos without active metabolites (lorazepam, oxazepam, temazepam) are preferred in patients with advanced liver disease

3 year old boy with high fevers and malaise for the past 4 days. Began limping yesterday and seems to refrain from using his right leg. Temp is 103. Passive ROM does not elicit pain and no joint effusion is seen but the patient refuses to bear weight on his RLE. Scintigraphy is most likely to reveal increased focal radio tracer uptake in what area?

Long bone metaphysis Hematogenous osteomyelitis is most common in children and usually affects the metaphysics of long bones due to the slow blood flow and capillary fenestrae in this region. Without proper treatment the infection can progress to chronic suppurative osteomyeliyis (a condition in which necrotic bone, sequestrum, serves as a reservoir for infection and becomes covered by a poorly conducted shell of new bone called involucrum). Epiphysis is usually spared unless acute infection isn't treated in which cause it can spread into the epiphysis and adjacent joint, resulting in septic arthritis. Note: vertebral body is the most common location for hematogenous osteomyelitis in adults

Dissection of the axillary lymph nodes can injure what nerve which results in weakness of the serrates anterior with winging of the scapula and impaired abduction of the shoulder past the horizontal

Long thoracic nerve

Most potent diuretics used as first line therapy for rapid relief of symptoms in patients with acute decompensated heart failure?

Loop diuretics (furosemide, torsemide, bumetanide)

75 year old male with an acute onset of a pruritic rash after eating strawberries. Has history of environmental allergies. Family reports he lives alone and his functional status has been declining. Walks with cane, has poor vision, and is frequently forgetful. Also has occasional dizziness when standing up and history of frequent falls. Most appropriate medication to treat his acute allergic reaction symptoms?

Loratadine or Cetirizine First generation antihistamines (hydroxyzine, promethazine, clorpheniramine, diphenhydramine) can cause significant side effects due to blockade of cholinergic, alpha-adrenergic, and serotonergic pathways. They should be avoided in older patients with cognitive or functional impairments. Newer generation antihistamines (loratadine, cetirizine) have minimal side effects and are preferred for these patients

Nephrotic syndrome is a hyper coagulable state. Sudden onset abdominal or flank pain, hematuria, and left sided varicoceles suggest renal vein thrombosis (known complication of nephrotic syndrome). What is responsible for the thrombotic and thromboembolic complications of nephrotic syndrome?

Loss of anticoagulant factors, especially antithrombin III

43 year old patient with amenorrhea & elevated prolactin levels is diagnosed with pituitary adenoma. Patient refuses medical treatment. If left untreated, patient is at risk of developing?

Loss of bone mass Hyperprolactinemia suppresses GnRH, which leads to reduced estrogen in women. Low estrogen levels are a risk for accelerated bone loss

Lesions of glossopharyngeal nerve (CN IX) result in?

Loss of gag reflex (afferent limb), sensation in the upper pharynx, posterior tongue, tonsils, and middle ear cavity, and loss of taste on the posterior 1/3 of the tongue

65 year old with right lower abdominal and groin pain and difficulty walking for the past week. Climbing stairs has been very difficult and he has fallen several times due to 'knee buckling'. PMH of DM, HTN, AF. Takes Warfarin. On PE, the right patellar reflex is decreased. Abdominal CT scan reveals a large fluid collection in the retroperitoneum lying anterior to the psoas muscle. What additional finding would you most expect on physical exam?

Loss of sensation on the anterior aspect of the right thigh CT findings are consistent with a spontaneous retroperitoneal hematoma most likely secondary to Warfarin use. Femoral nerve descends through the fibers of the psoas major muscle and emerges laterally between the psoas and iliacus muscle. It then runs beneath the inguinal ligament to the thigh. Lesions of the femoral nerve can occur due to trauma, nerve compression, stretch injury, or ischemia. Patients develop weakness of the quadriceps muscle, loss of patellar reflex, and loss of sensation over the anterior and medial thigh and leg.

Adverse effects of secondhand smoke exposure include increased risk of?

Low birth weight, asthma, middle ear disease, SIDs. Up to half of all SIDS cases are due to tobacco exposure, likely from impaired arousal and abnormal cardiovascular responses to stimuli

Patients with congenital adrenal hyperplasia due to 21 hydroxylase deficiency have defective conversion of 17 hydroxyprogesterone to 11 deoxycortisol which impairs cortisol synthesis. Decreased cortisol levels are sensed by hypothalamus and caused increase in ACTH secretion by the anterior pituitary. This results in stimulation of adrenal cortex and androgen overproduction. Treatment of congenital adrenal hyperplasia?

Low doses of exogenous corticosteroids to suppress excessive ACTH secretion & reduce stimulation of adrenal cortex (decreases androgen production)

Bartonella henselae causes cat scratch disease in immunocompetent patients, bacillary angiomatosis in immunocompromised patients, and culture negative endocarditis. Cat scratch disease is characterized by?

Low grade fever, lymphadenopathy proximal to cat scratch (axillary is most common) and a self limited course

Heparins are ideal anticoagulants for most patients with thromboembolic disease in pregnancy as they do not cross the placenta and therefore the risk of fetal bleeding or teratogenicity is low. Preferred heparin therapy?

Low molecular weight heparin (enoxaparin) with transition to unfractionated heparin at term LMWH is preferred as it has a relatively long half life and doesn't require routine lab testing, but it can't be used in patients with severe renal insufficiency. Unfractionated heparin has a has a short half life and requires frequent lab draws, but it can be used in patients with renal insufficiency. Unfractionated heparin is used in place of LMWH at term (37 weeks gestation) as it can be discontinued at the onset of labor to minimize hemorrhagic risk

The risk of venous thromboembolism (PE or DVT) in hospitalized patients can be greatly reduced with the administration of prophylactic anticoagulation, usually with?

Low molecular weight heparin (ex: enoxaparin) Preferred in most patients due to proven efficacy and relatively low risk of adverse effects

56 year old man in the ICU with a fever. Has history of HIV and hematologic malignancy. Last chemo was 2 weeks ago. Blood cultures grow pseudohyphae producing yeast species with an ability to form germ tubes. Deficiency of what cell type most likely contributed to the fungemia in this patient?

Low neutrophil count Local defense against Candida is performed by T cells, whereas systemic infection is prevented by NEUTROPHILS. For this reason, localized candidiasis is common in patients who have HIV, but neutropenic individuals are more likely to have the systemic form of the disease

Hemoglobin S aggregates in the deoxygenated state. HbS polymers form fibrous stands that reduce RBC membrane flexibility and promote sickling. Sickling occurs under conditions associated with anoxia including?

Low pH (high H+) and and high levels of 2,3 biphosphoglycerate. Organs with high metabolic demands (brain, muscles, placenta) promote sickling by extracting more oxygen from the blood (oxygen unloading). The inflexible erythrocytes predispose to microvascular occlusion and micro infarcts

Delirum may manifest as acute changes in cognition and behavior. Medication of choice to treat the behavioral and psychotic manifestations of delirium?

Low-dose antipsychotics (Haloperidol)

Sudden upward stretching on the arm at the shoulder (such as grabbing a tree branch) can damage what structure, resulting in finger clumsiness with total claw hand deformity

Lower trunk of the brachial plexus (carries nerve fibers from C8 & T1 spinal levels that are responsible for innervating all of the intrinsic muscles of the hand via median and ulnar nerves) Note: involvement of T1 nerve root proximal to sympathetic trunk can cause contaminant Horner's syndrome (ipsilateral ptosis, miosis, anhidrosis)

A fall on an outstretched hand can cause lunate dislocation. Location of lunate?

Lunate is the more medial of the two carpal bones that articulate with the radius. It lies immediately medial to the scaphoid bone. Pneumonic: So long to pinky, here comes the thumb

22 year old man involved in a fight. BP is 88/60 and HR is 114. PE shows a penetrating right sided stab wound just above the clavicle between the midclavicular and lateral sternal lines. Course of the wound appears to be perpendicular to the skin. Structure most likely injured?

Lung pleura Patient likely has tension pneumothorax. Increased air accumulates in pleural space, lungs and mediastinum deviate to opposite side of chest. Increased pressure within chest causes decreased systemic venous return to heart, leading to decreased CO. Signs of tension pneumothorax include tachycardia, hypotension, tachypnea, hyperemia, and absence of breath sounds and hyper resonance to percussion on the affected side. Treat with emergent needle thoracoctomy of chest tube The lung apices extend above the level of the clavicle and first rib through the superior thoracic arpeture. Penetrating injury in this area may lead to pneumothorax, tension pneumothorax, or hemothorax

Dermatomyositis is an autoimmune inflammatory disease that affects the muscles and the skin. Characterized by proximal muscle weakness as well as a helpicotrope rash in the periorbital area and cheeks & Gottron's papule which are raised erythematous plaques over the joints and bony prominences of the hands. Muscle biopsy is diagnostic and show mononuclear perimysial infiltrates, perifasciular atrophy, and patchy necrosis. Has a strong association with malignancy particularly?

Lung, colorectal, ovarian, and non-hodgkin lymphoma

Graph depicts a vascular bed in which the arterial/arteriolar resistance decreases as the tissue oxygen content increases. Most likely organ?

Lungs The pulmonary vascular bed is relatively unique in that tissue hypoxia results in vasoconstrictive response. Such hypoxic vasoconstriction occurs in the small muscular pulmonary arteries to divert blood flow away from under ventilated regions of the lung toward better ventilated areas. This relationship between tissue hypoxia and vascular resistance is reversed in the systemic circulation, ensuring that hypoxic tissues receive increased blood flow. Mechanism underlying hypoxic pulmonary vasoconstriction involves an increase in pulmonary artery smooth muscle cytosolic Ca2+ levels secondary to hypoxia induced modulation of K+ channels and/or decreased production of ROS.

54 year old man presents with 2 months of progressive, generalized weakness and easy fatiguability. He reports abdominal discomfort and early satiety. On exam he has pallor, abdominal dissension, and massive splenomegaly with the spleen crossing the midline. Peripheral blood count shows pancytopenia. Bone marrow is attempted but no marrow can be aspirated. Most likely description of lymphocytes in this patient?

Lymphocytes with cytoplasmic projections Hairy cell leukemia is an indolent B cell neoplasm predominantly found in middle aged men and characterized by bone marrow failure and infiltration into the reticuloendothelial system, causing massive splenomegaly. Other typical features include 'dry tap' & presence of lymphocytes with cytoplasmic projections

Initial infection is associated with a painless genital ulcer, but patients typically present after disease has spread to regional lymph nodes resulting in painful lymphadenitis. Very rare in USA, occurs mostly in tropical and subtropical regions

Lymphogranuloma venereum caused by L serovars of Chlamydia trachomatis

Autosomal dominant disease caused by defective DNA mismatch repair/mutations in MSH2 & MLH1?

Lynch syndrome (hereditary nonpolyposis colon cancer)

Pyruvate dehydrogenase complex deficiency is an inherited inborn error of metabolism causing lactic acidosis & neurologic defects. Cannot convert pyruvate to acetyl-CoA resulting in shunting of pyruvate to lactic acid. Metabolism of what AA can provide energy in form of acetyl-CoA without increasing lactate production?

Lysine & leucine (only purely ketogenic AA) Note: Ketogenic AA metabolism generates the ketone body precursor acetyl Co-A. Lysine & leucine cannot be metabolized to pyruvate so won't increase lactic acid!

65 year old male suffered from MI, extensive atherosclerosis and intermittent claudication. Autopsy findings of the brain reveal a cystic cavity surrounded by gliosis. What most likely produced this brain lesion?

Lysosomal digestion of the tissue Liquefactive necrosis is characterized by complete digestion and removal of necrotic tissue with formation of a cystic cavity. Hypoxic CNS injury is often followed by liquefactive necrosis. Note: abscess formation due to bacterial or fungal infection is another example of this type of necrosis

Phosphorylation of specific mannose residues to ensure proper transit through the Golgi apparatus is required for proteins destined for what cellular location?

Lysosome

Vinca alkaloids & taxanes are specific to what cell cycle phase?

M-phase

Occlusion of what artery typically leads to contralateral hemiplegia of the face and upper limb with relative preservation of the lower limb

MCA. If the occluded MCA is in the dominant (usually left) hemisphere, aphasia may occur too

40 year old man with refractory peptic ulceration. Labs show moderate iron deficiency anemia, occult blood int stool, and markedly elevated gastrin level. Upper GI endoscopy shows multiple ulcer in duodenum. multi-organ disease is suspected. Patent should be queried for a family history of what disorder?

MEN type 1 (genetic defect involves MEN 1 gene and results in hyperparathyroidism, endocrine pancreatic tumors, and pituitary tumors)

Characterized by medullary carcinoma of the thyroid, pheochromocytoma, marfanoid habits, and oral and intestinal mucosal neuromas

MEN2B

Structures arising from neural crest?

MOTEL PASS: melanocytes, odontoblasts, tracheal cartilage, enterocromaffin cells, laryngeal cartilage, parafollicaular C cells of the thyroid, adrenal medulla & all ganglia, Schwann cells, spiral membrane

68 year old woman with ESRD on intermittent dialysis presents with back pain that has been progressively worsening and associated with malaise and fatigue. No numbness or weakness. Treated several weeks ago for staphylococcal bacteremia associated with the dialysis catheter. Fever and tenderness over upper lumbar vertebrae without overlying skin changes on exam. Next best step in management?

MRI of spine Vertebral osteomyelitis should be suspected in patients with new or worsening back pain, fever and recent endocarditis or bacteremia (especially S. aureus). Should also be suspected if there are new neurologic findings and fever with or without back pain. MRI is preferred for diagnosis

38 year old man evaluated for bloody stools, weight loss, anemia. Father died of colon cancer at age 40. Younger sister suffers from endometrial cancer. Colonoscopy shows right ulcerative colon mass which reveals adenocarcinoma on biopsy. No polyps noted. Mutation in which gene is most likely responsible for patient's condition?

MSH2 Hereditary nonpolyposis colon cancer (Lynch syndrome) is an autosomal dominant genetic predisposition to colon cancer at a young age. Family history reveals high incidence of colon cancer and occasionally extra intestinal cancers (endometrial CA) in first degree relatives. Mutations in DNA mismatch repair genes (MSH2, MSH1) are responsible for HNPCC

Kidney stone more common in women, usually due to UTI caused by urease positive organisms (Proteus, Klebsiella) that have potential to form staghorn calculi?

Magnesium ammonium phosphate (struvite)

Patient's blood cultures grow MRSA and is started on a one daily IV antibiotic. A week later patient is found to have elevated creatine phosphokinase level. Symptoms improve within days of stopping the antibiotic. Agent used to treat this patient's infection affects what bacterial process?

Maintenance of membrane potential Daptomycin is a lipopeptide antibiotic with activity limited to gram positive organisms, including MRSA. It causes depolarization of the bacterial cellular membrane and inhibition of DNA, RNA, and protein synthesis. Daptomycin is associated with increased creatine phosphokinase levels and increased incidence of myopathy Vanomycin & Linezolid are two other antibiotics used for treatment of MRSA. Vancomycin- bind to D-alanyl-D-alaine portion of cell wall precursors, blocking glycopeptide polymerization & inhibiting bacterial cell wall synthesis. FIRST LINE for MRSA bacteremia. Adverse effects are nephrotoxicity & red man syndrome Linezolid- inhibits bacterial protein synthesis by binding to 50S subunit. Adverse effects are thrombocytopenia, optic neuritis, high risk of serotonin syndrome

Severe subtype of unipolar major depression characterized by symptoms meeting the criteria for a major depressive episode and the presence of delusions and/or hallucinations

Major depressive disorder with psychotic features

Patients with classic, salt wasting 21 hydroxylse deficiency have deficient cortisol and aldosterone synthesis combined with adrenergic androgen overproduction. Males and females present differently. Presentation in each sex?

Males- normal genitalia; present 1-2 weeks after brith with vomiting, hypotension, hyponatremia, hyperkalemia Females- present at birth with ambiguous genetalia

50 year old woman with periodic reddening of her skin that involves mainly her face/neck and is accompanied by mild warmth. Also has persistant watery diarrhea and abdominal cramping for past several months. PE shows several purple, vascular lesions surrounding her nose. Urinary excretion of 5-HIAA over 24 hours is increased. Abdominal imaging shows tumor in small intestine. What is most likely responsible for her symptoms?

Malignant carcinoid (extraintestinal metastasis of intestinal carcinoid tumors is required to produce carcinoid symptoms) Carcinoid tumors confined to the intestine DO NOT cause carcinoid syndrome as their secretory products are metabolized by the liver before enter the systemic circulation. Intestinal carcinoids that metastasize to the liver and extraintestinal carcinoids release vasoactive substance that avoid first pass metabolism resulting in carcinoid syndrome

Severe infection causing exquisite ear pain and drainage most commonly seen in elderly diabetic patients. Granulation tissue seen in ear canal with intact TM.

Malignant otitis externa

Purposeful falsification of symptoms for obvious external benefit

Malingering

Tear in gastric mucosa near the gastroesophageal junction. Typically results from repetitive forceful vomiting in alcoholics which can cause metabolic alkalosis

Mallory-Weiss tear

Legan determination that involves situations in which the treatment provided is below the accepted standard of practice and has resulted in injury or death to the patient. Not a category of medical error but rather refers to the consequence of many different types of errors that result in harm

Malpractice

Thiamine (vitamin B1) deficiency acutely leads to Wernicke encephalopathy (ataxia, nystagmus, ophthalmoplegia, anterograde amnesia) and chronically leads to Korsakoff psychosis (anterograde and retrograde amnesia, apathy, lack of insight, confabulation). The neural structure most frequently affected in patients with Wernicke encephalopathy is?

Mamillary body (part of the Papez circuit- involved in cortical control of emotion and memory). Thiamine is a coenzyme for PDH and deficiency results in the brains inability to properly metabolize glucose and turn it into energy. Alcoholic or malnourished patients should revise IV thiamine supplementation BEFORE IV dextrose because giving dextrose without prior thiamine can precipitate Wernicke encephalopathy by causing an acute drop in their thiamine level.

What passes through the foramen ovale?

Mandibular branch of trigeminal nerve (CN V3)

Tempomandibular disorder can arise from problems involving the TMJ, muscles of mastication, and nerves that supply the jaw. TMD can presents with both otologic symptoms and jaw pain because of what nerve that supplies the middle ear and the muscles of mastication?

Mandibular division of the trigeminal nerve (V3)

What nerve is responsible for general sensation anterior 2/3 of tongue

Mandibular divsion of trigeminal nerve (V3)

Restriction of branched chain amino acids (valine, leucine, isoleucine) is used to treat what two conditions?

Maple syrup urine disease (branched chain alpha ketoacid dehydrogenase deficiency) & propionic acidemia (propionyl-CoA carboxylase deficiency)

Defects in fibrillin-1 gene results in abnormal fibrillin protein which is a major component of microfibrils that form sheath around elastin. What disorder? Mode of inheritance?

Marfan syndrome. Autosomal dominant.

Agressive, ulcerating squamous cell carcinoma that presents in an area of previously traumatized, chronically inflamed, or scarred skin. Malignant transformation often occurs long after the initial trauma, usually more then 10 years later

Marjolin's ulcer

Strongest risk factors maternal to child transmission of HBV infection?

Maternal viral load and HBeAg status (virtually all infants born to HBeAg positive mothers will be infected perinatally) Infants born to HBeAg positive mothers have a high risk of acquiring prenatal hepatitis B virus infection. Infected neonates have high levels of HBV replication and are HBeAg positive & are at very high risk of chronic infection and progression to cirrhosis and hepatocellular carcinoma, but are usually asymptomatic or have only mild elevated LFTs. Administration of HBV vaccine and immunoglobulin ASAP after delivery can help prevent chronic infection.

Cellular extract containing purified nucleic acids is incubated along with short sequences of repeated deoxythymidine residues fixed to latex beads. What nucleic acid would bind strongest to the latex beads?

Mature mRNA (poly-A tail on mature mRNA is most likely to bind to the beads because the adenine residues will form complementary base pairs with repeated deoxythymidine resides fixed to beads) Poly-A tail is added as a post-transcriptional modification (not transcribed from DNA) downstream of the consensus sequence (AAUAAA) located near 3' end of the mRNA molecule; it protects mRNA from degradation within cytoplasm after it exits the nucleus

A linear skull fracture at the junction of the frontal, parietal, temporal, and spenoid bones (pterion) is noted on head CT scan. Branch of what artery is most likely severed?

Maxillary artery Middle meningeal artery is a branch of the maxillary artery, which enters the skull at the foramen spinosum and courses intracranially deep to the pterion. Skull fractures at this site may cause laceration of the middle meningeal artery leading to an EPIDURAL hematoma (must be treated rapidly as it can lead to elevated ICP, brain herniation, death).

What nerve passes through foramen rotundum?

Maxillary division of trigeminal nerve (CN V2)

Characterized by a triad of fibrous dysplasia of the bone, endocrine abnormalities (precocious puberty), and cafe-au-lait spots that are large, unilateral with an irregular border. Results from an activating station in the G protein/cAMP/adenylate cyclase signaling pathway

McCune Albright syndrome

Have amino acid substitution at the C terminal end of the B chain & a rapid onset of action, mimicking physiologic postprandial insulin secretion

Mealtime insulin analogs (lispro, aspart, glulisine)

True diverticulum that consists of all layers of the intestinal wall (mucosa, submucosa, muscularis). Remnant of the omphalomesenteric (vitelline) duct. Connected to ilium & located 2 feet from ileocecal valve. Often contains acid secreting ectopic gastric tissue which may cause ulceration of adjacent mucosa and lower GI bleeding

Meckel's diverticulum

Drain via the medial track which runs along the long saphenous vein up to the superficial inguinal lymph nodes bypassing the popliteal nodes so ONLY INGUINAL lymphadenopathy occurs

Medial lower extremity lesions

Syndrome characterized by contralateral hemiparesis (damage to lateral corticospinal tract, contralateral hemisensory loss (dorsal column medial lemniscal pathway) & ipsilateral tongue paralysis (hypoglossal nucleus)

Medial medullary syndrome (ischemic stroke syndrome caused by occlusion of anterior spinal artery)

Location of hypoglossal nucleus?

Medially on the floor of the fourth ventricle at the level of the medulla Nerve exits at the rostral end of the preolivary sulcus

In a positively skewed distribution the mean is the most shifted in the positive direction (to the right) followed by the median and then the mode. In such a situation what component reflects the central tendency the best?

Median reflects central tendency better then the mode does in positively skewed distribution

26 year old smoker with persistent dry cough presents complaining of recent onset headaches and dyspnea. Also complains of having a puppy face for 2 weeks. PE reveals symmetrical facial swelling and conjunctival edema. No shoulder pain. Pupils are equal, round, and reactive to light. Dilated vessels are seen over neck and upper trunk. His condition is most likely cause by what?

Mediastinal mass Intrathoracic spread of bronchogenic carcinoma may lead to compression of the SVC causing impaired venous return from the upper part of the body. Symptoms include dyspnea, facial swelling, and dilated collateral veins in the upper trunk. Headache, dizziness, and confusion can occur due to cerebral edema and elevated ICP. Lung CA followed by non-Hodgkin lymphoma are most common causes of SVC syndrome. Note: SVC can sometimes occur with Pancoast (superior sulcus) tumors that arise at the lung apex, but shoulder pain (due to compression of brachial plexus) and Horner's syndrome are more frequent manifestations

Patient with HTN started on valsartan and hydrochlorothiazide. No change in BP or plasma renin activity 3 weeks later. Most likely explanation?

Medication noncompliance (diuretics, & angiotensin II receptor blockers both increase plasma renin activity) Plasma renin activity is measure of amount of angiotensin I generated per unit of time. Antihypertensives block the RAAS pathway causing feedback increase in serum renin. Diuretics decrease RBF which increases renin and activates RAAS. Low Na+ also increases renin.

Elderly woman with dementia has recently moved to nursing home and had several falls. Work up for syncope is negative. Medications include amlodipine, ibuprofen, donepizil, diazepam. Most effective strategy in managing this patient's fall risk?

Medication review Medication related falls are a common problem in elderly nursing home patients. Optimal management should include a careful medication review and discontinuation of agents associated with increased fall risk

Condition characterized by lethargy, seizures, & hypoketotic hypoglycemia following period of fasting

Medium-chain acyl-CoA dehydrogenase deficiency

What nephron segment responds to vasopressin by increasing absorption of a specific solute that is important for generating a high medially concentration gradient?

Medullary segment of the collecting duct ADH acts on the medullary segment of the collecting duct to increase urea and water reabsorption, allowing for the production of maximally concentrated urine. Secretion of urea into the thin loop of Henle allows urea to recirculate and concentrate in the tubular system, further increasing its contribution to the medullary concentration gradient

Characterized by medullary cysts lined by cuboidal or urothelial epithelium. Cystic dilation of the collecting ducts is diagnostic

Medullary sponge kidney

Thyroid cancel that appears histologically as polygonal or spindle shaped cells with slightly granular cytoplasm that stains for calcitonin. Extracellular amyloid deposits consisting of calcitonin polypeptide may be seen. Component of MEN 2A & 2B

Medullary thyroid cancer

MEN2B is characterized & caused by?

Medullary thyroid cancer (enlarging thyroid nodule with elevated calcitonin level), pheochromocytoma (paroxysmal HTN, flushing, diaphoresis, headaches), mucosal neuromas, and marfanoid habitus (arm span greater than height, long fingers, joint laxity). Caused by germline mutations in RET proto-oncogene

34 year old with 6 days of fatigue and intermittent fevers who recently returned from Central Africa. Examination of peripheral blood smear with Giemsa staining reveals RBCs with multiple small rings called trophozoites & banana shaped gametocytes. Best treatment for this patient?

Mefloquine RBC's filled with multiple, smaller rings suggest Plasmodium infection (malaria). Chloroquine is treatment of choice for malaria acquired in areas known to have chloroquine sensitive organisms. Many African species are chloroquine resistant & thus should be treated with Mefloquine. P. vivax and P. ovale have dormant liver forms (hyponozoites) that require additional Primaquine therapy. Atovaquone-proguanil and artemisinins are also useful in chloroquine resistant organisms.

Vitamin B12 is obtained through the diet solely from animal sources, which places strict vegans at risk for dietary deficiency. Deficiency takes years (4-5) to develop due to the large hepatic B12 reserve and presents with?

Megaloblastic anemia & potentially irreversible neurologic deficits (paresthesias, weakness, ataxic gait)

Deficiency of B12 or folate is associated with what type of anemia?

Megaloblastic, macrocytic

Chronic alcoholics are likely to develop what type of anemia?

Megaloblastic, macrocytic due to nutritional deficiency in folate (within months) or B12 (within years. Impaired synthesis or purine and pyrimidine bases!

Short acting glucose lowering medications that act by closing the ATO-dependent K+ channel in the pancreatic beta cell membrane, including depolarization and stimulating insulin release. Usually given with meals and can reduce postprandial excursions

Meglitinides (repaglinide, nateglinide)

10 year old girl with short stature, no breast buds, and no axillary or pubic hair. Has a low hairline, a short and wide neck, broad chest, and widely spaced nipples. Most likely underlying mechanism for patient's condition?

Meiotic nondisjunction Turner syndrome is a genetically heterogeneous condition most commonly due to meiotic nondisjunction during gametogenesis resulting in a missing X chromosome in all of the patient's cells (45, X). Results in a missing SHOX gene which normally promotes long bone growth. In a smaller number of patients, nondisjunction occurs during mitosis in early embryogenesis resulting in a missing X chromosome in only some cells (mosaic 45X, 46XX). A minority have both X chromosomes but one is structurally abnormal and missing some genetic material (46 XX with partial deletion of one X chromosome).

1 day old infant with flat facial profile, protruding tongue, small ears, and upslanting palpebral fissures, and harsh holosystolic murmur. Born 39 weeks to 36 year old woman who deferred prenatal screening. No family history of genetic or chromosomal disorders. What likely occurred prior to conception?

Meiotic nondisjunction of anaphase I (increased incidence with age greater than 35) Down syndrome is most commonly caused by maternal meiotic nondisjunction, a process by which the fetus receives 3 full copies of chromosome 21.

14 year old with borderline high PTH level, mild hypercalcemia, low urinary calcium, and normal vitamin D. Mutation of what type of receptor is responsible for his lab abnormalities?

Membrane bound receptor coupled with a G protein Calcium sensing receptors are G protein coupled receptors that regulate the secretion of PTH in response to changes in circulating Ca2+ levels. Familial hypocalcuric hypercalcemia is a benign autosomal dominant disorder caused by defective Ca2+ sensing receptors in the parathyroid glands and kidneys

Oculomotor dysfunction, ataxia, and confusion form the triad of wernicke syndrome. Most of these symptoms usually resolve after thiamine administration except for?

Memory loss Korsakoff syndrome is a complication of wernicke encephalopathy, the hallmarks of which are permeant memory loss and confabulation

Characterized by tinnitus, vertigo, and sensorineural hearing loss. Pathogenesis is related to an increased volume and pressure of endolymph in the vestibular apparatus

Meniere disease

Most common presentation of Cryptococcus neoformans (budding yeasts with thick capsule) infection?

Meningeoencephalitis Occurs in immunosuppressed and can be diagnosed by India Ink staining of CSF. Although lung infection occurs first, it is usually ASYMPTOMATIC. In cases where it is symptomatic, cryptococcal pneumonia is diagnosed by mucicarmine staining of the lung tissue and bronchoalveolar washings.

Nasopharyngeal specimen is obtained and cultured on a selective medium containing vancomycin, colistin, nystatin, and trimethoprim. The organism most likely to be isolated is typically associated with what condition?

Meningitis Thayer Martin medium is used to isolate pathogenic Neisseria species (N. meningitides, N. gonorrhoeae). It is a chocolate sheep blood agar that contains vancomycin to inhibit the growth of gram positive organisms, colistin and trimethoprim to inhibit gram negative bacteria (other than Neisseria) and nystatin to inhibit yeasts.

Heamophilus influenza type b (Hib) vaccine induces anticapsular antibodies that facilitate complement mediated phagocytosis of the bacterium which has greatly reduced the incidence of invasive disease cause by Hib. Infant not vaccinated would be more at risk for what diseases?

Meningitis, bacteremia, pneumonia, epiglottitis Note: polyribosylribitol phosphate capsule is conjugated with protein toxoid (tetanus, diphtheria) which induces T cell dependent immune response. Production of opsonizing anticapsular antibodies by B-lymphocytes allows for efficient phagocytosis of bacterium

Prevents hemorrhagic cystitis due to cyclophosphamide or isosfamide. Binds acrolein, the toxic metabolite formed by these agents

Mesna

Patient with breast cancer is treated with systemic chemotherapy. One month later she develops frequent urination, suprapubic pain, dysuria, and progressive hematuria. UA shows numerous red blood cells but no leukocyte esterase or bacteria. What substance could have prevented patients condition?

Mesna Hemorrhagic cystitis during therapy with cyclophosphamide or isofosfamide is caused by the urinary excretion of the toxic metabolite acrolein. It can be prevented by aggressive hydration, bladder irrigation, and administration of Mesna, a sulfhydryl compound that binds acrolein in the urine

Zinc containing enzymes that degrade the ECM, participate in normal tissue remodeling, and in tumor INVASION through the basement membrane and connective tissue

Metalloproteinases Note: tumor cells DETACH from each other by decreased E-cadherin & they ADHERE to basement membrane through increased laminin

Liklihood of atherosclerotic plaque rupture is related to plaque stability rather than plaque size or the degree of luminal narrowing. Plaque stability largely depends on the mechanical strength of the fibrous cap. Inflammatory macrophages in the intima may reduce plaque stability by secreting what substance?

Metalloproteinases (degrade ECM proteins such as collagen)

Oocyte is arrested in what stage of meiosis immediately prior to fertilization?

Metaphase of meiosis II Primary oocytes are completely developed in female embryos by the 5th month of gestation, at which point they are arrested in prophase of meiosis I. Normal menstrual cycle hormones stimulate the primary oocyte to resume differentiation & then the secondary oocytes are arrested in metaphase of meiosis II prior to fertilization.

Drug that inhibits hepatic gluconeogenesis & increases peripheral glucose utilization. Lactic acidosis is rare complication but has increased risk with underlying renal insufficiency

Metformin

2 day old girl in newborn nurse with persistent crying, tremors, tachypnea, sneezing, and diarrhea. She was born vaginally and was breast feeding well until several hours ago when she became tachypneic. Her mother has poorly controlled schizophrenia and didn't receive prenatal care. Mother also had a positive hepatitis C antibody test during postnatal lab testing. On PE, newborn has increased tone in all extremities. She is irritable during exam but quiets when swaddled. CXR shows normal lung fields. Most appropriate pharmacotherapy for the newborn's symptoms?

Methadone Neonatal abstinence syndrome presents with neurologic, gastrointestinal, and autonomic symptoms. Tremors, irritability, diarrhea, and poor feeding are classic findings. Opiate replacement therapy is treatment of choice; dose is titrated to patient's symptoms and the patient is slowly weaned over several weeks. At risk infants are those born to mothers with poor mental health, no prenatal care, and hepatitis C infection.

Homozygous mutation in the methylene tetrahydrofolate reductase gene that leads to decreased enzymatic activity. Patient most likely has impairment in converting homocysteine to which of the following?

Methionine Elevated levels of plasma homocysteine are an independent risk factor for thrombotic events. Homocysteine can be metabolized to methionine via remethylation or to cystathionine via transsulfuration. Hyperhomocysteinemia is most commonly die to genetic mutations in critical enzymes or deficiencies of Vitamin B12, B6, and folate

Methemoglobinemia presents with gray or blue colored skin, SOB, chocolate colored blood. Treatment?

Methylene blue (acts as an artificial electron transporter for reduction of methemoglobin through NADPH pathway)

26 year old minor league baseball player with comedonal and inflammatory nodular eruption on his face, chest, and back. Most likely cause of these findings?

Methyltestosterone use Androgens stimulate follicular epidermal hyperproliferation and excessive sebum production, promoting acne development. Anabolic steroid misuse is a known cause of acne, especially in competitive athletes. 4 major elements that underlie the pathogenesis of acne: 1) follicular epidermal hyperproliferation 2) excessive sebum production 3) inflammation 4) Propionibacterium acnes

Dopamine antagonist with pro kinetic and antiemetic properties that can be used to treat GU motility disorders (gastroparesis) and nausea/vomiting

Metoclopramide

Antibiotic that binds to DNA within susceptible cells, unraveling the chemical structure of DNA and breaking its strands causing protein synthesis to be inhibited and cell death to occur

Metronidazole

Side effects of what antibiotic are gastrointestinal (nausea, vomiting, crampy abdominal pain) and neurologic (dizziness, paresthesias) & when taken with alcohol causes a disulfuram-like reaction?

Metronidazole

Bacterial vaginosis is caused by disruption of the vaginal bacteria flora and is characterized by a 'fishy smelling' thin, grayish vaginal discharge and clue cells (epithelium cells covered with gram variable rods) on wet mount microscopy. Treatment?

Metronidazole and clindamycin

56 year old with a one week history of hemiplegia and expressive aphasia has CT scan that reveals an area of hypo density in his brain. Tissue section from the region is obtained. Histologic staining after staining for lipid is shown. Cells marked most intensely by the lipid stain represent what cell type?

Microglia During the first week after an acute stroke, CT will reveal ischemia area to be hypodense and poorly delineated from the surrounding tissue. Edema and loss of distinction of gray-white matter junction are also seen. Microglia move to the area of ischemic infarct 3-5 days after the onset of ischemia and phagocytize the fragments of neurons, myelin, and necrotic debris. The abundance of lipids seen in the cytoplasm of the microglia results from the extensive phagocytosis of the myelin breakdown products. A cystic space replaces the necrosis, and astrocytes (migrate to the area of necrosis within 2 weeks of injury) form a glial scar along the periphery. Neutrophils dont phagocytize myelin and will not stain for lipids!

Individuals who demonstrate increased activity of what specific intracellular enzyme are more susceptible to developing benz(o)pyrene-induced lung cancer?

Microsomal monooxygenase (metabolizes steroids, alcohol, toxins, and other foreign substances by rendering them soluble and easier to excrete but also converts pro-carcinogens to carcinogens capable of causing mutations in human DNA) Most chemical carcinogens enter the body in an inactive state and are then converted into active metabolites by the cytochrome P450 oxidase system. Individual susceptibility to chemical carcinogens depends on the activity of these P450 enzymes, which is genetically determined.

Reye syndrome occurs in children with febrile illness treated with salicylates (ASPIRIN). It consists of hepatic failure and encephalopathy. Characteristic histological finding of this syndrome?

Microvesicular steatosis of hepatocytes without inflammation & cerebral edema (hyperammonia causes cerebral edema)

54 year old patient with well-articulated, nonsensical speech & lack of language comprehension. Branch of what artery is most likely for condition?

Middle cerebral artery. Fluent aphasia is produced by lesion in Wernicke's area, located in auditory association cortex within posterior portion of superior temporal gyrus in dominant temporal lobe. MCA supplies both Broca's (superior division of MCA) and Wernicke's (inferior division of MCA) areas.

Thyroid is formed from evagination of the pharyngeal epithelium and descends to the lower neck & can reside anywhere along the thyroglossal duct's usual path including the tongue (lingual thyroid) due to failure of what?

Migration Note; lingual thyroid can lead to obstructive symptoms (dysphagia, dysphonia, dyspnea) during times of heightened thyroid stimulation (puberty, pregnancy)

What diuretics cause regression of myocardial fibrosis and improve ventricular remodeling?

Mineralocorticoid receptor antagonists (spironolactone, eplerenone) as they block the deleterious effects of aldosterone on the heart They are recommended in addition standard heart failure therapy (ACE inhibitors & beta blockers) as they improve survival in patients with CHF and reduced LV ejection fracture, but should NOT be used in patients with hyperkalemia (increase Na+ and H20 excretion and conserve K+ in renal tubules) or renal failure. Most common side effects are hyperkalemia & gynecomastia (spironolactone >>> eplerenone). Note: Loop diuretics improve symptoms of HF but do not provide survival benefit (improve morbidity but not mortality)

What parameter best correlates with the potency of an inhaled anesthetic?

Minimal alveolar concentration MAC is the concentration of the anesthetic in the alveoli that renders 50% of the patients unresponsive to painful stimuli (ED50). Potency is inversely proportional to the MAC- the lower the MAC, the more potent the anesthetic. MAC depends on body temperature and decreases with increasing age.

Alveolar concentration that prevents movement in 50% of patients exposed to noxious stimuli

Minimal alveolar concentration (Note: potent anesthetics have low MACs)

Type of bias that occurs when either the exposure of the outcome is not identified correctly

Misclassification bias Note: random misclassification bias affects all groups to the same extent (ex: using the wrong size BP cuff on all groups)

5 year old patient with symptoms of acute epiglottitis (rapidly progressing fever, severe sore throat, drooling, progressive airway obstructive, cherry red epiglottis, & stridor) most likely?

Missed a vaccination (Hib)

Hemoglobin S cause?

Missense mutation in which valine replaces glutamate in B-globin chain (decreases negative charge of hemoglobin molecule but not as much as Hemoglobin C)

Hemoglobin C cause?

Missense mutation resulting in glutamate residue being substituted by lysine in the B-globin chain; results in overall decrease in negative charge of hemoglobin molecule

Site of beta-oxidation of fatty acids, citric acid cycle, & carboxylation of pyruvate (gluconeogenesis)?

Mitochondria

Small circular DNA molecules that resemble a bacterial chromosome & codes for proteins, tRNA, and rRNA likely originates from what cellular structure?

Mitochondria

Mature erythrocytes are found that are unable to synthesize heme even though they contain detectable levels of cytoplasmic enzymes involved in heme synthesis. Lack of what cellular organelle explains this?

Mitochondria Maturing erythrocytes lose their ability to synthesize heme when they lose their mitochondria which are necessary for the first and final 3 steps of heme synthesis

An opening snap is an early diastolic sound heard after S2 in patients with what conditions?

Mitral or tricuspid stenosis. The S2 to opening snap interval is the time between the closure of the aortic valve and the abrupt halting of the leaflet motion during opening of a stenotic mitral valve; a shorter interval is indicative of more severe mitral stenosis

Holosystolic murmur at the apex with radiation to the axilla is consistent with?

Mitral regurgitation

Early and large V wave is the characteristic pressure tracing finding in patients with what condition?

Mitral regurgitation (due to markedly elevated left atrial pressure during left ventricular systole/atrial diastole)

Under normal circumstances, pulmonary capillary wedge pressure closely reflects left arterial and left ventricular end diastolic pressure. Condition that leads to an increase in LA pressure that is reflected as an elevated PCQP during pulmonary artery catheterization?

Mitral stenosis Note: left ventricular filling may be normal resulting in an increased pressure gradient between LA and LV during diastole (LA pressure sufficiently high to overcome resistance between LA and LV resulting in normal left ventricular end diastolic pressure)

Mid-systolic click is caused by sudden tensing of the chord tendineae as they are pulled taut by the valve leaflets ballooning into the LA. Timing of click varies with LV volume; occurs earlier in systole with physical maneuvers that decrease LV volume. It is characteristic of?

Mitral valve prolapse

24 year old healthy college student enrolls in research study investigating factors affecting blood oxygen content. Partial pressure go oxygen in the blood is measured as it flows through the circulation. What factor explains the decrease in partial pressure of O2 from the alveolar capillary blood to the systemic arterial blood?

Mixture with deoxygenated blood The partial pressure of O2 in the left atrium and ventricle is over than that in the pulmonary capillaries due to mixing of oxygenated blood from the pulmonary veins with deoxygenated blood from the bronchial circulation and thebesian veins (small cardiac veins that drain into LA)

An outlier is defined as an extreme and unusual observed value in a dataset. It can affect measures of central tendency (mean, median, mode) as well as measure of dispersion (standard deviation, variance). What tends to be resistant to outliers?

Mode (outliers are not likely to affect the most frequently observed value)

Guillain-Barre syndrome is caused by?

Molecular mimicry. Antecedent event such as infection provokes immune response that cross reacts with Schwann cells a myelin sheath of peripheral nerves

CD14 is surface marker of what?

Monocyte-macrophage cell lineage

Situations in which treatment doesn't require parental consent to encourage minors to seek medical treatment?

Most minors can consent to prenatal care, diagnosis or treatment of STI, contraception, and drug and alcohol rehabilitation without emancipation or parental consent or notification

Renal potassium excretion with high dietary K+ load

Most of the K+ filtered by the glomeruli is resorbed in the proximal tubule (65%) & loop of Henle (30%)- even in hyperkalemic states! The late distal and cortical collecting tubules are the primary sites for regulation of K+ concentration in urine. K+ depletion simulates alpha intercalated cells to reabsorb extra K+. Principal cells secrete K+ under conditions of normal or increased K+ load

Oral thrush, interstitial pneumonia, and severe lymphopenia during first year of life is consistent with?

Mother to child vertical transmission of HIV-1 (antiretroviral therapy during pregnancy reduces risk of perinatal transmission to 1-2% and is recommended in all pregnant women with HIV& should be continued as long as they are breastfeeding)

Fluoroquinolone used mainly for treatment of bacterial infections that cause exacerbation of COPD, sinusitis, and community acquired pneumonia

Moxifloxacin

Unresponsive patient with history of drug abuse presents with constricted pupils is given IV medication and regains consciousness 2 minutes later and states he wants to go home. Drug used to treat this patient has the greatest affinity for what receptor?

Mu Naloxone is a pure opioid receptor antagonist used to treat opioid intoxication/overdose. Although it binds to mu, kappa, and delta opioid receptors, it has the greatest affinity for mu receptors making it an ideal agent for treating opioid intoxication

Facial pain, headache, and black necrotic eschar in nasal cavity in patient with diabetic ketoacidosis is highly suggestive of?

Mucormycosis

Acute calculous cholecystitis is an acute inflammation of the gallbladder initiated by gallstone obstruction of the cystic duct. Subsequent steps in pathogenesis after obstruction include?

Mucosal disruption by lysolecithins, bile salt irritation of the luminal epithelium, prostaglandin release with transmural inflammation, gallbladder hypomotility, increased intraluminal pressure causing ischemia, and bacterial invasion. Presents with persistent RUQ pain, fever, and leukocytosis. Murphys sign is positive on exam. Symptoms are preceded by episodes of transient abdominal pain and nausea after fatty meals due to temporary obstruction of cystic duct

4 year old boy with decreased appetite, abdominal cramps, and diarrhea over the last 3 days. Temperature is 100.4. Stool is positive for fecal leukocytes and occult blood. Fecal cultures grow non lactose fermenting gram negative rods on MacConkey agar. The bacteria ferment glucose without gas production (produce acid), do not generate hydrogen sulfide when grown on triple sugar iron agar, and cannot replicate on refrigeration temperatures. What bacterial factor is most important during pathogenesis of this patient's disease?

Mucosal invasion Shigella is a non-motile, non-lactose fermenting organism that does not produce hydrogen sulfide when grown on triple sugar agar. Mucosal invasion of the M cells that overlie Peyer's patches is an essential pathogenic mechanism for Shigella. Shigella then escapes the phagosome and spreads laterally to other epithelial cells via actin polymerization

Outbreak of waterborne gastroenteritis reported in Latin America is caused by oxidase positive, gram negative, comma shaped rods that can survive on alkaline media. Stool microscopy in infected patients would demonstrate?

Mucus and some sloughed epithelial cells ('rice water stool') Organism is vibrio cholerae which secretes toxins that modify electrolyte handling by enterocytes but do not invade or cause cell death so no erythrocytes or leukocytes are found on stool microscopy. Toxin induces mucin ejection by goblet cells resulting in flecks of mucus and sloughed epithelial cells

Characterized by 46 XX female with variable uterine development and no upper vagina (short vagina). As the uterus is hypo plastic or absent, patients cannot menstruate (primary amenorrhea). Normal ovaries are present which secrete estrogen normally and enable regular development of secondary sexual characteristics (breast, pubic hair).

Mullerian aplasia (Mayer-Rokitansky-Kuster- Hauser syndrome) Note: all females with mullerian defects should undergo renal ultrasound, as up to 50% will have a coexisting urologic abnormality (such as renal agenesis)

Absence of a uterus is due to effects of what substance in the fetal period?

Mullerian inhibitory factor Embryonic testis secrete testosterone and MIF. MIF is responsible for regression of the paramesonephric (mullein) ducts that normally give rise to the internal genitalia in the female fetus. Testosterone mediates development of male internal genitalia and DHT mediates development of external genitalia

CD7

Multi-chain complex T-cell marker

Type of inheritance of spina bifida?

Multifactorial

Term used to describe a complex process by which numerous genetic and environmental factors interact to determine phenotypic expression

Multifactorial inheritance Examples: cleft lip/palate, DM, spina bifida, CAD, HTN Note: the closer a relative is to the affected person, the more likely the relative is to develop the trait

64 year old man with persistent back pain, constipation, easy fatiguability for the last several months. Dry MM. Labs show elevated serum protein, BUN, and creatinine. Renal biopsy shows atrophic tubules which contain large, obstructing intensely eosinophilic casts. Most likely diagnosis?

Multiple myeloma Easy fatiguability, constipation, back pain, elevated serum protein, and azotemia in an elderly patient should raise suspicions for MM. Large eosinophilic casts composed of bence jones proteins are seen in the tubular lumen in myeloma cast nephropathy

First generation antihistamines are nonspecific and act as antagonists on multiple receptors including what receptor that results in flushed cheeks, fever, urinary retention, decreased sweating, altered mental status, and dilated pupils?

Musarinic acetylcholine receptor

32 year old man with fever, agitation, and disorientation. Family states patient has been acting strangely and seeing and hearing things that don't exist. He drools excessively and has severe, painful throat spasms when he attempts to eat or drink. PE shows confused & combative patient with mydriasis and neck rigidity. What acts as an entry for the causative agent of this patient's condition?

Muscle acetylcholine receptors Rabies virus has a bullet shaped envelope with know like glycoproteins that bind to nAch receptors. Once transmission occurs (usually due to bite from infected animal), the virus replicates locally in muscle tissue for several days/weeks before spreading in a retrograde fashion through the peripheral nerve axons to the CNS

24 year old patient's medication is switched from chlorpromazine to fluphenazine. Compared to chlorpromazine, fluphenzaine is more likely to cause?

Muscular rigidity First generation antipsychotics can be classified according to high or low potency and have characteristic side effect profiles. Low potency antipsychotics are most likely to cause non-neurological side effects (sedation, anticholinergic side effects, orthostatic hypotension). High potency antipsychotics are more likely to cause extrapyramidal symptoms (Potency = extraPyramidal).

Autoimmune disase caused by autoantibodies directed against nicotinic AcH receptors on postsynaptic membrane of NMJ. Typically presents with fluctuating weakness that worsens over the day and involves extra ocular muscles (ptosis, diplopia) & bulbar (fatiguable chewing, dysphagia) muscles

Myasthenia gravis

34 year old woman with blotches of skin on her appears that appear to be different colors and tingling sensation in her hands that have developed over the past several months. She is refugee from East Africa. On exam there is patchy areas of anesthesia and hypopigmentation on UE. Many oganisms can be seen invading Schwann cells on microscopy. Most likely organism causing her condition?

Mycobacterium leprae The severity of leprosy depends on the strength of the cell mediated immune response with tuberculoid leprosy representing the mild form (mild skin plaques with hypopigmentation, hair follicle loss, and focally decreased sensation- intact Th1 cell mediated immune response) and lepromatous leprosy, the more severe form (weak cell mediated immune response in which macrophage signaling to kill M. leprae is limited leading to mycobacterial dissemination that causes diffuse skin thickening, plaque like hypo pigmentation with hair loss, leonine facies, paresis, regional anesthesia, testicular destruction & blindness)

Causative agent of walking pneumonia, an infection typically characterized by a nagging, nonproductive cough, low grade fever and malaise. CXR open suggests a severe pneumonia even though the patient appears relatively well. Require cholesterol supplementation to grow in artificial media

Mycoplasma pneumonia

Enzyme found predominantly in neutrophils. Serum levels can increase following inflammation & infection

Myeloperoxidase

Found in neutrophils and converts hydrogen peroxide to hypochlorous acid, a bactericidal compound that causes oxidative damage to host cells

Myeloperoxidase

Deficiency of what enzyme causes failure of muscle glycogenolysis, resulting in decreased exercise tolerance, muscle pain and cramping, and myoglobinuria with physicial activity?

Myophosphoryalase deficiency (McCardle disease/glycogen storage disease type V)

As part of an experiment, radio labeled ATP is injected into skeletal muscle. During muscle contraction, the labeled ATP is observed attached to the sarcomere. The attachment causes immediate....?

Myosin head detachment from the actin filament Note: if ATP is not available, the cross bridge between actin and myosin will persist (rigor mortis)

Nitrates, via conversion to nitric oxide, activate guanylate cyclase and increase intracellular levels of cyclic guanosine monophosphate (cGMP). Increased levels of cGMP lead to?

Myosin light chain dephosphorylation (via activation of myosin light chain phosphatase and decreased activity of myosin light chain kinase), resulting in vascular smooth muscle relaxation

Autosomal dominant disorder caused by increased number of trinucleotide repeats (CTG) on myotonia-protein kinase gene. Sustained muscle contraction (myotonia) along with weakness and atrophy is common. Cataracts are seen in almost all patients and frontal balding & gonadal atrophy are other common features.

Myotonic dystrophy Abnormally slow relaxation of muscles is called myotonia. Classic symptoms of myotonic dystrophy are difficulty loosening ones grip after a handshake or inability to release the doorknob. Anticipation is a characteristic of this disease. Atrophy pf muscle fibers is prominent on LM- Type 1 fibers are most affected.

MOA of phencyclidine (PCP)

N-methyl-D-aspartate receptor ANTAGONIST with lesser effect on reuptake inhibition on biogenic amines (norepinephrine, dopamine, serotonin)

Ruptured ectopic pregnancy can present with abdominal pain, bloody vaginal discharge, orthostatic hypotension, and a positive pregnancy test.. PID significantly increases the risk of ectopic pregnancy an dis caused by?

N. gonorrhoeae or C. trachomatis

8 year old with exercise intolerance. Fatigue, muscle pain, cramps during exercise with severe muscle stiffness following strenuous activity. Blood shows no rise in lactate levels. Absent lactate dehydrogenase (LDH) activity. Strenuous exercise leads to inhibition of glycolysis in skeletal muscle due to intracellular depletion of what substance?

NAD+ In glycolysis NAD+ is needed for glyceraldegyde-3-phosphate to be converted to 1-3-biphosphoglycerate by the enzyme glyceraldehyde 3 phosphate dehydrogenase. In anaerobic glycolysis (exercising muscle) NAD+ is regenerated from NADH when pyruvate is converted to lactate by LDH. With LDH deficiency glycolysis is inhibited in strenuously exercising muscle as muscle cells cannot regenerate NAD+ leading to pain, muscle breakdown, fatigue. Note: under aerobic conditions pyruvate is concerted to acetyl CoA, NAD+ is converted to NADH in TCA cycle, NADH is reconverted to NAD+ in electron transport chain as energy in NADH is utilized to synthesize ATP

Catalyzes reduction of molecular oxygen to superoxide free radicals, aiding bacterial destruction by phagocytic cells

NADPH oxidase (NOTE: deficiency leads to chronic granulomatous disease in which those affected cannot destroy catalase positive organisms)

Ulcer in DISTAL duodenom; fasting serum gastrin concentration is at upper limit of normal and RISES in response to IV secretion. Likely due to?

NEOPLASM (Zollinger-Ellison Syndrome) NOTE: secretin inhibits the release of gastrin from normal gastric G cells

Number needed to harm represents the number of people who must be treated before 1 additional adverse event occurs. Equation?

NHH= 1 /absolute risk increase Subtract the adverse event rate in the control group from the adverse event rate in the treatment group to get the absolute risk increase

Are sensitivity and specificity affected by disease prevalence?

NO! (positive predictive value & negative predictive value are)

Is tobacco use associated with oral thrush (localized Candida infection)

NO. Tobacco use can cause leukoplakia a precancerous lesion which cannot be scraped off easily

Negative predictive value represents the probability of not having a disease given a negative test result. Relationship to prevalence?

NPV is inversely proportional to the prevalence of disease. When a patient has characteristics similar to the overall population (age, sex, risk factor status), the disease prevalence is a valid estimate of the pretest probability of disease. NPV= number of true negative/total number of negative tests

Foundation for management for rheumatoid arthritis is disease modifying antirheumatic drugs, which alleviate pain and inflammation and reduce long term joint destruction, however the response to treatment may take several weeks. Best option to provide rapid symptom relief in the interim?

NSAIDs and glucocorticoids

In cardiac muscle cells, a decrease in cytoplasmic calcium level occurs immediately preceding relaxation. Most likely responsible protein for this observed change in electrolyte levels?

Na+/Ca2+ exchanger Calcium efflux from cardiac cells prior to relaxation is primarily mediated via the Na+/Ca2+ (NCX) exchange pump and sarcoplasmic reticulum Ca2+-ATPase pump (SERCA). NCX uses the large extracellular Na+ concentration gradient to help pump Ca2+ out of the cell and in the process removes one intracellular Ca2+ in exchange for 3 Na+ ions. In contrast. SERCA is a Ca2+ATPase pump that actively transfers calcium from the cytoplasm to the lumen of the SR at the expense of ATP hydrolysis

First line pharmacological treatment for moderate to severe alcohol use disorder that is an opioid antagonist & reduces the craving for alcohol?

Naltrexone Note: Disulfuram does not reduce craving & can be used only in abstinent patients with strong motivation to maintain abstinence

Chronic sleep disorder characterized by excessive daytime sleepiness, cataplexy (sudden loss of muscle tone that occurs in response to intense, usually positive emotions), and REM sleep related phenomena including hypnagogic (upon falling asleep) and hypnopompic (upon awakening) hallucinations & sleep paralysis (inability to move immediately after waking up). Results from depletion of hypocretin-secreting neurons in the lateral hypothalamus that are involved in maintaining wakefulness.

Narcolepsy Diagnosis can be confirmed by low CSF levels of hypocretin-1 or shortened REM sleep latency on polysomnography

6 year old boy with persistent nasal bleeding. He picks his nose frequently and has had several nosebleeds in the past that have stopped spontaneously after pinching the nose. Exam shows continuous blood trickle from his right nostril. Silver nitrate cautery is performed and the bleeding stops. Cautery was most likely applied to what location in the patient's nasal cavity?

Nasal septum Epistaxis is commonly caused by irritation of the highly vascular mucosa at the anterior nasal septum. The anterior nasal septum contains the Kiesselbach plexus. The anterior ethmoidal, sphenopalantine, and superior labial arteries anastamose in this region. Note: branches of the sphenopalatine artery supply the posterolateral wall and the posterior choanae & are the most likely course of posterior nosebleeds. Posterior epistaxis is usually severe and can't be treated with cautery

Medical error that is recognized before any harm is done to the patient

Near miss Example: a patient is prescribed a lethal dose of a medication and the error is caught by the pharmacist

3 week old baby in NICU develops abdominal dissension, vomiting, and blood streaked stools. Had previously tolerated formula feeds and had normal stool and urine output. Born at 27 weeks gestation. Pregnancy was complicated by premature rupture of membranes and preterm delivery. Abdominal X-ray shows thin curvilinear areas of lucency that parallel the bowel and wall lumen. Most likely diagnosis?

Necrotizing enterocolitis One of the most common GU emergencies affecting newborns. Characterized by bacterial invasion and ischemic necrosis of the bowel wall and is associated with prematurity and initiation of enteral feeding. Abdominal X-ray showing pneumatosis intestinalis (air in bowel wall) confirms diagnosis.

To offset cost and avoid losing patients to follow up, a department is considering recommending the empiric use of penicillin for the treatment of suspected bacterial pharyngitis. This would be expected to decrease the need for what after long term implementation of this guideline?

Need for cardiac surgery (mitral valve disease from RF eventually requires valvotomy/other surgical procedures) Acute rheumatic fever is a complication of untreated group A streptococcal pharyngitis. Rheumatic heart disease is the most common cause of acquired valvular heart disease and cardiovascular death in developing countries. The incidence of ARF and RHD has been reduced in industrialized nations with prompt treatment of strep pharyngitis with penicillin. Note: PSGN can NOT be prevented with early antibiotic therapy

64 year old man with acute knee pain. Developed redness, swelling, and severe pain affecting the right knee yesterday. 3 similar episodes in the past year that resolved quickly with OTC analgesics. History of polycythemia era but has been noncompliant with phlebotomy treatments. VS normal. Right knee is red and warm and there is severe pain with movement of the joint. Needle aspiration is performed. Most likely finding on analysis of synovial fluid?

Needle shaped, negatively bifringent crystals Myleoproliferative disorders cause increased urate production and predispose to gout

Probability of not having a disease when the test result is negative?

Negative predictive value (calculated as the proportion of true negatives divided by the total number of negative tests- both true and false negatives) Remember NPV DOES VARY with prevalence of disease in the target population!

Most likely organism causing septic arthritis in a young, sexually active adult?

Neisseria Gonorrhoeae (gram negative diplococci). Causes triad of polyarthritis, vesiculopustular skin rash, and tenosynovitis

Neonate born prematurely experiences severe respiratory distress. Distress resolves with treatment and by two weeks go age, the patient has adequate respiratory function. Ophthalmoscopy later shows abnormal retinal vascularization that extends into the vitreous. Retinal findings are most likely related to?

Neonatal oxygen supplementation Use of concentrated oxygen therapy for neonatal respiratory distress syndrome may be complicated by retinopathy of prematurity. Temporary local hyperopia in the retina is thought to induce changes that use up-regulation of proangiogenic factors such as VEGF upon return to room air ventilation. Can result in vascular proliferation, retinal detachment, and blindness (retinopathy of prematurity/retrolental fibroplasia). The abnormal retinal neovascularization is a major cause of blindness in developed nations

Major toxicity of Cidofovir?

Nephrotoxicity manifesting as proteinuria or elevated creatinine

Endogenous netiuretic peptides promote vasodilation and natriuresis and provide a beneficial counter-regulatory mechanism in patients with HF. They are metabolized by what enzyme?

Neprilysin (inhibition of neprilysin improves outcomes in patients with chronic systolic HF)

20 year old with neck lump. History of a recently diagnosed and successfully removed pheochromocytoma. Father died of thyroid cancer in his 30s. Ultrasound reveals 3cm nodule in right lobe of thyroid and DNA is consistent with a subtype of thyroid cancer. Most likely histological finding of this thyroid cancer?

Nests or sheets of polygonal or spindle shaped cells with congo red positive deposits (medullary thyroid cancer) Patient likely has MEN2 which is associated with medullary thyroid cancer. MTC is a neuroendocrine tumor that arises from parafollicular calcitonin secreting C cells. Amyloid deposits are derived from calcitonin secreted by the neoplastic C cells and stain with congo red.

72 year old man with a week of progressive confusion and lethargy. Has had persistent cough for the past several weeks and 2 episodes of blood in the sputum. Wheezing in left lung. Found to have hyponatremia with features suggestive for SIADH secretion. CXR reveals mass in upper lobe of left lung. Histopathology confirms an aggressive lung cancer. What is most likely to be seen in the biopsy sample?

Neural cell adhesion molecule (CD56), neuron specific enolase, chromogranin, and synaptophysin. Small cell carcinoma of the lung (oat cell carcinoma) is the most aggressive type of lung cancer and has a neuroendocrine origin (tumor cells express neuroendocrine markers and contain neurosecretory granules in the cytoplasm). Some express neurofilaments. Frequently synthesize hormones or hormone like substances (Vaspressin leading to SIADH).

Neurofibromatosis type I (Von Recklinghausen disease) is an autosomal dominant disorder caused by mutations in the NF1 tumor suppressor gene. Patients characteristically develop numerous cutaneous neurofibromas composed mostly of Schwann cells which are embryologically derived from?

Neural crest

Patient with family history of skin cancer with a menlaocytic lesion seen on skin examination and a 1 week history of focal neurological deficit suggests her seizure activity results from malignant melanoma metastatic to the SNA. Melanoma is a malignancy of melanocytes which are derived from what embryological origin?

Neural crest cells Note: melanoma most commonly metastasizes to brain, GI tract, bone, liver, and lungs. Lung cancer and renal cancer are the two other most common malignancies that metastasize to brain.

Valproic acid during first trimester of pregnancy is associated with?

Neural tube defects (failure of neural tube closure at 4 weeks gestation)

Patient diagnosed with influenza and started on Oseltamivir. MOA of action of this drug?

Neuroaminidase inhibitor useful in the treatment and prevention of both influenza A and B virus infections. Impairs the release of newly formed virions from the infected host cells and impairs viral penetration of mucous secretions that overlie the respiratory epithelium. Must be taken within 48 hours of onset of symptoms!

Seizure in immunocompetent patient from Central or South America with cystic brain lesion on MRI- most likely diagnosis?

Neurocysticercosis caused by Taenia Solium (pork tapeworm). **Note: cerebral toxoplasmosis is only likely in immunocompromised patients

38 year old female with 4 hours of abdominal pain, nausea, and vomiting. Diagnosed with acute appendicitis. Further evaluation reveals abnormal islands of small cells that are uniform in shape and size. These cells are most likely derived from what source?

Neuroendocrine cells Carcinoid tumors are composed of islands or sheets of uniform cells with eosinophilic cytoplasm and oval to round stippled nuclei. These tumors are often derived from neuroendocrine cells in the GI tract. Appendiceal carcinoids typically have a benign course, but may cause appendicitis or rarely carcinoid syndrome if they metastasize to the liver

Autosomal dominant nervous system tumor syndrome in which patients commonly develop bilateral CN VIII schwannomas and multiple meningiomas

Neurofibromatosis type 2

Bilateral acoustic neuromas are associated with what condition?

Neurofibromatosis type 2, an autosomal dominant condition cause by a mutation on the NF-2 gene on chromosome 22

Woman with refractory nausea and vomiting despite taking Ondansetron around the clock. Recently diagnosed with ovarian cancer and started on highly emetogenic adjuvant chemotherapy. Additional therapy blocking what receptors would be helpful?

Neurokinin 1 (-pitant) 5-HT3 and dopamine receptor antagonists are particularly useful for chemotherapy induced vomiting. When these don't control symptoms NK 1 receptor antagonists (prevent substance P release) can be considered in order to prevent both acute vomiting & delayed emesis associated with chemotherapy

Progressively weakening diaphragmatic contractions during maximal voluntary ventilation with intact phrenic nerve stimulation indicates what type of pathology?

Neuromuscular junction pathology (myasthenia gravis) and/or abnormally rapid diaphragmatic muscle fatigue (restrictive lung or chest wall disease) Note: in a young previously healthy woman with worsening dyspnea who is postpartum think about myasthenia gravis Note: impairment of respiratory control centers in the brain step can result in decreased frequency and/or amplitude in INVOLUNTARY respirations

31 year old female with new onset tonic clonic seizures is found to have an intracranial mass detected on head CT. Biopsy reveals neoplastic cells that stain positive for synaptophysin and negative for GFAP. The neoplastic cells most likely originate from the same precursor tissue as what?

Neurons Synaptophysin is a protein found in the presynaptic vesicles of neurons, neuroendocrine, and neuroectodermal cells. CNS tumors of neuronal origin frequently stain positive for synaptophysin. Neoplasms of glial origin (astrocytomas, glioblastoma multiforme, peripheral nerve sheath tumors, ependymomas, oligodendrogliomas) stain positive for GFAP. Meningiomas don't stain positive for GFAP or synaptophysin as they are derived from arachnoid villi cells.

Vasopressin and oxytocin are synthesized within neurons found in the paraventricular and supraoptic nuclei and are released into the circulation from axon terminals in the posterior pituitary. Name of substance that are involved in the posttranslational processing and stabilization of oxytocin and vasopressin within neurosecretory vehicles during transport to the posterior pituitary?

Neurophysins

Dose limiting side effect of vincristine?

Neurotoxicity (finger numbness & tingling) This toxicity results from failure of microtubule polymerization in neuronal axons. Other notable adverse effects classically associated with chemotherapeutic agents include pulmonary fibrosis and flagellate skin discoloration with BLEOMYCIN. CHF with DOXORUBACIN. Hemorrhagic cystitis with CYCLOPHOSPHAMIDE

Main dose limiting side effect of Vincristine?

Neurotoxicity (typically peripheral neuropathy)

The second generation antipsychotic clozapine is the only antipsychotic that has consistently shown superior efficacy in treatment resistant schizophrenia and schizophrenia associated with persistent suicidality. Major adverse effects of this medication?

Neutropenia & the potential for life threatening agranulocytosis. Treatment guidelines require enrollment in a centralized program that regularly monitors the patients absolute neutrophil count. Seizure and myocarditis are other important adverse effects.

Significant adverse effect of ganciclovir & incidence is increased with co-administration of zidovudine or trimethoprim-sulfamethoxazole

Neutropenia (all three meds are associated with bone marrow supression)

Primary cells responsible for intense inflammatory response seen in patients with gout?

Neutrophils Neutrophils phagocytose urate crystals causing release of various cytokines & inflammatory mediators that lead to further neutrophil activation & chemotaxis

12 year old boy with gait instability and pruritic skin rash for past several weeks. Mom reports he has been irritable and has had loose stools during this time. Patient has had several episodes of similar skin rash that has resolved spontaneously. Exam shows scaly, erythematous skin lesions in sun exposed areas and cerebellar ataxia. Labs show increased levels of neutral amino acids in the urine. Patient's symptoms would most likely benefit from what substance?

Niacin Hartnup disease is an autosomal recessive condition caused by impaired transport of neutral amino acids (particularly tryptophan) in the small intestine and proximal tubule of the kidney. Tryptophan is an essential amino acid and precursor for niacin, serotonin, and melatonin. Conversion of tryptophan to niacin is responsible for up to have of NAD+ required for redox reactions so disease manifestation of Hartnup are primarily due to niacin deficiency. Symptoms include pellagra like skin eruptions and cerebellar ataxia, which occur as a result of the niacin deficiency. Diagnosis can be confirmed through detection of excessive amounts of neutral amino acids in the urine. Treat with high protein diet and daily niacin or nicotinamide supplementation.

56 year old man with severe right foot pain is diagnosed with acute gouty arthritis. PMH is significant for hyperlipidemia treated with several modifications. What antihyperlipidemic drug is most likely to have precipitated patient's new onset gout?

Niacin (decreases hepatic synthesis of triglycerides and VLDL and reduces clearance of HDL; can decrease renal excretion of uric acid precipitating acute gouty arthritis)

Subarachnoid hemorrhage occurs due to rupture of saccular (berry) aneurysm or arteriovenous malformation. Severe vasospasm 4-12 days after the initial insult is the major cause of morbidity and mortality in patient revering from SAH. Presents with new onset confusion and or focal neurological deficit. Medication often prescribed to prevent vasospasm?

Nimodipine (selective Ca2+ channel blocker) Note: CT may fail to show vasospasm, so transcranial color doppler is needed Note: rebleeding is anther dangerous, common complication of SAH which presents with sudden development of severe headache, severe nausea and vomiting, change in level of consciousness and appearance of new neurological deficits (can be detected on CT)

Substance in neurons that corresponds to the rough endoplasmic reticulum

Nissl substance

Most important mediatory of coronary vascular dilation in large arteries and pre-arteriolar vessels?

Nitric oxide It is synthesized from arginine and oxygen by endothelial cells and causes vascular smooth muscle relaxation by guanylate cyclase mediated cGMP second messenger system. Adenosine (a product of ATP metabolism) acts as a vasodilatory element in the small coronary arteries.

Unique neurotransmitter that freely diffuse across cell membranes, does not need to interact with other neurons via a synapse, and is thought to participate in formation of new memories?

Nitrous oxide

54 year old woman with progressive fever and headache. Also has had dyspnea, productive cough, fatigue, and night sweats. Underwent renal transplant 2 years ago and is on immunosuppressive therapy. Temperature is 101. On exam she is lethargic with patchy lung crackle. CXR reveals several nodules, but no parenchymal infiltrate. MRI shows a 1.2 cm ring enhancing focal lesion with surrounding edema in the frontal lobe. Sputum gram stain shows branching gram positive organisms. Organism that is most likely cause of this patients condition?

Nocardia asteroides Nocardia species mainly cause pulmonary, CNS, or skin disease in immunocompromised patients. Pulmonary nocardiosis can present as a cavitary pneumonia (often misdiagnosed as TB); branching gram positive organisms are seen on sputum examination Note: Toxoplasmosis can cause encephalitis and pneumonitis in patients with HIV and is typically diagnosed with serology; histopathology would show cysts or tachyzoites

68 year old man with persistent back pain for past several months. Focal tenderness over 10th rib and L1 and L2 vertebral region. Radionuclide bone imaging reveals increased activity in multiple vertebrae and ribs, corresponding to patient's painful areas. Radiographs show sclerotic lesions matching the bone scan areas of increased uptake which are highly suspicious for metastatic cancer. What additional findings are most likely to be seen in this patient?

Nodular prostate and elevated prostate specific antigen level Bony pain in an older man with osteoblastic (sclerotic) lesions on imaging is highly suspicious for prostate cancer. Note: Osteolytic (lucent) lesions are due to osteoclast stimulation and tend to represent aggressive cancer. Osteoblastic (sclerotic) lesions are due to osteoblast stimulation and indicates a more indolent course.

Sporothrix schenckii is a dimorphic fungus that causes a subcutaneous mycosis. Its often transmitted by a thorn prick. How does it present?

Nodules that spread along the lymphatics

Drug class that exacerbates hypoglycemia and masks its adrenergic symptoms (tremulousness, palpitations, anxiety/arousal) mediated by norepinephrine and epinephrine & should therefore be used with caution in patients with DM?

Non selective beta blockers (propanalol, timolol, nadolol) Blockade of B2 adrenergic receptors inhibits hepatic gluconeogenesis and peripheral glycogeneolysis & lipolysis. Use B1 selective antagonists (pindolol, acebutolol) instead in diabetics!

Polycythemia vera is a myeloproliferative disorder characterized by uncontrolled erythrocyte production. Patients frequently present with nonspecific symptoms (headache, weakness, diaphoresis), aquagenic pruritus, facial plethora (reddish complexion) and splenomegaly. Virtually all patients with this disease have a mutation in JAK2 which is what type of receptor?

Non-receptor (cytoplasmic) tyrosine kinase (associated with the EPO receptor) Note: labs show increased erythrocyte mass, thrymbocytosis, leukocytosis, and LOW EPO levels

Constipation, bradycardia, atrioventricular conduction block (negative chronotropic effect), and worsening of heart failure in patients with reduced left ventricular function (negative inotropic effect) are common side effects of what drug class that are frequently used for treatment of HTN, angina pectoris, and supra ventricular arrhythmias (atrial flutter, afib, paroxysmal supraventricular tachycardia)?

Nondihydropyridine calcium channel blockers (diltiazem, verapamil)

Common findings in down syndrome include cognitive impairment, facial dysmorphism, and cardiac defects. Down syndrome can be caused by what 3 events?

Nondisjunction (95% of cases- resulting in 3 copies of chromosome 21), unbalanced translocation, mocaisism

The prescription of medications likely to do more harm than good should be avoided, even if requested by patients, based on what bioethics principle?

Nonmaleficence (first, do no harm)

Complication that cannot be prevented given the current state of medical knowledge

Nonpreventable adverse event Example: an allergic reaction to a medication in a patient with no known history of drug allergies

86 year old woman hospitalized for UTI due to E. coli and is being treated with Cefrtiaxone. On 5th day of hospitalization she seems agitated and had three episodes of diarrhea. Temperature is 101. IN addition to appropriate hand hygiene, what equipment is necessary before examining this patient?

Nonsterile gloves and a gown (sterile gloves are used for minor procedures or surgeries, but not required for contact precautions) Patient's fever and diarrhea a few days after antibiotic initiation are concerning for C. difficile. Handwashing with soap and water, gown for any patient contact, and non sterile gloves that are to be changed after contact with contaminated secretions are required. In addition, a dedicated stethoscope and BP cuff should be left in patient's room. Alcohol based sanitizers don't kill the bacterial spores! Note: simple face mask is required for pathogens that are transmitted by droplets that remain suspended in air for limited periods such as N. meningitides, influenza, B. pertussis, Mycoplasma, and RSV when within 6-10 feet of affected patients, but gloves and gown are not required.

Autosomal dominant disorder sometimes referred to as the male version of Turner syndrome. Associated with congenital heart disorders (commonly pulmonary valve stenosis), short stature (due to IGF-1 abnormalities), flattened nasal bridge, scoliosis, lordosis, macrocephaly, & developmental delay, various hematologic abnormalities

Noonan syndrome

Intense self consciousness, concerns about appearance and social acceptance, moodiness, increasing need for privacy, & transient emotional outburst are in a 13 year old girl who is doing well in school and has friends are characteristic of what?

Normal adolescent behavior Assessment of severity, persistence, and degree of social and academic impairment can help differentiate normal adolescent behavior from behavioral changes requiring further evaluation

Achalasia is caused by reduced number of inhibitory ganglion cells in the esophageal wall. Patients experience dysphagia, regurgitation, and retrosternal chest pain. Barium esophagram typically shows dilation of the esophagus with distal narrowing. Esophageal manometry in achalasia shows?

Normal contraction of the upper esophageal, decreased amplitude of peristalsis in the mid esophagus with increased tone and incomplete relaxation at the lower esophageal sphincter

Patient with severe chest pain, diaphoresis, and palpitations. Dies two hours after onset of symptoms and autopsy reveals 100% occlusion of left anterior descending artery. LM of affected myocardium at the time of death would likely show?

Normal myocardium Cytoplasmic eosinophilia and nuclear pyknosis are the earliest signs of coagulative necrosis but they take at least 4 hours to develop. Edema and puntactate hemorrhages in infarcted myocardium also take about 4 hours to show up on LM.

Presents with enlarged ventricles with NORMAL ICP. Impaired absorption of CSF is compensated for by decreased CSF production. Most commonly found in adults.

Normal pressure hydrocephalus

Acute respiratory distress syndrome is characterized by hypoxia and bilateral pulmonary infiltrates and is associated with trauma, pneumonia, sepsis, and pancreatitis. The associated pulmonary edema is noncardiogenic in nature and the way to differentiate this from cariogenic pulmonary edema is?

Normal pulmonary capillary wedge pressure (6-12 mmHg) Pulmonary insults (direct or indirect) cause excessive cytokine release, leading to endothelial activation, neutrophilic migration to the lungs, and degranulation with release of toxic mediators. This worsens pulmonary alveolar and endothelial injury leading to increased pulmonary capillary permeability, leakage of fluid into the alveoli, and pulmonary edema

52 year old presents for checkup. Reports she has been down since her youngest child let for college 2 months ago. Worried about her daughter being away from home for the first time and whether she will be successful at school. At work she occasionally has lapses of concentration when thinking about her daughter, but it has NOT AFFECTED HER PRODUCTIVITY. Exercising and eating well and still enjoys going out with her husband and socializing with friends, even though all they do is talk about their kids, she says jokingly. Has occasional insomnia and tension headache, but states they are nothing new. Most likely explanation?

Normal sadness Mild or brief sadness without significant interference in psychosocial functioning is consistent with normal sadness. Must take into account severity, duration, and degree of functional impairment!

Major clinical manifestations of factor V leiden are DVT (and pulmonary emboli), cerebral vein thrombosis, and recurrent pregnancy loss. Two mechanisms by which factor V leiden causes hypercoagulability?

Normally activated protein C restricts clot formation by inactivating factors Va and VIIIa. Factor V leiden has reduced suceptibility to cleavage by activated protein C. Va is a cofactor in the conversion of prothrombin to thrombin persistently circulating factor Va leiden results in increased thrombin production. Factor V leiden is unable to support activated protein C anticoagulant activity. So there is increased coagulation and decreased anticoagulation.

Hematologic malignancies (leukemia, lymphoma, multiple myeloma) tend to be associated with what type of anemia?

Normochromic, normocytic

Most common cause of viral gastroenteritis?

Norovirus. Outbreaks common (schools, cruise ships, nursing homes. Symptoms are vomiting and watery diarrhea.

Best method to determine if a gene is being transcribed?

Northern blot (detects mRNA) Southern- DNA Southwestern- DNA bound proteins Western- protein Note: Elisa test is used to measure amount of protein in bod y fluids (example: measuring plasma insulin levels)

Neurologic damage associated with B12 deficiency classically includes subacute combined degeneration of the dorsal columns (loss of position and vibration sense, positive Romberg sign) and lateral corticospinal tracts (spastic paresis). Identify lesions on spinal cord

Note: patients also may develop atrophic glossitis, which appears as a smooth, shiny, erythematous tongue. Megaloblastic anemia and pancytopenia are also characteristic.

Acute cholecystitis is most often caused by gallstones obstructing the cystic duct. The diagnosis can be made by identifying signs of gallbladder inflammation (wall thinking, pericholecystic fluid) on ultrasound. When ultrasound is inconclusive, what test can be used to assess cystic duct patency and make the diagnosis?

Nuclear medicine hepatobiliary scanning (cholescintigraphy) In acute or chronic cholecystitis, the radio tracer is injected and is then taken up by the liver with progressive excretion into the common bile duct and proximal small bowel, but the gallbladder will not be visualized due to the obstruction.

Bacteria express a lipopolysaccharide on their outer membrane surface that stimulates toll like receptors in the inflammatory cells. This leads to degradation of IkB inhibitor protein which normally binds to a latent transcription factor found in the cytoplasm. What factor is most likely to be directly activated by the removal of this protein?

Nuclear-factor kappa B NF-kB is a transcription factor with a critical role in the immune response to infection. NF-kB is normally present in the cytoplasm in a latent, inactive state bound to its inhibitor protein, IkB. Extracellular substances such as lipopolysaccharide can initiate a signal cascade that results in the destruction of IkB and translocation of free NF-kB to the nucleus

Primary site of ribosome synthesis and assembly?

Nucleolus (all ribosomal RNA except 5S rRNA is transcribed in the nucleolus).

5 year old girl that 'sunburns too easily'. Mom says patients skin becomes red and scaly with minimal sun exposure. PE shows hyperpigmented skin with a few nevi on hands that have been enlarging rapidly. Disorder is due to a primary defect involving what process?

Nucleotide excision repair Xeroderma pigmentosum is autosomal recessive disorder characterized by increased sensitivity to UV radiation. Skin is normal at birth, but then present during first year of life with sun sensitivity. Later, skin shows atrophy, telangiectasis, & intermingling areas of hypo and hyper pigmentation due to chronic UV damage. Skin malignancies can develop as early as 5-6 years old. Note: MISMATCH repair is HNPCC/Lynch syndrome

Thymidine dimers are removed by?

Nucleotide excision repair by enzyme UV-specific endonuclease

Eukaryotic pre-mRNA undergoes significant post transcriptional processing including 5'-capping, poly A tail addition, & intron splicing before leaving what location?

Nucleus

Notochord becomes what in adults?

Nucleus pulpous of intervertebral disk

Drug of choice for oropharyngeal candidiasis in patients without advanced immunodeficiency?

Nystatin (a polyene antifungal) Acts by binding to ergosterol in the fungal cell membrane causing the formation of pores and leakage of fungal cell contents. Not absorbed from the GI tract and is administered as an oral 'swish and swallow'

Strain of E. Coli that does not ferment sorbitol and does not produce a glucuronidase

O157:H1

Pancreatic inflammation (pancreatitis, pancreatic cancer) can cause a blood clot in the splenic vein which can increase pressure in the short gastric veins and lead to gastric varices located where?

ONLY in the FUNDUS (rest of stomach and esophagus not affected)

Odds Ratio Formula

OR=ad/bc

Caused by restricted expansion of the chest wall due to severe obesity. Leads to hypoventilation with a chronically elevated PCO2 and reduced PO2.

Obesity hypoventilation syndrom (Pickwickian syndrome)

4 year old girl with dark red blood on her toilet tissue. Sequential imaging using Tc-pertechnetate scintigraphy demonstrates focal radio tracer accumulation in the right lower quadrant. What embryologic process most likely failed in this patient?

Obliteration of the omphalomesenteric (vitelline) duct Meckel diverticulum results from failed obliteration of the vitelline duct and usually presents with spontaneous but painless lower gastrointestinal bleeding. Tc-pertechnetate localizes ectopic partial cells of gastric mucosa and its increased uptake is diagnostic for Meckel diverticulum. Ectopic gastric mucosa in the Meckel diverticulum secretes acid and causes local ulceration and bleeding. Meckel diverticulum is a potential lead point for intussusception which can present with colicky abdominal pain and currant jelly stools.

Due to relaxation of the oropharyngeal muscle tone with the occlusion of the upper airway. Symptoms include daytime sleepiness, headaches, depression. Complications include systemic and pulmonary HTN, right heart failure, & an increased risk for cardiac events

Obstructive sleep apnea

Deaf patient who communicates through sign language presents with worsening depression and anxiety. Patient is able to read lips, write, and communicate through gestures. Friend is with him who communicates with him though sign language and can speak. Post appropriate course of action for evaluation of this patient?

Obtain assistance from a qualified sign language interpreter to ensure optimal medical care

3 year old boy diagnosed with meningitis. Mom request to take child home and states that she intends to treat him with homeopathic remedies. She has 4 other kids at home who 'never needed vaccinations & are strong and healthy'. After explaining risks of meningitis she remains unconvinced of need for hospitalization. Best course of action?

Obtain court injunction to mandate the continued treatment Parents' authority to make medical decisions for their children can be challenged in cases in which the child is at significant risk of harm. Physicians are justified in obtaining a court injunction to proceed with life saving medical treatment of the child

54 year old male with sore on right shoulder. PE demonstrates an ulcer with central black eschar surrounded by edema. Exudative microscopy reveals chains of large gram positive rods. What would be most helpful in making the diagnosis?

Occupation Cutaneous anthrax leads to formation of a necrotic skin wound with an erythematous and edematous border and a necrotic center after inoculation of spores of Bacillus anthracis into the skin. Anthrax is most commonly acquired occupationally by those who handle livestock that have not been immunized for the disease as well as those who handle the hides of such animals. Used as a biological weapon due to the near 100% mortality of the pulmonary form (if cutaneous anthrax is suspected in patient without occupational exposure then public health authorities should be contacts as potential for bioterrosim should be suspected).

55 year old with acute onset headache and difficulty with vision. She becomes unconscious en route to hospital. Head CT scan demonstrates an acute hemorrhage in the left temporal lobe & compression of the anterior medial temporal lobe against the free margin of the tentorium cerebelli. What cranial nerve most likely to be compromised?

Oculomotor Transtentorial (uncal) herniation is a complication of an ipsilateral mass lesion, such as a hemorrhage or brain tumor. First sign of uncal herniation is a fixed and dilated pupil on the side of the lesion (CN III compression). Ipsilateral paralysis of the oculomotor muscles, contralateral or ipsilateral hemiparesis, and contralateral homonymous hemianopsia with macular sparing may also occur

Case control is used to compare the exposure status of people with the disease (cases) to the exposure status of people without the disease (controls). Main measure of association of a case control study?

Odds ratio

What does odds ratio represent?

Odds that an outcome occurred in the presence of a particular exposure compared with the odds that the outcome occurred in the absence of that exposure

Where do aspergillomas develop?

Old lung cavities (from TB, emphysema, lung CA, sarcoidosis). Usually upper lobes and NO systemic symptoms Colonizes the cavity forming a 'fungus ball' WITHOUT tissue invasion. Appears on CXR as radiopaque structure that shifts when patient changes position. Can be asymptomatic or cause cough with hemoptysis

Slow growing tumors of adults that typically involve the white matter of the cerebral hemispheres. Appear as well circumscribed gray masses with calcification on gross exam

Oligodendroglioma

Both enterocysts and Meckel diverticula result from failure of obliteration involving the?

Omphalomesenteric duct (aka vitelline duct- normally obliterates during the 7th week of embryonic development)

Explanation for the synchronous production of B-galactosidase & galactoside permease in response to lactose in E. coli colonies?

One mRNA coding for both enzymes Note: Bacterial mRNA can be polycistronic (one mRNA codes for several proteins). Transcription & translation of E. coli lactose metabolism (bacterial lac open) proteins is regulated by single promotor, operator, and set of regulatory elements

Manic episodes are characterized by elevated/irritable mood, hyperactivity, decreased need for sleep, pressured speech, and grandiosity and may occur with psychotic features. Bipolar I disorder is diagnosed in patients with how many lifetime episodes of mania?

One or more

42 year old man with severe headaches and oliguria. BP is 240/150. Papilledema bilaterally. What is the pathologic process associated with his condition?

Onion like concentric thickening of arteriolar walls Hyperplastic (not hyaline!) arteriolosclerosis in renal arterioles can result from and perpetuate malignant HTN. Pathologic lesion is an onion line concentric thickening of arteriolar walls as a resultt of laminated smooth muscles cells and reduplicated basement membranes in the renal vasculature and elsewhere

Results in progressive, painless, bilateral vision loss with optic disc cupping on fundoscopy (due to optic disc atrophy)

Open angle glaucoma

Most effective way to begin a medical interview?

Open ended questions Once the patient provides an initial history, the physician can use closed ended and follow up questions to clarify certain points and fill gaps in the history

What passes through optic canal (medial to superior orbital fissure)?

Optic nerve (CN II) & ophthalmic artery

Causes monocular vision loss over several weeks with painful eye movement. Can be associated with afferent pupillary defect with hyperemia and swelling of the optic disc on fundoscopy. Frequently associated with multiple sclerosis

Optic neuritis

Prospective cohort study

Organized by selecting group of individuals (cohort), determining their exposure status, and then following them over time for development of disease of interest

Transport of what substance into the mitochondria is necessary for proper function of the urea cycle, the major disposal pathway for waste nitrogen generated by catabolism of amino acids?

Ornithine

Overuse injury of the secondary ossification center (apophysis) of the tibial tubercle. Very common cause of knee pain in young adolescent athletes after a recent growth spurt. Presents as pain and swelling at the tibial tubercle, the insertion point of the patellar ligament.

Osgood-Schlatter disease The quadriceps muscle group is connected to the patella, which in turn is attached to the tibial tubercle by the patellar ligament. Repetitive quadriceps contraction & chronic avulsion cause the proximal patella tendon to separate from the tibial tubercle leading to OSD disease, which is characterized by focal pain and swelling at the tibial tuberosity

Autosomal dominant inheritance of congenital telangiectasis. Rupture of the telangiectasis may cause epistaxis, GI bleeding, or hematuria.

Osler-Weber-Rendu syndrome (hereditary hemorrhagic telangiectasia)

Characterized by osteophyte formation leading to hard bony enlargement of the DIP (Heberden nodes) and PIP (Bouchard nodes). Brief morning stiffness may be present

Osteoarthritis

Blue sclerae, slightly protruding eyes, early hearing loss, frequent fractures, slight spinal curvature, poor muscle tone are characteristic of what disease which is often confused with child abuse?

Osteogenesis imperfecta

In patients with prolactinoma, high levels of circulating prolactin suppress GnRH secretion from the hypothalamus, leading to reduced secretion of LH and subsequent hypogonadism, anovulation, & amenorrhea. The resulting estrogen deficiency can cause?

Osteoporosis with an increased risk of fragility fractures & lead to vaginal dryness and atrophy, hot flashes, and dysparenunia

Insoluble Ca2+ requires an acidic environment for proper absorption. Strong acid suppressing medications such as proton pump inhibitors may decrease the absorption of dietary Ca2+ and therefore long term use of these medications is associated with an increased risk of?

Osteoporotic fractures

Students return from cave exploration trip to central USA develop fever, cough, and malaise. Pulmonary infiltrates and hilarious aden-patchy are present on CXR. Lung tissue specimens would show?

Ovoid cells within macrophages (Histoplasma capsulatum is endemic to Mississippi and Ohio river valleys and associated with bird/bat droppings)

34 year old male is trapped underground following collapse of an access shaft. It takes two days to reach him. Upon being rescued patient reports feeling dizzy and weak and hasn't eaten anything in over 30 hours. Fingerstick glucose is 78 g/dL. What biochemical reaction is responsible for maintaining his current blood glucose levels?

Oxaloacetate --> phosphenolpyruvate The 2 major processes that maintain plasma glucose between meals are glycogenolysis (first 12-18 hours of fasting) and gluconeogenesis (once hepatic glycogen stores are depleted). Gluconeogenesis uses many of the glycolytic enzymes, but hexokinase, phosphofruktokinase, and pyruvate kinase need to be bypassed as they are unidirectional. The initial steps of gluconeogenesis involve the conversion of pyruvate to oxaloacetate and oxaloacetate to phosphoenolpyruvate by pyruvate carboxylase and phospheonylpyruvate carboxykinase respectively. These two enzymes work together to bypass pyruvate kinase. The other 2 unique gluconeogenic enzymes are fructose 1,6 biphosphatase (bypasses phosfructokinase) and glucose 6 phosphatase (bypasses hexokinase)

67 year old male with severe, ongoing drug resistant HTN, dies of intracranial hemorrhage. At autopsy, right kidney is significantly shrunken & left kidney appears grossly normal. Mechanism that most likely explains patient's renal morphology?

Oxygen and nutrient deprivation Pathogenesis of patient's HTN and ICH are most likely renovascular, specifically unilateral renal artery stenosis. Most common cause of renal artery stenosis is obstruction by an atheromatous plaque at origin of renal artery. Ischemic kidney secretes high levels of renin, causing HTN, and eventually the stenosis can cause renal atrophy due to O2 and nutrient deprivation. Risk factors include male sex, DM, and increased age.

Distinct foci found within eukaryotic cells that are involved in mRNA regulation and turnover. Play a fundamental role in translation repression and mRNA decay & contain numerous proteins including RNA exonucleases, mRNA decaying enzymes, & constituents involved in mRNA quality control & microRNA induced mRNA silencing. Also seem to function as form of mRNA storage.

P bodies (Note: interaction between mRNA & P bodies ONLY OCCURS IN THE CYTOPLASM)

What drug causes dissociative & anesthetic effects with psychosis and severe agitation leading to violent trauma, ataxia, horizontal & vertical nystagmus, and delirium?

PCP

68 year old man with a history of advanced melanoma that is unresectable and resistant to adjuvant regimens. He recently began receiving monoclonal antibody infusions that were approved for an advanced melanoma management. The monoclonal antibodies block a specific cell surface receptor found on T lymphocytes to facilitate destruction of cancer cells by T cells capable of recognizing tumor antigens. What cell surface receptor is most likely blocked by the treatment?

PD-1 The binding of programmed death receptor 1 (PD-1) to its ligand (PD-L1) down regulates the immune response against tumor cells by inhibiting cytotoxic T cells. Many types of cancers evade the immune system by increasing expression of PD-L1. Monoclonal antibodies blocking PD-1 help prevent T cell inhibition and promote apoptosis of tumor cells. Anti-PD1 therapy is used in advanced melanoma and in certain types of lung cancer

Esmolol infusion produces rapid symptom relief and HR slowing. What portions of the ECG will be affected most significantly by this medication?

PR interval Beta blockers decrease AV nodal conduction, leading to an increased AV nodal refractory period. This correlates to PR interval prolongation on ECG. PR interval is the period from the beginning of atrial depolarization (start of P wave) to the beginning of ventricular depolarization (start of QRS complex)

Best way to monitor warfarin?

PROTHROMBIN TIME/INR Warfarin is an ORAL anticoagulant that inhibits the CARBOXYLATION of Vitamin K dependent coagulation factors II, VII, IX, X. Its used in atrial fibrillation, DVT, and pulmonary thromboelbolism Skin necrosis can occur during first few days of Warfarin treatment due to transient hyper coagulable state (reduction in protein C anticoagulant activity before affecting other factors' procoagulant activity). DO NOT USE warfarin in pregnancy- teratogenic and can cause fetal bleeding. a PTT is used to monitor unfractionated Heparin. Bleeding time used to monitor platelet function.

Hypercalcemia in sarcoid is caused by?

PTH independent formation of 1,25 dihydroxyvitamin D by activated macrophages leading to increased intestinal absorption of calcium

Disease characterized by eczematous exudate over nipple and areola due to ductal spread of malignant cells to nipple surface, occasionally associated with bloody nipple discharge

Paget disease of nipple

Biopsy showing a mosaic pattern of lamellar bone with bone pain and elevated alkaline phosphatase level is diagnostic of?

Paget's disease of bone

With with smoking history, upper limb pain/weakness, & ipsilateral ptosis and mitosis are suggestive of?

Pancoast tumor Pancoast tumors are non small cell lung cancers (squamous cell carcinoma, adenocarcinoma) that arise near the superior sulcus. Patients may have ipsilateral shoulder pain, upper limb paresthesias, and areflexic arm weakness due to involvement of brachial plexus. Horner's syndrome (ipsilateral partial ptosis, miosis, anhydrosis) can occur due to involvement of cervical sympathetic ganglia

34 year old man presents due to frequent clumsiness. PMH notable for recurrent renal stones, for which he underwent definitive treatment with neck surgery 3 years ago. PE reveals bitemporal visual fiend defects. In addition to brain imaging patient should be screen for what tumor?

Pancreatic (most common is gastrinoma, followed by insulinoma; often metastatic tumors and are a frequent cause of death in MEN1) Multiple endocrine neoplasia type 1 is characterized by tumors of pituitary, parathyroid gland, and pancreas. Caused by mutations in MEN1 (menin) tumor suppressor gene. Primary hyperparathyroidism is often the initial manifestation and usually presents with asymptomatic hypercalcemia or recurrent renal stones

The dorsal pancreatic bud forms the majority of the pancreatic tissue (body, tail, and most of the head). The ventral pancreatic bud is the precursor for the uncinate process, inferior/posterior portion of the head, and the major pancreatic duct (of Wirsung). Failure of the dorsal and pancreatic buds to fuse leads to?

Pancreatic divisium in which the pancreatic ductal systems remain separate with the accessory duct draining the majority of the pancreas via the minor papilla. The smaller ventral duct (of Wirsung) opens into the major papilla and functions to drain the inferior/posterior portion of the head and uncinate process

14 year old Caucasian male with chronic diarrhea, failure to gain weight and recurrent respiratory infections with Pseudomas aeruginosa. Brother died from severe respiratory infection at age 9. Agent that would be most like to improve this patients condition long term?

Pancreatic lipase Symptoms are suggestive of cystic fibrosis which causes steatorrhea and failure to thrive to due malabsorption secondary to obstruct fibrosis and pancreatic insufficiency (can be corrected by pancreatic enzyme supplementation)

Faconi anemia presentation?

Pancytopenia, hypo/hyperpigmented patches, short stature, hypo plastic thumbs (remember these kids have increased risk of malignancy such as AML!)

Histopathology of this type of thyroid cancer reveals branching papillary structures with concentric calcifications (psammoma bodies) and ground glass or empty appearing nuclei (Orphan Annie eye nuclei)

Papillary thyroid cancer

Most common type of thyroid cancer. Characteristic findings on histopathology include large cells with nuclei containing finely dispersed chromatin, giving an empty or ground glass appearance (Orphan Annie eye) and intranuclear grooves or inclusions & psommoma bodies can be found within tumor.

Papillary thyroid cancer

Lymph from testes drains into what nodes?

Para-aortic nodes

Infant with facial dysmorphia, cleft palate, right to left shunt, and absent thymic shadow on x-ray who experiences frequent and recurrent sinopulmonary infections. Lymph nodes will show poor development of what structure?

Paracortex (poorly developed in DiGeorge syndrome due to deficiency of mature T lymphocytes) Note: agammaglobulinemia causes an absence of B cells preventing primary lymphoid follicles and germinal centers from forming in the lymph node cortex

Brassy, barking cough & dyspnea, stridor, fever, and recent history of URI infection in a child are suggestive of viral laryngotracheitis (croup). Most common cause of croup?

Parainfluenza virus

32 year old brought by ambulance due to severe abdominal pain and undergoes an emergency appendectomy. Following a successful operation, patient questions whether appendectomy was necessary and starts doubting surgeon's explanation that the surgery was potentially lifesaving. He suggests hospital is taking advantage of his excellent insurance and makes threats to sue. No psychiatric history. Lives alone and prefers to work for himself, claiming his previous bosses were always trying to set him up for failure. Most likely diagnosis?

Paranoid personality disorder Individuals with paranoid personality disorder exhibit lifelong pattern of pervasive suspicion and distrust. Unlike patients with psychotic disorder, they don't have fixed delusions and other psychotic symptoms

Muscle biopsy from healthy volunteer consists mainly of myoglobin rich, glycogen poor fibers with many mitochondria. Most likely biopsy site?

Paraspinal muscles Postural skeletal muscles (soleus, paraspinal muscles) contain predominantly Type I slow twitch, red muscle fibers that derive ATP primarily form oxidative aerobic metabolism Note: type II fast twitch fibers are specialized for generating rapid forceful pulses of movement & derive ATP energy through anaerobic glycogenolysis and subsequent glycolysis

45 year old woman with 4 month history of headaches and blurred vision. CT shows 2 cm left sided adrenal mass. Adrenalectomy performed. Assuming timor cells are functionally similar to cells from the outermost adrenal cortex, patient would likely have presented with what additional symtpoms?

Parasthesias and muscle weakness Excess production of aldosterone causes renal Na+ retention and excess urinary K+ and H+ secretion resulting in HTN, hypokameia, and metabolic alkalosis. Significant ECF volume expansion doesn't occur due to aldosterone escape

Where are H2 receptors found?

Parietal cells in the gastric mucosa (H2 antagonists block gastric acid secretion by parietal cells)

Decreased dopamine in the substantia nigra and striatum is found in what disease?

Parkinson's disease

In patients with sickle cell anemia and other chronic hemolytic disorders, the most common viral cause of an aplastic crisis is infection of erythroid progenitor cells with?

Parovirus B19, a non enveloped single stranded DNA virus

Presents with triad or hemolytic anemia, hypercoagulability, & pancytopenia. Results from acquired mutation in PIGA gene that causes absence of glycosylphosphatidylinositol anchor and associated deficiency of CD55 & CD59 complement inhibitor proteins

Paroxysmal nocturnal hemoglobinuria

Stem cell defect from deficiency of compliment regulatory proteins CD55 & CD59 (decay accelerating factor). PIG-A (phosphatidylinositol class A) gene is defective leading to over activation of complement which destroys erythrocytes intravascularly. Decrease in pH during sleep leads to hemolysis, pancytopenia, iron deficiency anemia & thrombosis. First urination of day is dark from hemoglobin. Only cure is allogenic bone transplant

Paroxysmal nocturnal hemoglobinuria

Patient exposed to nitrates develops cyanosis that is not improved by administration of supplemental oxygen. Analysis of a sample of arterial blood drawn prior to O2 supplementation would reveal normal what?

Partial pressure of oxygen in the arterial blood. Nitrates cause poisoning by inducing conversion of Heme iron (Fe2+) to oxidized ferric (Fe3+ state) leading to formation of methemoglobin which cannot bind oxygen leading to functional anemia. Blood partial pressure of O2 is a measure of oxygen dissolved in the plasma which is unrelated to hemoglobin function so it will remain unchanged.

Anti-phospholipid antibodies are found in patients with SLE and in antiphospholipid antibody syndrome. Antiphospholipid antibody syndrome causes a hyper coagulable state and patients may suffer recurrent miscarriages. What is paradoxically increased in these patients despite the propensity towards thrombosis?

Partial thromboplastin time (PTT)

25 year old has sill birth at 18 weeks gestation. Only symptom during pregnancy was pain in both knees and feet, which she attributed to being on her feet all day as a teacher. Pain lasted a week and resolved without medication. Fetal autopsy shows pleural effusion, pulmonary hypoplasia, and ascites. What infection was the most likely etiology of the stillbirth?

Parvovirus (nonenveloped, single stranded DNA virus) Infection in adults presents with acute, symmetric arthralgia/arthritis with or without rash. Infected fetus can develop hydrops fetalis (sever anemia, heart failure, pleural effusions, pericardial effusions, and ascites)

Intervention most likely to improve prognosis in a patient with diphtheria?

Passive immunization Treatment of acute C. diphtheriae infection in order of importance: 1) Diphtheria antitoxin (passive immunization) 2) Penicillin or erythromycin 3) DPT vaccine (active immunization) Diphtheria infection is associated with a 5-10% mortality rate, especially in younger patients or those with myocarditis. Cardiomyopathy is the most common cause of death.

Cause of soft tissue infection that develops within 24 hours following a dog or cat bite and has a characteristic mouse like odor. Management includes would care and antibiotics targeted against the organism (amoxicillin-clavulantate)

Pasteurella multocida

Common in preterm infants & presents with continuous murmur, widened pulse pressures, & signs of cardiovascular strain.

Patent ductus arteriosus (shunts fetal right ventricular output away from lungs and into aorta) Note: indomethacin or ibuprofen can inhibit PGE2 synthesis and accelerate closure

Precordial continuous machine like murmur indicates what cardiac condition which produces isolated pulmonary artery SpO2 elevation?

Patient ductus arteriosus

Characterized by a continuous murmur best heard in the left infraclavicular region with maximal intensity at S2. Occurs most commonly in patients born prematurely and those with cyanotic congenital heart disease

Patient ductus arteriosus Note: a small PDA is often asymptomatic and is usually detected incidentally during routine cardiac auscultation

Patient with dermatomal Herpes Zoster rash has serum testing positive for varicella gig antibodies? This suggests what?

Patient has reactivation of varicella zoster virus (NOTE: varicella IgG antibodies do NOT confer immunity against herpes zoster because they are ineffective against latent virus in dorsal root ganglia)

Patient with eruptive and palmar xanthomas and premature atherosclerosis. Lack of ApoE3 & ApoE4 in circulating lipoproteins. What is impaired in this patient?

Patient would have decreased clearance of chylomicrons & VLDL. Patient has familial dysbetalipoproteinemia (type III hyperlipoproteinemia), an autosomal recessive disorder characterized by high cholesterol and triglycerides

Specific model of primary care in which patients have access to a personal physician who coordinates care and sees the patient through all aspects of care including preventative services and acute & chronic disease management

Patient-centered medical home

Niacin deficiency results in?

Pellagra (dermatitis, dementia, diarrhea, eventual death if untreated)

Term used to describe the proportion of people with a given genotype who express its associated phenotype

Penetrance

Patient started on an antiretroviral regimen that includes an agent that selectively binds to the HIV gp41 protein. This drug directly inhibits what viral process?

Penetration into target cells Antiretroviral agents that selectively bind to the HIV envelope transmembrane glycoprotein gp41 precent the conformational changes necessary for the viral membrane to fuse with the target cell membrane. HIV unable to gain entry into uninfected CD4+ T cells. These agents are 'fusion inhibitors' (Enfuviritide)

Gram positive bacteria are treated with antibiotics. Several days later, the bacteria assume a spherical configuration when placed in an isotonic solution and disintegrate rapidly when placed in a hypotonic solution. What antibiotic was most likely used?

Penicillins, cephalosporins and vancomycin are able to disrupt the peptidoglycan cell wall of gram positive and gram negative organisms. The peptidoglycan cell wall of these organisms gives them the ability to survive osmotic stress & this ability is lost after treatment with these antibiotic agents

Presentation of Zollinger-Ellison Syndrome?

Peptic ulcers (especially distal duodenal), diarrhea, and heartburn. Patients will have gastrinomas (often malignant) located in the small intestine/pancreas

Effects on ventilation and perfusion when going from a supine position to a standing position?

Perfusion greatly increases from the apex to the base of the lung. Ventilation slightly increases from the apex to the base so the V/Q (ventilation/perfusion) ratio decreases in the lung form the apex to the base while the blood flow increases

Syndrome characterized by upward gaze palsy, absent pupillary light reflex, & impaired convergence

Perinaud syndrome (midbrain stroke at level of superior colliculus)

Midline episiotomy most likely involves what structure?

Perineal body (tendinous center point of the perineum that separates the urogenital & anal triangles). Midline episiotomy is a vertical incision from the posterior vaginal opening to the perineal body (transects the vaginal lining & submucosal tissue, but not external anal sphincter or rectal mucosa)

Side effects are unusual of Lamivudine, unlike other NRTIs, but consist of?

Peripheral neuropathy and lactic acidosis

X-inactivation occurs in genetically normal females and results in conversion of the inactivated X-chromosome into compact heterochromatin (Barr body). Heterochromatin is condensed chromatin composed of heavily methylated DNA in tight associated with deacetylated histones. It has a low level of transcriptional activity. Where is heterochromatin typically located?

Periphery of the nucleus

Type of effect cortisol exerts on many hormones to improve the response to a variety of stressors?

Permissive effect (occurs when one hormone allows another to exert its maximal effect, but itself does not have the intrinsic effect vs. additive and synergistic). Cortisol increases vascular and bronchial smooth muscle reactivity to catecholamines and increases glucose release by the liver in response to glucagon

4 month old being evaluated for hereditary metabolic disorder after presenting with failure to thrive and developmental delay. Was born outside USA and had several seizures during neonatal period. PE reveals abnormal head shape with a large anterior fontanelle and widely spaced cranial cultures. Profound hypotonia and hepatomegaly noted. Elevated levels of VERY LONG CHAIN FATTY ACIDS and phytanic acid are found. Cultured skin fibroblasts show impaired ability to oxidize VLCFAs. Patient's condition is best explained by dysfunction in what cell structure?

Peroxisomes Build up of VLCFAs due to impaired beta oxidation & some branched chain FA (phytanic) due to impaired alpha oxidation is the hallmark of peroxisomal disorders (Zellweger syndrome, adrenoleukodystriphy). Accumulation of these FA in the brain results in permanent neurologic dysfunction. Zellweger syndrome- caused by defective peroxisomal biogenesis; typically presents in early infancy with craniofacial abnormalities (widened sutures, large anterior fontanelle), hepatomegaly, and profound neuro defects (hypotonia, seizures, developmental delay); death within months of initial presentation X-linked adrenoleukodystrophy- caused by defective transport of VLCFAs into peroxisomes; patients present in early childhood or adulthood with neurological deterioration and adrenal insufficiency as VLCFAs accumulate in the CNS and adrenal glands

3 week old boy with discharge from umbilicus. Postnatal course was uncomplicated with shriveling of cord at 14 days of life. Area reveals a small reducible umbilical hernia, minimal clear to straw colored dischagre from the umbilicus, and erythema around the area. Most likely cause of condition?

Persistence of allantois remnant Urachus is a remnant of the allantois that connects the bladder with the yolk sac during fetal development. Failure of urachus to obliterate at birth results in patent urachus which can facilitate discharge of urine from umbilicus Note: Leukocyte adhesion deficiency involves decreased expression of neutrophil cell surface adhesion proteins, B2 integrins. Neutrophils fail to migrate to infected sites but are increased in blood. Delated separation of the umbilical cord (>30 days).

Characterized by chronic depressed mood and at least two other depressive symptoms (fatigue, hopelessness, etc.) lasting for at least 2 years

Persistent depressive disorder (dysthymia)

2 year old boy evaluated for recurrent ear, skin, and pneumonia infections. Incision and drainage of his skin infection infections revealed S. aureus on bacterial culture but no purulence. His infections usually resolve with prolonged antibiotic courses. Lab testing reveals absent CD18 antigens on the surface of leukocytes. He is at greatest risk for?

Persistent leukocytosis Leukocyte adhesion deficiency is an autosomal recessive disorder due to absence of CD18 antigens necessary for the formation of integrins. Clinical features are caused by failure of leukocyte chemotaxis and include recurrent skin & mucosal infections without purulence, delayed separation of the umbilical cord, and persistent leukocytosis.

Virulence factors of the gram negative coccobacillus Bordetella pertussis?

Pertactin: forms the basis of the acellular pertussis vaccine; promotes B. pertussis adherence to the ciliated upper respiratory epithelium Tracheal cytotoxin: promotes local tissue destruction, resulting in cough Pertussis toxin: causes excessive adenylate cyclase activity preventing effective phagocytosis and allows organism to persist in alveolar macrophages and ciliated epithelial cells, leading to prolonged disease course; produces lymphocytosis

Normal grief has similarities to symptoms of major depressive episode such as sadness, insomnia, decreased appetite, and weight loss. Normal grief can be differentiated from major depressive episode by lack of?

Pervasive anhedonia (inability to experience pleasure), worthlessness, and suicidality

Non-pathogenic strains of Corynebacterium diphtheriae can acquire pathogenicity and thus the ability to cause severe pseudomembranous pharyngitis though what mechanism?

Phage conversion permitting exotoxin production (acquiring the Tox gene via lysogenization by a temperate bacteriophage; Tox gene codes for the diphtheria AB exotoxin). Lysogenization is the process by which a bacteriophage infects a host bacterium and integrates its genome into the host bacterium's genome

Route taken by Neisseria meningitids to reach the meningitis?

Pharynx --> blood --> choroid plexus --> meningies N. Meningitides is transmitted through aerosolized droplets and subsequently colonizes the nasopharyngeal epithelium. Penetration of the epithelium can lead to blood infection. Spread to the meningitis occurs via transcellular penetration of the cerebral capillary endothelium or at the entry of the cored plexus

45 year old man with recurrent palpitations and chest discomfort associated with SOB. He is started on Dofetilide to maintain normal sinus rhythm. This medication exerts is main effect on what portion of the cardiac myocyte action potential?

Phase 3 (late repolarization) Class III antiarrhythmic drugs (amiodarone, stall, dofetilide) predominately block K+ channels and inhibit the outward potassium currents during phase 3 of the cardiac action potential, thereby prolonging repolarization and total action potential duration. Class III antiarrhythmics are often used for maintenance of sinus rhythm in patients with paroxysmal atrial fibrillation.

In an experiment investigating vasoconstriction of the arterial wall, two samples of isolated porcine arterial vessels are studied. Vascular tone is measured in the control vessel during infusion of increasing doses of NE. The other vessel is pretreated with drug A prior to infusion of NE. A graph of the study reveals a decrease in Vmax in the vessel treated with Drug A. Drug A is most similar to what agent?

PheNOxybenzamine = NO reversible Alpha 1 adrenergic receptors mediate arterial vasoconstriction whereas beta 2 receptors mediate vasodilation. NE is an agonist at alpha 1 and beta 1 but has significantly less action at beta 2 so its s potent vasoconstrictor Phenoxybenzamine is an irreversible alpha 1 and alpha 2 adrenergic antagonist that effectively reduces the arterial vasoconstriction induced by NE. Because phenoxybenzamine is an irreversible antagonist, even very high concentrations of NE, such as those seen in pheochromocytoma, cannot overcome its effects. Note: Phentolamine is a reversible, competitive alpha adrenergic antagonist used in the management of catecholamine induced hypertensive crises (pheochromocytoma, MAO inhibitor toxicity, cocaine intoxication) & Labetalol is a reversible competitive antagonist of alpha 1 and beta adrenergic receptors with partial beta 2 agonist activity used to treat HTN; since they are reversible, competitive antagonists high doses of NE can overcome the alpha adrenergic inhibition of these drugs. Reversible competitive inhibitors DO NOT CHANGE VMAX, they increase Km! Note: Propanalol is a nonselective beta adrenergic antagonist and pretreatment with a beta antagonist would potentiate the vasoconstrictive effects of NE due to unopposed alpha adrenergic stimulation

Hallucinogen that causes dissociative symptoms, agitation, hallucinations and violent behavior. Ataxia, nystagmus, and memory loss are the other distinguishing symptoms of its abuse

Phencyclidine

67 year old with pneumonia develops hypotension and lactic acidosis. He's started on NE IV drip. A few hours later, the antecubtial vein being used for the infusion blanches and the tissues surrounding the IV site become hard, cold, and pale. Local injection of the affected tissue with what agent will have greatest benefit?

Phentolamine Blanching of a vein into which NE is being infused together with induration and pallor of the tissues surrounding the IV site are signs of NE extravasation &resulting vasoconstriction. Tissue necrosis is best prevented by local injection of an alpha 1 blocking drug such as Phentolamine.

Phenylketonuria results from the failure to convert phenylalanine to tyrosine & leads to intellectual disability if left untreated. What is deficient enzyme?

Phenylalanine hydroxylase

3 year old boy has not yet begun to walk or speak. Assessment reveals severe intellectual disability. He dies six months later from refractory seizure resulting in respiratory failure. Autopsy shows pallor of substantia nigra, locus ceruleus, and vagal nucleus dorsalis. Underlying condition responsible for this patient's death is most likely caused by a deficiency of what enzyme?

Phenylalanine hydroxylase Deficiency of phenylalanine hydroxylase or its cofactor tetrahydrobiopterin (BH4) causes accumulation of phenylalanine in body fluids and the CNS. Homozygous infants are normal at birth but gradually develop severe intellectual disability and seizures if left untreated. Hypopigmentation of the skin, hair, eyes, and catecholaminergic brain nuclei is also frequently seen.

Patient given medication that increased peripheral vascular resistance, increases systolic blood pressure, decreases pulse pressure, and decreases heart rate. What medication?

Phenylepherine (selective alpha 1 adrenergic receptor agonist) The BP increase elicits a baroreceptor mediated increase in vagal tone which results in a decreased stroke volume and HR. Pulse pressure (systolic pressure minus diastolic pressure) is decreased because of the decrease in SV and increased afterload Note: Epinephrine- increases pulse rate and pulse pressure & decreases PVR (stimulates alpha and beta but has stronger effect on B2 than alpha1) Isoproterenol- nonselective beta agonist which decreases PVR and diastolic BP & increases cardiac rate and output and pulse pressure Dobutamine- increases HR and contractility with little effect on PVR

Cortisol increases the conversion of norepinephrine to epinephrine in the adrenal medulla by increasing the expression of what enzyme?

Phenylethanolamine-N-methyltransferase (PMNT) Remove pituitary --> low ACTH --> low cortisol --> decreased PNMT activity --> reduced conversion of NE to E

Increased citrate concentrations decrease glycolysis because its a powerful inhibitor of what enzyme?

Phosphofructokinase-1 Note: citrate is formed from condensation of acetyl CoA + oxaloacetate in first step of TCA cycle

6 month old boy not developing normally. PE reveals delayed developmental milestones and hypotonia. Two years later, child is found to have involuntary movements and demonstrates a tendency to aggressively bite his own lips and fingers. Labs show elevated blood uric acid level. Activity of what enzyme is most likely increased as a result of this patients condition?

Phosphoribosyl pyrophosphate amidotransferase Lesch-Nyhan syndrome is an X linked recessive disorder caused by a defect in hypoxanthine-guanine phosphoribosyltransferase (HGRPT) that is characterized by development of dystonia, choreoathetosis, self mutilation, and hyperuricemia within the first few years of life. This results in failure of the purine salvage pathway, leading to increased degradation of hypoxanthine and guanine to uric acid. De novo purine synthesis must increase to replace the lost bases, necessitating an increased in phosphoribosyl pyrophosphate (PRPP) amidotransderase activity

Progression of hemochromatosis in women is slowed by what?

Physiologic iron loss through menstruation & pregnancy

Occurs in premature infants due to liver immaturity & associated immaturity of UDP glucuronyl transferase production which is necessary for bilirubin conjugation. Uncongugated bilirubin accumulates in tissues and responds to light therapy

Physiologic jaundice

78 year old patient admitted for bronchoscopy is premeditated with intramuscular atropine (anticholinergic to decrease respiratory mucous secretions & promote bronchodilaton) and becomes acutely restless, disoriented, and combative. Pupils are widely dilated and nonreactive to light. ECG shows sinus tachycardia. What agent will reverse all of this patient's signs and symptoms?

Physiostigmine (cholinesterase inhibitor with a tertiary ammonium structure that can reverse both the central and peripheral nervous system symptoms of anticholinergic toxicity). Others that can cross the BBB: galantine, donepezil, rivastigmine. Elderly are at high risk of anticholinergic toxicity due to decreased renal and hepatic clearance Neostigmine, edrophonium, and pyridostigmine have a quaternary ammonium structure that limits central nervous system penetration & causes them only to reverse peripheral symptoms

50 year old presents with blurred vision and confusion after clearing and burning foliage weeds on his property. Temperature of 102. Disoriented to time, person, place PE shows flushed skin, dry mucosa. Pupils dilated and nonreactive to light and accommodation. Bowel sounds decreased. What drug should be administered to reverse condition?

Physiostigmine (increases acetylcholine levels by preventing its degradation by cholinesterase) Patient has anticholinergic toxicity likely from jimsonweed (Datura stramonium) while working in his yard. Jimsonweed contains large concentrations of anticholinergic comopounds atropine, scopolamine, and hyoscyamine.

Compulsive consumption of a nonfood and/or non-staple food. Common in pregnancy and often seen in associated with iron deficiency anemia. Pregnancy patients should be assessed throughout pregnancy for it, especially if they experience unexplained weight loss

Pica

Primary virulence factor that allows Neisseria meningitidis to initially attach to and colonize the nasopharyngeal epithelial surface?

Pilli (undergo significant antigenic variation which makes them a difficult vaccination target)

4 year old boy with headaches that have progressively worsened over past one month and are most severe when laying down. Had had non blood, non bilious emesis for 2 weeks that typically occurs early in the morning. Has had intermittent blurry vision, especially when looking up. PE shows bilateral papilledema, inability to gaze upwards, and bilateral eyelid retraction. Brain mass is suspected. Most likely location of this lesion?

Pineal gland. Germinomas (arises from embryonic germ cells) are the most common pineal gland tumor and present with obstructive hydrocephalus from aqueduct stenosis (causing headache, papilledema, vomiting) and dorsal midbrain (perinaud) syndrome due to compression of the pretectal region of the midbrain. Perinaud syndrome is characterized by limitation of upward gaze with a downward gaze preference, bilateral eyelid retraction, and pupils that react to accommodation but not to light. Germinomas in the suprasellar region cause endocrinopathies (central precocious puberty, diabetes insipidous) due to pituitary/hypothalamic dysfunction & do not cause perinaud syndrome

Organisms that ferment lactose appear what color on MacConkey agar?

Pink (non-lactose fermenters appear white)

Patient with poorly controlled DM with a non healing ulcer of left foot. Started on broad spectrum antibiotics. 24 hours later blood cultures grow beta lactamase producing Bacteroides species. Most appropriate antimicrobial coverage?

Piperacilin-tazobactam (combination of extended spectrum penicillin with beta lactamase inhibitor) as it is effective against most gram negative enteric rods (including Pseudomonas aeruginosa) and against Bacteroides fragilis (gram negative anaerobic rods that produce B-lactamase. Chronic non healing ulcers (areas of necrosis & diminished blood supply) facilitate growth of anaerobes. Other antibiotics with anaerobic activity: metronidazole, carbapenems, & clindamycin

Exam of patient reveals possible entrapment of the sciatic nerve in the greater sciatic foramen. What structure passes through the foramen and occupies most of its volume?

Piriformis muscle Piriformis muscle passes through greater sciatic foramen and is involved with external hip rotation. Muscle injury or hypertrophy can compress the sciatic nerve in the foramen causing piriformis syndrome. Piriformis can be tender with deep palpation or on stretching with adduction and internal rotation

Cushing syndrome with elevated ACTH levels that are suppressed by high dose but not low dose dexamethasone is cause by?

Pituitary adenoma

29 year old woman with bilateral intermittent white nipple discharge for the past several weeks. Menses stopped 6 months ago. B-HCG negative. Most likely diagnosis?

Pituitary adenoma Galactorrhea results from direct stimulatory effect of prolactin on mammary glands & amenorrhea results from inhibitory effect of prolactin on GnRH secretion

Patient having difficulty walking. He was installing new shingles on his roof when he fell to the group and injured his right leg. On PE, right foot is dorsiflexed and everted and patient is unable to stand on his tiptoes. Patient is most likely to have sensory loss over what area?

Plantar foot Tibial nerve may be injured at the level of the popliteal fossa due to deep penetrating trauma or knee surgery. Patients typically have weakness on foot plantar flexion, foot inversion, and toe flexion with sensory loss over the sole

Primary thrombotic microangiopathy syndromes (TTP-HUS) present with hemolytic anemia with schistiocytes, thrombocytopenia, and organ injury (brain, kidneys heart). Bleeding time is prolonged but PT & PTT are normal (unlike DIC). What is expected on renal biopsy?

Platelet rich thrombi in glomeruli & arterioles (thrombotic microangiopathy syndromes cause platelet activation & diffuse microthrombosis in arterioles & capillaries)

What provides major proliferative stimuli for the cellular components of atherosclerotic plaques?

Platlets Release of PDGF by locally adherent platelets, endothelial cells, and macrophages promotes migration of smooth muscle cells from the media into the intimate and their subsequent proliferation

Term that describes multiple phenotypic manifestations (often in different organ systems) from a mutation in a single gene

Pleiotropy

73 year old man with exertion dyspnea. Exercise tolerance has decreased over the last year and he can barely walk a few blocks without stopping. Also reports occasional episodes of lightheadedness and palpitations. PE reveals cardiac murmur. Point on pressure tracing that most likely corresponds to the peak murmur intensity in this patient?

Point B (on right side of picture; left is normal) The murmur of aortic stenosis is a systolic ejection type, crescendo-decrescendo murmur that starts after the first heart sound and typically ends before the A2 component of the second heart sound. The intensity of the murmur is proportional to the magnitude of the LV to aorta pressure gradient during systole. The maximum pressure difference between the LV and aortic pressure tracings is noted at point B and corresponds to the peak intensity of a cardiac murmur during auscultation.

Plan that requires patients to have a primary care provider and obtain referrals for specialty consultations. Differ from HMO plans in that they allow patients to see providers outside the network but at higher out of pocket costs

Point of service plan

Mutations in what gene are responsible for acquired resistance to HIV reverse transcriptase inhibitors & HIV protease inhibitors?

Pol gene

Androgenetic alopecia is the most common cause of hair loss in both males and females. What is its inheritance pattern?

Polygenic with variable expressivity (pattern & severity of baldness depends on both hormonal, aka circulating androgens, and genetic factors & vary between males and females)

Inflammatory disorder that affects patients over 50 and causes subacute pain and stiffness in the shoulder, hips, weight loss, fever, and malaise

Polymalgia rheumatica

20 year old woman presents with 3 days of vaginal pain with intercourse and dysuria. Reports low grade fevers and mild headache. Sexually active & recently began a monogamous relationship with a new partner. Her boyfriend uses condoms inconsistently. Genital exam reveals multiple painful, shallow ulcers with an erythematous base on left labia. No discharge or cervical motion tenderness. Bilateral tender inguinal lymphadenopathy present. Best diagnostic test?

Polymerase chain reaction for viral DNA Multiple painful genital ulcers and constitutional symptoms in sexually active patient is highly suggestive of primary genital herpes simplex virus infection. Diagnose with PCR, direct fluorescence antibody testing, viral culture, or Tzank smear. The characteristic vesicles are often absent!

Genetic mutations in cardiac K+ channel proteins are seen in patients with congenital long QT syndrome. A decrease in depolarizing K+ current leads to prolongation of QT interval which predisposes to the development of ?

Polymorphic ventricular tachycardia (torsades de pointes) & sudden cardiac death

Some relapsed breast cancer patients who were treated with Tamoxifen were found to have lower serum concentrations of endoxifen and 4-hydroxytamoxifen, the active metabolites of tamoxifen. Most likely cause of the drug's ineffective in these patients?

Polymorphism of cytochrome P450 enzymes Patients with genetic polymorphisms resulting in poor CYP2D activity (metabolizes Tamoxifen to its active compounds) are exposed to decreased levels of the active metabolite and have higher risk of disease relapse

43 year old male with muscle weakness for the past 6 weeks that is most pronounced in the hips and shoulders. Having difficulty rising from chairs and combing hair. Muscle biopsy reveals MHC class I molecule over expression on the sarcolemma with CD8+ lymphocyte infiltration. Most likely diagnosis?

Polymyositis Polymyositis causes symmetric proximal muscle weakness. Muscle biscay reveals inflammation, necrosis, and regeneration of the muscle fibers. Over expression of MHC class I proteins on the sarcolemma leads to infiltration of CD8+ T lymphocytes and myocyte damage Note: polymyositis is similar to dermatomyositis but lacks the typical skin findings (dusky red, rash on face/eyelids)

5 year old boy with severe respiratory distress. Has fever, stridor, dyspnea, and markedly swollen and erythematous epiglottis. Major virulence factor of pathogen causing his infection?

Polyribosylribitol phosphate (major virulence factor of H. influenzae is its polysaccharide capsule which is composed of this polymer) The polyribosylribitol capsule protects the bacterium against phagocytosis and complement mediated lysis by binding factor H, a circulating regulator protein that normally prevents complement (C3b) deposition on host cells. Note: epiglotitis is confirmed by direct visualization of radiographic evidence of a swollen epiglottis ('thumbprint' sign)

S. pneumoniae, the most common cause of community acquired pneumonia, are gram positive diplococci that exhibit partial alpha hemolysis on blood agar (green colonies) and are bile soluble & optochin sensitive. Presents with acute onset of fever, cough, and lobar consolidation on CXR. Primary virulence factor of Streptococcus pneumoniae (without which it cannot cause disease) is?

Polysaccharide capsule that inhibits phagocytosis Other virulence factors of S. pneumoniae are IgA protease (inactivates secretory IgA), adhesins (necessary for adhesion to epithelial cells) & pneymolysin (cytotoxin that causes pore in cell membrane and cell lysis) Capsule swells and appears as a halo around the blue stained bacterial cells when specific anti capsular antibodies & methylene blue dye are added ('quelling reaction')

Presents in early infancy with marked cardiomegaly, severe generalized hypotonia, macroglossia, & hepatomegaly. NORMAL blood glucose levels. Muscle biopsy reveals accumulation of glycogen in lysosomes

Pompe disease (glycogen storage disease type II)- deficiency of acid maltase (alpha glucosidase)

Patient with itchy rash that appeared this morning, recently retuned from Cancun. Husband has similar symptoms. Diffuse, pruritic, papulopustular rash on trunk & extremities. Micro analysis reveals oxidase positive gram negative rods that produce pigment on culture medium. Most likely course of infection?

Pool water (due to Pseudomonas- gram negative, oxidase positive, non lactose fermenting, motile rods that produce green pigment) "Hot tub folliculitis' is a superficial and self limited Pseudomonas aeruginosa infection of hair follicles that occurs in small outbreaks following exposure to pool or spa in which chemicals have not been maintained at appropriate concentrations.

Homogenous deposition of eosinophilic hyaline maternal in the intimal and media of small arteries and arterioles characterizes hyaline arteriolosclerosis. This is typically produced by?

Poorly controlled HTN and/or DM

28 year old woman with acute onset abdominal pain, nausea, and confusion. Reports doesn't smoke or drink because they make her feel sick. CT abdomen shows no abnormalities. A sample of her urine is reddish in color and darkens on standing for 24 hours. IV dextrose is administered and her symptoms improve significantly. Dextrose infusion most likely improved this patient's condition by affecting what pathway?

Porphyrin synthesis Acute intermittent porphyria is an autosomal dominant condition caused by porphobilinogen deaminase deficiency. Most patients remain asymptomatic but a minority present with acute attacks of abdominal pain and vomiting, peripheral neuropathy, neuropsychological symptoms, and red-brown urine. Treatment is with IV glucose of heme preparations with downregulate ALA synthase activity.

Superior vesical artery & its branches supply what structures?

Portions of urinary bladder and ductus deferent

Proportion of individuals with a positive test result who actually have the disease

Positive predictive value (depends on specificity, sensitivity, and PREVALENCE of disease in population being tested!)

Characterized by intrusive thoughts, nightmares, flashbacks, avoidance of trauma reminders, hypervigilance, and sleep disturbance lasting greater than or equal to one month

Post traumatic stress disorder Note: acute stress disorder involves the same symptoms that last 3 days to 1 month after trauma exposure. Symptoms may remit within 1 month or progress to PTSD.

Artery that branches off of the basilar artery and supplies CN III and IV and other structures in the midbrain. Also supplies thalamus, medial temporal lobe, splenium of corpus callosum, and occipital lobe

Posterior cerebral artery

Occlusion of what artery produces contralateral homonymous hemianopia with macular sparing?

Posterior cerebral artery (lesions og occipital cortex)

Left knee pain after playing basket ball. Coronal MRI viewed from posterior aspect is shown. What ligament is marked by the asterisk?

Posterior cruciate (attaches to the posterior part of the intercondylar area of the tibia and the anterior part of the lateral surface of the medial epicondyle on the femur) POD = PCL goes Out and Down (to lateral part of tibia)

Prevents posterior displacement of the tibia relative to femur. Originates from the anterolateral surface of the medial femoral condyle and inserts into the posterior intercondylar area of the tibia. Its integrity can be tested by using posterior drawer test.

Posterior cruciate ligament POD = PCL goes Out and Down (to lateral part of tibia)

Patient involved in MVC sustains a pelvic fracture. He is unable to void and complains of a full bladder sensation. Foley catheter placement is attempted, but the procedure is aborted once resistance is encountered and the patient begins to complain of pain. Most likely to have an injury of what portion of the urogenital tract?

Posterior urethera at the bulbomembranous junction (membranous segment is relatively unsupported by adjacent tissues and is the weakest point of the posterior urethra). Note: injury to the posterior urethra is associated with pelvic fractures & the anterior urethra is most commonly damaged in straddle injuries. In ability to void with a full bladder sensation, a high riding boggy prostate (caused by hematoma formation below the gland), and blood at the urethral meatus are suggestive of urethral injury, particularly in the presence of a pelvic fracture. If urethral injury is suspected, placement of foley catheter is contraindicated

PTH, vitamin D, & calcium axis

Posteroperative hypoparathyroidism is common after thyroidectomy. decreased PTH results in decreased calcium & phosphate resorption from bone & decreased calcium reabsorption by kidneys

12 year old boy with fatigue. 1+ protein and RBC casts in urine. Light microscopy shows enlarged hyper cellular glomeruli. Most likely?

Poststreptococcal glomerulonephritis Associated with elevated ASO titers, anti-DNAse B titers, decreased C3 and total complement levels, and the presence of cryoglobulins. C4 usually normal

4 year old boy with tender personal papule and pustules, some of which are broken down and covered with golden yellow crusts. Exudate microscopy reveals gram positive cocci in chains. What clinical syndrome is most likely to follow this infection?

Poststreptococcal glomerulonephritis (facial edema, dark cola-colored urine, HTN, hematuria, nephritic range proteinuria, and urinary red blood cell casts Impetego can be caused by S. aureus (most common) or S. pyogenes (as in this question- gram + cocci in chains). Acute rheumatic fever and PSGN are post infectious complications of group A step infections. PSGN can follow wither impetigo or streptococcal pharyngitis whereas ARF is only associated with pharyngitis!!

RNA interference is an important mechanism by which short (20-30 base pair) non-coding RNA sequences (SIRNA & miRNA) induce what by base pairing with complementary sequences within target mRNA molecules?

Posttranscriptional gene silencing (interferes with mRNA translation)

What should be administered to patients exposed to iodine-131 to prevent tissue damage/development of radiation induced thyroid carcinoma?

Potassium iodide (competitively inhibits thyroid uptake of radioactive iodine isotopes)

Refers to the dose of a drug required to produce a given effect. Primarily affect by the affinity of the drug for its receptor & the amount of drug able to reach the target tissues. Compare with ED50 (dose required to produce 50% of maximum response)

Potency

Investigators are designing a randomized control trial to test the hypothesis that drug B will decrease the mortality associated with acute ST elevation MI compared to standard of care. To ensure investigators will not miss a difference between drug B & standard of care (if a difference truly exists) what would they want to maximize?

Power (1-beta) Power is the probability of rejecting a null hypothesis when it is truly false. It is typically set at 80% and depends on sample size and difference between outcomes

When treating patients who have been referred for specialty care or for a second opinion, it is imperative not to undermine the patient's relationship with the primary physician. A physician should avoid making negative comments about the quality of acre rendered by that practitioner unless?

Practices are imminently dangerous or far outside acceptable standards of care

Ability of a test to reproduce identical or similar results with repeated measurements

Precision/Reliability

35 year old man with acute onset of coffee ground emesis and lightheadedness. Was started on Naproxen and cyclobenzaprine three weeks ago after developing low back pain with stiffness. Has an allergy to PCN that causes a rash and pruritus. Upper GI endoscopy reveals gastric mucosal erythema erosions. Type of drug reaction that best explains patients current symptoms?

Predictable drug reaction Patient has upper GI hemorrhage which is likely an adverse reaction to Naproxen (NSAID that inhibits prostaglandin mediated gastric mucosal protection leading to mucosal erythema, erosions, ulcerations). Changes to mucosa can be seen within one week. Note: most adverse drug reactions are predictable due to known pharmacologic properties of the drug & unpredictable reactions are less common and are due to genetic differences, immune interactions, etc.

Characterized by unwanted episodes of early ejaculation accompanied by a sense of lack of control. Diagnostic criteria is ejaculation within one minute of penetration most of the time for at least 6 months

Premature ejaculation Note: evaluation of any sexual disorder requires taking careful medical and substance abuse histories and assessing psychosocial stressors and cormorbid psychiatric conditions

46 year old gardener with knee pain. Began 3 days ago and improved only minimally with acetaminophen. Patient spends hours on his knees several times a week while working. No erythema or abrasions, but there is a sharp localized pain on palpation of the knee. Severe pain when walking around the room or climbing onto the exam table. Most likely affected bursae in this patient?

Prepatellar A bursa is a fluid filled synovial sac that serves to alleviate pressure from bony prominences and reduce friction between muscles and tendons. Acute trauma or chronic repetitive pressure can cause injury leading to localized pain and tenderness. Prepatellar bursitis causes anterior knee pain and is usually due to repetitive or prolonged kneeling.

Presbyopia and skin wrinkles are both age related changes. Pathogenesis?

Presbyopia occurs due to denaturation of structural proteins within the lens, leading to loss of lens elasticity. Decreased synthesis and increased breakdown of collagen and elastin contribute to the development of skin wrinkles

Best indicator of severity of mitral regurgitation (holosystolic murmur at apex with radiation to axilla)?

Presence of an audible S3 (absence of S3 can be used to exclude severe chronic MR) This murmur is generated by regurgitant blood flow from the LV back to LA during systole. Results in elevated pressure and blood volume in the LA which increases the amount of blood reentering the LV during diastole. An audible S3 gallop occurs when LV cannot accommodate excess flow (generated by the sudden cessation of blood flow into the LV during passive filling phase of diastole). Remember S3 can be normal in healthy young adults

MOA of liver injury in HBV?

Presence of viral HBsAg and HBcAg on the cell surface stimulate the host's cytotoxic CD8+ T lymphocytes to destroy the infected hepatocytes. HBV HAS NO DIRECT CYTOTOXIC EFFECT.

46 year old man complaining of double vision, ptosis, difficulty swallowing, nausea, and dry mouth. Wife has similar symptoms. Ate home canned potatoes yesterday. What step in neuromuscular transmission is affected?

Presynaptic exocytosis of Ach vesicles. Patient has manifestation of both nicotinic (diplopia, dysphagia) and muscarinic (dry mouth) blockade. Botulism inhibits both muscarinic and nicotinic cholinergic nerves. Botulism presents as a descending paralysis that first manifests with CN abnormalities (dysphonia, dysphagia, diplopia). Preformed neurotoxin commonly occurs after eating contained home canned food. Toxin is highly potent (small quantity can be lethal without ventilatory support) but heat labile (so prevent illness by heating food).

Changing disease prevalence in a steady state population with a constant incidence means there is an additional factor affecting duration of condition because?

Prevalence equals incidence rate multiple by the average disease duration. A factor that prolongs disease duration (such as improved quality of care) will increase disease prevalence, as affected patients survive longer

Injury to a patient by an act of commission or omission rather than from the underlying disease and are the result of failure to follow evidence based best practice guidelines

Preventable adverse event

Mycobacterium tuberculosis grows in long, serpentine cords due to the presence of cord factor, a surface glycolipid, on the cell wall. Cord factor is a primary virulence factor of M. tuberculosis that functions by?

Preventing macrophages from being bactericidal due to the inhibition of phagolysosome acidification and also leads to the formulation of caseating granulomas

Hypophosphorylated retinoblastoma protein performs what function?

Prevention of G1/S cell cycle transition Rb tumor suppressor gene encodes the Rb protein, which regulates the cell cycle. Active (HYPOphosphorylated) Rb protein prevents damaged cells from proceeding past the G1 to S phase checkpoint, while the inactive (HYPERphosphorylated) Rb protein allows the damaged cell to enter mitosis. Abnormal phosphorylation of Rb protein results in its activation

Psychological disorder characterized by increased intake of free water, leading to hyponatremia & production of a large volume of dilute urine. Water restriction corrects serum sodium levels & leads to increased urine osmolality. No significant increase in urine osmolality on ADH administration because at that stage of the test patient has maximal endogenous ADH effect.

Primary (psychogenic) polydipsia

Small, fibrotic focus in the upper portion of the lower lobe of the right lung and a calcified right hilarious lymph node are most consistent with what stage of a tuberculosis infection?

Primary M. tuberculosis infection The Gohn complex describes the two initial sites (lower lobe of lung, ipsilateral hilarious lymph node) of primary tuberculosis infection. Over time, these sites become calcified and fibroses and can be visualized on gross pathology and radiographic imagining (Ranke complex)

Chronic autoimmune liver disease characterized by lymphocytic infiltrates & destruction of small and mid-sized intrahepatic bile ducts with resulting cholestasis. Anti-mitochondrial antibody titer positivity & elevated alkaline phosphatase. Most common in middle aged women and associated with other autoimmune disorders. Clinical features are fatigue, itching, hepatosplnenomegaly.

Primary biliary cholangitis Note: similar morphologic findings are seen in hepatic graft vs. host disease (highlighting the immunologic etiology of the disorder)

45 year old female recently diagnosed with Sjogrens syndrome complains of severe pruritic that slowly progresses over the last year. She says pruritis is especially prominent over palms and soles and is unbearable during the night. Alkaline phosphatase is elevated. Most likely diagnosis?

Primary biliary cirrhosis Primary biliary cirrhosis is a chronic liver disease characterized by autoimmune destruction of the intrahepatic bile ducts and cholestasis (elevated alkaline phosphatase). Most common in middle aged women with severe pruritis (especially at night) one of the first symptoms reported. Fatigue, hepatosplenomegaly, and xanthomatous lesions in the eyelids or in the skin and tendons are common. With progression jaundice, steatorrhea, portal HTN, and osteopenia may develop. Lab findings include elevated alkaline phosphatase and cholesterol as well as IgM. Confirmed with anti-mitochondrial antibodies in serum.

Chronic autoimmune liver disease characterized by destruction of intrahepatic, interlobular bile ducts by granulomatous inflammation. Typically presents insidiously with pruritus and fatigue in middle aged women & patient may eventually develop signs of cholestasis (jaundice, pale stool, dark urine) and hypercholesterolemia (xanthelasma)

Primary biliary cirrhosis Note: fatigue & high alkaline phosphatase in a may with longstanding UC is suggestive of primary sclerosis cholangitis. Histo findings include diffuse inflammation/fibrosis of large intra and extra hepatic bile ducts with periductal concentric fibrosis ('onion skin pattern') & obstruction of small ducts

Presents with a solitary, painless genital ulcer with heaped up boarders and a clean base a few weeks after unprotected intercourse.

Primary syphilis (syphilitic chancre) Cause by Treponema palladium, a gram negative spirochete that cannot be cultured or detected on gram stain due to small size. Humoral antibody response takes 4 weeks to develop so false negative initial serologic testing (with rapid plasma reagin, VDRL) is common early in course of disease. Direct visualization of organism by dark field microscopy (showing motile, helical spirochetes) or fluorescent antibody testing is gold standard.

Stages of syphilis

Primary: CHANCRE - painless ulceration at inoculation site with raised, indurated borders that develops 1-3 weeks after contact & resolves in 3-6 weeks Secondary: CONDYLOMATA LATA- large, gray wartlike growths appearing in genital/perineal region & diffuse macular rash encompassing palms and soles; develops 5-10 weeks following chancre resolution Latent phase- asymptomatic period Tertiary: GUMMAS- painless, indurated granulomatous lesions that progress to white gray rubbery lesions that may ulcerate & are most commonly cutaneous but also appear in other organs; develops many years after infection of untreated individuals; cardiovascular involvement with ascending aortic aneurysms with calcification visible on x-ray and aortic valve insufficiency (murmur, high pitched S2 heart sound) Neurosyphilis (meningoencephalitis, tabes dorsalis) can occur in ANY stage but is a common manifestation of late syphilis

Polymerase chain reaction is used to amplify small fragments of DNA (genes, eons, noncoding regions) by repeated replication. What does it require to be performed?

Primers that are complementary to the regions of DNA flanking the segment of interest (exact nucleotide sequence of target region does NOT need to be known), thermostable DNA polymerase, deoxynucleotide triphosphate, and a source of DNA template strand

Anti-histone antibodies are found in drug induced lupus. Symptoms are similar to SLE but without renal or CNS involvement. Common causes of drug induced lupus?

Procainamide, hydralazine, isoniazid, D-penicillamine

Drug induced lupus erythematous is characterized by abrupt onset of lupus symptoms (fever, arthralgia, pleuritic) with positive anti-histone antibodies. Genetically predisposed individuals who are slow acetyators are at greater risk for developing it. Has been linked to drugs metabolized by N-acelyation in the liver such as?

Procainamide, hydralizine, isoniazid Clinical improvement is rapid on discontinuation of causative agent!

Patient started on an integrate inhibitor. What step of viral replication is most likely to be inhibited as a result of this drug?

Production of viral messenger RNA Integration of double stranded HIV DNA into the host cell's chromosomes is necessary to induce viral gene expression and prevent degradation of the viral genome.' -gravir's' are integrate inhibitors that disrupt HIV genome integration, preventing synthesis of viral mRNA

Hormone that prevents lactation in pregnancy?

Progesterone In pregnancy, progesterone is secreted by the corpus luteum and later by the placenta. Although prolactin secretion increases as pregnancy progresses, high progesterone levels inhibit lactation through negative feedback on prolactin in the anterior pituitary Note: insulin and thyroxine contribute to the normal gestational mammary gland hypertrophy

Patients with advanced metastatic cancers or other terminal illness and a life expectancy of how many months should be evaluated for hospice care?

Prognosis less than or equal to 6 months

Appendicitis causes dull visceral pain at the umbilicus due to afferent pain fibers entering at the T10 level in the spinal cord. What changes the quality of pain in appendicitis?

Progressive inflammation in the appendix irritates the parietal peritoneum and abdominal wall to cause more severe somatic pain shifting from the umbilicus to McBurney point (2/3 of the distance from the umbilicus to the ASIS)

Long term treatment of Parkinson's disease with levodopa can be complicated by periodic and sometimes unpredictable fluctuations in motor function. This on-off phenomenon is thought to be a consequence of what?

Progressive nigrostriatal neurodegeneration leading to a decreased therapeutic window for levodopa Before the advanced stages of parkinson disease, the fluctuations appear to correlate with serum drug levels, however, they become independent of medication dosing/unpredictable with advanced disease

32 year old man presents with visual problems. Multiple 'near-misses' while driving being nearly hit by cars coming from both left and right sides. Complains of frequent headaches that respond to acetaminophen. Decreased lateral peripheral vision in PE. Which of following hormones most likely elevated in this patient?

Prolactin Most common pituitary adenomas are prolactinomas. Symptoms of elevated prolactin in men are mild and nonspecific usually and patients may not seek care until tumors are large and cause mass effect (headaches). Bitemporal hemianopsia occurs due to compression of optic chiasm. Hypopituitarism occurs due to compression of surrounding pituitary. Prolactinomas present with galactorrhea and amenorrhea in women & hypogonadism/infertility, impotence, decreased libido in men.

28 year old woman with worsening SOB over the past week. Had a recent episode of hemoptysis. Repots ongoing vaginal bleeding after an uncomplicated vaginal deliver of her son 9 weeks ago. On exam uterus is enlarged. Labs show markedly increased B-hCG. CXR show multiple bilateral lung nodules. Most likely to be found on endometrial curettage in this patient?

Proliferation of cytotrophoblasts and syncytiotrophoblasts Choriocarcinoma is a malignant form of gestational trophoblastic disease composed of anaplastic cytotrophoblasts and synctiotrophoblasts without villi. It often presents as dyspnea/hemoptysis due to pulmonary metastasis from hematogenous spread. Most commonly preceded by normal pregnancy but can occur following any pregnancy (molar, ectopic, aborted). Causes abnormal vaginal bleeding, uterine enlargement, & significantly increased B-hCG levels. Bulky intrauterine mass thats soft and yellow white, with extensive areas of necrosis and hemorrhage.

Bleeding due to Heparin toxicity is treated with?

Promatine sulfate (specific antagonist of Heparin)

Classic triad of fever, stiff neck, and altered mental status should raise suspicion for acute bacterial meningitis. Evaluation requires?

Prompt blood cultures, empiric antibiotics, and lumbar puncture with CSF analysis

52 year old woman with hand tremor that has been persistent for several months. Most prominent when she is carrying out simple daily activities such as drinking from a glass or pouring from a tea kettle. Reports tremor subsides somewhat when she drinks a small amount of alcohol. Mother had similar problems. Exam shows bilateral tremor in the UE. Best treatment for this patient?

Propanalol Essential tremor is the most common movement disorder. Patients experience a slowly progressive, symmetric postural and/or kinetic tremor that most commonly affects the upper extremities. Essential tremor is often inherited in an autosomal dominant fashion (referred to as familial tremor). First line treatment is nonspecific beta adrenergic antagonist Propanalol.

Physician prescribes the appropriate medication to lower severe hypertriglyceridemia and explains that the patient is likely to experience skin flushing and warmth after taking the pulls. Primary agent mediating these side effects?

Prostaglandin Niacin is used in the treatment of hyperlipidemia. It increases HDL levels and decreases LDL levels and triglycerides. Niacin causes cutaneous flushing, which is mediated by prostaglandins and can be diminished with pretreatment with aspirin.

43 year old male diagnosed with hepatic cirrhosis due to prolonged heavy alcohol consumption. Develops massive ascites and lower extremity edema that responds well to furosemide therapy. A week later he receives high dose ibuprofen for joint pains and soon develops worsening abdominal distension. Blunting of the diuretic response in this patient is due to interruption of what substance?

Prostaglandins Furosemide is a loop diuretic that works by inhibiting NA/K/2Cl supporters in the loop of hence effectively causing increased Na, Cl, and fluid exertion. Additionally, loop diuretics also stimulate prostaglandin release. By stimulating renal prostaglandin release, loop diuretics also increase renal blood flow leading to increased GFR and enhanced drug delivery. Thus concurrent use of NSAIDs with loop diuretics can result in a decreased diuretic response.

70 year old main with 3 weeks of terrible low back pain. Reports having to strain to urinate. Imaging reveals several osteoblastic lesions in lumbar vertebrae. What structure was most likely involved during spread of disease?

Prostatic venous plexus Skeletal system is a common site of metastasis due to hematogenous seeding (not lymphatic spread!). Cancers of the pelvis, including the prostate as in this patient, spread to the lumbosacral spine via the vertebral venous plexus which communicated with a number of venous networks (including the prostatic venous plexus which receives venous supply from prostate, penis, bladder)

72 year old female is started on warfarin for stroke prevention. Two days later she is hospitalized with severe skin and subcutaneous fat necrosis. Drug effects on what process are most likely responsible for her skin findings?

Protein C activity Warfarin induced skin necrosis is a rare but important complication of warfarin initiation that is thought to be due to a transient hyper coagulable state that can occur in the first few days of warfarin therapy. Warfarin inhibits protein C and S (natural anticoagulants in the blood via proteolysis and deactivation of factors V and VIII). Protein C has a a short half life so its anticoagulant activity is reduced quickly (VII has short half life as well). II, IX, and X have longer half lives. The combination of these two leads to procoagulant state. Thrombosis and clot can interrupt flow flow to the skin causing necrosis. For this reason overlap with Heparin when Warfarin is started. Risk of skin necrosis is increased in patients with preexisting protein C deficiency and in lose started on a large loading dose of Warfarin

Why does Haemophilus influenza serotype b vaccine consist of capsular polysaccharide conjugated to carrier protein (tetanus toxoid protein or outer membrane protein of Neisseria meningitidis)?

Protein conjugation causes a T cell mediated immune response leading to long term immunity through production of memory B-lymphocytes. Children less than 2 have immature humoral immunity so vaccines with polysaccharide antigen alone are ineffective (humeral response is amplified through T cell recruitment)

Insulin is an anabolic hormone that acts via receptor tyrosine kinase signaling to increase the synthesis of glycogen, proteins, FA and nucleic acids. Tyrosine kinase/PI3K stimulation promotes glycogen synthesis by activating what enzyme that dephosphorylates and activates glycogen synthase?

Protein phosphatase

Urea cycle defects typically cause neurological damage due to accumulation of ammonia. Nutritional restriction of what substances can improve the condition?

Protein restriction (reduces the amount of amino acid turnover)

Metabolism of protein, carbs, fat, and ethanol yield how many calories per gram?

Protein- 4 Carbs- 4 Ethanol- 7 Fat- 9

Peritrichous flagella refers to flagella distributed uniformly over the entire surface of a bacterial cell and are characteristic of highly motile organisms such as?

Proteus mirabilis

Most duodenal peptic ulcers are caused by H. pylori infection. Most effective method to prevent disease reoccurrence is to to eradicate the infection with antibiotics (tetracycline, metroizazole) typically in combination with what other drugs?

Proton pump inhibitors & bismuth subsalicylate

Drug class that block the final common pathway of gastric acid secretion from parietal cells, which is stimulated by acetylcholine, histamine, & gastrin

Proton pump inhibitors (omeprazole, lansoprazole)

At low pO2 hemoglobin molecule is fully deoxygenated and binding of the first O2 is difficult. After O2 binds to one of the 4 heme moieties, the oxygen affinity of the other subunits increases. Additional O2 molecules bind as oxygen partial pressure increases. What do hemoglobin molecules release as they bind oxygen?

Protons In the lungs, the binding of oxygen to hemoglobin drives the release of H+ and CO2 from hemoglobin (haldane effect). In peripheral tissues, high concentrations of CO2 and H+ facilitate unloading of O2 from hemoglobin (Bohr effect)

Patient is requesting an abortion because caring for a child is the last thing she needs right now. The gynecologist prefers not to perform abortions on moral grounds but does have training in the procedure. Most appropriate course of action of gynecologist?

Provide patient with resources for providers who will perform abortions Physicians are not required to provide medical services against their personal beliefs but should provide referral to a provider who will perform the procedure

Typical presentation of Zika virus?

Pruritic maculopapular rash, low grade fever, arthralgias, conjunctivitis

Mimic immunologic reactions but occursr via non-immunologic mechanism (ex: direct immune cell activation, inhibited prostaglandin synthesis). Examples include rhinitis with NSAIDS, anaphylaxis to radiocontrast media, and pruritus with opiates

Pseudoallergic drug reaction

Patient with acute mono or oligoarticular arthritis characterized by pain, warmth, erythema, and swelling. Synovial fluid analysis reveals elevated WBC count with neutrophilic predominance and rhomboid shaped calcium pyrophosphate crystals. Diagnosis?

Pseudogout (these crystals are positively bifringent under polarized light & blue when aligned parallel and yellow when aligned perpendicular; gout is negatively bifringent & needle shaped) Knee is involved in over 50% of cases whereas with regular gout the MTP joint is more frequently involved

Nonlactose-fermenting, oxidase positive motile gram negative rod

Pseudomonas aeruginosa

Organism that most commonly causes malignant otitis externa?

Pseudomonas aeruginosa

Wound tissue of burn patient grows oxidase positive colonies that do not ferment lactose on MacConkey agar. Most likely organism?

Pseudomonas aeruginosa (gram negative rods) Only few antibiotics can treat it! MacConkey agar selects for gram negative organisms.

Presents with elevated ICP & classical occurs in young women who are overweight. Related to cerebral venous outflow abnormalities due to elevated ICP

Pseudotumor cerebri

26 year old man with fever, right flank pain, and difficulty walking for the past 3 days. Was treated with cephalexin for furunculosis one week ago. Was diagnosed with T1DM ten years ago and takes SQ insulin. Patient lies supine with his right knee flexed and externally rotated. He resists simultaneous extension of the leg and thigh particularly at the hip. The pathological process occurring in this patient most likely involves what muscle?

Psoas major A psoas abscess can occur as the result of hematogenous or lymphatic seeding from distant site or by spread from an adjacent site. Patients can present with fever, back or flank pain, inguinal mass, and difficulty walking. Inflammation of the psoas muscle causes pain with the hip is extended (psoas sign). Risk factors include DM, IV drug use, HIV, other forms of immunosuppression.

28 year old male with bilateral skin lesions on elbows. Biopsy demonstrates thin stratum granulosum and prominent parakeratotic stratum corneum with occasional foci of neutrophil accumulation. Most likely diagnosis?

Psoriasis Characterized by hyperparakeratosis, ancanthosis, rete ridge elongation, mitotic activity above the epidermal basal cell layer, and a reduced or absent stratum granulosum. Neutrophils may for spongiotic clusters in the superficial dermis and the parakeratotic stratum corneum (MUNRO microabscesses). Epidermal cell layer superior to dermal papillae may be thinned and contain dilated blood vessels, a finding responsible for the pinpoint bleeding evident when the silvery scale is removed from the salmon colored plaque (Auspitz sign)

28 year old seen in postpartum clinic one week after spontaneous vaginal delivery under epidural analgesia that was complicated by prolonged second stage labor. Patient has had frequent fecal incontinence since delivery as well as mild perineal pain relieved by ibuprofen. Healing perineal laceration is noted on exam. Rectal exam reveals decreased anal sphincter tone and loss of anal wink reflex. Bilateral LE reflexes are 2+ with normal tone. Most likely cause of fecal incontinence?

Pudendal nerve injury Stretch of the pudendal nerve may occur due to stress placed on the pelvic floor during labor. Pudendal nerve injury leads to weakness of the perineal musculature causing fecal and urinary incontinence, perineal pain, and sexual dysfunction

Obstructive sleep apnea presents in obese individuals with excessive daytime sleepiness and signs of nocturnal upper airway obstruction (snoring, gasping). Its associated with systemic HTN and prolonged, untreated disease can also cause?

Pulmonary HTN and right heart failure Reduced ventilation causes transient hypercapnia and hypoxemia which results in reflexive systemic and pulmonary vasoconstriction, endothelial dysfunction, abnormal venous return & cardiac output, and sympathetic cardiac stimulation.

Caused by inhalation of spores most commonly while working with goat hair or hides. Symptoms initially consist of myalgia, fever, and malaise, but rapidly process to hemorrhagic mediastinitis (widened mediastinum on CXR), bloody pleural effusions, septic shock, & death.

Pulmonary anthrax aka 'woolsorters disease' Caused by Bacillus anthracis that forms long chains describes as 'serpentine' or 'medusa head' in appearance on microscopy. Produces D-GLUTAMATE antiphagocytic capsule that is required for pathogenicity.

A significant mismatched defect on a ventilation-perfusion scan is a specific finding for?

Pulmonary embolism (which is usually the result of a DVT and part of a continuum of the same disease, known as pulmonary thromboembolism)

Reduction in the slope of the curve depicting lung volume vs. distending pressure indicates decreased lung compliance which is the hallmark of what condition?

Pulmonary fibrosis

What structure could be pierced by a penetrating injury to the 2nd intercostal space at the left sternal border?

Pulmonary trunk

12 year old found to have a wide, fixed splitting of the second heart sound (S2) on PE. If present, the congenital heart disease in this patient may require surgical repair to prevent irreversible changes in what structure?

Pulmonary vessels Wide fixed splitting of the 2nd heart sound is a characteristic osculatory finding in patients with ASD. A hemodynamically significant ASD can produce chronic pulmonary HTN as a result of left to right intracardiac shunting. Eisenmenger syndrome is the late onset reversal of a left to right shunt due to pulmonary vascular sclerosis resulting from chronic pulmonary HTN. Closure of the ASD may be required to prevent irreversible pulmonary vascular sclerosis and a permanent Eisenmenger syndrome (associated with late onset cyanosis with clubbing and polycythemia) Note: Pulmonary HTN due to an ASD can result in RVH and RA enlargement, however these changes are not necessarily irreversible. If pulmonary HTN is corrected the heart can revert to a more normal morphology

In normal physiology, venous return to the right side of the heart increases during inspiration resulting in delayed closure of the pulmonic valve in relation to aortic valve closure. Inspiratory splitting of the aortic and pulmonic components of S2 can be appreciated on auscultation. What condition delays pulmonic valve closure even further and accentuate the normal splitting of S2?

Pulmonic valve stenosis

Anovulation due to hypogonadotropoc hypogonadism is a common form of infertility that can be treated with?

Pulsatile administration of GnRH Stimulates FH and LSH release and ovulation occurs in 10-20 days. Signs of ovulation are loss of dominant follicle on US, surge in urine LH levels, rise in serum progesterone, and rise in basal body temperature. Constant infusion of GnRH (or a long acting analog) suppress FSH and LH and suppress gonadal function (used for prostate cancer, endometriosis, precocious puberty, premenopausal breast cancer)

74 year old with an episode of bright red blood. On colonoscopy, he is found to have several small pouches protruding from the lumen of the sigmoid colon. The walls are composed of mucosal and submucosal layers that are covered by serosa. Patients diverticula are best characterized as what term?

Pulsion Colonic diverticula often involve the sigmoid and develop due to increased intraluminal pressure (pulsion) created during strained bowel movements (due to chronic constipation, for example) causing outputting of the mucosa and submucosa through areas of focal weakness in the muscularis (false diverticula). Individuals (typically >60) may be asymptomatic or present wit hematochezia or diverticulitis. Note: traction diverticula are created by inflammation and subsequent scarring of the gut wall, which typically results in pulling and outpouching of all gut wall layers (true diverticula). This may occur in the midesophagus due to mediastinal lyqmohadenitits caused by TB or fungal infection

Describes the fact that a researcher's beliefs in the efficacy of treatment can potentially affect the outcome

Pygmalion effect

X-linked agammaglobulinemia is characterized by low or absent circulating CD19 and CD20 B cells & panhypogammaglobulinemia. Affected patients have increased susceptibility to?

Pyogenic bacteria, enteroviruses, and Giardia lamblia due to the absence of opsonizing and neutralizing antibodies

Vitamin that serves as a cofactor in AA transamination & decarboxylation reactions?

Pyridoxal phosphate (B6) Note: Transamination reactions occur between AA and alpha keto acid. Amino group from AA is transferred to alpha keto acid, and alpha keto acid becomes AA. Example is glutamate (AA) reacts with oxaloacetate (alpha-ketoacid) to form aspartate (resulting AA) and alpha ketogluterate (resulting alpha ketoacid)

Homocystinuria is most frequently caused by an autosomal recessive deficiency of cystathionine beta-synthase which requires what cofactor?

Pyridoxine (B6) Note: 50% of patients respond to high doses of pyridoxine which improves residual enzyme activity & reduces plasma homocysteine levels. Additional treatment includes dietary restrictions of methionine.

HIV patient with new onset seizures & multiple ring enhancing lesions with mass effect likely has cerebral toxoplasmosis. First line treatment?

Pyrimethamine & sulfadiazine along with leucovorin Lack of clinical or imaging response after 1-3 weeks of empiric treatment for toxoplasmosis requires further eval (especially if toxoplasmosis serologies are negative). Primary central nervous system lymphoma (composed of B-lymphocytes) is second most common cause of ring engaging lesions with mass effect in HIV.

Syndrome characterized by poor feeding, developmental delay, seizures, abnormal eye movements, ataxia, and mental delays. Elevated levels of lactic acid due to inability to convert pyruvate to acetyl CoA.

Pyruvate dehydrogenase deficiency (Leigh syndrome)

Enzymes that require Thiamine and are impaired in alcoholics?

Pyruvate dehydrogenase, alpha ketoglutarate dehydrogenase, and transketolase. Administration of glucose to a thiamine deficient patient can result in Wernicke encephalopathy (confusion, ophthalmoplegia, ataxia) due to increased thiamine demand. Metabolism of ethanol increases the NADH to NAD+ ratio inhibiting the entire citric acid cycle, but in the setting of wernicke's thiamine dependent dependent enzymes are especially effected due to the lack of NAD+ AND thiamine

Patient with acute mesenteric ischemia has elevated lactic acid. Decreased activity of what enzyme best explains this acid/base disorder?

Pyruvate dehydrogenase. Under hypoxic conditions intracellular accumulation of NADH inhibits pyruvate dehydrogenase & pyruvate is converted to lactate by LDH

In glycolysis pyruvate is formed from phosphenylpyruvate by what unidirectional enzyme?

Pyruvate kinase

32 year old with myalgia, anorexia, skin rash. Has been consuming large amount of raw egg white for past several months. PE shows macular dermatitis of extremities. Water soluble vitamin deficiency is suspected. What biochemical conversions would use the deficient vitamin as a cofactor?

Pyruvate to oxaloacetate Biotin (B7) is an important cofactor for several CARBOXYLASE enzymes. Biotin acts as a CO2 carrier on the surface of carboxylase enzymes and is essential cofactor for numerous reactions, including the conversion of pyruvate to oxaloacetate and FA metabolism. Excess ingestion of avidin found in egg whites has been associated with biotin deficiency which presents with mental status changes, myalgias, anorexia, macular dermatitis, and lactic acidosis (increased conversion of pyruvate to lactic acid)

Path of blood circulating in kidney?

R - S - I - A - I - A - G - E - V/P - V Rainbow stinks in all I Average gorillas eat poop, vomit renal artery --> segmental artery --> interlobar arteries --> arcuate arteries --> radial arteries --> afferent arterioles --> glomerular capillaries --> efferent arterioles --> peritubular capillaries --> interlobar veins

Renal blood flow equation

RBF= renal plasma flow/(1-hematocrit) Renal plasma flow = PAH clearance = (urine [PAH] x urine flow rate)/plasma [PAH]

Medullary thyroid cancer is a neuroendocrine tumor that arises from calcitonin secreting C (parafollicular) cells. Its often seen in the context of MEN 2A and 2B, although majority of cases are sporadic. Both sporadic and familial medullary thyroid cancers are associated with mutations in what protooncogene?

RET

What enzyme functions exclusively in nucleolous?

RNA polymerase I

Patient complaining of abdominal pain, vomiting, severe watery diarrhea. Recently returned from camping trip where ehe ate wild mushrooms. Jaundiced on PE with enlarged liver. Elevated ALT, AST, and bilirubin. Toxin directly inhibits what?

RNA polymerase II (halting mRNA synthesis) Toxin ingested was amatoxin which are found in a variety of poisonous mushrooms (Amanita phalloides, known as death cap).

25 year old male fractured his right tibia. His right leg is now in a case and requires underarm crutches to maculate. Two weeks later he develops right upper extremity weakness and numbness. On exam there is diminished strength on extension of the hight wrist and absent triceps reflex. Injury to what nerve is most likely responsible for his symptoms?

Radial nerve Radial nerve injury can occur with repetitive pressure/trauma at the axilla (improperly cited crutches). Findings include weakness of the forearm, hand, and finger extensors (wrist drop, absent triceps reflex) and sensory loss over the posterior arm, dorsolateral hand, and dorsal thumb. More distal lesions spare the triceps brachii.

What drug is most appropriate for decreasing risk of bone fractures and breast cancer?

Raloxifene (has estrogen agonist activity on bone which decreases bone resorption and improves bone density and has an estrogen antagonist on breast tissue & can decrease risk of breast cancer) Note: Tamoxifen also has estrogen antagonist activity in the breast and is used in treatment of ER+ breast cancer and can reduce risk of osteoporosis, however, it has agonist activity on the uterus and increases risk of endometrial hyperplasia/cancer so its not appropriate for routine use in osteoporosis. Raloxifine acts as an estrogen antagonist in the uterus and does not increase risk of endometrial cancer!

Melatonin agonist that is safe & effective drug for reducing time to sleep onset in elderly?

Ramelteon

Antianginal agent that exits its effect by inhibiting the late phase of the inward sodium channels in ischemic cardiac myocytes. Also blocks K+ channels and causes a dose dependent increase in QT interval (however, torsades has not been reported)

Ranolazine

Where are serotonin releasing neurons located in the CNS?

Raphe nuclei of brainstem (these neurons disseminate widely to synapse on numerous structure in the CNS & are the target of many antidepressants such as TCAs, SSRIs, SNRIs which inhibit serotonin reuptake) Note: Caudate nucleus & putamen- cholinergic & GABA neurons Locus ceruleus- norepinephrine neurons Nucleus basalis of Meynart- cholinergic neurons Substantia nigra- dopaminergic neurons Red nucleus neurons participate in motor coordination of the UE

How to calculate attack rate?

Ratio of the number of people who contract an illness divided by the number of people who are at risk of contracting the illness For example attack rate for developing N/V after eating potato salad is number of people who ate potato salad (alone or in combo with other food) who developed N/V divided by the total number of people who ate potato salad

In chronic bronchitis there is progressive thickening of the walls of the bronchi and bronchioles. Severity is largely dependent upon the extent to which the luminal diameter of the bronchi and bronchioles is decreased. Major contributor to this wall thickening is submucosal gland enlargement which can be quantified by?

Ratio of thickness of the mucous gland layer to thickness of wall between the epithelium and cartilage (Reid index). Reid index is normally 0.4. Elevations of Reid index above normal value of 40% correlate well with the duration and severity of chronic bronchitis.

Defense mechanism that involves replacing unacceptable feelings and impulses with their extreme opposites

Reaction formation

Transformation of an unacceptable impulse or emotion into its opposite

Reaction formation

17 year old man with a painful subcutaneous nodule on his left forearm. Two weeks earlier, he suffered a laceration to his forearm while playing soccer and had sutures placed. A biopsy of the lesion is taken and a granulomatous response is seen. Most likely responsible for this patient's condition?

Reaction to foreign body Granulomatous inflammation is a form of chronic inflammation characterized by aggregates of activated macrophages that assume an epitheliod appearance. Foreign bodies (retained sutures) can elect a granulomatous response. Persistent granulomatous inflammation with subsequent fibrosis can cause organ dysfunction which is seen in a number of granulomatous diseases.

44 year old man with several weeks of difficulty walking and frequent falls. Repots episodes of sharp, stabbing pain in his extremities. Has HIV acquired from unprotected sexual contact and is not adherent with treatment. 5/5 motor strength in all extremities. DTRs absent at knee and ankle bilaterally. Proprioception and vibration are reduced throughout LE. Has a wide based gait and positive Romberg sign. What diagnostic finding would be most associated with his current symptoms?

Reactive VDRL tests on CSF samples Tabes dorsalis, a late form of neurosyphilis, causes progressive degeneration of the dorsal columns and dorsal roots of the spinal cord. Manifestations include loss of proprioception and vibratory sensation, severe lancinating pains, and sensory ataxia (wide based gait, positive Romberg sign). Many patients also have Argyll Robertson pupils in which small irregularly shaped pupils fail to constrict to light but constrict normally to accommodation.

Young man with acute lower extremity arthritis & sterile joint effusion following GI infection. Associated symptoms are conjunctivitis, urethritis, and kertoderma blennorrhagicum. Most likely diagnosis? Preferentially affects what individuals?

Reactive arthritis preferentially affects HLA-B27 positive individuals. Occurs following infection with Chlamydia, Clostridium difficile, Campylobacter, Salmonella, Shigella, Bartonella, or Yersinia. Presents with sterile arthritis due to deposition of immune complexes

Postpartum blues is a benign self limited condition that begins several days postpartum and resolves without intervention. Symptoms include mild depressive symptoms such as sadness, insomnia, tearfulness, irritability, anxiety and impaired concentration. Symptoms peak at 5 days and resolve usually within 10 days. If woman calls who has given birth five days earlier complaining of these symptoms what should you do?

Reassure her that postpartum mood changes are common and encourage her to call in a few days if the symptoms don't remit spontaneously

Bias that results from study participants' inaccurate recall of past exposure and occurs most often in retrospective studies such as case control studies.

Recall bias Note: people who have experienced an adverse event are more likely to recall risk factors than those who have not experienced an adverse event

67 year old man with right tibial pain for the past 3 months that has gotten worse over time. Has also had progressive hearing impairment for the past year. PE reveals local tenderness and a lumpy protuberance over the right tibia. Bone biopsy reveals numerous multinucleated giant cells, some containing over 100 nuclei. What factor is essential for differentiation of the cells described in biopsy?

Receptor activator of nuclear factor kappa-B ligand Patient has deformity of long bone with hearing loss due to deformity of the skull, typical fractures of Pagets disease of the bone. Osteoclasts originate from hematopoietic progenitor cells. Macrophage colony stimulating factor and RANK-L play an important role in osteoclast differentiation. Paget's disease of bone is characterized by increased numbers of abnormal osteoclasts, excessive bone turnover, and disorganized bone remodeling. Note: Osteoprotegerin is a physiologic decoy receptor that decreases binding of RANK-L to RANK (reduces differentiation and survival of osteoclasts resulting in increased bone density; osteoprotegerin loss of function mutations cause juvenile pagets disease. Monoclonal antibody, Denosumab, inhibits RANK-L/RANK interaction leads to increased bone density and is commonly used for treatment of osteoporosis

Refers to gene exchange that occurs through the crossing over within homologous regions of 2 nonsegmented double stranded DNA molecules (subsequent progeny have recombined genomes with traits from both parent viruses)

Recombination (example two HSV types would undergo recombination since they are nofsegmented) Note: reassortment is the mixing of whole genome segments in SEGMENTED viruses that affect the same cell Note: interference is when a virus inhibits replication and/or release of a second virus that is infecting the same cell Note: Phenotypic mixing occurs when a host cell is coinfected with 2 viral strains and progeny virions contain parental genome from one stain and nucleocapsid (or envelope) proteins from the other strain (progeny can acquire additional traits), but the genome is unchanged and subsequent progeny would not retain these traits. Note: transformation is the uptake of viral DNA (also describes incorporation of viral DNA into a host cell chromosome aka lysogeny); alters genetic composition of the host cell but doesn't typically call genomic change in progeny viruses

The presence of anti-HBc and anti-HBs antibodies in the serum without detectable viral antigens indicates?

Recovery from acute hepatitis B infection Note: patients vaccinated against hepatitis B will have anti-HBs antibodies without detectable levels of anti-HBc & chronic hepatitis B is indicated by persistent levels of HBsAg and HBV DNA in the serum (see phone for pics)

Why do drugs that undergo extensive first pass metabolism have increased bioavailability when administered rectally?

Rectal administration is capable of partially bypassing first pass metabolism as the region BELOW the dentate line drains into the SYSTEMIC circulation via the middle (drains to internal iliac vein) & inferior (drains to internal pudendal vein) rectal veins. (NOTE: venous drainage ABOVE dentate line is to the portal venous system via the superior rectal veins which drain to inferior mesenteric vein)

Gold standard for diagnosing Hirschsprung disease/congenital ganglionic megacolon in which neonates classically present with failure to pass meconium in the first 48 hours or life (less severe disease may present later with chronic constipation and failure to thrive)?

Rectal suction biopsy

Its suggested that some unknown factors may specifically interrupt the normal migration of neural crest cells to the intestine. This migration process is typically complete by the 12th week. If progress is interrupted during the final week of migration, what organ is most likely to be affected?

Rectum Hirschsprung disease is a result of abnormal migration of neural crest cells during embryogenesis. These cells are the precursors of ganglion cells of the intestinal wall plexi. Since neural crest cells migrate caudal, the rectum is always affected in Hirschsprung disease.

The valsalva maneuver increases vagal tone and can be used to abolish paroxysmal supraventricular tachycardia. Most important muscle in achieving the increased intraabdominal and intrathroacic pressure of the valsalva maneuver?

Rectus abdominis

Primary infection with HSV-2 typically presents with fever and a painful vesicular genital rash. HSV-s primarily infects the sacral dorsal root ganglia and can be reactivated to cause?

Recurrent genital lesions (tends to be localized and less severe due to humoral immunity from prior infection)

During an attempt to ligate inferior thyroid artery, a nervous structure in close proximity is inadvertently damaged. Most likely injured structure?

Recurrent laryngeal nerve (causes hoarseness) Note: the external branch of the superior laryngeal nerve may be militated during thyroid surgery due to its close proximity to the superior thyroid artery

COPD is characterized by markedly reduced expiratory airflow velocity, increased residual volume, and limitation of tidal volume due to overinflation. Chronic bronchitis and/or emphysematous destruction of inter alveolar walls are responsible for the airflow limitation. Pattern on respiratory flow volume curve?

Red line on picture

Direct effects of opiate induced analgesia on spinal cord neurons?

Reduced Ca2+ influx presynaptically & increased K+ efflux postsynaptically Opiate analgesics reduce pain by binding to mu receptors and inhibiting synaptic activity in the CNS> Activation of presynaptic mu receptors on the primary afferent neuron leads to closure of voltage gated Ca2+ channels and reduced expiatory neurotransmitter release. Binding to mu receptors on postsynaptic membrane causes opening of K+ channels and membrane hyper polarization Note: local anesthetics reduce pain transmission by blocking voltage gated Na+ channels

Bias that occurs when the case and control populations differ due to admission or referral practices (example: studying involving asbestos as a risk factor for lung cancer conducted at a hospital specializing in treating asbestosis may select patients with lung cancer from the respiratory department and a control group without lung cancer from other departments and may therefore erroneously report a stronger association than actually exists)

Referral (admission) bias

Patient with DKA is started on IV fluid and IV infusion of recombinant human insulin without amino acid modification. This same type of insulin will be used subcutaneously in long term management of this patient. Type of insulin?

Regular insulin DKA is treated with IV infusion of regular insulin that lowers serum glucose and ketone levels within minutes. Regular insulin can be used subcutaneously in long term management of DM, usually in combo with along acting insulin analog. When given SQ is starts working in 30 minutes and lasts 5-8 hours. Note: regular insulin and NPH are the only human recombinant insulins without any amino acid modifications. NPH starts working in about 2 hours.

Patient with heart failure hospitalized and on diuretics. Hematocrit increases from 44% to 55% on day 3 of admission. ABG shows PaO2 of 70 mmHg. Normal red blood cell mass. Most likely cause of polycythemia?

Relative erythrocytosis (plasma volume contraction from diuretics) Erythrocytosis is defined as hematocrit >52% in men and >48% in women Normal RBC mass = plasma volume contraction Increase in RBC mass = absolute erythrocytosis Primary erythrocytosis = low EPO (polycythemia vera) Secondary erythrocytosis = high EPO (chronic hypoxia from high altitudes, smoking, COPD or abnormal secretion of EPO) Hypoxic secondary erythrocytosis = SaO2 < 92% aka PaO2 <65 mmHg

Most common trigger for DIC in pregnancy?

Release of tissue factor (thromboplastin) from an injured placenta into maternal circulation

7 year old boy with growth retardation. MRI shows 4cm multiloculated, cystic, suprasellar lesion bulging into the flood of the 3rd ventricle and base of the brain. Calcifications are present. Which structure is this mass most likely derived from?

Remnants of Rathke pouch Craniopharyngiomas arise from Rathke's ouch remnants in the anterior pituitary & characteristically have solid, cystic, and calcified components. Present between ages 5 and 10 with mass effect and visual defects. Anterior pituitary is derived from an outputting of the pharyngeal roof and is called Rathke's pouch. Posterior pituitary arises from extension of hypothalamic neurons

Patient's with SLE can have autoantibodies directed against snRNPs (anti-Smith antibody) which impairs what cellular function?

Removal of introns from RNA transcripts snRNA is synthesized by RNA polymerase II in the nucleus and complexes with specific proteins form small nuclear ribonucleoproteins (snRNPs), which are essential components of splicosomes.

Patient involved in MVC in persistent vegetative state. Feeding tube placed while efforts are made to contact with who is currently away. When she arrives she says patients doesn't have a wall but reports that he didn't want tubes in him to keep him alive. Children are demanding everything be done to keep him alive although they acknowledge he did not wish to live with tubes. Best course of action?

Remove tube in accordance with patient's wishes as described by wife When patents are unable to make end of life decisions and no written advanced directive exists, responsibility for decisions falls to next of kin. All decisions must be based on best estimation of what patient would have wanted (autonomy)

Newborn with tachypnea and hypoxia. Born at 39 weeks gestation via cesearan section due to variable decelerations. PE shows flattened nose and bilateral club feet. Breast sounds markedly diminished bilaterally. Intubated and mechanically ventilated but dies one hour later. Most likely biopsy finding?

Renal agenesis Potter sequence results from a renal anomaly that cases decreased fetal urine output leading to oligohydramnios. Lack of amniotic fluid causes compression of the fetus (facial and limb abnormalities) and pulmonary hypoplasia, which is the most common cause of death in affected infants

What artery supplies blood to the proximal portion of the ureter?

Renal artery Note: distal ureter is supplied by the superior vesicle artery. Circulation to the middle portions of the ureter is variable and anastomotic (gonadal, common iliac, internal iliac, aorta, and uterine can be involved)

57 year old man with several episodes of red colored urine over the last month. Has smoked 1PPD for the past 30 years. Enlarged firm prostate is palpated on DRE. Hematocrit is 56% and UA shows hematuria. CT scan shows left sided renal mass. Most likely diagnosis?

Renal cell carcinoma Extensive smoking history, painless hematuria, polycythemia (elevated hematocrit from increased EPO produced by tumor), abdominal mass, flank pain, and weight loss are classic signs of RCC.

79 year old man with confusion for the past 2 days. Reports nausea, decreased appetite and poor oral intake for several weeks. Mentions everything around him has a 'yellow tint'. PMH significant for long standing HF with reduced EF and atrial fibrillation. On Metoprolol, lisinopril, digoxin, and apixaban. Age related changes in what factor most likely accounts for his symptoms?

Renal clearance Digoxin toxicity classically presents with visual changes (yellow tinting) and GI disturbances. It is a cardiac glycoside used to treat HF and afib that is predominantly cleared by the kidneys. Elderly patients exhibit age related renal insufficiency even in the presence of normal creatinine levels! Dose of digoxin must be reduced in elderly due to this!

47 year old man is treated for bacterial sinusitis with ampicillin. A week later develops fever and a skin rash. He also reports low urine output. PE shows a diffuse maculopapular rash. Serum creatinine is 2.4. Urine sediment microscopy reveals 3-4 RBCs/hpf, 5-10 WBCs/hpf, and 3-5 eosinophils/hpf. Pathologic process affecting this patient's kidneys most likely involves what structure?

Renal interstitium (causes interstitial edema and leukocyte infiltration; inflammatory cells commonly infiltrate the tubular epithelium and granuloma formation may be seen) Fever, maculopapular rash, and symptoms of acute renal failure (elevated creatinine, oliguria) 1-3 weeks after beginning treatment with a beta-lactam antibiotic or other specific drugs (quinolone antibiotics, NSAIDs, sulfonamides, rifampin, diuretics) are highly suggestive of drug induced acute interstitial nephritis. Peripheral eosinophilia and eosinophiluria are important clinical clues. Symptoms resolve completely after cessation of the offending medication.

Classically presents with gross hematuria, acute flank pain, and passage of tissue fragments in urine. Most commonly seen in patients with sickle cell disease or trait, DM, analgesic nephropathy, or severe obstructive pyelonephritis

Renal papillary necrosis Note: abrupt onset of gross hematuria in an otherwise healthy patient with a family history of sickle cell disease suggests renal papillary necrosis due to underlying sickle cell trait

Most notorious side effect of the polyene antifungal, Amphotericin B?

Renal toxicity (severe hypokalemia & hypomagnesemia are commonly seen during therapy and often require daily supplementation.

Patient with disseminated histoplasmosis is started on Amphotericin B. A day later develops palpitations and weakness and ECG reveals premature ventricular beats. If these new findings are related to Amphotericin B toxicity, they are likely related to dysfunction located where?

Renal tubular dysfunction Most dangerous adverse effect of Amphotericin B is nephrotoxicity which is due to both a decrease in GFR and direct effects on the tubular epithelium. Nephrotoxicity can lead to anemia (decreased EPO production) and electrolyte abnormalities. Hypokalemia and hypomagnesemia result due to increase in membrane permeability of the distal tubule. Hypokalemia leads to weakness and arrhythmias. EKG findings in hypokalemia include T wave flattening, ST segment depression, prominent U waves, and premature atrial and ventricular contractions. Profound hypokalemia can cause VT or v fib.

Combination of changes in repose to Enalapril?

Renin: increased AT 1: increased AT2: decreased Aldosterone: decreased Bradykinin: increased

Elevated serum aldosterone levels can manifest with HTN, hypokalemia, and muscle weakness. Increased levels of both renin and aldosterone are indicative of secondary hyperaldosteronism which can be caused by?

Renovascular disease, malignant HTN, & renin secreting tumors

Restoration of blood flow after clot lysis in acute MI using fibrinolytic drugs can lead to?

Reperfusion arrhythmia (typically accelerated idioventricular rhythm and BENIGN)

K+ ion permanence is highest during which phase of the action potential?

Repolarization

34 year old man undergoes pulmonary function testing for workup of dyspnea. PFT compared with a normal individual reveals and increased lung volume at the end of maximal exhalation. What would you expect to be increased in this patient?

Residual volume/total lung capacity ratio (indicating air trapping in the lungs. RV is the lung volume that remains after maximal expiration. RV is increased in COPD. Note: although the total lung capacity increases in obstructive lung disease, it does so primarily though expansion of the RV compartment and as a result the TLC/RV ratio would likely be decreased in obstruction lung disease.

24 year old African American woman with sickle cell disease complaining of exquisite pain in her right thigh. Patient has fever. RI reveals abnormal imaging results and antibiotics are promptly initiated. Blood cultures grow non lactose fermenting, oxidase negative, motile organisms. Virulence mechanism most likely contributing to this patient's current presentation?

Resistance to opsonization Salmonella is a non lactose fermenting, oxidase negative, motile, gram negative organism with a special capsule called the 'Vi antigen' that protects it from opsonization and phagocytosis. S. aureus is another common cause of osteomyelitis in patients with sickle cell disease, so antibiotic coverage should be directed against both types of bacteria

Acute salicylate intoxication presents with nausea/vomiting, confusion, dizziness, tinnitus, fever, & tachypnea. Two different acid base abnormalities that are characteristic are?

Respiratory ALKALOSIS occurs first since salicylates directly stimulate the medullary respiratory center resulting in increased ventilation and loss of CO2 in the expired air. Anion gap metabolic ACIDOSIS begins to develop 12 HOURS LATER since high concentrations of salicylates increase lipolysis, uncouple oxidative phosphorylation, and inhibit TCA cycle. Organic acids (ketoacids, lactate, pyruvate) accumulate in blood and bind bicarbonate increasing the calculated anion gap. ABG in salicylate toxicity often shows a pH in normal range due to this mixed respiratory alkalosis/metabolic acidsosis, very low PaCO2 (due to respiratory alkalosis & compensation for metabolic acidosis), and low HCO3- (due to metabolic acidosis).

23 year old man with progressive weakness and confusion. Roommate says he has been drinking water all the time and using the bathroom constantly since he became sick with a cold 2 days ago. Temperature is 100, BP is 96/58, pulse 112. PE reveals dry mucous membranes, and urine has a strong fruity odor. Na+ 134, K+ 3.8, Cl- 100, Bicarb is 14, glucose is 498, pH is 7.17, paCO2 is 40. Creatining is 1.0. What is most likely occurring in this patient based on lab findings?

Respiratory failure Patient has diabetic ketoacidosis. Anion gap metabolic acidosis confirms the diagnosis (134- [100+14] = 20 & normal is 10-14). Normally metabolic acidosis is partially compensated for by respiratory alkalosis by hyperventilating/dropping PaCO2. When the steady state PCO2 (in this patient its 40) persists above the range given by the winter formula (PCO2 = [1.5 *HCO3-] + 8 +/- 2) which in this patient would be 27-31, then the patient has superimposed respiratory acidosis (respiratory failure) interfering with normal respiratory compensation.

Manifests with painless transient monocular vision loss (amaurosis fugax) and is most commonly caused by atherosclerotic emboli originating from the ipsilateral carotid artery. Fundoscopy may show embolic plaques and retinal whitening. Typically affects older patients with vascular risk factors (HTN, hyperlipidemia, DM)

Retinal ischemia

External stimulus applied to a cell increases the activity of several enzymes, including dihydrofolate reductase and DNA polymerase. What immediately precedes the observed effect?

Retinoblastoma protein phosphorylation Proliferation signals activate CDK4, resulting in hyperphosphorylation of the Rb protein and release of E2F transcription factor. Because hyperphosphorylated Rb is inactive, cells are allowed to transition unchecked from the G1 phase to the S phase in the cell cycle

71 year old with 3 hours of persistent retrosternal CP and dyspnea. Diagnosed with acute MI and undergoes cardiac catheterization. During cannulation of the right common femoral artery, the arterial wall is penetrated superior to the right inguinal ligament. PCI performed. Soon after procedure, patient becomes cold, clammy, and hypotensive. PE shows 5cm region of ecchymosis surrounding femoral puncture site. Internal bleeding is suspected. Most likely location of the blood collection?

Retroperitoneal space Optimal site for obtaining vascular access in the lower extremity during cardiac catheterization is the common femoral artery BELOW the inguinal ligament. Cannulation above the inguinal ligament can significantly increase the risk of retroperitoneal hemorrhage, as the external iliac artery (continuation of femoral artery) lies directly inferior to the peritoneum at this site

Syndrome characterized by loss of speech and motor skills, deceleration of head growth, and stereotypic hand movements (wringing hands repeatedly) after a period of normal development until age 5-18 months. Affects mainly girls and is associated with mutations in the MECP2 gene.

Rett syndrome

Supplemental oxygen administration in patients with COPD can lead to increased CO2 retention (oxygen induced hypercapnia) resulting in confusion and depressed consciousness. Major cause of this?

Reversal of hypoxic pulmonary vasoconstriction, which increases physiologic dead space as blood is shunted away from well ventilated alveoli (and towards poorly ventilated alveoli/damaged from COPD)

Parallel shift to the right in the log-dose response cure indicating increase in ED50 & no change in maximum effect (Emax) are changes expected for the effect of what type of antagonist?

Reversible competitive antagonist Competitive antagonist--> change ED50 --> shift right Noncompetitive antagonist --> change Emax --> shift down

Strongyloides stercoralis infection begins following skin penetration by filariform (infectious) larva and can be diagnosed by finding what in the stool?

Rhabditiform (noninfectious) LARVAE in the stool. Rhabditiform larvae can mature into filariform larva in the human GI tract, precipitating an auto infection cycle that occurs entirely within the affected individual. This can result in a hyperinfection syndrome characterized by massive dissemination of the organism, leading to multi organ dysfunction and septic shock

Scarlet fever can predispose to?

Rheumatic fever and glomerulonephritis (Treat with Penicillin V to prevent development of RF!)

Characterized by diffuse fibrous thickening and distortion of the mitral valve leaflets along with commissural fusion at the leaflet edges. Patients often present with diastolic murmur, dyspnea, and fatigue and are at increased risk of atrial fibrillation and thromboembolism/stroke

Rheumatic mitral stenosis Cardiac auscultation in mitral stenosis reveals a loud first heart sound (S1), an early diastolic sound (opening snap), followed by a mid-diastolic murmur from turbulent flow across the mitral valve

Muscle innervated by dorsal scapular nerve (C5). Originates on spinous processes of T2-T5, inserts on medial border of the scapula, and draws scapula upward and medially

Rhomboid major

Treatment of chronic HCV?

Ribavirin and interferon alpha

23 year old presents after having a tonic clonic seizure. Has a history of seizures and takes phenytoin. Blood tests reveal her phenytoin plasma level is low. Patient reports being compliant with her medication. What drug could be possible for her condition if co-administered with phenytoin?

Rifampin P450 inducers: barbiturates, rifampin, carbamazepine, griseofulvin, and chronic alcohol consumption P450 inhibitors: isoniazid, cimetidine, macrolides, azole antifungals, grapefruit juice, omemprazole Phenytoin metabolism depends on the function of hepatic P450 oxidases and is dose dependent. Drugs that induce hepatic microsomal enzymes enhance phenytoin metabolism and decrease its serum concentration

Patient on Warfarin has a subtheraputic INR resulting in a recurrent pulmonary embolism. Possible medications she is concurrently taking with the Warfarin?

Rifampin, phenobarbital, phenytoin (universal enhancers of the cytochrome P-450 pathway which decrease the efficacy of Warfarin)

Forms the right lateral cardiac border on frontal CXR along with the superior vena cava

Right atrium

Patient with lobar pneumonia involving the right middle lobe. The mediastinal contour obscured by this lung consolidation on CXR would be primarily formed by what structure?

Right atrium Right middle lobe is adjacent to the right border of the heart which is primarily formed by the right atrium. Consolidation in the right middle lobe can obscure the x-ray silhouette of the right heart border

59 year old with crushing chest pain, sweating and lightheadedness. ECG shows bradycardia and ST segment elevation in II, III, avF. Occlusion of what coronary artery?

Right coronary artery In most individuals, occlusion of the RCA can result in transmural ischemia of the inferior wall of the LV producing ST elevation in II, III, aVF as well as possible sinus node dysfunction (this patient's bradycardia). Occlusion of the proximal LAD would be expected to result in anteroseptal transmural ischemia, with ST elevations in V1-V4. Occlusion of LCX would produce transmural ischemia of the lateral wall of the LV with ST elevations mainly in v5 and v6, possibly also I and aVL.

Forms the anterior (sternal) surface of the heart and the majority of its inferior border. A penetrating injury at the left sternal border in the 4th intercostal space would puncture this chamber.

Right ventricle

46 year old with chest pain and dyspnea on exertion. Left and right cardiac catheterization is planned. During procedure, catheter records period pressure changes with a maximum of 25mm Hg and a minimum of 2 mmHg. Catheter is advanced further and then shows periodic pressure changes with a maximum of 25 mmHg and a minimum of 10 mmHg. Assuming results are normal, first set of readings was most likely obtained from what location?

Right ventricle Right sided pressures in the heart are lower than left sided pressure due to lower resistance in the pulmonary vasculature. RV diastolic pressure is similar to right atrial/central venous pressure (1-6mm Hg) whereas pulmonary artery diastolic pressure is slightly higher (6-12 mmHg) due to resistance to flow in the pulmonary circulation Normal hemodynamic findings Right atrium: Mean 1-6 Right ventricle: Systolic 15-30 & Diastolic 1-6 Pulmonary artery: Systolic 15-30 & diastolic 6-12 Left atrial/PCWP/LVEDP: Mean 6-12 Left ventricle: Systolic 90-140 & diastolic 6-12

Major determinant of symptom severity in tetralogy of fallot?

Right ventricular outflow tract obstruction (infants with significant LVOT obstruction shunt more deoxygenated blood across the VSD and to the aorta and are more cyanotic)

Vasopressin and desmopressin cause a V2 receptor mediated increase in water and urea permeability at the inner medially collecting duct. Effect of increased urea permeability?

Rise in urea reabsorption (decrease urea clearance) enhances the medullary osmotic gradient, allowing the production of maximally concentrated urine

A married couple comes to the physician for prenatal counseling. Husband is 3 feet tall with disproportionately short upper and lower extremities, a large head, and a prominent forehead. Spouse is average height. They are concerned about unborn child's height. Best response to their concerns?

Risk of child to be short is about 50% Achondroplasia is an autosomal dominant disorder that results in gain of function mutation of the FGFR3 gene. Most individuals affected by autosomal dominant disorders have a 50% chance of transmitting the mutation to their offspring

Monoclonal antibody directed against the CD20 antigen which has improved the prognosis of some lymphomas

Rituximab

Quality improvement measure that identifies what, how, and why a preventable adverse outcome occurred. First step involves collecting data mainly through interviewing multiple individuals involved in the steps leading to the outcome

Root cause analysis (should be utilized when solving recurrent problems and preventing further harm to patients)

Location of oculomotor nucleus?

Rostral midbrain at the level of the superior colliculus and red nucleus Nerve exits at the interpenducular fossa

Rare autosomal recessive disorder characterized by asymptomatic conjugated hyperbilirubinemia that results from numerous defects in hepatic uptake and excretion of bilirubin pigments?

Rotor syndrome (jaundice but normal life expectancies)

Impaired collagen synthesis resulting from vitamin C deficiency (scurvy) can lead to fragile vessels, predisposing to gingival bleeding, ecchymosis, and petichaie. The hydroxylation of proline and lysine residues in collagen helps it attain its maximum strength and requires vitamin c as a cofactor. Where does this hydroxylation occur?

Rough endoplasmic reticulum

Cryptococcus neoformans causes meningoencephalitis in patients with untreated AIDS. Diagnosis can be made by detecting the polysaccharide capsule in CSF using the latex agglutination test. India ink stain of CSF is also used for diagnosis and would demonstrate?

Round or oval budding yeast

Most common renal malignancy is clear cell carcinoma (commonly presents with gross painless hematuria in older adult) which arises in renal proximal tubular cells. Findings on histopathology?

Rounded or polygonal cells with abundant clear cytoplasm seen on LM. 'Clear cells' have high glycogen or lipid content (see photo) that dissolves during routine tissue preparation.

Rotavirus less common than Norovirus in developed countries due to?

Routine childhood vaccine

Antimetabolies Methotrexate and 5-FU are specific to what cell cycle phase?

S-phase

Topoisomerase I (irinotecan) and II (etoposide) inhibitors are specific to what cell cycle phase?

S-phase

Protein A is a virulence factor produced by what organism which has the ability to bind to the Fc portion of IgG and prevent opsonization and complement mediated killing of the bacteria?

S. Aureus

Rapid onset nausea, vomiting, and abdominal cramping after eating mayonnaise containing product is like due to?

S. aureus heat stable 'PREFORMED EXOTOXIN'

Influenza infection alters the respiratory epithelium and can increase the risk of secondary bacterial pneumonia. Leading pathogens?

S. pneumoniae, S. aureus, H. influenzae The elderly are affected most commonly, but S. aureus can cause secondary pneumonia in young, previously healthy patients

14 year old girl with dark urine and facial puffiness. 4 weeks ago she had pustular skin lesions that broke down over a few days to form thick scabs on the low extremities. Microscopic exam of urine sediment shows RBC casts. Organism responsible for this patient symptoms is pyrrolidonyl arylamidase positive and is?

S. pyogenes

Low frequency diastolic sound that occurs during atrial kick of ventricular diastole & reflects flood colliding with a stiff ventricular wall. Can be normal in healthy older adults, but can also indicate hypertrophic cardiomyopathy or concentric LVH (due to aortic stenosis or HTN)

S4

Second most common cause of this disorder (after X-linked) is an autosomal recessive deficiency of adenosine deaminase, an enzyme necessary for the elimination of excess adenosine within cells. Toxic levels of adenosine accumulate within lymphocytes leading to lymphocyte cell death and resultant cellular & humoral immune deficiency. Treatment option to 'infect' patient stem cells with gene coding for adenosine deaminase is being researched.

SCID

N. Gonarrhae sample is placed on an antibiotic containing medium and bacterial colonies are cultures. Best term to describe this medium?

SELECTIVE (Thayer-Martin VCN include vancomycin, colistin, nystatin, and trimethoprim which inhibits growth of contaminants such as gram positive organisms, gram negative organisms other than Neisseria, and fungi) Note: Differential medium- identify cultured organisms based on their metabolic and biochemical colonies (organisms that ferment lactose will be pink MacConkey agar and black on EMB agar) Enrichment medium- contain special growth factors (Factor X and V for Hemophilus or anaerobic medium for Clostridium) Reducing medium- remove oxygen (used to culture anaerobic organisms) Synthetic- chemically defined medium in which all chemical contents are known

Arteriovenous concentration gradient reflects overall tissue solubility of an anesthetic. Anesthetics with high tissue solubility have LARGE arteriovenous concentrations and SLOW OR FAST onsets of action?

SLOW. (Note: a large amount of anesthetic is taken up from arterial blood if tissue solubility is high resulting in low venous concentration and more anesthetic is needed to replace whats absorbed by tissues aka blood saturation takes longer so brain saturation is delayed)

Most appropriate pharmacotherapy for PTSD?

SSRI Combine with trauma focused CBT!

28 year old man diagnosed with panic disorder who is not acutely symptomatic and has a history of alcohol abuse. Preferred treatment?

SSRI Pharmacotherapy of panic disorder includes SSRIs and benzodiazepines (used for severely symptomatic and functionally impaired patients who require rapid relief). SSRIs are often preferred to benzos due to the lack of physiological dependence and abuse potential

24 year old woman with copious green vaginal discharge and burning on urination for 3 days. History of yeast infection 1 year ago treated with OTC medication. Recently became sexually active with new partner & occasionally uses condoms. PE reveals yellow-green frothy vaginal discharge and diffuse vaginal erythema but no lesions. Best test to confirm diagnosis?

Saline microscopy (wet mount) Trichomonas vaginitis is caused by a motile protozoan which can be seen on wet mount microscopy of vaginal discharge. It presents with yellow green vaginal discharge and vaginal burning and is sexually transmitted.

Patients with sickle cell disease are at increased risk of septic arthritis due to?

Salmonella

Most common cause of osteomyelitis in children with sickle cell disease?

Salmonella Note: Staph aureus and E. coli are the next most common causes

43 year old African American man complains of malaise, nocturnal fevers, and cough for the past several weeks. Has lost 10 pounds over the last month. Works as a respiratory therapist in a hospital. Palpable lymph node in supraclavicular bed is biopsied and reveals well formed, noncaseating granulomas. Most likely diagnosis?

Sarcoidosis African American presenting with constitutional symptoms, bilateral hilar adeonpathy (arrows on CXR), and pulmonary complaints is concerning for sarcoid. Biopsy showing non-caseating granulomas confirms diagnosis of sarcoid (distinguishes from TB which would show caseating granulomas and acid fast bacilli)

35 year old woman with 3 month history of progressive dyspnea on exertion, nonproductive cough, and fatigue. Smokes 4 cigarettes a day. Mother with RA. CT lung guided biopsy performed and reveal aggregates of epithelioid macrophages and multinucleated giant cells. Most likely diagnosis?

Sarcoidosis Sarcoidosis often presents in young African American women with insidious onset of respiratory symptoms (cough, dyspnea, chest pain) accompanied by fatigue, fever, and weight loss. Characteristic histopathologic feature is non-caseating granulomas which consist of aggregates of epitheloid macrophages and multinucleated giant cells

Highly contagious disease that presents with prutitic rash (usually worse at night) in flexor surfaces of wrist, lateral surfaces of fingers, and finger webs. Excoriations with small, crusted, red papules scattered around affected areas are common

Scabies (diagnosis confirmed by skin scrapings from excoriated lesions that show mites, ova, and faces under light microscopy)

15 year old boy with right arm numbness. Has had fluctuating tingling and numbness involving the right shoulder, arm, and hand for the past several weeks. Also recently developing dull pain in the right little finger and the hand. Symptoms worsen with overhead activities and when throwing a baseball. Has been told that her has an extra rib, but has no other medical problems. On PE there is decreased sensation over the medial 2 fingers and hypothenar eminence of the right hand. What structure is most likely contributing to his condition?

Scalene muscles Thoracic outlet syndrome is due to compression of the lower trunk of the brachial plexus in the scalene triangle, the space bordered by the anterior and missile scalene muscles and the first rib. Symptoms include UE numbness, tinging, and weakness. Can be caused by an anomalous cervical rib, scalene muscular abnormalities, or injury (repetitive overhead arm movements, trauma).

Characterized by fever, pharyngitis, sandpaper-like rash, circumoral pallor, & strawberry tongue

Scarlet fever (caused by group A streptococcus that produce pyrogenic exotoxins)

Neisseria gonorrhoeae and Chlamydia trachomatis cause mucopurulent cervicitis which can progress to PID if untreated. Most serious complications of PID?

Scarring of the fallopian tubes leading to ectopic pregnancy & infertility

Best management of somatic symptom disorder (preoccupation with unexplained medical symptoms & excessive health care)?

Schedule regular visits with PCP who can monitor condition & AVOID unnecessary diagnostic testing and specialist referrals (mental health referral only once physician-patient relationship is well established!!!). Focus on functional improvement rather than symptom elimination (stress reduction, healthy behaviors)

Schistocytes (helmet cells) are fragmented erythrocytes that occur secondary to mechanical trauma from microangiopathic hemolytic anemias or prosthetic heart valves. Additional characteristic findings of intravasular hemolytic anemias?

Schistiocytes, decreased serum haptoglobin levels, increased LDH, increased bilirubin Note: haptoglobin is a serum protein that binds to free hemoglobin and promotes its uptake by the reticuloendothelial system

Patient currently exhibiting symptoms of mania (elevated mood, decreased need for sleep, hyper sexuality, grandiose delusion). Has a history of many similar episodes over the past ten years as well as a three week period delusions and hallucinations occurring in the absence of major mood episode. Most likely diagnosis?

Schizoaffective disorder Lifetime history of at least 2 weeks of psychotic symptoms without significant mood disturbance (manic or depressive) is a KEY requirement for the schizoaffective diagnosis, however, mood symptoms must be present for most of the illness. Note: in bipolar and major depression with psychotic features, the psychotic symptoms occur exclusively during manic or depressive episodes and when the patient's mood is euthymic, there are no psychotic symptoms; in schizophrenia significant mood symptoms (meeting criteria for mania or depressive episodes) are absent so if you have MANIA its schizoaffective not schizophrenia!!!!

2 month history of progressive social withdrawal, paranoid ideation, and auditory hallucinations. Most likely diagnosis?

Schizophreniform disorder Same symptoms as schizophrenia (delusions, hallucinations, disorganized speech and behavior, negative symptoms) but for shorter time and functional decline is not required for diagnosis

Characterized by a long standing pattern of eccentric behavior, odd beliefs, perceptual distortions, and social anxiety despite familiarity

Schizotypal personality disorder

Intracranial mass removed from 33 year old patient. Mass is composed of elongated cells that have regular, oval nuclei. Biphasic pattern of growth is identified consisting of areas of dense cellularity interspersed with less dense myxoid regions. Cells show S-100 immunoreactivity. Most likely diagnosis?

Schwannoma Schwannomas present histologically with a biphasic pattern of cellularity (Antoni A and B areas) and S100 positivity (indicating NEURAL CREST origin). Schwannomas can arise from the peripheral nerves, nerve roots, and cranial nerves (except CN II). Acoustic neuromas are most common and are located at cerebellopontine angle at CN VIII.

Benign, tan or brown epidermal tumor commonly identified on the skin of middle aged or elderly individuals. Can be found almost anywhere on the body. Tumors may have variable appearance from nearly flat macule to raised wart like lesions ranging in size from a few millimeters to several centimeters. Typically have a velvety to greasy surface and a well demarcated border, and are often described as having a 'stuck-on' appearance

Seborrheic keratosis On microscopic exam, the lesions are composed of small cells resembling basal cells with variable pigmentation, hyperkeratosis (thickening of the stratum corneum), and keratin containing cysts. Rapid onset of numerous seborrheic keratoses is often associated with an undiagnosed internal malignancy (Leser-trelat sign)

Refers to interrupting the disease process before symptoms develop

Secondary prevention Examples: Case finding- looking for disease that may be asymptomatic (age & gender appropriate screening such as Pap smear) Community screening- community wide (measuring BP at state fair)

Fever, artralgia, diffuse lymphadenopathy, widespread maculopapular rash. Elevated aminotransferase levels. Extensive clumping and flocculation when patient's serum added to mixture of cardiolipin, cholesterol, and lecithin. Likely diagnosis?

Secondary syphillis Note: clumping indicates anticardiolipin antibodies present, but this positive nontreponemal screening requires diagnostic confirmation with treponemal serologic testing looking for antibodies that directly target T. Pallidum (confirmation of syphillis requires 2 forms of serologic testing!)

Hormone that inhibits release of gastrin from normal gastric G cells and increases bicarbonate secretion from pancreas

Secretin

Giardia lamblia causes injury to the duodenal and jejunal mucosa by adhering to the intestinal brush border and releasing molecules that induce a mucosal inflammatory response. Major component of adaptive immunity against G. lambdlia?

Secretory IgA (impairs adherence) Note: conditions causing IgA deficiency predispose patients to chronic giardiasis (IgA deficiency, X-linked agammaglobulinemia, common variable immunodeficiency)

First step in management when a physician suspects child abuse after the parent and child have been interview together?

Seek permission from the parents to interview the child alone Following a full evaluation, physician should contact child protective services immediately with any child abuse concerns

Guillain Barre syndrome is an acute demyelinating polyneruropathy. Its thought to be due to molecular mimicry and is commonly preceded by a URI or gastroenteritis. What are the findings on light microscopy?

Segmental demyelination of the peripheral nerves and an endoneural inflammatory infiltrate Macrophages strop the myelin sheath from the axon and lipid laden macrophages are seen following the engulfment of myelin

14 year old girl brought to ED following suicide attempt with theophylline tablets. Patient should be carefully monitored for what complications?

Seizures (major cause of morbidity and mortality) & tachyarrhythmias Note: treat with activated charcoal to reduce GI absorption & beta blockers are used for the drug induced tachyarrhythmias & benzos/barbituates for seizures

Case control study

Select patients with particular disease (case) and those without disease (control) and then determine previous exposure status

In prospective studies, disproportionate loss to follow up between the exposed and unexposed groups creates the potential for attrition bias, which is a form of what type of bias?

Selection bias Note: investigators always try to achieve high patient follow up rates in prospective studies because of this

How do ACE inhibitors reduce GFR?

Selective efferent arteriolar dilation & decrease in renal perfusion pressure. In the kidney, angiotensin II preferentially constricts the efferent arteriole, maintaining the GFR within normal range. ACE inhibitors promote efferent arteriolar dilation causing GFR reduction. Angiotensin II functions: systemic vasoconstriction, preferential constriction of glomerular efferent arteriole, enhancement of adrenal aldosterone secretion. ACE inhibitors decreased circulating levels of angiotensin II

Bias that occurs in case control studies when cases are selected from the entire disease population instead of from just those that are newly diagnosed. For example, a study on cancer survival that is not limited to newly diagnosed patients could contain a high proportion of relatively benign malignancies, as these patients generally live longer

Selective survival bias

Inhibitor of MAO type B that is used clinically to delay the symptoms of Parkinson disease

Selegline Note: many neurologist favor the use of combinations of selegiline, anticholinergics, and amatadine until they no longer provide control of symptoms and only then are levodopa/carbidopa introduced

Ability of a test to identify the true presence of disease

Sensitivity

Romberg test is a test of PROPRIOCEPTION in which patients are observed for unsteadiness as they stand with their feet close together, arms to sides, and eyes closed. It is used to distinguish what?

Sensory from cerebellar ataxia. Positive romberg sign indicates sensory ataxia which may be caused by defects in the posterior column or peripheral nerves (ex: tabes dorsalis, B12 deficiency)

Injury to the axillary nerve most commonly occurs in the setting of shoulder trauma (anterior dislocation, humeral fracture) and presents with?

Sensory loss over the lateral shoulder and weakness on shoulder abduction (due to denervation of the deltoid muscle)

Unexpected occurrence involving death or serious physical or psychological injury that requires immediate investigation

Sentinel event Example: inpatient suicide, death of full term infant, retained object after surgery

Consists of anxiety (greater than 4 weeks in children and greater than 6 months in adults) due to separation from those whom the patient is attached. Physical symptoms and nightmares may accompany anxiety

Seperation anxiety disorder

Aspergillus histology?

Septate narrow hyphae with sharp-angle branching

What should be closely monitored after D&C is performed on patient with a complete hydatidiform mole?

Serial measurements of B-hCG (persistently elevated or rising levels may signify development of an invasive mole or choriocarcinoma) Note: complete moles are p57 negative due to absence of maternal genome

5 month old boy with concerns of developmental delay. Comprehensive lab evaluation reveals impaired tetrahydrobiopterin (BH4) synthesis. What is most likely deficient in this patient?

Serotonin Tetrahydrobiopterin (BH4) is a cofactor used for hydroxylse enzymes in the synthesis of tyrosine, dopamine, and serotonin. PKU can result from BH4 deficiency due to dihydropteridine reductase deficiency. Intellectual disability is the hallmark of this condition and results in neurotransmitter (serotonin) deficiency and hyperphenylalanemia. Treatment involves a low phenylalanine diet and BH4 supplementation.

Ondansetron inhibits what type of receptors and is used primarily to treat nausea and vomiting following chemotherapy

Serotonin (5-HT3) 5-HT3 receptors are located peripherally in the presynaptic nerve terminals of the vagus nerve in the GI tract. These receptors are also present centrally in the chemoreceptor trigger zone and the solitary nucleus and tract.

Neurotransmitter formed by hydroxylation and decarboxylation of tryptophan by tryptophan hydroxylase

Serotonin (5-hydroxytryptamine)

Medication treatment of OCD targets what neurotransmitter?

Serotonin (SSRIs are treatment of choice)

Originates on the first 8 ribs & inserts on the medial border of the scapula. Rotates the scapula upward, allowing abduction of the arm over the head. Paralysis (due to injury of long thoracic nerve) results in impaired abduction & winging of the scapula but NOT shoulder droop

Serratus anterior

Patient with pleural effusion has a tube placed to drain the fluid via an incision at the 5th intercostal space in the midaxillary line. What muscle is most likely to be dissected during the procedure?

Serratus anterior Chest tube for drainage of pleural effusion is placed through the skin & subcutaneous fat into 4th or 5th intercostal space in the anterior axillary or midaxillary line. Tube traverses through the serrates anterior muscle, intercostal (external, internal, innermost) muscles, and parietal pleura to reach the pleural cavity.

26 year old female with new facial hair and a deeper voice over the past few months. LMP was 5 months ago. PE is significant for coarse facial hair and pelvic exam reveals clitoromegaly with a large adnexal mass. Pregnancy test is negative. Pelvic US confirms a large ovarian cyst. Most likely diagnosis?

Sertoli-leydig tumor Sertoli-leydig cell tumors of the ovary arise from the sex cord storm and secrete testosterone. Typical features include a large ovarian mass, amenorrhea, and signs of virilization. Tubular structures lined by round Sertoli cells and surrounded by a fibrous storm are seen on pathology

23 year old male with 2 months of fatigue, malaise, abdominal discomfort. Has tender hepatomegaly and elevated LFTs. Never been vaccinated against hepatitis. No raw/undercooked foods recently and no sick contacts. No history of blood transfusion. Has had several episodes of unprotected sex with different female partners over the past year. What lab finding is likely to be present in this patient?

Serum HBsAg Main modes of transmission of HBV are sexual (homo & heterosexual) and percutaneous (IV drug use, needlesticks, transfusions). Note: Risk of sexual transmission of HCV is very low and it is usually asymptomatic!

Expected lab values in patient with osteoporosis?

Serum calcium, phosphorous, and PTH levels are typically in the normal range (low bone mass results in susceptibility to fractures occurring with minimal or no trauma) Note: osteoporosis is defined as bone density 2.5 standard deviations below the mean bone mineral density for young adult women based on none density measurement using dual x-ray absorptiometry

What test should be ordered before starting Metformin?

Serum creatinine

Life threatening condition that presents in infancy with severe bacterial & viral infections, mucocutaneous candadiasis, persistent diarrhea, & failure to thrive. Labs reveal absent T cells and hypogammagloblinemia. Thymic shadow is usually absent due to severe T cell deficiency

Severe combine immunodeficiency

13 month old boy presents with increased work of breathing and cough. Tachypneic with personal cyanosis on PE. Bilateral interstitial opacities on CXR. Has recurrent history of OM, pneumonia, and thrush as well as chronic diarrhea and failure to thrive. Analysis of bronchoalveolar lavage shows Pneumocystis jiroveci. Most likely diagnosis?

Severe combine immunodeficiency SCID is caused by a genetic defect in T cell development, leading to loss of both cellular and humoral immunity. Patients present in infancy with recurrent bacterial, viral, fungal, and opportunistic infections as well as failure to thrive and chronic diarrhea.

Toxin that inactivates 60S ribosomal subunit in human cells leading to an inhibition of protein synthesis and eventual cell death

Shiga toxin (Shigella dysenteriae) & Shiga-like toxin (EHEC)

A temporary course of benzodiazepines is sometimes used during the initiation period for SSRIs if there is significant increase in insomnia or other anxiety related symptoms. What benzodiazepines are preferred for this use at bedtime as they minimize undesirable daytime side effects?

Short acting (triazolam, oxazepam, midazolam) or intermediate acting (alprazolam, lorazepam, temazepam)

High systolic intraventricular pressure and wall stress of the left ventricle prevent myocardial perfusion during systole, therefore the majority of left ventricle myocardial perfusion occurs during diastole. Major limiting factor for coronary blood supply to the left ventricular myocardium during periods of exercise (tachycardia)?

Shorter duration of diastole

53 year old male presenting with dyspnea on exertion, nodular densities on CXR most prominent in the apical regions, calcified hilar lymph nodes (eggshell calcifications), and bifringent particles surrounded by fibrous tissue on biopsy. Most likely diagnosis?

Silicosis

28 year old female with history of T1DM presents with malaise, abdominal pain, and vomiting for a day. MM dry. Patient develops involuntary rhythmic jerking movement in RUE that last one minute while remaining conscious. Reports that she has had several similar episodes prior to arrival. Urine is positive for glucose and ketones and blood glucose is 450 mg/dL. Type of seizure patient had and due to what condition?

Simple partial (focal) seizure (no loss of consciousness and originating in a localized region of one cerebral hemisphere) due to metabolic derangements (hyperglycemia, anion gap metabolic acidosis) associated with diabetic ketoacidosis.

Cross sectional study (prevalence study)

Simultaneous measurement of exposure and outcome

Neurofibromatosis type 1 occurs due to a mutation of the NF1 gene located on chromosome 17. Cafe-au-lait spots, multiple neurofibromas, and Lisch nodules are the most common symptoms. Inheritance pattern?

Single gene autosomal dominant disorder

For a purified RNA molecule to induce viral protein synthesis in a host cell what type of RNA virus must it be?

Single stranded positive sense RNA (able to act directly as mRNA using the host's intracellular machinery for translation; example: rhinovirus) purified (+) sense single stranded RNA can be infectious but (-) sense single stranded RNA and double stranded RNA are not infectious as they require additional enzymes)

As part of patient's post transplant immunosuppressive regimen, he takes a medication that inhibits lymphocyte proliferation by blocking IL-2 signal transduction. This medication is most similar to what drug?

Sirolimus Sirolimus binds to the imminophilin FK-506 binding protein (FKBP) in the cytoplasm, forming a complex that binds and inhibits mTOR. Inhibition of mTOR signaling blocks IL-2 signal transduction and prevents cell cycle progression and lymphocyte proliferation

62 year old woman with dilated, tortuous veins at the calves and ankles bilaterally, with no erythema or edema. This condition increases the risk for?

Skin ulcerations Varicose veins are dilated, tortuous veins most commonly found in the superficial veins of the leg. They are caused by chronically increased intraluminal pressure and/or loss of tensile strength in the vessel wall, leading to incompetence of the venous vales. Common complications include edema, stasis dermatitis, skin ulcerations, poor wound healing, and infections

Epidural hematoma occurs due to tear of the middle meningeal artery. Clinical presentation is characterized by a 'lucid interval' followed by loss of consciousness. Its often associated with a temporal bone fracture an is located between what structures?

Skull bone and dura mater

Fixed LV outflow tract obstruction (valvular aortic stenosis) can cause pulsus parvus et tardus which is palpable as?

Slow rising low amplitude pulse due to diminished stroke volume (pulsus parvus) & prolonged LV ejection time (pulsus tardus)

Histopathology is most likely to show injury predominantly in what structure in chronic lung transplant rejection?

Small airways Chronic lung transplant rejection is due primarily to progressive scarring of the small airways, leading to bronchiolitis obliteratans. Manifestations occur months or years after transplantation and include obstructive lung disease (reduced FEV1) with dyspnea and dry cough

72 year old man with left hilarious mass, hyponatremia, decreased serum osmolality, & urine osmolality >100. Most likely diagnosis?

Small cell lung cancer (neuroendocrine malignancy that is the most common cause of SIADH due to ectopic secretion of ADH)

Thickened bronchial walls, lymphocytic infiltration, mucous gland enlargement, and patchy squamous metaplasia of the bronchial mucosa are features of chronic bronchitis. Leading cause of chronic bronchitis?

Smoking

Most effective preventative intervention in almost all patients, especially in those with diabetes?

Smoking cessation

Initial step in synthesis of steroid hormones is the conversion of cholesterol to pregnenolone in the mitochondria. Where does the remainder of steroidogensis occur?

Smooth ER (steroid producing cells contain a well developed smooth ER)

3 week old infant with projectile non-bilious vomiting after every meal. Abdominal exam reveals prominent peristalsis in the epigastrium and an olive sized mass is felt on deep palpation of the right upper abdomen. The mass most likely represents?

Smooth muscle hypertrophy Congenital pyloric stenosis arises secondary to hypertrophy of the pyloric muscularis mucosae. Stenosis is relieved by surgical splitting of the muscle. 3:1 ratio of males to females.

Involves excessive fears of scrutiny or embarrassment in social or performance situations, resulting in significant distress and functional impairment

Social anxiety disorder

Theory of personality development that emphasizes importance of observing and imitating behaviors, attitudes, & emotional reaction of others

Social learning

55 year old man being treated for depression is brought to ED with severe confusion and hallucinations. His wife found him when she came home from work. She found an empty bottle of pills in the bathroom. BP is 80/60 and pulse is 100. Mouth is dry & face is flushed. ECG shows QRS prolongation (120 msec) and frequent premature ventricular beats. Labs are obtained and hydration started. What agent would best correct patient's cardiac abnormalities?

Sodium bicarbonate TCA overdose can present with mental status changes, seizures, prolonged QRS duration, ventricular arrhythmias, and anticholinergic findings. Symptoms of overdose are caused by blockage of cardiac fast sodium channels & inhibition of muscarinic acetylcholine, histamine, and alpha 1 adrenergic receptors. Sodium bicarbonate is used to treat associated cardiac toxicity and works by increasing serum pH and extracellular sodium (alleviating fast sodium channel blockade).

30 year old woman with generalized weakness and paresthesias. BP is 150/95. Labs show low plasma renin. CT reveals solitary, hypotenuse, 1.5cm right adrenal mass. What would sodium, potassium, and bicarbonate electrolyte patterns would you expect to see in this patient?

Sodium: normal. Potassium: low. Bicarbonate: high. Primary mineralocorticoid excess (hyperaldosteronism) causes increased renal sodium reabsorption leading to HTN, hypokalemia, and metabolic alkalosis. Despite increase in Na+ reabsorption, hypernatremia and pedal edema are rarely observed due to aldosterone escape. Increased intravascular volume causes increased RBF and augmented release of ANP. Primary hyperaldosteronism is frequently caused by adrenal adenoma or bilateral adrenal hyperplasia.

Drug that inhibits nonstructural protein 5B (NS5B), an RNA dependent RNA polymerase needed for HCV replication. Adverse effects include fatigue and nausea.

Sofosbuvir

Most cases of esophageal squamous cell carcinoma occur in men above age 50 with a history of prolonged smoking and alcohol use. Patients typically present with solid food dysphagia as the tumor gradually obstructs the esophageal lumen, which can progress to liquid dysphagia. Retrosternal discomfort/burning and significant weight loss are also common. Chronic GI blood loss may result in iron deficiency anemia with fatigue. Prognosis for esophageal cancer is poor as many patients present with incurable, locally advanced or metastasis disease. Histopathologic features of esophageal SCC?

Solid nests of neoplastic squamous cells with abundant eosinophilic cytoplasm and distinct borders. Areas of keratinization and presence of intercellular bridges are also characteristic.

Patient with Turner syndrome karyotype reveals 40% of cells have 45X genotype and remain 60% have a 46XX genotype. Most likely cause of this condition?

Somatic mosaicism (patient has manifestations of Turner syndrome & peripheral cells with multiple distinct genotypes; germline mosaicism would not cause clinical manifestations & would not be detectable on karyotyping of blood sample) Mosaicism is the presence of multiple genetically different cell lines within the body which can result from chromosomal nondisjunction or mutations during 1st stages of embryonic development. Somatic mosaicism affects forming the body causing disease manifestations to develop while germline mosaicism affects cells giving rise to gametes allowing the affected genes to pass to offspring (affected parent does not develop clinical manifestations). Somatic mosaicism results in mixture of normal & mutated somatic cells, often leading to milder form of disease

Excessive and persistent health anxiety and preoccupation with multiple somatic symptoms

Somatic symptom disorder

VIPomas are pancreatic islet cell tumors that hyper secrete vasoactive intestinal peptide, which increases intestinal chloride loss into the stool and causes excess losses of accompanying water, sodium, and potassium. Presents with MANY episodes a day of tea colored, odorless, water stools. VIP also inhibits gastric acid secretion. Treatment with what substance can promptly relieve the symptoms of a VIPoma?

Somatostatin (inhibits the secretion of VIP) Also known as WDHA syndrome (pancreatic cholera)- watery diarrhea, hypokalemia, achlorhydria

65 year old Vietnamese speaking man admitted with small bowel obstruction & needs surgery. Surgeon talks with daughter who speaks both Vietnamese and English and asks her to translate information for patient & assist with translating the consent form. Most appropriate course of action by the resident?

Speak with attending surgeon privately about using a trained Vietnamese language interpreter Must use medical interpreter to promote adequate patient understanding & participation in decision making process, ESPECIALLY during informed consent process. Family members may only be used in urgent cases in which no interpreter is available!!!!

Ability of a test to identify the true absence of a disease

Specificity

Nerve that is vulnerable to injury in the posterior triangle of the neck. Injury results in weakness of trapezius muscle which presents with drooping of the shoulder, impaired abduction of the arm above horizontal (weakness in rotating the glenoid upward), & winging of scapula

Spinal accessory nerve (CN XI)

Disorder caused by mutations in SMN1 (survival motor neuron) gene resulting in impaired assembly of snRNPs in lower motor neurons. Results in infants with flacid paralysis due to impaired SPLICOSOME function & degeneration of anterior horn cells in spinal cord

Spinal muscular atrophy snRNPs are important components of the splicosome which removes introns from pre-mRNA during processing within the nucleus

Medical therapy treatment for Conn's syndrome (aldosterone excess that causes HTN, hypokalemia, metabolic alkalosis, depressed renin)?

Spironolactone or eplerenone (aldoesterone antagonists) Note: HYPOaldosteronism is the cause of type IV renal tubular acidosis

Most common organ injured with blunt abdominal trauma?

Spleen (predisposes to infections with encapsulated organisms such as S. pneumonie, H. influenze, & N. meningitidis in the future as splenic opsonizing antibody is important for their removal)

35 year old man with persistent abdominal pain who suffered a full speed collision with another player. Injury involves an organ that is supplied mainly by an artery of the foregut, though the organ itself is not a foregut derivative. Most likely injured organ?

Spleen. Spleen is most common intra-abdominal organ injured by blunt trauma. Spleen is intraperitoneal organ derived from mesoderm of dorsal mesentery. Although it is supplied from the splenic artery of the celiac trunk, a foregut derivative, it is not a foregut derivative itself. Venous return of the spleen courses through splenic vein to return to portal circulation.

26 year old presents after being involved in MVC. Was driving while wearing seatbelt when his care what hit on the left by another car. Complains of persistent left shoulder pain since accident associated with nausea and hiccups. BP is 90/60. Pulse is 115. Has a bruise on left lower chest wall with tenderness to palpation. Normal heart sounds and equal breath sounds. Rigid and tender abdomen. Most likely diagnosis?

Splenic laceration Rigid abdomen with associated left should pain, hypotension, and tachycardia suggest spleen laceration and hemoperitoneum. Shoulder pain is referred pain due to peritoneal irritation (kehr sign). Any abdominal process (ruptured spleen, peritonitis, hemoperitoneum) irritating the phrenic sensory fibers around the diaphragm can cause referred pain to the C3-C5 shoulder region. Phrenic nerve irritation can also cause hiccups due to spasmodic diaphragmatic contraction pulling air against a closed larynx.

Portion of the spleen that destroys aged/abnormal erythrocytes, stores emergency blood cells and platelets, and aids in clearing circulating bacteria?

Splenic red pulp

Achalasia is an esophageal motility disorder characterized by failed relaxation of the lower esophageal sphincter resulting in food retention, dilation of the esophageal body, and symptoms of solid/liquid dysphagia. Achalasia increases risk for what cancer?

Squamous cell carcinoma of the esophagus

16 year old with difficulty walking. On PE he has thoracic scoliosis and multiple small scars on his hands from frequent accidental burns because he can't feel the heat. Neuro exam reveals 1+ bicep reflexes and 3+ patellar reflexes bilaterally and decreased muscle strength in the bilateral UE and LE. Hand muscles are atrophic and position and vibration senses in feet are impaired. Most likely cause of patients problems?

Sryingomyelia Combination of chronic loss of upper extremity pain and temperature sensations, upper extremity lower motor neurons signs (weakness, hyporeflexia), and/or lower extremity upper motor neuron (weakness, hyperreflexia) signs in the setting of scoliosis is consistent with syringomyelia. In this condition a central cystic dilation in the cervical spinal cord (a syrinx usually C8-T1) slowly enlarges, causing damage to the ventral white commissure and anterior horns. As cyst expands it can affect the lateral corticospinal tract and produce loss of position/vibration sense in feet due to involvement of the posterior columns. Scolisos occurs due to paresis of paravertebral muscles.

Patient on Warfarin for atrial fibrillation suffers from an thromboembolic stroke. Recently started taking new OTC drug recently but doesn't remember name. Most likely started taking what drug recently?

St John's wort (induces CYP450 hepatic microsomal enzymes and will lower plasma concentrations and decreased efficacy of Warfarin)

73 year old with epigastric pain than begins 30 minutes after meals and does not respond to antacids. Patient has lost more than 10 pounds over the last few months which he states is due to him eating less due to his fear of the pain. History of HTN, hyperlipidemia, CABG, right sided carotid endarterectomy. Has smoked 1PPD for 32 years. Upper GI endoscopy shows no abnormalities. Underlying pathophysiology of this patient's disease process is similar to what condition?

Stable angina Postprandial epigastric pain and associated food aversion/weight los in the setting of generalized atherosclerosis is consistent with chronic mesenteric ischemia (atherosclerosis of the mesenteric arteries results in diminished flow to the intestine after meals when more blood is needed for digestion/absorption of nutrients). Analogous to stable angina, chest pain that occurs with physical exertion due to increased myocardial oxygen demand

Fixed atherosclerotic plaques obstructing more than 75% of the lumen of a coronary artery are associated with what type of angina?

Stable angina (exacerbated by exercise and relieved by rest and/or sublingual nitro within 5 minutes) Plaques occluding less than 75% of coronary artery lumen are usually asymptomatic. Acute coronary syndromes (unstable angina, MI, and SCD) result form acute plaque change, often with superimposed thrombosis. An ulcerated atherosclerotic plaque with a partially obstructive thrombus is associated with unstable angina or subendocardial infarction. A ruptured atherosclerotic plaque with a fully obstructive thrombus is associated with transmural MI.

Most common pathogen causing cystitis and acute pyelonephritis is E. coli. Second most common cause of UTI in sexually active women is?

Staph Saprophyticus Note: UTIs most commonly effect women because of their short urethra; recurrent UTIs in boys or ben require investigation for urinary tract obstruction or reflux

10 year old bot with high fever and chills for the last few days. Complains of a dull pain just about his left knee. NO trauma. Point tenderness 3cm above knee cap without joint effusion. Imaging shows soft tissue swelling and periosteal reaction over lower end of femur. Organism most likely responsible for symptoms?

Staphylococcus aureus Hematogenous osteomyelitis is predominately a disease of children that most frequently affects the long bones. S. Aureus is implicated in most cases secondary to a bacteremic event. S. pyogenes (group A strep) is the second most common cause of hematogenous osteomyelitis

19 year old with pelvic pain, fever, and chills since last night. LMP was 12 weeks ago and she had a surgical pregnancy termination 2 days ago. Was diagnosed with influenza two weeks ago which resolved with rest. Temperature is 101, BP 92/60, pulse 102. PE shows diffuse lower abdominal tenderness. Speculum exam reveals open cervical os with foul smelling tissue in vaginal canal. Bimanual exam reveals moderate uterine tenderness without any adnexal masses. Most likely cause of patient's condition?

Staphylococcus aureus Septic abortion typically presents with fever, abdominal pain, uterine tenderness, and/or foul smelling discharge after pregnancy temrination. Common offending pathogens are S. aureus, Group B strep, and E. coli due to seeding of the uterine cavity during instrumentation

Most common cause of septic arthritis?

Staphylococcus aureus (catalase positive gram positive cocci in clusters). Most cases in young adults are linked to injection drug use.

Most common cause of infective endocarditis in patients with prosthetic valves and septic arthritis with prosthetic joints. Susceptible to novobiocin, catalase positive, coagulase negative.

Staphylococcus epidermidis Note: S. aureus can ferment mannitol, is coagulase positive, & produces yellow pigment; S. saprophyticus is novobiocin resistant

Statins inhibit the intracellular rate limiting step of cholesterol biosynthesis via competitive inhibition of HMG-CoA reductase. This leads to enhanced hepatic LDL receptor recycling and increased LCL clearance from the circulation. Most common complication of statin use?

Statin-induced myopathy (myalgia, elevated creatinine kinase)

Critical shortening in telomere length can signal for programmed cell death. Telomerase is a reverse transcriptase (RNA dependent DNA polymerase) that lengthens telomeres by adding TTAGGG repeated to the 3' end of chromosomes. What cell type have long telomeres due to high amounts of telomerase activity?

Stem cells (such as epidermal basal cells) Allows them to proliferate indefinitely in a controlled manner

Lamotrigene can be used to treat partial and generalized seizures as well as bipolar disorder and works by blocking voltage gated sodium channels. Most serious adverse reaction of Lamotrigene?

Stevens-Johnson syndrome (less than 10% of body surface area) & toxic epidermal necrolysis which is greater than 30% of body surface area (rare, life threatening adverse effects characterized by flu like symptoms followed by widespread mucocutaneous epidermal necrosis)

28 year old with a 4 month history of amenorrhea and whitish nipple discharge from both breasts. Negative pregnancy test. Has also had increased fatigue, depressed mood, and weight gain over the past 4 months. Exam reveals thinning of outer third of eyebrows and a enlarged but contender thyroid. Elevated TSH, low T4, and antithyroid peroxidase antibodies are present. Most likely mechanism causing elevated prolactin?

Stimulation of lactotrophs by TRH Prolactin production is regulated primarily by inhibitory effects of hypothalamic dopaminergic pathways. However, prolactin secretion is stimulated by TRH & in patients with primary hypothyroidism, the increased TRH by the hypothalamus can lead to hyperprolactinemia

28 year old male with abdominal discomfort and loose stools over the past year who has lost 22 pounds despite having a normal appetite. Well balanced diet, no travel, no sick contacts. Lab tests are normal. Next best step for assessing for impaired nutrient absorption?

Stool microscopy with Sudan III stain (noninvasive and quickly & easily identifies unabsorbed fat) Malabsorption can present with steatorrhea, diarrhea, weight loss, fatigue, or abdominal discomfort. Fats are typically the earliest and most severely affected macronutrient in generalized malabsorption so testing stool for fat is most sensitive strategy for screening for malabsorption disorders.

35 year old with 2 week history of nasal congestion. Has used a topical decongestant every few hours since symptoms began. Experienced relief for 1 week but the nasal congestion returned. PE shows nasal mucosa that appears edematous and red with a few areas of punctate bleeding. Next best step in management of this patient?

Stop the decongestant Topical preparations of alpha adrenergic agonists cause vasoconstriction of the nasal mucosa vessels and are used as decongestants. Overuse of these drugs causes negative feedback, resulting in decreased norepinephrine synthesis and release from nerve ending, diminishing the degongestants effect and resulting in rebound rhinorrhea (tachyphylaxis) Note: nitroglycerin is another medication associated with the phenomenon of tachyphylaxis; decrease in effect is due to diminished release of NO from target cells and therefore drug free intervals of 10 hours should be maintained during use

Acute tubular necrosis can be caused by decreased renal perfusion due to severe hypovolemia, shock, or surgery. Most commonly affected portions of the nephron by acute tubular necrosis?

Straight proximal tubules and thick addending limb of heel's loop in the outer medulla. Muddy brown casts are pathognomonic for cute tubular necrosis

Endocarditis and bacteremia associated with what organism are associated with gastrointestinal lesions (colon cancer) in 25% of cases?

Streptococcus gallolyticus (S. bovis) When S. gallolyticus is cultured in the blood, workup for colonic malignancy with colonoscopy is essential

Bacterial meningitis presents with confusion, headache, fever an nuchal rigidity and causes an increased in CSF neutrophil count and protein concentration as well as a decrease in CSF glucose. Leading cause of community acquired meningitis in adults?

Streptococcus pneumoniae (lancet shaped gram positive cocci in pairs)

Due to loss of pelvic floor support and incompetent of the urethral sphincter. Increased intraabdominal pressure (coughing, sneezing, vigorous effort) greater than urethral sphincter pressure can cause brief involuntary urine loss

Stress incontinence

Aldolase B deficiency, or hereditary fructose intolerance, is an autosomal recessive disorder that leads to accumulation of toxic metabolite fructose 1 phosphate. Patients have hypoglycmemia and vomiting when fructose or sucrose is consumed. Treatment?

Strict removal of both fructose and sucrose from diet

Rare congenital neurocutanous disorder characterized by the presence of cutaneous facial angiomas as well as leptomeningeal angiomas. Skin involvement typically overlies V1 and V2 distributions of the trigeminal nerve. Associated with mental retardation, seizures, hemiplegia, and skull radio opacities. Skull radiographs may show characteristic 'tram-track' calcifications

Sturge-Weber Syndrome (encephalotrigeminal angiomatosis)

Thyroid disease characterized by a disruption of follicles & a mixed cellular infiltrate with occasional multinuclear giant cells. Typically appears after viral URI with fever, painful & tender thyroid

Subacute granulomatous (de Quervain) thyroiditis

Symmetrical enlargement of the ventricles is characteristic of communicating hydrocephalus which usually occurs secondary to dysfunction or obliteration of what structure?

Subarachnoid villi (usually a sequelae of meningeal infections, including tuberculosis meningitis, or subarachnoid/intraventricular hemorrhage)

Occurs due to rupture of the CORTICAL BRIDGING VEINS. In young patients, it results from a fall or MVC, and manifests with GRADUAL onset of headache and confusion. In elderly patients it may occur after MINOR trauma and present with a variety of neurologic symptoms.

Subdural hematoma (accumulation of blood between the dura mater and arachnoid) (BE ABLE TO RECOGNIZE ON CT- crescent shaped mass)

During ventricular systole, the coronary vessel supplying the LV are compressed by surrounding muscle so the majority of the LV blood flow occurs during diastole. Systolic reduction in coronary blood flow is greatest in what region making this portion of the LV most prone to ischemia and infarction?

Subendocardial region (where wall pressures are the highest)

6 month old with persistent constipation and abdominal distension. Barium enema shows narrow rectum and rectosigmioid area. Rest of colon is significantly dilated. Biopsy with sampling from what clan area would be most useful in making diagnosis?

Submucosa of the narrow part Submucosal (Meissner) and myenteric (Auerbach) autonomic plexi are absent in the affected segment of the bowel in Hirschsprung disease. The submucosa of the narrow area is the most superficial later where the absence of ganglion cells can be seen

46 year old woman with recurrent renal colic. Past history of peptic ulcer disease. Lab results show elevated calcium and low phosphorous. Most likely change in bone structure?

Subperiosteal resorption with cystic degeneration (patient has primary hyperparathyroidism in which classic symptoms are bone pain, renal stones, GI disturbances such as PUD, and psychiatric disorders) Osteitis cystica is the most characteristic skeletal manifestation of primary hyperparathyroidism. Present with bone pain, subperiosteal erosions affecting the phalanges of the hand, a 'salt and pepper' skull, and brown tumor bone cysts. Skeletal findings most commonly involve cortical (compact) bone in pectoral girdle, pelvic girdle, and limbs. Other bone morphology diseases: Paget disease- disorganized lamellar bone structure in a mosaic pattern Vitamin D deficiency- osteoid matrix accumulation around trabeculae Osteopetrosis- persistence of primary spongiosa in medullary cavity with no mature trabeculae ('marble bone disease') Osteoporosis- trabecular thinning with fewer interconnections

There is a specific bacterial product that when injected locally into the muscles of patients with relentless focal dystonias such as torticollis, produces a dramatic but temporary relief of symptoms. This substance is produce by bacteria that demonstrate?

Subterminal spore formation Clostridia are gram positive spore forming anaerobic rods. C. botulinum is the bacteria responsible for botulism, a toxin mediated disease (synthesizes its neurotoxins intracellularly and releases them by autolysis). Local injections of botulinum toxin into muscle are used to treat focal dystonias, achalasia, and spasms. Toxin prevents release of acetylcholine which lasts about three months before regeneration of the nerve terminal occurs.

65 year old man diagnosed with Parkinson disease 10 years ago is interested in discussing surgical options dye to developing dyskinesia & motor fluctuations. Patient would most likely benefit from high frequency deep brain stimulation targeted at suppressing neuronal activity in what brain structure?

Subthalamic nucleus Nigrostriatal degeneration in Parkinson disease reduces activity of the thalamus and its projections to the cortex, resulting in bradykinesia and rigidity. Patients with medically intractable symptoms of Parkinson disease may benefit from high frequency deep brain stimulation of the globus pallidus interns or subthalamic nucleus as it promotes thalami-cortical DISINHIBITION with improved mobility Note: high frequency stimulation of the ventral intermediate nucleus of the thalamus may reduce thalamic excitation and lessen excessive movements in patients with medially intractable ESSENTIAL TREMOR

45 year old quadriplegic with suspected bacterial pneumonia develops respiratory failure requiring mechanical ventilation. Skeletal muscle relaxant is administered prior to intubation and patient subsequently goes into cardiac arrest. Cardiac monitor shows VFIB. Serum K+ is 10. Administration of what agent most likely responsible for his condition?

Succinylcholine Succinylcholine can cause significant K+ release and life threatening arrhythmias in patients at high risk for hyperkalemia, including those with burns, myopathies, crush injuries, and denervating injuries or disease. Because the nAch receptors are nonselective ion channels, their opening not only allows Na+ influx but also K+ release. Use vecuronium or rocuronium in these patients!

Fast acting, depolarizing neuromuscular blocking agent used for rapid sequence intubation that causes equal reduction of all 4 twitches during train of four stimulation (phase I blockade)- prevents repolarization in phase 1. Prolonged administration or use in patients with abnormal plasma cholinesterase activity causes transition to phase II (non depolarizing) block seen as progressive reduction in each of the 4 twitches- Ach receptors become desensitized & inactivated in phase II

Succinylcholine Note: nondepolarizing NMJ clockers (ex: pancuronium, tubocurarine) competitively inhibit postsynaptic Ach receptors & result in less Ach being released with each subsequent impulse so they always show a progressive reduction in each of the 4 responses of the train of 4 stimulation

Promotes peptic ulcer healing by binding to he base of mucosal ulcers and providing physical protection against gastric acid

Sucralfate

Psychogenic causes of erectile dysfunction account for about 10% of cases and include performance anxiety, sexual partner dissatisfaction, and marital problems. Important clues that point towed psychogenic impotence?

Sudden onset and the presence of morning erections

24 year old with harsh systolic ejection murmur dies suddenly. Father and uncle died suddenly in their 30s. What would have increased the intensity of the patient's murmur?

Sudden standing Hypertrophic cardiomyopathy is characterized by asymmetric ventricular septal hypertrophy and variable dynamic left ventricular outflow tract obstruction. Maneuvers that increase preload or after load will decrease murmur intensity (sudden standing, valsalva, nitroglycerin administration) by increasing the LV end diastolic volume and lessen the outflow tract obstruction. Squatting, sustained hand grip, or passive leg raise will increase prelaod and/or after load thereby decreasing murmur intensity. Phenylepherine, a selective alpha 1 agonist, administration would increase SVR (after load) and decrease murmur intensity

Drug classes that improve blood glucose levels in patients with T2DM by increasing secretion of insulin and c-peptide (useful marker for endogenous B-cell insulin secretion) from pancreatic beta cells?

Sulfonylureas (chlorpropamide, tolbutamide, glyburide, glipizide, glimepiride) & meglitinides (nateglinide, repaglinide)

Lymph from scrotum drains into what nodes?

Superficial inguinal nodes

Pernicious anemia is an autoimmune disorder caused by the cell mediated destruction of parietal cells in what layer of the gastric body and fundus?

Superficial upper glandular layer Parietal cells are responsible for the secretion of HCl and intrinsic factor, a glycoprotein needed for absorption of B12. Deficiency leads to megaloblastic anemia and neurologic dysfunction

'Apple peel atresia' occurs when what artery is obstructed?

Superior mesenteric artery Results in blind ending proximal jejunum, a length of absent bowel and mesentery, and terminal ileum spiraled around and ileocolic vessel

During open aneurysm repair the inferior mesenteric artery is ligated, the diseased portion of the aorta is dissected and the graft is placed from below the renal arteries to the bifurcation of the aorta. Collateral circulation from what vessel is most likely responsible for preventing ischemia of the descending colon?

Superior mesenteric artery SMA and IMA are the 2 main vessels supplying the small and large intestines. They are connected by a pair of anastomoses: marginal artery of Drummond, which is the principal anastomosis and the inconsistently present arc of Riolan (mesenteric meandering artery)

Oculomotor nerve (CN III), ophthalmic nerve (CN V1) branches, trochlear nerve (CN IV), abducens nerve (CN VI), and superior ophthalmic vein enter orbit via?

Superior orbital fissure

Venous components of internal hemorrhoids (not painful) drain into what vein?

Superior rectal vein which communicates with inferior mesenteric vein.

Patient with extensive smoking history with hemoptysis, numbness in right forearm extending up to tips of 4th and 5th fingers, and right shoulder pain. Most likely diagnosis?

Superior sulcus tumor (Pancoast tumor) These tumors located at the lung apex can invade surrounding structures and lead to ipsilateral Horner syndrome, rib destruction, shoulder pain, weakness/atrophy of hand muscles, UE edema, spinal cord compression/paraplegia, and pain in the distraction of C8, T1, and T2 nerve routes.

Structure embryonically derived from the common cardinal veins?

Superior vena cava (and other constituents of the systemic venous system) In early embryonic development, the body's veins fall into 3 main groups; umbilical, vitelline, and cardinal. Umbilical vein degenerates. Vitelline veins form the portal system. Cardinal veins form systemic circulation. Common cardinal veins of the developing embryo drain directly into the sinus venosus, which then drains into the primitive atrium of the developing heart.

Where does lymph from superior and inferior portions of the bladder drain?

Superior- external iliac nodes Inferior- internal iliac nodes

Patient received deep intramuscular injection and developed problems walking later that day. When patient is asked to walk across the room, his right hip drops every time his right foot is raised off the ground. What location did he most likely receive the injection?

Superomedial quadrant of the left buttock (superior gluteal nerve/gluteus medium injury cause Trendelenburg gait) Injections given in the superomedial part of the buttock risk injury to the superior gluteal nerve. Injections into the superiomedial, inferomedial, inferolateral regions of the buttock risk injury to the sciatic nerve. The superolateral quadrant of the buttock is a relatively safe area for intragluteal injections, but the anterolateral gluteal region (Von Hochstetter triangle) is preferred

Due to gravity, supine patients typically aspirate into which lobe? Patients who are upright (or semi-recumbent) tend to aspirate into which lobe? Aspirated material is more likely to travel down which main bronchus?

Supine patients typically aspirate into the posterior segments of the upper lobes and the superior segments of the lower lobes. Patients who are upright (or semi-recumbent) tend to aspirate into the basilar segments of the lower lobes. Aspirated material is more likely to travel down the right main bronchus (has a larger diameter, is shorter, and more vertically oriented than the left main bronchus)

How do high circulatory levels of cortisol result in hypogonadotropic hypogonadism?

Suppress release of gonadotrophin releasing hormone

Interferons alpha and beta are produced by most human cells in response to viral infections which function to?

Suppress viral replication by halting protein synthesis and promote apoptosis of infected cells, limiting the ability of viruses to spread through the tissues Note: virally infected cells don't secrete interferon gamma (improves intracellular killing ability of macrophages), only interferon alpha and beta!

Bitemporal hemianospsia, amenorrhea, & enlargement of pituitary gland on brain imaging suggestive of prolactin secreting pituitary adenoma. What embryonic layer did lesion originate?

Surface ectoderm

After exposure to mutagens several strains of enveloped RNA viruses isolated from nasal exudate of experimental animals acquire the ability to infect human epithelial cells. What viral encoded protein is most likely to have mutated in this circumstance?

Surface glycoprotein Changes in host range are most commonly caused by a mutation in the viral encoded surface glycoprotein that mediates vision attachment to target host cell plasmalemma receptors. Mutations in viral encoded capsid proteins, RNA polymerases, endonuclease or proteases could nonspecifically effect viral replication in the standard host cell but would be less likely to significantly alter the range of host cell types that an enveloped RNA virus could successfully infect.

According to LaPlace's Law, as the radius of a sphere with a constant surface tension decreases, the distending pressure increases, thus smaller spheres collapse before larger ones. Role of surfactant in this process?

Surfactant counteracts alveolar collapse by decreasing surface tension as the alveolar radius decreases & it also prevents the unchecked expansion of larger alveoli (ensures all alveoli expand at similar rates) LaPlace's Law: P=2T/r

Effects of androgen abuse

Suspect androgen abuse in patient with elevated hematocrit, testicular atrophy, & acne

Exceptions to confidentiality?

Suspected child or elder abuse, knife or gunshot wounds, diagnosis of a reportable communicable disease, patients at risk of physically harming themselves or others

What structure should be ligated to avoid excessive bleeding during an oophorectomy?

Suspensory ligament of the ovary (contains ovarian artery, vein, lymphatics, nerves & ovarian artery is major blood supply to the ovary) Ovaries are suspended by the mesoovarium superiorly, ovarian ligament medially, and suspensory ligament (infundibulopelvic ligament) of the ovary laterally. Suspensory ligament is a fold of perineum that attaches the ovary to the pelvic sidewall. Note: ureter is in close proximity to the suspensory ligament of the ovary (medial at the level of bifurcation of the common iliac artery) & must be identified prior to ligation to avoid inadvertent ureteral transection Note: transverse cervical ligament (cardial ligament) extends from the cervix & lateral fornix of the vagina to the lateral pelvic walls; uterine artery courses in its superior portion necessitating ligation during a HYSTERECTOMY

Patient with advanced dementia and stroke with residual facial droop and hemiparesis is hpospitalized due to septic shock. He is treated and improving. Voice is soft and breathy. Rhonchi heard over right lower lung. CXR demonstrates dense air space opacities in the superior region of the right lower lobe. Most likely underlying etiology of patient's hospitalization?

Swallowing muscle dysfunction Elderly patients with dementia or hemiparesis may have dysphagia which is a risk factor for aspiration pneumonia. Dependent lung consolidation is commonly seen with aspiration pneumonia. Patient developed septic shock secondary to pneumonia.

Amniotic fluid embolism is a rare and catastrophic pregnancy complication that results from amniotic fluid (containing arachidonic metabolites that results in an anaphylactoid reaction) entering the maternal circulation. Common signs include hypoxia, hypotensive shock, and DIC (from tissue factor aka thromboplastin being released from amniotic fluid). Histologic evaluation of the pulmonary vasculature reveals?

Swirls of fetal squamous cells and mucin

Primary function of nucleolus?

Synthesis and assembly of immature 60S and 40S ribosomal subunits that are exported from the nucleus to fully mature in the cytoplasm

1 year old both with high fever, irritability, & photophobia who recently underwent ventriculoperitoneal shunt placement for congenital hydrocephalus. On PE temperature is 101 F and passive flexion of neck results in spontaneous flexion of hips & knees. Blood cultures grow coagulase NEGATIVE Staphylococcus. Most important virulence factor by which this bacterium causes disease?

Synthesis of an extracellular polysaccharide matrix Staphylococcus epidermidis, a component of normal skin flora, is one of the most common causes of infections associated with foreign bodies (including VP shunts) due to its ability to form adherent biofilms.

Patient has been treated with phenelzine for his depression. Physician wants to switch to sertraline. What process occurs during the 2 week washout interval to allow sertraline therapy to be initiated safely?

Synthesis of monoamine oxidase Phenelezine is an MAO inhibitor and Sertraline is an SSRI. Coadministration of these can produce excessive synaptic serotonin levels & serotonin syndrome due to decreased degradation and reuptake. Wash out period allows sufficient time for MAO regeneration.

Function of RNA polymerase II?

Synthesize mRNA, snRNA & miRNA

MIlrinone infusion is is initiated in a patient with refractory CHF due to LV systolic dysfunction. What response would most likely to be seen in addition to increased contractiliy?

Systemic arterial and venous dilation Phosphodiesterase-3 inhibitors such as milrinone and inamrinone lead to an increase in intracellular cyclic adenosine monophosphate (cAMP) concentration, which promotes intracellular Ca2+ influx & increases cardiac contractility. An increase in cAMP concentration in vascular smooth muscle cells also causes systemic vasodilation, which limits the use of these two medications in severely hypotensive patients

Poison ivy dermatitis is a form of allergic contact dermatitis which is a type IV hypersensitivity reaction mediated primarily?

T lymphocytes (CD8+ T cells are the primary effector cells and directly destroy keratinocytes expressing haptenated proteins) Manifests as intensely pruritic erythematous papule, vesicles, or bullae that often form linear patterns

8 year old boy with one week history of fever and throat pain presents with severe dyspnea, tachypnea, and inspiratory stridor. Has also experienced worsening dysphagia with solid foods over the last two weeks. Labs reveal many immature hematopoietic cells (blasts) in the peripheral blood smear. The neoplastic cells causing patients condition normally give rise to what type of cell?

T-lymphocytes ALL is most common malignancy of childhood. B-cell ALL is responsible for approximately 70-80% of all cases whereas T-cell is responsible for 15-17% of cases of ALL. T-cell ALL presents as a mediastinal mass that can cause respiratory symptoms, dysphagia, or superior vena cava syndrome

Promotor region that binds transcription factors and RNA polymerase II during the initiation of transcription. Located approximately 25 bases upstream from the beginning of the coding region

TATA box

Formula for total filtration rate

TFR = GFR (inulin clearance)x plasma concentration

Keloids results from excessive collagen formation during the remodeling phase of wound healing. They present as raised, painful, and pruritic nodules that groups beyond the wound borders and are due to what factor being produced excessively without regulation?

TGF-beta (promotes differentiation of fibroblasts into myofibroblasts) Note: hypertrophic scares are limited to the wound area and may regress spontaneously, in contrast to keloids that extend beyond the original wound, do not regress, and often recur after excision

Neonates born to mothers with poorly controlled diabetes during pregnancy are exposed to high maternal glucose levels in utero resulting in excessive fetal insulin production and insult hyperplasia. Fetal hyperinsulinemia persists for several days following both and predisposes the infant to?

TRANSIENT hypoglycemia

Protein kinase A is responsible for the intracellular effects of the G protein mediated adenylate cyclase second messenger system. Hormone receptors that use this system include?

TSH, glucagon, PTH receptors

Administering liothyronine (T3) supplementation to a patient with hypothyroidism would have what effect on TSH, T3, reverse T3, T4?

TSH- decrease T3- increase T4-decrease Reverse T3- decrease (T3 cannot be converted to reverse T3, only T4 can be converted to reverse T3) Note: synthetic T3 is not recommended as it has short half life and patients can experience wide fluctuations in plasma T3 levels, use T4 (levothyroxine) instead!

Marijuana contains tetrahydrocannabinol, which stimulates cannabinoid receptors to produce mild euphoria with inappropriate laughter, increases appetite, slowed reaction time/motor speed, and cognitive impairment. Two most immediate physical symptoms of marijuana use?

Tachycardia and conjunctival injection

Describes decreased drug responsiveness in a short period following one or more diseases (rapidly developing tolerance)

Tachyphylaxis

Large vessel vasculitis predominantly involving the aorta & its proximal branches associated with weak upper extremity pulses

Takayasu arteritis

15 year old with concerning learning abilities and behavior. Reading and writing skills are significantly impaired compared with other classmates and patient often misbehaves despite receiving multiple detentions. Neuropsychological assessment shows mild intellectual disability and cytogenetic studies show a karyotype containing 47 chromosomes. What would you expect to find on further evaluation?

Tall stature, gynecomastia, sparse facial/body hair, cryptorchidism, azoospermia (Klinefelter syndrome 47 XXY). Usually not diagnosed until puberty when characteristic physical signs begin to develop. Severity generally increases with each additional chromosome.

Lateral ankle sprain due to inversion of a plantar flexed foot and most commonly involves what ligament?

Talofibular ligament

65 year old female with uncontrollable movements. Symptoms become worse with emotional stress and fatigue. PMH includes HTN, hyperlipidemia, osteoarthritis, and schizophrenia. Exhibits facial grimacing, lip smacking, and twisting movement around fingers during exam. Has no delusions or hallucinations. Most likely diagnosis?

Tardive dyskinesia (movement disorder associated with long term treatment with dopamine antagonist medications)

Autosomal recessive disorder caused by Beta hexosaminidase A deficiency, which results in GM2 ganglioside accumulation. Clinical features include progressive neurodegneration and cherry red macular spot. No splenomegaly (in contrast to Neimann Pick)

Tay Sacchs disease

22 year old with a positive pregnancy test at home. Five weeks ago she was diagnosed Lyme disease and was treated with the recommended first line antibiotic. Patient didn't know she was pregnant at the time and is concerned about potential side effects for her baby. Most common finding in infants exposed to this antibiotic in utero?

Teeth staining Tetracyclines (Doxycycline) are the first line treatment for Lyme disease, but are contraindicated in pregnancy due to the potential for fetal tooth discoloration. Amoxicillin should be given instead for treatment of Lyme disease in pregnant women. Tetracyclines localize in the enamel and dentin of developing teeth causing enamel hypoplasia and yellow, ray, or brown staining of the deciduous teeth. If the exposure is near term, permanent teeth can also be stained. For this reason tetracyclines are contraindicated in kids less than 8.

Corresponds to treating an established condition with the goal of minimizing its progression or complications

Tertiary prevention

The cutoff value of a quantitative diagnostic test determines whether a given result is interpreted as positive or negative. Using a lower cutoff point typically causes more patients with the disease to test positive decreasing the number of negatives and therefore increases?

Test sensitivity Note: more patients without the disease will also test positive, resulting in an increased number of false positives and decreased specificity.

Syndrome that develops due to a defect in the testosterone receptors in peripheral tissues. Such patients are phenotypic female who present with primary amenorrhea. Uterus is absent and the vagina ends in a blind pouch, but breast development is normal due to the production of estrogen by aromatase.

Testosterone unresponsiveness (androgen insensitivity syndrome)

Abnormal neural crest cell migration leading to anterior and cephalad DEVIATION of the INFUNDIBULAR SEPTUM during embryologic development resulting in maligned VSD and overriding aorta causes?

Tetralogy of Fallot

Cyanotic spells that improve with squatting, prominent right ventricular impulse, systolic murmur are consistent with?

Tetralogy of Fallot Note: squatting increases the peripheral systemic vascular resistance (afterload) and decreases right to left shunting across VSD

27 year old woman undergoes laparotomy for RLQ pain and is found to have an inflamed terminal ileum. The involved segment of the intestine is removed. Lesion observed on LM reveals a noncaseating granuloma. What cell type most likely mediated the formation of this patient's lesion?

Th1 cells Granulomas are characterized by a large number of epitheloid macrophages that may fuse together to form multinucleated cells (Langhans giant cells) surrounded by a band of lymphocytes. Granuloma formation involves chronic Th1 and macrophage activation in response to a difficult to eradicate infection

Reason patient's vaccinated against H. influenzae can still get otitis media, sinusitis, & bronchitis caused by H. influenzae?

The strain of H. influenzae responsible for these infections does not produce a capsule (nontypeable) Nontypeable H. influenzae is part of normal upper respiratory tract flora and since these strains don't form a polysaccharide capsule immunity is not conferred by vaccination with Hib.

Preferred method of slowing progression of hemochromatosis?

Therapeutic phlebotomy

34 year old female presents with small bluish lesion under the nail of her right index finger. Lesion is extremely tender to touch. If lesion is a tumor, cells of origin are most likely to have what function?

Thermoregulation A benign gloms tumor (glomangioma) can produce a very tender, small (a few mm) red-blue lesion under the nail bed. This type of tumor originates from the modified smooth muscle cells that control the thermoregulatory functions of dermal glomus bodies. Note: subungual melanomas are composed of melanocytes and present the same way

Coenzyme for multiple important dehydrogenase enzymes including transketolase, alpha ketoglutarate dehydrogenase, and pyruvate dehydrogenase

Thiamine (B1)

78 year old with one month history of dyspnea, generalized weakness, fatigue, and palpitations. Reports numbness and tingling in both lower limbs. Daughter states he has not been taking care of himself since wife died. Cardiovascular exam shows displaced apical impulse at 6th intercostal space, third heart sound, and high volume collapsing carotid pulses. Bilateral crackles, 2+ pitting edema, and mild hepatomegaly present. Neuro exam shows decreased light touch and vibration sense in feet with decreased knee and ankle reflexes bilaterally. Deficiency of what nutrient is most likely responsible for his symptoms?

Thiamine (Vitamin B1) Thiamine deficiency causes beriberi and Wernicke-Korsakoff syndrome. Dry beriberi is characterized by symmetrical peripheral neuropathy. Wet beriberi includes the addition of high output CHF

Effect of thiazide diuretics on distal tubular calcium reabsorption

Thiazide diuretics cause hypercalcemia by increasing detail tubular reabsorption of filtered calcium. Increase in serum calcium results in suppression of PTH (distinguishing from primary hyperparathyroidism)

Hypercalciuria is a risk factor for calcium stone formation. What medication can help prevent calcium stone formation by decreasing urine Ca2+ exertion?

Thiazide diuretics. (contraindicated in hypercalcemia!) Via 2 mechanisms: 1. Inhibition of the NA/CL transporter on apical side of DCT decreases intracellular Na+ concentrations activating basolateral Na/Ca2+ anti porter which pumps Na+ into cell in exchange for Ca2+. The resulting decrease intracellular Ca2+ enhances luminal Ca2+ reabsorption across apical membrane 2. Hypovolemia induces by thiazides increases Na+ and H2O reabsorption in proximal tubule leading to passive increase in paracellular Ca2+ reabsorption

Drug class used for T2DM that activate peroxisome proliferator activated receptor gamma & exerts their glucose lowering effect by decreasing insulin resistance

Thiazolidinediones (poglitazone & rosiglitazone)

64 year old male loses consciousness near the entrance of an ER. A physician rushes to the patient and palpates a strong pulse along the inner side of the SCM muscle. Vessel palpated by the physician is a derivative of which aortic arch?

Third (common carotid artery)

35 year old complinainnjg of weakness, fatigue and pallor. PE is unremarkable except for conjunctival pallor. B12 and folate WNL. Bone marrow biopsy shows absence of erythroid precursors but preserved myeloid and megakaryocytic elements. Further work up may show?

Thymic tumor Patient has low erythrocyte count, low percentage of reticulocytes, and low hemoglobin. Pure red cell aplasia is a rare form of marrow failure characterized by severe hypoplasia of marrow erythroid elements in the setting of normal granulopoeisis and thrombopoeisis. Pure red cell aplasia is associated with THYMOMA, lymphocytic leukemia, and parvovirus B19 infection Note: selective absence of erythroid precursors in the bone marrow would not occur with renal cell carcinoma, but can cause erythropoietin production/polycythemia. Uterine fibroids, cerebellar hemangioblastoma, RCC< and HCC can all produce erythropoietin and cause polycythemia, but not pure red cell aplasia

Enzyme that catalyzes oxidation of iodide to iodine, ionidation of thyroglobulin tyrosine residues & iodotyrosine coupling reaction that forms T3 & T4

Thyroid peroxidase (antibodies against this in Hashimoto)

BBB only permits passage of substances from the blood to the brain via transcellular movement across the endothelial plasma membrane, which is limited by diffusion or carrier mediated transport. What is the BBB formed by?

Tight junctions between nonfenestrated capillary endothelial cells (prevents paracellular passage of fluids and solutes)

Patient given single bolus of propofol which provides adequate sedation for procedure and he becomes clinically alert several minutes after administration. What mechanism explains rapid recovery from anesthesia?

Tissue redistribution of the drug. Propofol and other highly lipophilic drugs readily diffuse across membranes, quickly accumulating in tissues receiving high blood flow accounting for their rapid onset of action. They are then redistributed to organs receiving less blood flow explaining their short duration of action

Local cutaneous adverse effects including atrophy/thinning of the dermis associated with loss of dermal collagen, drying, cracking, and/or tightening of skin, telangiectasis, and ecchymoses are due to what chronic treatment of eczematous dermatitis?

Topical corticosteroids

Formula for net excretion rate

Total filtration rate minus total tubular resorption rate Net excretion rate= [(inulin Clearance)(Plasma concentration of substance)]-(Tubular resorption of substance)

Equal to one divided by the sum of the inverse values for resistance of each of the contributing vessels

Total peripheral resistance for a group of vessels in PARALLEL (Note: for vessels in series you just add up all the values; total body circulation is best described as parallel circuit while circulation in individual organ is best described as series)

Common childhood neuropsychiatric disorder characterized by multiple motor and at least one vocal (phonic) tic. Tics characteristically wax and wane and can be suppressed temporarily and are preceded by a premonitory urge. More common in boys. Exacerbated by fatigue and stress. Associated with ADHD and OCD.

Tourette disorder Note: chronic tic disorder involves one or more motor OR verbal tics (but not both) for greater than or equal to 1 year

Clostridium perfringens causes food poisoning and clostridial myonecrosis (gas gangrene). The food positioning is characterized by?

Toxin-mediated, late onset, and causes transient watery diarrhea (occurs when large quantities of clostridial spores are ingested, germinate in the digestive tract, and then elaborate toxin- explaining the late onset)

42 year old with an episode of generalized tonic clonic seizure. Has no history of seizures but has had headaches for the past several weeks. No pets. No travel history. PE is notable for postictal state, oral thrush, and several enlarged cervical and inguinal lymph nodes. MRI of brain with contrast is shown. Most likely cause of his symptoms?

Toxoplasmosis Patient likely has HIV given oral thrush (usually seen with CD4 count <200), cervical and inguinal lymphadenopathy, and brain lesions. In patients with HIV (particularly those with low CD4 counts), headaches, seizures, and multiple ring enhancing CNS lesions on MRI suggest toxoplasmic encephalitis. Cat exposure is not essential for diagnosis, as it can be transmitted through contaminated food. Note: M. tuberculosis and T. solium can cause multiple brain lesions but are unlikely without travel history

Function of RNA polymerase I?

Transcribes 45S pre-rRNA gene into single template which is processed into mature 18S, 5.8S, and 28S rRNAs

Function of RNA polymerase III?

Transcribes DNA to form tRNA and 5S rRNA (exception rRNA gene- located outside nucleolus)

TATA and CAAT boxes are promotors of transcription in eukaryotic cells and are located about 25 and 75 bases upstream from the transcription start site respectively. They promote initiation of transcription by serving as binding sites for?

Transcription factors and RNA polymerase II

Patient with T2DM is started on pioglitazone. Reason for delay of onset of this medication?

Transcription modulation Thiazolidinediones decrease insulin resistance by binding to peroxisome proliferator activated receptor gamma, a transcriptional activator of genes involved in glucose and lipid metabolism. Since the medication is dependent on alteration of gene expression and subsequent protein synthesis, there can be a delay in action of several days to a few weeks. Important genes that are upregulated are GLUT-4 (insulin responsive transmembrane glucose transporter expressed on adipocytes and muscle cells that increases glucose uptake) and adiponectin (cytokine secreted by fat tissue that increases number of insulin responsive adipocytes and regulates fatty acid oxidation)

Process by which a bacteriophage (virus) transfers DNA from one bacterial cell to another. While replicating within the host bacterium, a bacteriophage may accidentally incorporate host bacterial DNA into phage particle. Once released, can then transfer DNA from previous host into newly infected bacterium (bacteria can acquire genes for virulence & antibiotic resistance)

Transduction

Process that streptococcus pneumoniae is able to undergo which allows the bacterium to take up exogenous DNA fragments & express the encoded proteins?

Transformation (through this method strains of S. pneumoniae that do not form a capsule can acquire the genes that code for the capsule and gain virulence) Other bacteria that can undergo transformation: Haemophilus, Bacillus, Neisseria

Fibroblasts overexpress what cytokine that is essential for wound healing in keloid formation?

Transforming growth factor B TGF-B is critical for fibroblast migration, proliferation, and connective tissue synthesis. Increased TGF-B activity is responsible for the hypertrophic/keloid scarring and fibrosis of the lung, liver, and kidney that occur with chronic inflammation

Enzyme of nonoxidative (reversible) branch of pentose phosphate pathway responsible for interconversion of ribose-5-phosphate and fructose-6-phosphate

Transketolase

Cystic fibrosis is an autosomal recessive disease caused by mutations in the CFTR gene that results in thick, plugging mucous and elevated sodium and chloride levels in sweat. What describes the CFTR protein?

Transmembrane ATP-GATED chloride channel (NOT VOLTAGE GATED!!!) Note: the channel pore opens after binding 2 ATP molecules which allows the transport of chloride ions down the electrochemical gradient.

Knockout mouse with a deletion in the platelet endothelial cell adhesion molecule 1 (PECAM-1) gene. Absent expression of this gene will most likely effect what neutrophil function?

Transmigration Margination: hemoconcentration & deceased shear wall stress; improved contact with endothelial lining Rolling: Sialyl Lewis X on leukocyte binds with L-selectin on neutrophils or E-selection or P-selectin on endothelial cells Activation: integrins activated Tight adhesion/crawling: CD18 binds ICAM Transmigration: PE-CAM1 Leukocyte adhesion deficiences (autosomal recessive): LAD1: absence of CD18 leading to inability to synthesize beta 2 integrins Mac-1 and LFA1 affecting tight adhesion, crawling and transmigration. Presents with recurrent skin infections without pus formation, delayed attachment of umbilical cord, and poor wound healing LAD2: more mild, no delay in separation of umbilical cord, less severe and fewer infections LAD3: similar to type 1; severe recurrent bacterial infections, delayed separation of umbilical cord, bleeding complications (due to affected beta 3 integrins on platelets)

An isolate of S. pneumoniae from a patient with meningitis is incubated with low dose radio labeled ceftriaxone and then subjected to protein electrophoresis. Five distinct bands are detected by radioautography. The bands most likely represent radio labeled ceftriaxone that is bound to what?

Transpeptidases Penicillins and cephalosporins function by irreversibly binding to penicillin-binding proteins such as transpeptidases that function to cross link peptidoglycan in the bacterial cell wall. Inhibition of transpeptidase (as occurs with ceftriaxone) leads to cell wall instability and bacteriolysis. Many bacterial species synthesize multiple different penicillin binding proteins (hence why there are 5 distinct bands in this study) Note: a change in the structure of penicillin binding proteins that prevents cephalosporin binding is one mechanism of bacterial resistance to cephalosporins

Results from linear, rather than spiral, development of aorticopulmonary septum in utero

Transposition of great vessels

Mobile genetic elements that can mediate DNA transfer from plasmids to a bacterial chromosome, move genetic material from one position to another along a bacterial chromosome, or transfer genes from a bacterial chromosome to a plasmid. Location of a gene in genome is important as it determines proximity to promotor or suppressor region

Transposons

Hereditary form of cardiac amyloidosis is associated with mutations in what gene?

Transthyretin Note: amyloid deposition in the myocardium results in infiltrative cardiomyopathy which presents in adults with progressive dyspnea, peripheral edema & ascites

Superior mesenteric artery syndrome occurs when what portion of the duodenum is entrapped between the superior mesenteric artery and the aorta causing symptoms of partial intestinal obstruction?

Transverse portion of duodenum This syndrome occurs when the aortomesenteric angle critically decreases to less than 20 degrees (normally 45) , secondary to diminished mesenteric fat (including low body weight, recent weight loss, severe burns, or other inducers of catabolism, and prolonged bed rest), pronounced lordosis, or surgical correction of scoliosis Note: SMA leaves aorta at level of L1 and supplies intestine from duodenum and pancreas to left colic flexure. Transverse portion of duodenum lies horizontally at L3.

Tubular network located at the junction of the A and I bands of striated myocytes

Transverse tubule (T-tubule) system Note: T tubules are open to the extracellular space and facilitate the spread of depolarization to the inside of the cell

Monoclonal antibody against HER2 that is used in patients with HER2 positive tumors

Trastuzumab

Genetic disorder resulting in abnormal development of the 1st and 2nd pharyngeal arches. Craniofacial abnormalities result in airway compromise & feeding difficulties. Absent/abnormal ossicles result in conductive hearing loss

Treacher-Collins Syndrome

23 year old male treated for infectious urethritis with ceftriaxone. A week later his symptoms have not improved. What is next course of action?

Treat with oral azithromycin Urethritis in young man most like to be due to N. gonorrhoeae or C. trachomatis, both of which cause dysuria & mucopurulent discharge. Standard treatment presumes infection by both organisms. Cefrtiaxone given for N. gonorrheae and Doxycycline or azithromycin given for C. trachomatis

Cell wall component and major virulence factor for M. tuberculosis. Protects M. tuberculosis from being killed by macrophages and stimulates granuloma formation

Trehalose dimycolate

Holosystolic murmur that increases in intensity on inspiration (Carvallo sign) most likely represents?

Tricuspid regurgitation The other holosystolic murmurs (mitral regurgitation, VSD) do not increase in intensity during inspiration

Infarcts involving the anterior portion of the medial pons can produce dysarthria and contralateral hemiparesis/lower facial palsy due to disruption of the ipsilateral corticospinal and corticobulbar tracts. Nerve that arises at the level of the middle cerebellar peduncle at the lateral aspect of the mid pons?

Trigeminal

Used to treat susceptible organisms that cause urinary tract, GI, and skin infections. Drug of choice for organisms such as Pneumocystis jiroveci, Toxoplasma gondii, Nocardia, and Stenotrophomonas maltophilia.

Trimethoprim-sulfamethoxazole

Newborn with fetal growth retardation, micrognanthia, low set ears, prominent occiput, hypertonia (clenched hands with overlapping fingers), rocker bottom feet and cardiac/GI/renal defects most likely has what chromosomal abnormality?

Trisomy 18/Edwards syndrome Most commonly the result of meiotic nondisjunction due to advanced maternal age

35 year old G1 P0 woman with abnormal laboratory results. Patient is 18 weeks gestation based on LMP. Results of quadruple screen performed yesterday reveal low maternal serum alpha fetoprotein, low estriol, high B-hCG, high inhibit A. US is scheduled to confirm gestational age and evaluate for fetal anomalies. Most likely cause of patient's results?

Trisonomy 21 (associated with low maternal serum AFP and estriol & increased levels of B-hCG and inhibit A)

Villous atrophy on diagnostic testing, recent TRAVEL, diarrhea, abdominal pain and flatulance is most likely?

Tropical spruce (treat with antibiotics)

Striated muscle fibers fail to contract in response to Ca2+ release from the sarcoplasmic reticulum. Deficiency in what substance best explains this finding?

Troponin Calcium binds to troponin C which shifts tropomyosin to expose the actin binding sites for myosin, allowing contraction to occur

35 year old woman at 12 weeks gestation presents for evaluation of genital warts. PE reveals several verrucous skin colored lesions over labia majora. Virus involved in this condition has an affinity to also affect what respiratory structure in her baby?

True vocal cords (only area in respiratory tract covered with stratified squamous epithelium). Patient has HPV which is a small DNA virus that has predilection for stratified squamous epithelium, which protectively lines anatomical structures that undergo frequent friction & abrasion including true vocal cords, cervix, & anus. Infants can acquire respiratory papillomatosis via passage through HPV infected birth canal leading to warty growths on true vocal cords and weak cry, hoarseness, stridor.

What enzyme degrades complex polypeptides to dipeptides, tripeptides, and amino acids while activating other pancreatic enzymes

Trypsin

6 month old female with skin rash and bouts of diarrhea. Skin reddens after exposure to sunlight. PE reveals dry scale well marginated eruptions of skin of forehead, cheeks, and back of hands. Deficiency in the absorption of what essential amino acid is likely?

Tryptophan Patient has Hartnup disease in which there is a defect in the intestinal absorption and renal tubular reabsorption of natural amino acids such as tryptophan. Tryptophan is precursor to formation of niacin so deficiency results in pellagra (diarrhea, photosensitive dermatitis, dementia).

26 year old woman with absence of her last 3 menstrual periods. Developed bilateral milky discharge. Takes a medication that prevents her from 'hearing voices'. B-hCG is negative. Interruption of what CNS pathway is most likely causing her symptoms?

Tuberoinfundibular pathway (developed amenorrhea and galactorrhea as an adverse effect of antipsychotic drug that blocks D2 receptors). Note: the major dopaminergic pathways in the brain include the mesolimbic and mesocortical pathways (regulate cognition and behavior), nigrostriatal pathway (regulates coordination & voluntary movements), and the tuberoinfundibulnar pathway (inhibits prolactin secretion). Antipsychotics used to regulate the mesolimbic/mesocortical pathways can affect the two other dopaminergic pathways resulting in side effects.

Autosomal dominant syndrome characterized by cortical & subependymal hamartomas, cutaneous angiofibromas (adenoma sebaceum), visceral cysts, and a variety of other hamaratomas as well as renal angiolipomas and cardiac rhabdomyomas. Seizures are a major complication.

Tuberous sclerosis

32 year old man hospitalized with multiple fractures and internal bleeding following MVC. He is successfully resuscitated and taken to operating room for left femoral fracture fixation and is stable postoperatively. He develops oliguria on the second day of hospitalization. Renal biopsy findings reveal loss of columnar epithelium & denudation of basement membrane, interstitial edema, and epithelial cell vacuolization. Most likely outcome of this patients renal injury?

Tubular re-epithelialization Patient developed ischemic acute tubular necrosis as a result of hypotension from hemorrhage. 3 phases: 1) initiation: original toxic/ischemic insult causes slight decrease in urine output and tubular cell damage begins (lasts 36 hours) 2) maintenance: oliguria, fluid overload, HYPERKALEMIA, metabolic acidosis, GFR stabilizes at level below normal, rise in creatinine, LM reveals tubular epithelial necrosis, denudation of basement membrane, casts containing degenerating cells/debris (lasts 1-2 weeks) 3) recovery: re-epithelization of tubules, GFR recovers quickly, tubular cells recover gradually resulting in transient polyuria and loss of electrylotes (HYPOKALEMIA) Most patients with acute tubular necrosis experience tubular re-epithelization and regain renal function. When ATN is associated with multi organ failure, renal function may be permanently impaired (foci of interstitial scarring can be seen on LM)

Cachexia is a syndrome that encompasses anorexia, malaise, anemia, weight loss, and generalized wasting due to an underlying systemic disease. What mediates paraneoplastic cachexia in humans by suppressing appetite and increasing basal metabolic rate?

Tumor necrosis factor alpha Note: IL-1B and IL-6 also contribute to cachexia

Fibrous tissue that immediately overlies the testicles (beneath the tunica vaginalis) and the corpora cavernous of the penis. Excess collagen formation within this tissue can cause significant pain and curvature of the penis in Peyronie disease

Tunica albuginea

2 week old boy with persistent swelling of left scrotum. No noticeable discomfort. Bilateral descended testicles with an enlarged, fluctuant left hemi-scrotum that transilluminates brightly. Most like cause of this patient's condition is swelling in what location?

Tunica vaginalis (communicating hydrocele) During embryogenesis testis descends through inguinal canal drawing with it a diverticulum of peritoneum (processus vaginalis) into the scrotum. Normally, communication between the processus vaginalis and peritoneum is obliterated and the tunica vaginalis is the remaining tissue overlying the testis and epididymis. Communicating hydrocele results when processus vaginalis remains patient and allows peritoneal fluid to accumulate in the tunica vaginalis, resulting in painless swelling that transilluminates on exam

Aortic coarctation in a child/young adult presents with lower extremity claudication (pain and cramping with exercise), blood pressure discrepancy between the upper and lower extremities, and delayed or diminished femoral pulses. Disorder that is associated with aortic coarctation?

Turner syndrome (10% of cases)

Manifests in a neonate with lymphedema, diminished femoral pulses, and cystic hygromas. Short stature, primary amenorrhea, and aortic anomalies are other important clinical features

Turner syndrome (45,X due to loss of paternal chromosome)

Patient suffered MI one year ago. Gross exam of heart after death for unknown reason shows scarring and enlargement of left ventricle. Type of collagen seen in heart is?

Type I collagen (most prevalent collagen in human body and primary collagen in mature scars)

Type of error that occurs when researchers reject the null hypothesis when the null hypothesis is really true (study finds a statistically significant difference between 2 groups when one does not truly exist)

Type I error Alpha is the maximum probability of making a type I error that a researcher is willing to accept

Myasthenia gravis results from what type of autoimmune hypersensitivity reaction against skeletal myocyte surface acetylcholine receptors?

Type II (antibody mediated) Note: goodpasture syndrome similarly involves autoantibodies against basement membrane collagen in the renal glomeruli and lungs

Type of error that occurs when researchers fail to reject the null hypothesis when it is truly false

Type II error Beta is the probability of committing a type II error

Pneumocytes that are the source of pulmonary surfactant and have the ability to proliferate& regenerate the alveolar lining in response to injury?

Type II pneumocystis

Teen with milky white skin, white hair with blue translucent irises. Most likely deficient enzyme?

Tyrosinase (converts tyrosine to melanin)

4 month old boy with intellectual disability, eczema, seizures, light pigmentation/hair, and a 'musty' body odor. What amino acid is most likely essential in this patient?

Tyrosine Phenylketonuria (PKU) results from inability to convert phenylalanine to tyrosine by phenylalanine hydroxyls system making tyrosine essential. Restrict phenylalanine intake!

51 year old woman is diagnosed with breast cancer than is estrogen, progesterone, and human epidermal growth factor 2 receptor positive. Adjuvant therapy with a monoclonal antibody is started. What is the most likely target of this drug?

Tyrosine kinase receptor Human epidermal growth factor receptor 2 (HER2) is a tyrosine kinase receptor that is over expressed in 20% of breast cancer patients. In such patients, HER2 blockade with the monoclonal antibody TRASTUZMAB down regulates cellular proliferation and promotes apoptosis. Note: ER and PR positive breast cancer is treated with anti estrogen medications (SERMs such as tamoxifen in premenopausal women & aromatase inhibitors such as anastrozole or letrozole in postmenopausal women), but these are NOT monoclonal antibodies!

Children with xeroderma pigmentosum usually have severe photosensitivity, hyperpigmentation in sun exposed areas, and greatly increased risk for skin cancer. It is an autosomal recessive disorder characterized by defective nucleotide excision repair often caused by a deficiency in what enzyme?

UV-specific ENDONUCLEASE

Protein that undergoes ATP dependent attachment to other proteins labeling them for degradation in the proteasome

Ubiquitin Note: impairment of the ubiquitin-proteasome system can contribute to development of neurodegenerative disorders such as Parkinson's and Alzheimer's (mutations in Parkin, PINK1, DJ-1 are associated with AR forms of Parkinson's that have early age of onset)

Process of degrading viral particles via ubiquitin proteome pathway?

Ubiquitin ligases recognize specific protein substrates and attach a ubiquitin tag. The proteins are degraded to an appropriate size via the proteasome and coupled with the MHC class I protein complex in the ER. They are presented on cell surface for recognition by cytotoxic CD8+ lymphocytes

Decrease in glucuronyl transferase result in what?

Unconjungated hyperbilirubinemia; improves bilirubin solubility for biliary excretion

Decreased numbers of functional T-tubules in muscle fibers leads to?

Uncoordinated contraction of myofibrils, which manifests as muscle weakness. Transverse tubules (T-tubules) are invagination of the sarcolemma that transmit depolarization signals to the sarcoplasmic reticulum to trigger the release of calcium and induce smooth muscle contraction. The uniform distribution of T-tubules in striated muscle fibers ensure that each myofibril contracts at the same time which is necessary for efficient contraction

When a specific ion channel opens, the respective ions will flow across the membrane in a direction that bring the resting membrane potential (-70mV) closer to that ion's equilibrium potential

Under normal psychologic conditions there is low concentration of Na+ and Cl- in the cell and high concentration of K+ in the cell. K+ moves out and Ca2+, Cl-, and Na+ move into cell.

Anticoagulant most effective in inactivating thrombin?

Unfractionated Heparin- it can bind to BOTH antithrombin and thrombin to allow antithrombin to inactivate thrombin

36 year old woman with frequent urination since exacerbation of MS 2 months ago. Has difficulty holding urine and has passed a small amount of urine while trying to reach bathroom on several occasions. No leakage with coughing or sneezing. Explanation for symptoms?

Uninhibited bladder contraction (urge incontinence) Patient's with MS most commonly develop urge incontinence due to loss of CNS inhibition of detrusor contraction in the bladder. As disease progresses, the bladder can become atonic and dilated, leading to overflow incontinence.

Portion of the brain that contains neural structures (optic nerve, pretectal nuclei, Edinger-Westphal nuclei, oculomotor nerve) that mediate direct and consensual pupillary light reflex?

Upper midbrain

Infants with hydrocephalus typically develop macrocephaly, poor feeding, muscle hypertonicity, and hyperreflexia. Untreated hydrocephalus can lead to developmental delay and seizures. Treatment requires surgical placement of a shunt to bypass the obstruction, most often via the ventriculoperitoneal route. Hypertonicity and hyperreflexia result from?

Upper motor neuron injury caused by stretching of the periventricular pyramidal tracts

Musculocutaneous nerve innervates the major forearm flexors (biceps brachii, brachialis) and coracobrachialis (flexes and adducts the arm) and provides sensory information to the lateral forum. It can be injured by trauma or strenuous upper extremity exercise and is derived from what portion of the brachial plexus?

Upper trunk (C5-C7)

Concentrations of PAH, creatinine, inulin, and urea increase as fluid runs along the proximal tubule, while concentrations of bicarbonate, glucose, and amino acids decrease. Inulin is not secreted into the renal tubules, while PAH is. Identify these substances on tubular fluid/plasma ultrafiltrate graph

Upward slope indicates increasing concentration of substance as fluid moves toward distal parts of the tubule (usually result of secretion or non-resorption of that substance). Downward slope indicates active reabsorption of substance in proximal tubule.

29 year old undergoes hysterectomy. Postoperatively, she has a fever and right sided back pain. She has been able to urinate without any difficulty. Injury to what structure is responsible for her postoperative symptoms?

Ureter (ureteral injury is a rare but serious complication of pelvic surgery presenting with flank pain and fever) Ureter can be injured during hysterectomy due to its close proximity to the uterine structures. The distal ureter may be severed during ligation of the uterine vessels because the ureter passes inferior and lateral to the uterine artery at the level oft the internal cervical os prior to entering the bladder (water under the bridge). Since the second ureter is uninjured, patient can still urinate normally. Bilateral ureteral injury is extremely unlikely and results in anuria and bilateral flank pain.

The ureteric bud & metanephric (blastema) mesoderm ultimately give rise to what structures?

Ureteric bud- Collecting system of the kidney including the collecting tubules and ducts, major and minor calyces, renal pelvis and ureters. Metanephric (blastema) mesoderm- glomeruli, Bowman's space, proximal tubules, loop of Henle, DCT

If fetal unilateral hydronephrsis is caused by obstruction, what is the most likely site?

Ureteropelvic junction (inadequate canalization of the uteropelvic junction, the connection between the kidney and the ureter, is the most common cause of UNILATERAL fetal hydronephrosis). Note: most common cause of BILATERAL fetal hydronephrsis in boys is posterior urethral valves (caused by an obstructive, persistent urogenital membrane at the junction of the bladder and the urethra). Note: Vesicoureteral reflux is a NONOBSTRUCTIVE cause of fetal hydronephrosis

53 year old male with 2 days of severe flank pain radiating to groin. KUB is normal. CT scan of abdomen and pelvis performed which shows stag horn calculus in renal calculus. Most likely type of stone?

Uric acid (most common radiolucent stones- negative KUB) Uric acid stones are associated with urine pH less than 5.5, high purine intake, or malignancy. Gout is another risk factor.

14 month old boy with failure to thrive and developmental delay. MCV is 114. Urine specimens contain large amounts of orotic acid crystals, Supplementation with what substance would benefit this patient?

Uridine Orotic aciduria is a rare autosomal recessive disorder of de novo pyrimidine synthesis that occurs due to a defect in uridine 5'-monophosphate (UMP) synthase. Children typically presents with physical and mental retardation, megaloblastic anemia, and large amounts of urinary orotic acid. Uridine supplementation can improve symptoms as uridine is concerted to UMP via nucleoside kinases. Note: differentiate from ornithine transcarbamylase deficiency in which patients have elevated orotic acid but no megaloblastic anemia and DO have symptoms of hyperammoniemia (encephalopathy) & presents within first few weeks of life decr OTC= OH, no M= orotic aciduria + Hyperammonemia, no Megaloblastic anemia decr UMP synthase = OM, no H= orotic aciduria + Megaloblastic anemia, no Hyperammonemia

72 year old man with altered mental status, skin flushing, and high fever. If blood cultures are positive for E. coli what is most likely site of infection leading to bacteremia?

Urinary tract E. coli is most common cause of UTI in both healthy adults and elderly patients. E. coli is part of normal GI flora, and special adhesive proteins (fimbriae) allow some strains to colonize and ascend the urinary tract. This can result in UTI, pyelonephritis, or bacteremia and sepsis following success to the blood stream. UTIs are most common cause of E. coli bacteremia.

4 day old girl with increasing difficulty feeding over the past 24 hours. Parents say she has been too sleepy to feed and has been vomiting. Infant has 3 healthy living siblings and an older brother who died in infancy from 'low sugar in his blood'. PE shows tachypnea & signs of dehydration. Patient is only responsive to painful stimuli. Urine reveals significantly elevated levels of methylmalonic acid. Lab values that would result from this patients condition?

Urine propionic acid: elevated Serum glucose: low Urine ketones: elevated Serum ammonia: elevated Methylmalonic acidemia is an organic academia due to complete or partial deficiency of mehtylmalonyl-CoA mutate. Complete deficiency classically presents with lethargy, vomiting, and tachypnea in a newborn. Labs show hyper ammonia, ketotic hypoglycemia, and metabolic acidosis. Diagnosis is confirmed by elevated urine methylmalonic acid and propionic acid. Build up of methylmalonic acid and prprionic acid leads to metabolic acidosis. Hypoglycemia results from increased metabolic rate leading to increased glucose utilization and direct toxic inhibition of gluconeogenesis by the organic acids. Presence of hypoglycemia leads to increased FFA metabolism that produces ketones (further increasing anion gap metabolic acidosis). The organic acids also directly inhibit the uric acid cycle leading to hyperammonemia. Note: propionic acidemia, a deficiency in proprionyl CoA carboxylase, also results in hyperammonemia, hypoglycemia, and metabolic acidosis, but it doesn't include elevated levels of methylmalonic acid

Main mechanism underlying all types of renal stones?

Urine supersaturation. Low fluid intake increases the concentration of stone forming agents, thereby promoting stone formation. All patients with nephrolithiasis should be advised to consume ample water and clear liquid

Transient hypersensitivity disorder characterized by pruritic erythematous plaques that arise suddenly and resolve over hours. Most often cause by IgE mediated degranulation of mast cells, leading to increased permeability of the microvasculature with edema of the superficial dermis.

Urticaria

82 year old woman diagnosed with pneumonia and started on empiric antibiotics. After several days of treatment her condition deteriorates and she is transferred to ICU. Sputum cultures were obtained on admission but were never checked by the team, resulting in a delay in starting more specific antibiotic treatment. What intervention would have most likely prevented this type of medical error?

Use standardized patient handoffs Multiple providers assume responsibility for a patient's care at different times of the day which can result in discontinuity of care. Communication failures between physicians during patient hand offs are a major cause of medical errors and can be reduced by use of a structured process. Note: applying systematic organization method is more effective than more detailed information so that essential, larger clinical picture doesn't get lost in the details.

Activation of muscarinic receptors results in peripheral vasodilation or vasoconstriction?

VASODILATION. (due to synthesis of NO in endothelial cells which leads to vascular smooth muscle relaxation/hypotension) NOTE: Muscarinic receptor activation in other sites causes smooth muscle contraction

Key growth factors that promote angiogenesis in neoplastic and granulation tissue?

VEGF & FGF Proinflammatory cytokines (IL-1, IFN-gamma) can indirectly promote angiogenesis through increased VEGF expression. The laminin in basement membranes may pose a physical barrier to the sprouting of new blood vessels.

During inspection of ear canal, a speculum is inserted into the meatus in close contact with the posterior wall, causing the patient to suddenly become lightheaded and faint. He recovers in a few minutes with no residual confusion. Most likely irritated nerve during procedure?

Vagus nerve Vagus nerve provides cutaneous sensation to the posterior external auditory canal via its small auricular branch. Sensation to the rest of the canal is from the mandibular branch of the trigeminal nerve V3

Nerve responsible for sensation and taste to the posterior area of tongue ROOT?

Vagus nerve (CN X)

6 year old African American male 'runs out of breath quickly'. He has missed several pediatric vaccinations and has been hospitalized twice with a 'chest infection' and once with abdominal pain. Patient states that sometimes his 'bones hurt'. What protein change accounts for his condition?

Valine substitution for glutamic acid (point mutation in the 6th codon of the beta-globin gene). HbS polymerizes at low oxygen tension causing sickling and hemolysis of erythrocytes and resultant vascular occlusion. Poor exercise tolerance and exertional dyspnea are due to anemia.

7 day old infant with progressive lethargy, vomiting, and poor feeding. On exam, infant is somnolent and dehydrated with decreased muscle tone. Labs reveal metabolic acidosis with elevated anion gap, ketosis, and hypoglycemia. Further eval reveals markedly elevated propionic acid level due to defective conversion of propionyl-CoA to methylmalonyl-CoA. This patient is unable to use what amino acids for energy production?

Valine, isoleucine, methionine, threonine Propionyl-CoA is derived from metabolism of valine, isoleucine, methionine, and threonine & odd chain fatty acids. Congenital deficiency of propionyl-CoA carboxylase, the enzyme responsible for the conversion of propionyl-CoA to methylmalonyl-CoA, leads to development of propionic academia. Hypoglycemia and ketosis develop secondary to acidosis. Presents with lethargy, poor feeding, vomiting, and hypotonia 1-2 weeks after birth. Treatment is low protein diet containing minimal amounts of valine, isoleucine, methionine, threonine

11 year old boy who presents after briefly losing consciousness. Mother saids she heard a loud noise then turned around to see him on the floor shaking and jerking. Mom says he often stares into space and doesn't respond to questions and doesn't respond. Patient is sleepy but oriented to time and place. Several hours later he is alert and wants to go home. Best long term treatment for this patient?

Valproate Older children with absence seizures may also develop generalized tonic-clonic or myoclonic seizures. Although ethosuximide is effective against isolated absence seizures, it doesn't suppress tonic-clonic seizures. VALPROATE is a BROAD spectrum anti epileptic that is effective for both types of seizures

Antibiotic that binds to the terminal D-alanine residues of cell wall glycoproteins and prevents transpeptidases from forming cross links

Vancomycin

Tri-cyclic glycopeptide that inhibits synthesis of peptidoglycan, main component of the cell wall of gram positive bacteria. Used against MRSA as well as C-diff

Vancomycin

Rapid administration of what antibiotic can cause histamine release resulting in flushing ('red man syndrome')?

Vancomycin Note: can also cause dose related nephrotoxicity and ototoxicity

Pressure in the left renal vein may become elevated due to compression where the vein crosses the aorta beneath the superior mesenteric artery. This 'nutcracker effect' can cause hematuria and flank pain. Pressure can also be elevated in left gonadal vein leading to formation of what?

Varicocele

52 year old man with several days of chest pain and cough. Immigrated to UA from Taiwan 20 years ago. PE reveals a decrescendo-type diastolic murmur over the RSB. Serum fluorescent treponemal antibody absorption testing is positive. CXR reveals mediastinal widening. Pathologic process most likely responsible for his symptoms starts with?

Vasa vasorum obliteration Tertiary syphilis can result in thoracic aortic aneurysm. If the aneurysm compresses adjacent structures and dilates the aortic valve ring, a murmur (aortic regurgitation) and mediastinal widening might be present. Pathogenesis begins with vasa vasorum endarteritis and obliteration, resulting in inflammation, ischemia, and weakening of the aortic adventitia. Fluorescent treponemal antibody absorption testing is specific for syphilis.

12 year old African American male with fever, chest pain, dyspnea. History significant for 2 prior hospitalizations for abdominal pain, which resolved with analgesics & hydration. Today labs reveal hematocrit of 23& and reticulocyte count of 9%. Dies several hours later. Autopsy reveals firm and brown spleen. Spleen findings are due to?

Vascular occlusion Patient with sickle cell anemia and acute chest syndrome. Major chronic pathologic changes in the spleens of patients with sickle cell anemia result from repetitive splenic infarctions caused by splenic microvessel occlusion. Fibrosis, brown discoloration, and eventual autosplenectomy result.

Intestinal astresias of the midgut (jejunum, ileum, proximal colon) are the result of?

Vascular occlusion in utero

34 year old female with progressive exertion dyspnea for the past 6 months. Mother died of pulmonary arterial hypertension at age 32. Loud second heart sound heart at left upper sternal border on exam. ECG shows right axis deviation. If patient's condition is inherited, what would be the cause of her disease?

Vascular smooth muscle proliferation Pulmonary arterial hypertension follows a 2 hit hypothesis. An abnormal BMPR2 gene acts as the first insult and predisposes to excessive endothelial and smooth muscle cell proliferation. A second insult (infection, drugs, ion channel defects) is then thought to activate the disease process, resulting in vascular remodeling, elevated pulmonary vascular resistance, and progressive pulmonary hypertension

Exercising muscles can receive up to 85% of the total cardiac output during periods of strenuous activity. Although sympathetic discharge during exercise causes increased cardiac output and splanchnic vasoconstriction, there is only a modest increase in mean blood pressure because?

Vasodilation within the active skeletal muscles significantly DECREASES the total SYSTEMIC VASCULAR RESISTANCE (mediated by local release of adenosine, K+ ions, ATP, CO2, and lactate)

Dynamic left ventricular outflow tract obstruction that occurs in hypertrophic cardiomyopathy worsens with decreased LV volume, which can be caused by reduction in cardiac preload and/or after load. Medications that should be avoided with this condition?

Vasodilators (dihydropyridine calcium channel blockers, nitroglycerin, ACE inhibitors as they decrease SVR & afterload and lower LV volumes) & diuretics (decreased preload) Negative inotropic agents such as beta blockers & nondihydropyridine calcium channel blockers (verapamil) & disopyramide reduce LVOT obstruction and are helpful in symptomatic patients with HCM HCM patients have a systolic murmur that accentuates with standing from a supine position

What changes do you expect to see regarding O2, CO2, and pH in venous blood at the peak of exertion?

Venous blood O2 content decreased because exercising muscles extract additional O2. CO2 content increased due to increased production & decreased pH. Note: homeostatic mechanisms maintain arterial blood gas levels and arterial pH near resting values

62 year old woman with difficulty walking that began 3 hours ago. Cannot feel right side of body. History of HTN and DM. Has smokes 1PPD for past 30 years. Father had MI at 60. PE shows loss of touch, temperature, and vibratory sensation affecting RUE and RLE. Sensation is diminished over right side of face. Muscle strength 5/5 throughout. Most likely suffered stroke affecting what brain structure?

Ventral posterior thalamus Ventral posterior lateral nucleus (receives input from the spinothalamic tract and dorsal columns) and ventral posterior medial nucleus (receives input from trigeminal pathway) of the thalamus send somatosensory projections to the cortex via thalamocortical fibers. Damage to these nuclei results in complete contralateral SENSORY (touch, pain/temp, vibration/proprioception) loss. Severe proprioceptive defects can cause unsteady gait Note: anterior 2/3 of the posterior limb of the internal capsule is mainly composed of motor fibers (corticospinal tract) & posterior 1/3 contains sensory fibers (thalamocortical tract); although very small lesions to posterior limb may cause contralateral pure sensory deficits, most lesions result in pure motor or combined sensorimotor deficits

What do ventral posterolateral nucleus, ventral posteromedial nucleus, lateral geniculate nucleus, and medial geniculate nucleus in the thalamus mediate?

Ventral posterolateral nucleus- somatic sensation of body Ventral posteromedial nucleus- facial sensation and tase Lateral geniculate nucleus- vision Medial geniculate nucleus- hearing

Most frequent mechanism of sudden cardiac death in the first 48 hours after an acute MI & is related to electrical instability in the ischemic myocardium

Ventricular fibrillation

Asymptomatic 11 year old bot with murmur associated with a harsh, low pitched, holosystolic murmur at the left sternal border. Accentuated during maneuvers that increase after load (handgrip maneuver).

Ventricular septal defect Note: A small VSD is usually asymptomatic and produces a louder murmur due to higher interventricular pressure gradient

4 distinct anatomic abnormalities of Tetralogy of Fallot?

Ventricular septal defect, overriding aorta over right and left ventricles, right ventricular outflow tract obstruction, right ventricular hypertrophy

5 year old boy with persistent food seeking behavior over the past 5 months. Physician becomes concerned about possible CNS involvement. Lesion in what area of the hypothalamus could be associated with increased food seeking behavior?

Ventromedial nucleus Hypothalamus is located in the diencephalon and is a key region responsible for homeostasis. Ventromedial nucleus is vital in monitoring blood glucose and is thought to be the center of satiety; lesions in the VMN lead to hyperplasia and obesity. Lateral nuclei signal hunger and lesions located in this region result in starvation in adults and failure to thrive in infants. Causes of hypothalamic lesions include disease, trauma, infection or tumors. In children, tumors of the hypothalamus are commonly gliomas & in adults they are usually metastatic in origin.

30 year old with severe headache and neck pain following an MVC. Neuroimaging is unremarkable except for left transverse foramina fracture at C2. Patient placed in cervical collar and admitted for further eval. While in hospital he suddenly develops dizziness and left facial numbness. Neurologic exam shows partial ptosis and miosis of the left eye as well as nystagmus. There is reduced sensation to pain and temperature on the left side of the face and the right side of the body. Patient is ataxic and leans to the left when he walks. Vascular imaging would most likely reveal an acute dissection to what artery?

Vertebral Posterior inferior cerebellar artery (branch of vertebral artery) occlusion causes lateral medullary (Wallenberg) syndrome, characterized by vertigo/nystagmus, ipsilateral cerebellar signs (fall to side of lesion), loss of pain/temperature sensation in the ipsilateral face & contralateral body, bulbar weakness (dysphonia, dysphagia), and ipsilateral Horner syndrome. This condition can occur in the setting of cervical spine trauma with dissection of the vertebral artery (vertebral arteries course through transverse foramina in the neck before entering the skull at the foramen magnum)

Sensation of excessive motion compared to physical reality. Commonly due to dysfunction within the vestibular system (such as inner ear)

Vertigo

23 year old woman with recurrent episodes of UTI. Has had 5 episodes of cystitis and an episode of pyelonephritis in the past year. Symptoms tend to occur a few days following sexual intercourse. Most likely predisposing factor for pyelonephritis in this patient?

Vesicoureteral urine reflux Suppression of endogenous flora, colonization of the distal urethra by pathogenic gram negative rods, and attachment of these pathogens to the bladder mucosa are the stages of pathogenesis in lower UTIs. Anatomic or functional vesicoureteral reflux is almost always necessary for the development of acute pyelonephritis (the normal vesicoureteral junction does not allow retrograde flow of urine)

34 year old man with persistent buzzing and decreased hearing in his right ear for the past 6 months. MRI reveals intracranial tumor. Lesion most likely arose from what structure?

Vestibulocochlear nerve (CN VIII) Acoustic neuromas are schwann cell derived tumors that typically arise from the vestibular portion of the vestibulocochlear nerve and are commonly located at the cerebellopontine angle (between cerebellum and lateral pons). Patients usually present with unilateral sensorineural hearing loss and tinnitus. Damage to vesticular portion can result in vertigo, dysequillibrium, & nystagmus. Bilateral acoustic neuromas are frequently seen in patients with neurofibromatosis type 2

42 year old Asian male who is a known HBV carrier presents with malaise, weight loss, and abdominal fullness. Evaluation reveals a liver mass and an increases serum alpha fetoprotein level. Current condition is most likely caused by what?

Viral DNA integration into the host genome Hepatocellular carcinoma is associated with increases AFP and is strongly associated with HBV infection. Integration of viral DNA into the genome of host hepatocytes triggers neoplastic changes. Other wise factors for hepatocellular carcinoma: HCV, alcoholic cirrhosis, aflatoxins, hemochromatosis

65 year old man presents with high fevers, severe headaches, seizures, & confusion. CSF reveals elevated opening pressure with hemorrhagic lymphocytic pleocytosis, increased protein, normal glucose. MRI of brain reveals abnormal sign in bilateral temporal loves. Drug that inhibits what enzyme would be most effective in this patients treatment?

Viral DNA polymerase Most effective treatment for HSV encephalitis is IV acyclovir whose MOA is complete inhibition of viral DNA polymerase

Defects in type I interferon release leads to increased susceptibility to what type of infections?

Viral infections (IFN-alpha and IFN-beta are released from virally infected cells and cause macrophage and cytotoxic T lymphocyte activation)

45 year old Caucasian woman unable to walk. Spinal cord shows symmetric myelin layer vacuolization and axonal degeneration involving the posterior columns and lateral corticospinal tracts. Most likely cause of this patient's condition?

Vitamin B12 deficiency Myelopathy associated with vitamin B12 deficiency is called subacute combined degeneration. 'Combined' refers to degeneration of both the ascending (dorsal columns) and descending (corticospinal tract) pathways). Loss of position and vibration sensation, taxis, and spastic paresis are common manifestations. Note: in tabes dorsalis damage only occurs to the dorsal columns/roots, not the lateral corticospinal tract Note: anterior spinal cord syndrome (compression/occlusion of artery that runs along the front of the spinal cord) usually presents with complete loss of strength below level of injury with loss of pain and temperature, but preserved vibratory senses

Vitamin deficiency characterized by sore throat, hyperemia of pharyngeal mucous membranes, edema of mucous membranes, cheilitis, stomatitis, glossitis, and seborrheic dermatitis.

Vitamin B2 Riboflavin (treat with riboflavin supplementation!) Can be associated with normocytic normochromic anemia

Isoniazid, used to treat TB, can result in what vitamin deficiency?

Vitamin B6 deficiency Clinical manifestations consist of stomatitis, cheilosis, atrophic glossitis, sideroblastic anemia, and peripheral neurpathy

Carnitine (AA derivative responsible for transporting FA into mitochondria for B-oxidation) is synthesized from lysine and methionine. What vitamin is essential for this synthesis?

Vitamin C

Necessary cofactor for hydroxylation of proline & lysine residues in collagen. Deficiency results in decreased strength of collagen fibers & causes scurvy

Vitamin C

Necessary for the hydroxylation of proline and lysine residues in pro-collagen. Deficiency is most often seen in severely malnourished individuals and leads to capillary bleeding, poor wound healing, and periodontal disease. In children, bony deformities and subperiosteal hemorrhages are also characteristic

Vitamin C

15 year old boy presents for follow up. Has suffered from recurrent respiratory infections and chronic diarrhea since early childhood. Has been hospitalized for parenteral antibiotic treatment. Currently on pancreatic enzyme therapy and multiple dietary supplements. PE shows decreased proprioception and hyporeflexia in LE. Labs are consistent with mild hemolytic anemia. Most likely cause of these findings?

Vitamin E deficiency Vitamin E deficiency can occur in individuals with fat malabsorption (such as this patient with CF). Deficiency of this fat soluble vitamin is associated with increased susceptibility of the neuronal and erythrocyte membrane to oxidative stress. Clinical manifestations include ataxia, impaired proprioception & vibratory sensation, and hemolytic anemia

Involved in post-translational conversion of glutamate to gamma-carboxyglutamic acid which is necessary for function of many clotting factors & regulatory proteins involved in coagulation cascade

Vitamin K

Disease associated with deletion of tumor suppressor gene on chromosome 3. Results in constitutive expression of HIF & activation of angiogenic growth factors. Presents with hemangioblastomas of the retina, cerebellum, and/or medulla, bilateral renal cell carcinoma, and other tumors

Von Hippel Lindau disease

Rare, autosomal dominant condition characterized by the presence of capillary hemangioblastomas in the retina and/or cerebellum as well as congenital cysts/neoplasms in the liver, kidney, and pancreases. Patients are also at risk of renal cell carcinoma which can be bilateral

Von Hippel-Lindau disease

Inherited peripheral nervous system tumor in which patients develop neurofibromas, optic nerve gliomas, Lisch nodules (pigmented nodules of iris), and cafe au lait spots (hyperpigmented cutaneous macules)

Von Recklinghausen's disease/Neurofibromatosis type 1

24 year old with chronic fatigue. Has a history of heavy menstrual periods since menarche and recalls frequent nosebleeds as a child. Hemoglobin is 9.2, MCV is 72, and she has decreased levels of ferritin. Most likely diagnosis?

Von Willebrand factor deficiency Microcytic anemia with low ferritin level is characteristic of iron deficiency anemia. Heavy menstrual bleeding and frequent nose bleeds further suggests von willebrand disease. Patients with von willebrand disease often present with a LIFELONG history of mucosal bleeding, including gingival bleeding, epistaxis, and/or menorrhagia. These patients have normal platelet levels but a prolonged bleeding time due to impaired platelet functioning. Von willebrand factor normally binds to COLLAGEN. It functions as a promotor of platelet adhesion at sites of vascular injury by binding platelet glycoproteins to sub endothelial collagen on injured blood vessel walls. It also acts as a protective carrier protein for circulating factor VIII

Syndrome characterized by loss of pain/temp over ipsilateral face & contralateral body (damage to spinal trigeminal and spinothalamic tracts), ipsilateral bulbar muscle weakness (nucleus ambiguous), vertigo (vestibular nuclei), ipsilateral limb taxis (inferior cerebellar peduncle), & Horner syndrome (descending sympathetic fibers

Wallenberg syndrome (results from injury to dorsolateral medulla- supplied by PICA)

Vitamin K and fresh frozen plasma can be used to reverse effects of what?

Warfarin Note: Vitamin K requires synthesis of new coagulation factors so it takes days to reverse. FFP should be used in acute life threatening bleeding

What is oral thrush associated with?

Wearing dentures, diabetes mellitus, immunosuppression

Due to degeneration of the anterior horn cells that is caused by a mutation in the SMN1 gene. Causes symmetric progressive proximal weakness that occurs in childhood

Werdnig-Hoffman syndrome (spinal muscle atrophy type 1)

Single strand flavivirus transmitted by mosquitoes (Culex species), most commonly in the summer. Most infections are asymptomatic or may present with a flu like illness, often with a maculopapular or morbilliform rash on back and chest. Neuroinvasive disease manifests as meningitis, encephalitis, or asymmetric flaccid paralysis & patients may have parkinsonian features (rigidity, bradykinesia, tremor, postural instability). Older age and malignancy/oral transplant are risk factors for neuroinvasive infection

West nile virus Note: acute onset ASYMMETRIC paralysis and concurrent tremor are highly suggestive of WNV. Polio could cause paralysis but its eradicated in the USA so VERY unlikely

Pyuria and bacteriuria are found in both upper and lower UTIs. What are only formed in the renal tubules and are pathognomonic for acute pyelonephritis when accompanied by symptoms of acute UTI?

White blood cell casts

An increase in effective stroke volume or ejection fraction is depicted on the left ventricular pressure volume relationship graph by what?

Widening of the loop with a shift in the isovolumetric relaxation line to the left (indicating less residual blood volume in the ventricle at end systole)

Syndrome that consists of triad of eczema, thrombocytopenia, and combined B & T lymphocyte deficiency. Onset is early in life with thrombocytopenia present at birth and eczema and repeated infections, particularly encapsulated organisms, following at 6-12 months of age

Wiskott-Aldrich syndrome (X linked recessive- only in males!)

X linked disorder characterized by immunodeficiency eczema, & thrombocytopenia.

Wiskott-Aldrich syndrome (immunodeficiency is a combined B and T cell disorder)

States that the first 2 nucleotide positions on the mRNA codon require transitional (Watson-Crick) base pairing whereas the third nucleotide position may undergo less stringent base pairing

Wobble hypothesis (genetic code is considered degenerate because more than 1 codon can code for a particular AA)

32 year old migrant ranch worker is brought to ED with painful paroxysmal involuntary muscle contractions involving the jaw, neck, and trunk. The causative substance most likely traveled by which route?

Wound --> motor neuron axons --> spinal cord Tetanospasmin is a potent neurotoxin produce by Clostridium tetani that is responsible for tetanus which causes uncontrolled muscle spasms and respiratory failure. TOXIN (not organism) travels within the motor neuron by retrograde transport into spinal cord where it causes inhibition of inhibitory interneurons and unregulated firing of primary motor neurons. Classic findings are jaw stiffness (truisms) & bizarre 'smiling' appearance (risus sardonicus).

Radial nerve provides sensory information to the skin of the posterior arm, forearm and dorsal lateral hand and provides motor innervation to all of the extensor muscles of the upper limb below the shoulder. Damage to the proximal radial nerve (at the axilla or mid shaft humerus) may result in?

Wrist drop BE ABLE TO IDENTIFY NERVES ON BRACHIAL PLEXUS

Ulnar nerve injury most commonly occurs at the elbow and may cause sensory loss in the medial one and a half digits of the hand. Patient's can have an 'ulnar claw deformity' with weakness of what movements?

Wrist flexion/adduction, finger abduction/adduction, and flexion of the 4th/5th digits. Note: Nerve injury at the wrist presents similarly

Immunodeficiency syndrome caused by insufficient production of mature B cells, which predisposes mainly to recurrent infections with encapsulated pyogenic bacteria (S. pneumoniae, H. influenzae type B)

X-linked (Bruton's) agammaglobulinemia

25 year old man experiences severe intolerance to certain medications & on 2 occasions his reactions to various drugs have required hospital admission. Pedigree shows the only males are affected by the drug intolerance & male offspring of unaffected parents are affected. This condition most likely exhibits what inheritance pattern?

X-linked recessive (most consistent with an X-linked recessive inheritance from an asymptomatic carrier female) In X-linked recessive inheritance affected males will always produce unaffected sons and carrier daughters and carrier female have a 50% chance of producing affected sons and carrier daughters. G6PD deficiency (which is what this patient has) follows this inheritance pattern and causes acute hemolytic anemia in response to oxidant drugs.

Yellow macule/papules found on the medial eyelids.

XanthElasmas = on the Eyelid (a type of xanthoma) Note: they are dermal accumulations of macrophages containing cholesterol and triglycerides and are generally associated with primary or secondary hyperlipidemia or dyslipidemia & thus when one is noted on exam serum lipids and blood glucose level should be checked to determine the patient's type of lipid disorder and to rule out underlying diabetes. An LDL receptor abnormality is the most common cause

6-mercaptopurine is mainly degraded in the liver by what enzyme?

Xanthine oxidase Note: allopurinol can increase concentration of 6-MP significantly. HGPRT is necessary for 6-MP activation

Chronic uric acid lowering therapy is recommended for patients with gout who have frequent gouty attacks, uric acid kidney stones, tophi, or chronic joint destruction from gout. Preferred treatment?

Xanthine oxidase inhibitors (allopurinol, febuxostat) to decrease uric acid production. Note: initiate during intercritical periods (between attacks) a acute changes in serum uric acid levels can worsen an acute attack. Note: uricosuric agents (probenecid) are second line drugs for chronic management of gout; they increase renal excretion of uric acid and are therefore contraindicated in patients with a history of renal stones

Does negative predictive value vary with pretest probability of a disease?

YES. A patient with high probability of having a disease (for example prior radiation to neck and thyroid cancer) will have a low NPV with a negative test but a patient with a low probability of having a disease (no risk factors) will have a high NPV with a negative test

Researcher investigating antibodies that disrupt the binding of actin to structural elements within the sarcomere. Where do the antibodies bind?

Z line. Sarcomere is defined as distance between two Z lines. Thin actin filaments in the I band bind to structural proteins at the Z line. Unbound elements of actin project into the middle of the sarcomere where they interact with thick myosin filaments. Thick myosin filaments in the A band are bound to structural proteins at the M line. A band- myosin filaments (always remains same length during muscle contraction). M line (not in picture) - center of A band where myosin filaments anchor to structural elements I band- actin does not overlap with myosin H band- myosin does not overlap with actin

Type of kinetics in which a constant amount of drug is eliminated for every unit time regardless of plasma concentration

Zero order kinetics

Short acting hypnotic medication used for the treatment of insomnia that is chemically unrelated to benzodiazepines. Has the same MOA as benzos (binds to GABA A receptor and enhances inhibitory action of GABA on the CNS), but has a much lower risk of tolerance and dependence

Zolpidem

Middle zone of adrenal cortex that contains foamy appearing cells in columns that secrete glucocorticoid hormones (primary cortisol)

Zona fasciculata

Outermost zone of the cortex composed of cells arranged in rounded or arched clusters that secrete mineralocorticoid hormones (primarily aldosterone)

Zona glomerulosa

Patient with hypertension, low plasma renin activity, hypokalemia, and metabolic alkalosis. Overactivity of what structure is most likely responsible for symptoms?

Zona glomerulosa of the adrenals Patient has primary hyperaldosteronism (typically as a result of bilateral nodular hyperplasia of the zona glomerulosa or an aldosterone producing adrenal adenoma)

Inner zone of adrenal cortex that contains basophilic cells in anastomosing cords that secrete androgens

Zona reticularis

On jugular venous pressure tracings, the first peak is the a wave which is generated by what and is absent in people with what condition? prominent in which condition?

a wave is generated by atrial contraction and is notably absent in people with atrial fibrillation Note: a rapid y descent that becomes deeper and steeper during inspiration is often observed with constrictive pericarditis Note: hypertrophic cardiomyopathy is associated with a prominent a wave

Congenital hypothyroidism

additional symptoms are umbilical hernia, non pitting edema (puffy face) and protruding tongue! Initially normal at birth- symptoms develop after maternal T4 wanes!

3 year old with high grade fever, vomiting, altered mental status. PE shows hypotension, tachycardia, neck stiffness, and a petechial rash all over trunk and lower extremities. Rapidly develops cardiovascular collapse and bleeding from venipuncture sites and dies. Autopsy would likely show?

adrenal hemorrhage Patient has bacterial meningitis with DIC due to sepsis. Subsequent shock associated with meningococcal septicemia can cause bilateral hemorrhagic infarction of the adrenal glands leading to acute adrenal crisis (waterhouse-friderichesen syndrome)

Blockade of what receptor results in increased release of norepinephrine from peripheral nerve terminals causing increased BP and HR

alpha 2 adrenergic receptors

Males with classic, non salt wasting 21 hydroxylase deficiency present at age 2-4 years old with early virilization, increased linear growth, and elevated levels of 17-hydroxyprogesterone and androgens while females with classic 21 hydroxylase deficiency (with or without salt wasting) present with?

ambiguous genitalia AT BIRTH

Sciatica is a nonspecific term for low back pain that radiates down the leg due to compression of the lumbosacral nerve roots (from lumbar disc herniation, spinal foramina stenosis, etc.). S1 nerve root is commonly involved resulting in pain/sensory loss down the posteriorr thigh and calf to lateral aspect of foot. Patients may also have weakness on thigh extension, knee flexion, and foot plantarflexion with abscence of what reflex?

ankle jerk reflex (S1)

Reason for lack of long lasting immunity against Neisseria gonorrhoeae?

antigenic variation

Most common cause of intracranial hemorrhage in children?

arteriovenous malformation

Damage to the brainstorm at what level results in decerebrate (extensor) posturing?

at or below the level of the red nucleus (midbrain tegmentum, pons). Note: Damage to the neural structures above the red nucleus (cerebral hemispheres, internal capsule) results in decorticate (flexor) posturing

Low dose dopamine infusion stimulates D1 receptors in the renal and mesenteric vasculature, resulting in vasodilation & increased blood flow to these sites. Increasing doses of dopamine stimulate what receptors resulting in increased cardiac output and elevated systemic vascular resistance?

beta 1 and alpha 1 receptors Dopamine receptor binding: D1 > B1 > alpha 1 Note: at the higher end of the dose range, the increase in afterload can result in decreased cardiac output

Elevated prolactin leads to amenorrhea (inhibition of GnRH release), galactorrhea, and breast soreness. How does Risperidone and other antipsychotics cause hyperprolactinemia?

blocking D2 receptors on lactotrophs

A protein mixture obtained from a cell culture is separated by gel electrophoresis and subsequently transferred to a filter membrane. Labeled double stranded DNA probes are then used to detect a specific protein of interest in the same. What protein was most likely to be detected by this method?

c-Jun or c-Fos (nuclear transcription factors that directly bind to DNA via a leucine zipper motif). Southwestern blots are used to identify and isolate proteins that bind DNA such as transcription factors, nucleases, and histones. In this technique, the target protein binds to a labeled dsDNA probe that is homologous to the protein's regulatory sequence.

Pheochromocytoma is a catecholamine secreting tumor that presents with episodic HTN, tachycardia, headaches, diaphoresis, and tremors. Detection of elevated levels of what substances in the plasma confirms the diagnosis?

catecholamines and metanepherines

Cerebrotendinous xanthomatosis is a rare genetic disorder characterized by abnormal cholesterol & bile acid metabolism that follows a progressive course including development of xanthomas, cataracts, & pervasive neurological changes (dementia, seizures). Genetic defect disrupts cholesterol 7-alpha-hydroxylase (rate limiting step in bile synthesis). This leads to an accumulation of bile precursors most notably of which is what?

cholestANol (forms xanthomas)

Right sided colon cancers usually grow as exophytic masses and present with occult bleeding and symptoms of iron deficiency anemia (weight loss, pallor, progressive fatigue) while left sided colon cancers tend to infiltrate the intestinal wall and encircle the lumen causing?

constipation and symptoms of intestinal obstruction Note: recotosigmoid involvement often causes hematochezia Note: tenesmus (ineffectual/painful straining on defecation) and small caliber stools are characteristic of RECTAL adenocarcinoma

Foreign bodies (chicken, fish bones) can become lodged in the piriform recess and may damage the internal laryngeal nerve, impairing what reflex?

cough reflex Internal laryngeal nerve is a branch of the superior laryngeal nerve (CN X) and mediates the afferent limb of the cough reflex above the vocal cords Note: the internal laryngeal nerve does not carry motor fibers and mediates sensation mainly from the larynx and epiglottis (stimulation in these areas induces coughing, not gagging which is why the gag reflex is not impaired)

Atenolol is prescribed for HTN in 47 year old man. What cAMP changes would you expect in vascular smooth muscle, juxtaglomerular cells, and cardiac tissue?

decreased cAMP in cardiomyocytes and juxtaglomerular cells, and no effect on cAMP in vascular smooth muscle B1 adrenergic receptors are found in cardiac tissue and on renal juxtaglomerular cells, but not in vascular smooth muscle. B1 receptor is a GPCR associated with Gs. Note: nonselective beta blockers such as propranolol would result in a decrease in cAMP in all three areas

Sequence that hepatitis B replicates by?

double stranded DNA --> +RNA template --> double stranded DNA progeny Note: although it is a DNA virus, HBV replicates via reverse transcription

Polyprotein product of what HIV gene is glycosylated to become gp160

env gene

A moderately elevated alkaline phosphatase of unclear etiology should be followed up with?

gamma-glutamyl transpeptidase Bone and liver are the primary sources of alkaline phosphatase. Gamma-glutamyl transpeptidase is an enzyme that is predominately present in hepatocytes and biliary epithelia and not in bone, therefore it is useful in determining whether an elevated alkaline phosphatase is of hepatic or bony origin

gp160 is proteolytically cleaved in ER and golgi to form what two products?

gp120 & gp41

Fibrinolytic therapy for acute ST segment elevation MI is a reasonable repercussion technique for patients with no contraindications for thrombosis (although PCI is preferred). Fibrinolytic agents such as alteplase bind to fibrin in the thrombus and activate plasmin which leads to thrombolysis. Most common effect of thrombolysis?

hemorrhage (gastrointestinal, cerebral) Note: intracranial hemorrhage can present with asymmetric pupils and an irregular breathing pattern in a comatose patient several hours after thrombolytic is administered

Patients with chronic kidney disease may develop renal osteodystrophy (osteopenia, osteomalacia) from secondary hyperparathyroidism which is caused by?

hyperphosphatemia and hypocalcemia Note: deranged signaling between renal cells and bone osteoblasts and osteoclasts (FGF-23, Klotho) also contributes to the skeletal changes that occur with CKD. Renal failure results in accumulation of uremic toxins which have been shown to decrease peripheral conversion of T4 to T3 and could cause functional hypothyroidism.

The differential diagnosis of anxiety includes medical conditions and substance induced etiologies (intoxication, withdrawal). Anxiety due to a general medical condition is diagnosed when symptoms are the physiological consequences of an underlying medical condition such as?

hyperthyroidism, hypoglycemia, pheochromocytoma, hypercortisolism

Ehlers-Danlos syndrome is a group of rare hereditary disorders characterized but defective collagen synthesis. Patients often have joint laxity, hyerextensible skin, and tissue fragility due to the formation of soluble collagen that foes not properly crosslink. It can be caused by pro collagen peptidase deficiency which results in?

impaired cleavage of terminal propeptides in the EXTRACELLULAR space

MOA of Heparin?

increases the effect of naturally occurring anticoagulant antithrombin III Note: aPTT is measured to monitor therapeutic effect of Heparin

Patient with rheumatoid arthritis, denies bleeding, fecal occult blood testing is negative. Most likely findings for anemia?

low MCV, low Fe2+, high ferritin, low TIBC (Increase of hepcidin in anemia of chronic disease)

76 year old woman with a new diastolic murmur. Multiple cultures drawn at different times grow Enterococcus. ECG reveals mitral valve vegetation suggestive of endocarditis. Started on additional antibiotic and a few days later develops tinnitus and hearing loss. Antibiotic most directly affects what process?

mRNA genetic code reading Aminoglycodsides are associated with ototoxicity (hearing loss, tinnitus) and nephrotoxicity. They inhibit genetic code reading and protein synthesis by binding to the prokaryotic 30S ribosomal subunit. Also impact translocation.

Reverse transcription polymerase chain reaction (RT-PCR) is used to detect and quantify levels of mRNA in a sample. It uses reverse transcription to create a complementary DNA template that is then amplified using the standard procedure. RT-PCR can be used to diagnose CML by identifying?

mRNA transcript containing both BCR and ABL exons in affected cells (NOT fusion PROTEIN containing BCR and ABL domains because RT-PCR doesn't detect protein!) Note: RT-PCR cannot detect changes in parts of the gene that are not transcribed (promotors, enhancers), but PCR that use chromosomal DNA can

Function of gp120

mediates viral attachment by binding to CD4 receptor

RNA interference

miRNA undergoes processing in nucleus to form double stranded precursor --> cytoplasm --> cleaved by dicer --> individual strands separated and incorporated into RNA induced silencing complex (RISC) --> miRNA associated with RISC binds complementary sequences on target mRNA --> exact match results in degradation & partial match causes translational repression by preventing ribosome & transcription factor binding

Majority of overdose deaths in the USA are cause by?

opioids (both prescription analgesics & heroin)

Electron dense deposits on the epithelial side of the glomerular basement membrane (subepithelial humps) seen in patients with which condition? What are those humps composed of?

poststreptococcal glomerulonephritis Immune complexes composed of IgG, IgM, and C3

Nongonococcal urethritis, conjunctivitis, and arthritis is the classic triad of reactive arthritis. It is an HLA-B27 associated arthropathy that occurs within several weeks following a genitourinary or enteric function. It belongs to the group of seronegative spondoloarthropathies and can cause what in 20% of cases?

sacroilitis Note: skin findings include keratoderma blennorrhagicum (hyperkeratotic vesicles on the palms and soles) and circinate balanitis (serpiginous annular dermatitis of the glans penis)

Since only eons code for proteins, introns must be spliced out in the nucleus prior to translation. This occurs via what particles?

snRNPs

Health care proxy is a person legally designated to make medical decisions in the event the patient loses decision making capacity. The proxy overrules all other possible surrogate decision makers, including family members. Family members serve as surrogate decision makers in what order in the absence of a health care proxy or advanced directive?

spouse, adult children, parents, siblings

43 year old man with recurrent episodes of fever and sore throat despite multiple courses of antibiotic therapy. He has felt 'run down' and fatigue all the time. Bruises easily and has had bleeding gums on several occasions. Temperature is 100.2. On exam he has mucosal pallor, pharyngeal erythema, and multiple ecchymosis on extremities. Peripheral blood smear is shown. Most likely chromosomal abnormality?

t(15, 17) Patient has recurrent infections (neutropenia), pallor (anemia), and ecchymosis (thrombocytopenia). Peripheral blood smear shows abnormal myelocyte precursors containing coarse rod shaped intracytoplasmic granules called AUER rods. M3 variant (acute promyelocytic leukemia) of acute myelogenous leukemia is characterized by presence of promyelocytes on smear & is associated with DIC (bleeding, thrombocytopenia, prolonged prothrombin and activated thromboplastin time). Affected cells contain t(15,17) which is a translocation between retinoid acid receptor alpha gene on chromosome 17 and promyelocytic leukemia gene on chromosome 15. Forms chimeric gene product PML/RAR-alpha which codes for abnormal retinoid acid receptor. Manage with ALL TRANS RETINOIC ACID

Acute promyelocytic leukemia is the M3 variant of acute myelogeous leukemia. Affects adult patients and may present with disseminated intravascular coagulation. What is cytogenetic abnormality and what does it lead to?

t(15,17) leads to formation of the promyelocytic leukemia-retinoic acid receptor alpha (PML/RARa) fusion gene, which is unable to signal for proper cellular differentiation, unlike the normal retinoic acid receptor. Treat with all trans retinoic acid- stimulates PML/RARa proteolysis & differentiation of myloblasts into granulocytes

Direct arteriolar vasodilators (hydrazine, minoxidil) lower blood pressure by reducing systemic vascular resistance but trigger reflex sympathetic activation and stimulate the RAAS axis resulting in common side effects of?

tachycardia and edema (sodium and fluid retention) To counteract such compensatory effect, these agents are often given in combination with sympatholytics and diuretics.

Aminoacyl tRNA synthetase is the enzyme responsible for loading the appropriate amino acid on what portion of the tRNA?

terminal hydroxyl group of the 3' CCA tail of tRNA (amino acid binding site)

HbF contains gamma globin instead of beta globin. Patients with beta thalassemia major are asymptomatic at birth due to the presence of gamma globins and HbF. Switching to the HbA production and the cessation of gamma globin synthesis precipitates symptoms of beta thalassemia. What is embryonic hemoglobin (gower) composed of?

two zeta and two epsilon chains

Humans acquire schistosomiasis via contact with freshwater sources that contain snails infected with Schistosoma larvae. S. japonica and S. mansioni cause intestinal and hepatic schistosomiasis and S. haematobium causes?

urinary schistosomiasis (terminal hematuria, dysuria, frequent urination, hydronephrsis, pyelonephritis, squamous cell carcinoma of the bladder)

HIV DNA integration into host genome is catalyzed by?

viral integrase

9 year old girl with prolonged epistaxis. Patient has had frequent nose bleeds that often last longer than 10 minutes. Family history is significant for a grandfather who had an unspecified bleeding disorder. Bleeding time and PTT are prolonged. Thrombin time and prothrombin time are normal. Most likely diagnosis?

von Willebrand disease vWD is most common inherited bleeding disorder. It has an autosomal dominance pattern of inheritance and variable penetrance. Absence of vWF leads to impaired platelet function (prolonged bleeding time) and coagulation pathway abnormalities due to decreased factor VIII activity (prolonged PTT)


Kaugnay na mga set ng pag-aaral

kinesiology ch 9 function of the movement of the hand

View Set

Texas Law of Agency: Mini-Quizzes

View Set

ATI: Chapter 22: Adolescents (12 to 20 years)

View Set

¿Qué hacen los estudiantes? use the pictures to figure out what each student brings to class

View Set

LEGL 2700 Test 1 (chp. 1, 3, and 4)

View Set

Finance and Investment Cumulative Quiz

View Set

CSULB-12A Introduction To Cybersecurity EXAM REVIEW

View Set

Sociology ch. 8 Social Stratification, class and inequality

View Set

Appropriate Quotation - MLA Style

View Set

Exam #5: The Central Nervous System

View Set